تغطية شاملة

أمر رئيس إحدى الجامعات في ولاية إنديانا المحاضر بالتوقف عن تدريس نظرية التصميم الذكي كعلم شرعي

وروج المحاضر، وهو أستاذ مساعد في جامعة BSU، والذي يدرس مقرر "حدود العلم"، بحسب مؤسسة حرية الدين، المنهج المسيحي على المنهج العلمي. وفي الرسالة التي وزعتها على أعضاء هيئة التدريس، قالت إن التصميم الذكي هو موقف ديني رفضه العلم منذ فترة طويلة

تمثال يعد أيضًا رمزًا لجامعة بيل ستيت في ولاية إنديانا. من ويكيبيديا
تمثال يعد أيضًا رمزًا لجامعة بيل ستيت في ولاية إنديانا. من ويكيبيديا

أعلنت رئيسة جامعة بول ستيت في ولاية إنديانا (جامعة بول ستيت)، جو آن جورا، يوم الأربعاء الماضي، أنه لا ينبغي للمدارس العامة تدريس التصميم الذكي أو نظرية الخلق في فصول العلوم أو التعامل معها على أنها حقيقة. يأتي ذلك عقب الكشف عن منكر للتطور بين أعضاء هيئة التدريس بجامعة إنديانا، حيث يدعو رئيس جامعة الولاية إلى عدم تدريس نظرية الخلق تحت ستار العلم.

إن مفهوم التصميم الذكي، الذي يرفض التطور، يرى أن القوة الذكية، وليس العمليات غير المنضبطة مثل الانتقاء الطبيعي، هي التي تقف وراء خلق الكون والمخلوقات الحية. وجاء بيان جورا ردًا على شكوى وردت بشأن أستاذ جامعي قام بتدريس التصميم الذكي كعلم. "يتم تعريف نهج التصميم الذكي من قبل المجتمع العلمي على أنه اعتقاد ديني وليس كنظرية علمية." جورا يكتب في رسالة إلى أعضاء هيئة التدريس في المؤسسة. "لذلك فإن التخطيط الذكي ليس محتوى مناسبًا لدروس العلوم."

جاء هذا الإعلان ردًا على رسالة من مؤسسة الحرية الدينية، التي أعربت عن قلقها من أن إريك هادين، الأستاذ المساعد في جامعة ولاية بنسلفانيا، انتهك الفصل الدستوري بين الكنيسة والدولة بالطريقة التي قدم بها التصميم الذكي في فصل العلوم المحترم. تحت عنوان "حدود العلم". وبحسب موقع "إنسايد للتعليم العالي"، تعارض المؤسسة أن يشارك دينغ معتقداته ويفرض وجهة النظر المسيحية على وجهات النظر الأخرى.

يمكن الآن مناقشة التصميم الذكي في جامعة BSU في دورات العلوم الاجتماعية والإنسانية التي تركز على الأديان، ولكن لا ينبغي إعطاء هذا النهج وزنًا مختلفًا عن المناهج الأخرى. إن السماح بعرض الموضوع في فصول العلوم كنظرية علمية صحيحة لا يمثل المجتمع العلمي، وبالتالي لا يفي بمتطلبات المعايير الأكاديمية. وقال جورا إن القضية إذن ليست قضية حرية أكاديمية، بل قضية نزاهة أكاديمية.

في النهاية، تقرر أن تظل دروس دين خالية من التحريف لمنهج التصميم الذكي ووافق دين على التعاون. لا يزال من غير الواضح ما إذا كان سيستمر في تدريس دورة "حدود العلوم".

في مؤسسة الحرية هناك دين عرقي، كما هو الحال مع خبير التطور البروفيسور جيري كوين، أستاذ علم البيئة والتطور في جامعة شيكاغو وصاحب مدونة "لماذا التطور صحيح". وفي معهد ديسكفري، من ناحية أخرى، يقول المبادر والمروج لهذا النهج، الذي له تأثير في جميع أنحاء العالم، بما في ذلك الحاخامات معنا، إن "موقف جورا ضد الحرية الأكاديمية، وهو أورويل في أقصى تطرفه، كما أنه يتنازل عن عضو هيئة التدريس الذي سُرقت مشترياته الروحية."

ما بعد النصي. على مكتب المتحدث الرسمي باسم وزارة التربية والتعليم منذ أكثر من أسبوع طلب من الموقع للرد على طلب جمعية الحرية التي تطلب تدريس التطور ليس فقط لأولئك الذين اختاروا 5 وحدات من علم الأحياء، ولكن لجميع الطلاب، نظرًا لأن هذه النظرية هي أساس علم الأحياء بأكمله، لكنهم ما زالوا يختارون الانتظار، ربما سأستسلم.

تعليقات 889

  1. شموليك

    أنت دائما الملام. مثلي.

    وكما ذكرنا، أرى أين يكمن خطأ أينشتاين المزعوم. سؤالي هو: كيف لم يرى؟

    ولم يتغير شيء أساسي في فهمنا لميكانيكا الكم واللامكانية منذ عام 1935. الشيء الوحيد الذي فعله بيل هو النظر بعمق في التفاصيل الصغيرة، حيث من المعروف أن الشيطان موجود. لكن لم يحدث أي تقدم فيما فعله، بما في ذلك نظرية المتباينة (وهي نظرية رياضية بحتة).

    أقول "خطأ واضح" لأن أينشتاين لم يُمنح الفرصة للدفاع عن نفسه. لقد مات معتقدًا أنه وجد الثغرة في نظرية الكم المكروهة. وكما أعرف الثعلب العجوز، فإنه سيخرج من هذه الأزمة ويجد حجة جديدة لضرب منافسه وصديقه العزيز نيلز بور.

    فيما يتعلق بالفوتون، يرجى ملاحظة: في QED، الفوتون ليس رصاصة تشق طريقها من البندقية إلى الهدف، ولكنها تمر عبر كل مسار ممكن تقريبًا في الطريق. "كل إلكترون بمفرده يسير بكل الطرق الممكنة في نفس الوقت: بطريقة لطيفة ومستقيمة.. فجأة يغير اتجاهه، ويقطع الطريق الطويل إلى مجرة ​​المرأة المسلسلة، حيث يستدير ويعود إلى الخلفية" براين جرين ، "الكون الأنيق" ص١٢٢.

    لذا، إذا كان الإلكترون - أو أي جسم كمي آخر مثل الفوتون - يكفي لزيارة مجرة ​​المرأة المسلسلة خلال رحلتها القصيرة من المصدر إلى الكاشف، فلا بد أنه يتحرك بسرعات كبيرة، وبالتأكيد أسرع من الضوء، أليس كذلك؟

    وفاينمان هو من يقول ذلك، وليس أنا.

    لإعطاء ميزة لفكرتي، قد ترغب في إلقاء نظرة على رابط منذ عامين:

    https://www.hayadan.org.il/maby-neutrino-didnt-pass-speed-of-light-181011/comment-page-7/#comment-312259

    كما تحب: قصص، أغاني، أمايات..

    ويوصف هناك ظاهرة قوس قزح، الذي يكون دائما على مسافة ثابتة من كل مساح، بغض النظر عن موقعه. من المستحيل الوصول إليه، ومن المستحيل الوقوف تحته، ومن المستحيل تجاوزه.

    يذكرنا قليلا بسرعة الضوء، أليس كذلك؟

    وفي حالة قوس قزح، فالجواب هو أن قوس قزح موجود بالفعل في كل مكان - بما في ذلك موقع المشاهد - ولكنه لا يستطيع رؤية الانعكاسات التي يتعرف عليها إلا على مسافة معينة وثابتة.

    هذا هو اتجاهي لجعل المسلمة 2 نظرية.

  2. ميخائيل

    أنا آسف حقًا إذا جرحتك بأي شكل من الأشكال، بالتأكيد لم تكن نيتي. من فضلك أرني أين وماذا، حتى أتمكن من التعلم والتصحيح في المستقبل.

    وطبعاً من حقك أن تنسحب من المناقشة، مع أن هذه هي المرحلة التي هي في أمس الحاجة إليك في رأيي.

    والسبب بسيط: إذا قرأت ردي من المقال قبل عامين، فقلت فيه: "أعتقد أنه من الممكن بناء نموذج، بالتأكيد في ذهني، ولكن أيضًا في الممارسة العملية، في المختبر، من شأنه أن يبني نموذجًا". تنتج الجاذبية (حسب ليساج) دون احتكاك (وهو ما أشار إليه فاينمان في نموذج ليساج)، والقصور الذاتي، والأهم من ذلك: موجة ستنتشر بنفس السرعة في كل إطار مرجعي.

    وإلى ذلك سأضيف: احتمال غير المحلة

    عندما أقول "النموذج العقلي" أعني نموذج الكمبيوتر.

    ومن هو الأنسب لبناء نموذج الكمبيوتر هذا؟ تخمين واحد... حتى لو كان ذلك فقط لإثبات كذبة إسرائيل وأنه على خطأ. إذا سألت، سأكون سعيدًا بإعطائك الافتراضات الأساسية للنموذج.

    الكشف الكامل - لست متأكدًا من أن النموذج سوف يؤدي إلى القصور الذاتي. وفي رأيي أنها بلا شك ستؤدي إلى الجاذبية حسب ليساج دون مشكلة احتكاك فاينمان، وإذا غيرنا توزيع التصادمات بين الجزيئات، فإن موجة تنتشر بنفس السرعة لكل قاس، بغض النظر عن سرعة الكيل.

    وهذا ما نبحث عنه، أليس كذلك؟

  3. وكما قلت، قررت أن أتوقف عن المشاركة في المناقشة لأن كل ما "أستفيده" منه هو الانزعاج من طريقة التعامل مع كلامي.
    سأضيف ذلك الآن فقط - بالإضافة إلى حقيقة أن جميع الاستنتاجات التي توصلت إليها إسرائيل حول النظريات الموجودة في رأيي خاطئة، فإنني أتراجع أيضًا عن توصيتي بأن يقدم نظريته إلى هيئة علمية ما.
    أمتنع عن كتابة السبب لأنني، كما ذكرت، سئمت النقاشات التي لا تنتهي والمعاملة التي تتلقاها كلماتي.

  4. إسرائيل،
    لأني مذنب...
    مرة أخرى، لماذا فكر أينشتاين وكل هذا ليس سؤالًا يمكن الإجابة عليه. ومن المفترض أنه افترض، كما فعلت أنت، أن المعلومات حول الدوران هي مع ذلك معلومات وبالتالي فإن "المعلومات" تنتقل بسرعة أعلى من سرعة الضوء وهذا يتناقض مع افتراض النظرية النسبية. ويحل الفيزيائيون اليوم جزءًا من المشكلة من خلال الادعاء بأنه لا توجد طريقة للتأثير على السببية من خلال هذا التأثير. ما اكتشفه بيل في الستينيات (!) هو أنه لا توجد نظرية متغيرة مخفية محليًا يمكن أن تكون جيدة مثل ميكانيكا الكم، وإذا قرأت الويكي عن المفارقة، فسوف ترى أنه لا يوجد في الواقع تفسير جيد للتشابك الظاهرة وهذا أمر رائع حقًا لأنه يعني أن لدينا الكثير لاستكشافه.

    ميكانيكا الكم تحيرنا مرارا وتكرارا لأنها تجبرنا على تغيير المفاهيم الكلاسيكية، سواء أردنا ذلك أم لا. قدمت المقالة (وليس فقط أينشتاين) ادعاءات حول ماهية "الواقع المادي" بناءً على حدسنا النيوتوني (وهو المغالطة) وأعطته ميكانيكا الكم "لا" ولكن بعد ذلك، لم تكن التجارب ذات الصلة قد أجريت بعد، وبالتالي فقد تم إجراءها كان جيدًا في تقديم الادعاءات التي تم تقديمها ولكن عندما تم إثبات النظرية مرة أخرى ومرة ​​أخرى، لم يعد أمامنا خيار سوى التخلي عن المفاهيم التي صاغها المقال. فمن ناحية، تتنبأ النسبية بنجاح بالعديد من الظواهر، لذلك نميل إلى التفكير فيها على أنها "صحيحة". ومن ناحية أخرى، نعلم أن "المفارقة" ليست مفارقة لأن النتيجة تم تأكيدها بالتجارب. والطريقة للتوفيق بين هاتين الحقيقتين هي ثني النظرية النسبية لأن الواقع يتفوق على أي نظرية. أود أن أذكر مرة أخرى أن أينشتاين، مثل كل الناس، يمكن أن يرتكب خطأ أو يستخدم مفاهيم سيجعلها المستقبل غير ذات صلة. هذا بالضبط ما حدث.

    فيما يتعلق ببيل، فإن الادعاء بأن شيئًا لم يتغير خلال 30 عامًا هو ادعاء كاذب، حيث يتم تحقيق اختراقات رياضية طوال الوقت، وكذلك الأمر بالنسبة للتقنيات الرياضية التي يتم تحسينها باستمرار. من المؤكد أنه ليس من التافهة الادعاء بأن نتيجة بيل كان من الممكن أن يتم التوصل إليها بالفعل في عام 1930، ومن الواضح تمامًا أن مفارقة EPR والنتائج المكتشفة نتيجة للرغبة في اختبار المفارقة هي الدافع وراء بيل. هكذا يتقدم العلم. بالمناسبة، في بداية الشهر كان هناك مؤتمر نوبل مثير للاهتمام للغاية والذي يمكنك مشاهدته على اليوتيوب. البروفيسور قال جيمس جيتس (الذي كانت محاضرته الأولى التي ألقاها في المؤتمر تقريبًا مثل بنك التنمية المتعددة الأطراف، كم هو رائع) ذلك بالضبط. لقد أجرى مقارنة لطيفة بين الموسيقى والرياضيات، وقال إنه مثلما يتم إنشاء أعمال جديدة طوال الوقت، كذلك في الرياضيات، كذلك مع الأسلوب. وقال إن الأسلوب الرياضي الذي كان يُمارس منذ 100 عام كان مختلفًا تمامًا عن الأسلوب الذي يُمارس اليوم. وهذا رابط المحاضرات مرة أخرى (معظمها رائع!) وسأكون سعيدا لو قام العالم بمتابعة بعض الأشياء التي قيلت هناك، مثل حقيقة أنه بعد الانفجار الكبير الكون تباطأ توسعه إلى نقطة معينة ثم انعكس الاتجاه. ليس رائعا؟
    http://www.youtube.com/watch?v=F1P0HNhQE68&list=PLHuAoPzfQhGGQzB58S1iVQvu43Xvbp2c4

    فيما يتعلق بشفرة أوكهام، فإن هذا المبدأ ليس مبدأ ملزمًا بأي حال من الأحوال، ولكنه على الأكثر قاعدة عامة.

    أحب أن تشرح بطريقة طويلة ومفصلة (أيضًا بطريقة سردية) كيف يمكن للفوتون أن يتحرك بسرعات لا نهائية، ومع ذلك أقترح مرة أخرى منتدى كراوس لأنه يبدو أنك وصلت إلى نوع من الإرهاق هنا، أو وصل مايكل . أعتقد أن اللغة الإنجليزية ليست عائقًا بالنسبة لك 

  5. أبي

    تم وصف النظرية في عدة تعليقات في المناقشة التي أجريتها مع ر. (وماذا عنه بالمناسبة؟ أين ذهب؟) وابدأ بهذا الرد:

    https://www.hayadan.org.il/astronomers-reach-new-frontiers-of-dark-matter-130112/comment-page-11/#comment-326491

    والمهم بالنسبة لنا هو الرد التالي:

    إسرائيل شابيرا

    لقد عدنا من الجبال.
    يكون الأمر سهلاً جدًا عندما يكون منزلك في الجبال. ما عليك إلا أن تفتح الباب، وستجد الجبال هناك.

    روبي
    لم يبدأ الكون من حالة من الفوضى الشاملة. على العكس من ذلك، وفقا لنظرية الانفجار الأعظم، كانت حالتها الأولية منظمة للغاية.

    اليوبيل.

    لست متأكدًا من أنك فهمت أوكهام أو المنهج العلمي بشكل صحيح.

    خذ الهندسة على سبيل المثال. بديهياتها بسيطة وأساسية، ولكن يمكنك استخدامها لبناء المزيد والمزيد من الجمل المعقدة، والتوصل إلى حسابات معقدة إذا لم تتعلم كيف تم التوصل إليها، ستعتقد أن الطبيعة لم تكن قادرة على بناء مثل هذا شيء معقد في حد ذاته (لماذا حجم الهرم حاصل ضرب المساحة في الارتفاع مقسومًا على 3؟ ومن أين جاء فجأة الـ 3؟).

    تتطور الفيزياء بطريقة مماثلة: من البسيط إلى المعقد.

    وخير مثال على ذلك هو الموضوع المعروض علينا: الموجات الجيبية. يبدو شيئًا معقدًا للغاية: موجة تنتشر في المكان والزمان بطريقة معقدة للغاية. المعادلة التي تصف انتشار الموجات الجيبية هي معادلة لعدة متغيرات، للزمان والمكان. يابرادي، يا لها من فوضى! هل يمكن أن تكون الطبيعة قد نظمت مثل هذا الشيء المعقد، أو ربما القصة بأكملها موجودة فقط في العقول المحمومة للفيزيائيين المختلين؟

    ومع ذلك، إذا قمنا بتجميد الموجة الجيبية في بعدين تتحرك للأمام عبر الزمن - من خلال تصوير مثل هذه الموجة في سلك - فسنرى الموجة الجيبية بكل مجدها. وهكذا قمنا بتحييد عامل الوقت.

    والآن ماذا سيحدث إذا حلقنا فوق الموجة وقمنا بالتقاط مقطع فيديو لها؟ ستظهر لنا الموجة كحركة توافقية بسيطة. وهكذا قمنا بتحييد عامل المسافة.

    ولكن ماذا عن الحركة التوافقية البسيطة؟ كما أنها معقدة للغاية، أليس كذلك؟ كيف خلقت الطبيعة البسيطة مثل هذا الشيء المعقد؟

    هذا كل شيء، إذا أخذنا عجلة دوارة، وقمنا بتصوير مقطع فيديو لإسقاط الضوء على نقطة معينة في العجلة على الطاولة، فسنحصل على حركة توافقية بسيطة.

    وهكذا وصلنا من موجة جيبية ثلاثية الأبعاد تتحرك عبر الفضاء كدالة لثلاثة أبعاد للمكان + البعد الزمني، إلى غطاء علبة النسكافيه التي يمكن تدويرها على الطاولة.

    ونفس الشيء مع المعالجة الرياضية للموجة: ما عليك سوى قطع الموجة المعقدة جزئيًا، حتى تصل إلى الصف السادس في الجبر.

    هذه هي الطريقة التي بنيت بها كل الفيزياء. نبدأ بـ F=MA، ونصل إلى هبوط مركبة فضائية غير مأهولة على المريخ.

    وفيما يتعلق بسؤالك: نموذج الطريق السريع هو ببساطة نموذج ماكسويل للنظام المفتوح.

    النظام المغلق هو نظام مثل غرفتك، حيث يكون الهواء في حالة سكون نسبة إلى الغرفة، أو نسبة إلى الهواء المتطاير، أو نسبة إلى الهواء. سيكون للموجة الصوتية التي تتحرك عبر الهواء في مثل هذا النظام سرعة ثابتة (سرعة الصوت) بالنسبة لذلك النظام المرجعي (الغرفة، الهواء، وما إلى ذلك).

    ولكن ماذا سيحدث لمثل هذه الموجة في نظام مفتوح مثل كوننا؟ نسبة إلى ما سوف تقدم؟ (بعد كل شيء، هذا هو بالضبط ما حاولت التجربة M-M العثور عليه).

    إجابتي هي أنه في النظام المفتوح ستتقدم الموجة بجميع السرعات، لأنه في النظام المفتوح ستتحرك جزيئات الهواء بجميع السرعات، من 0 إلى ما لا نهاية. لكننا كقياسيين لا نستطيع قياس سوى سرعة واحدة: سرعة الصوت بالنسبة إلينا.

    استبدل جزيئات الهواء بجزيئات ليساج، سرعة الصوت مع سرعة الضوء، راجع الشرح التفصيلي الذي قدمته في المقال السابق لماذا أعتقد أن هذا ضروري، وإليك نموذج الطريق السريع.

    وشيء آخر ينبثق من وصف النظام المفتوح: كيف يمكن أن تنجح تجربة MM؟ إذا كان الكون لا نهائيًا متناحي الخواص ومتجانسًا، ألا يعادل العثور على نظام سكون للموقع إيجاد مركز خط مستقيم لا نهائي، وهو أيضًا لانهائي متناحٍ ومتجانس؟

    وقيل إن مايكلسون هو من وجدها، وأنها كانت تتحرك بالنسبة إلينا بسرعة 4576 كم/ث باتجاه المرأة المسلسلة. فلماذا هذا؟ ماذا عن التجانس؟ ألا يعادل هذا إيجاد مركز الخط المستقيم اللانهائي الذي يبعد عنا 4576 كيلومتراً في الاتجاه الموجب للمحور السيني؟

    4 أبريل 2012

    والشيء الرئيسي:

    ولا يتحرك الفوتون بسرعة واحدة فقط - وهي سرعة الضوء - ولكن بكل السرعات، من سالب ما لا نهاية إلى ما لا نهاية.

    ونحن، باستخدام حواسنا وأدواتنا، نستطيع قياسها بسرعة واحدة فقط: سرعة الضوء.

    لقد شرحت السبب في هذا المقال الطويل. يسعدني أن أشرح مرة أخرى إذا طلب منك ذلك.

    ميكانيكا الكم تدعم هذا النهج تمامًا. إن التأكيد على أن الفوتون - أو أي جسم كمي - ليس له موقع محدد قبل الانهيار، ويمكن أن يكون في أي مكان في الكون مع احتمال معين، يتطلب سرعة أعلى من سرعة الضوء، في الواقع لا نهائية.

    وغير محلية أيضا.

    هلوسه؟ ربما. والبديل هو قطارات يبلغ طولها مليمترات دون علم الركاب بداخلها، يراقبون المستقبل ويؤثرون على الماضي.

    نأمل أن يكون الرد ناجحا، اي فون.

  6. ميخائيل

    تستعد العائلة بأكملها لرحلة إلى بالم سبرينغز، لذلك سيكون من الصعب علي الرد بطريقة منظمة في اليومين المقبلين. وفي هذه الأثناء، إليك سؤال:

    انت كتبت:

    "في رأيي (الذي سبق أن عبرت عنه دون جدوى) أن ما أزعج أينشتاين بشأن اللا محلية هو إمكانية نقل المعلومات عبر مسافة وانتهاك مبدأ السببية".

    إذا كنت على دراية بتجربة الاختيار المؤجل لـWeiler

    https://www.hayadan.org.il/quantun-philospy-part-b-07121/

    إذن، هذا هو بالضبط ما يتحدث عنه: التأثير على الماضي من المستقبل.

    وهذا اقتباس من مقال غالي:

    "إذا قال ألبرت أينشتاين عن التشابك الكمي إنه "عمل شبحي عن بعد"، فإن تجارب الاختيار المتأخرة تطاردها الأشباح..."

    العبرية غير منتظمة بعض الشيء، ولكن المعنى لا يزال واضحا. ألا تنتهك هذه التجربة مبدأ السببية؟

    إذا كنت توافق على ذلك، وإذا قبلت أن أينشتاين (حسنًا، وليس أوكهام) كان سيغمى عليه على الفور من التفسير الحالي للتجربة، وإذا كنت توافق أيضًا على أن أي شيء أفضل من عكس السبب والنتيجة، فيمكننا المضي قدمًا إلى نظريتي.

    في رأيي، الأمر أقل خطورة بكثير.

  7. إسرائيل:
    لن أتعاون مع دور الفزاعة الذي ألقيته عليّ في كل رد تقريبًا عندما تعيد اختراعي.
    أنت تأخذ كل كلمة أقولها تقريبًا خارج سياقها وتحاول تحريفها.
    كما أنني لست مستعدًا لنسيان المرات العديدة التي طلبت منك فيها تقديم نظريتك (بما في ذلك الوقت الحالي، الذي كان يمكن لأي شخص عاقل أن يفسره على أنه طلب لتفصيل النظرية).

  8. "من الجميل أن تعترف بأن التغيرات في المسافة والزمن هي نتيجة للافتراض الذي وضعته النسبية حول ثبات سرعة الضوء - وهو الافتراض الذي تم إثباته في مئات التجارب."

    أعتقد أنك قرأت نصف التعليق فقط.

    هنا هو الأصلي:

    "ومن ناحية أخرى، يبدو أنه لا مفر من هذه النتائج الغريبة، لأنه إذا سلمنا أن سرعة الضوء ثابتة دائما، والسرعة هي مقياس المسافة مقسوما على الزمن، فإن ما يتغير حتما هو المسافة والزمن.

    وسؤالي هو: هل هذا صحيح؟

    كان من الممكن أن يكون استنتاجك "من الجيد أن تعترف بذلك" صحيحًا لولا ظهور الكلمتين "يُزعم" و"في الواقع" بوضوح في الفقرة.

    "إن العرض كما لو لم يتم قبول بعضهم في التجربة هو بالطبع مضلل".

    ولا علم لي بتجربة تؤكد تقصير الطول. حول تأكيد النسبية الذي قدمته قبل بضعة أسابيع، يقول أنه لا توجد مثل هذه التجربة. هل يمكنك الإشارة إلى مثل هذه التجربة؟ ربما بعض التجارب حيث يتم ضغط قطار يبلغ طوله كيلومترًا إلى ملليمتر؟

    "والأصح أن نقول إنه لم يتم بطلان أي منها قط (وهو الاختبار العلمي لنظرية ما)"

    ولم يتم دحض وجود الله أبدًا. أيضا هبوط كائن فضائي والتواصل مع الموتى.

    "في رأيي (الذي سبق أن عبرت عنه دون جدوى) أن ما أزعج أينشتاين بشأن اللا محلية هو إمكانية نقل المعلومات عبر مسافة وانتهاك مبدأ السببية".

    المعلومات تمر بلا شك. دوران الإلكترون أو استقطاب الفوتون.

    "إنه لم يفكر في حقيقة أن الأمر لا يتعلق بنقل المعلومات."

    لم أفكر؟ أينشتاين في أهم ورقته منذ النسبية العامة؟ ؟؟؟؟

    "هذا هو الفرق بينه وبين علماء الفيزياء اليوم."

    لا يوجد اختلاف كبير في المعرفة اليوم عن تلك التي كانت موجودة في عام 1935. إذا كنت أتذكر بشكل صحيح، فإن إيهود يدعي أن تفسير إنقاذ النسبية من تجربة أسبكت ضيق للغاية.

    "بافتراض أنك تقبل الرأي العام حول موضوع تجربة EPR، فأنا مهتم بمعرفة ما إذا كان لا يزال هناك، في رأيك، تجربة تتعارض مع النظرية النسبية."

    لا. ولا أدعي أن هناك أي خطأ في النسبية. وأزعم أنه يمكن أن يكون هناك تفسير آخر ليس مسلمة تترك تحديد سرعة الضوء في جميع الأنظمة المرجعية، وأن إطالة الزمن لا تتوافق مع الزمن المطلق للانفجار. دعونا نتحدث عن الانفجار. أين نهاية الكون؟ ماذا كان قبل الانفجار؟

    "بدأت المناقشة بأكملها بادعاءات بوجود اتصالات في إشعاع الخلفية وليس لها علاقة بـ EPR."

    بدأت المناقشة من التخطيط الذكي في ولاية إنديانا. لقد انتقل إلى EPR بسبب اهتمام شموليك، ولكن في الواقع يرتبط EPR تمامًا بإطالة الزمن.

    "هل يشير تسربه إلى EPR إلى أنك تلقيت تفسيرات للأخطاء في التجارب الفكرية مع عمر الكون وإشعاع الخلفية؟"

    لا سمح الله. إذا كان هناك تمدد زمني وكان هناك بالفعل عمر للكون، فمن الممكن نظريًا تجاوز هذا العمر.

    "قل فقط إذا كنت قد فهمت أن التجارب التي اقترحتها بشأن عمر الكون كانت مبنية على قطار لا يمكن بناؤه خلال العمر المحدود للكون."

    في كل قطار يمر بنا، وبافتراض أن عمر أول سيارة هو عمرنا، فبحسب طول المدة تكون السيارة الأخيرة أكبر منا. حتى في قطار الوادي.

    "إن إشعاع الخلفية عبارة عن ساعة مثل كل الساعات."

    هل هناك ساعة واحدة من بين كل تلك الساعات عمرها أعلى من عمر إشعاع الخلفية؟ لأنه بحسب النسبية هناك عدد لا يحصى من الساعات التي يقل عمرها عن عمر الإشعاع (زوفر عوفر، الفوتونات المندفعة، التوائم).

    إذا كانت الإجابة على السؤال بنعم، يرجى الإشارة إلى هذه الساعة العتيقة.

    إذا كانت سلبية، فأنت نفسك خصصت ساعة إشعاع الخلفية كأحدث ساعة.

    "أنت تشير إلى أن هناك احتمال آخر إلى جانب النسبية".

    التلميحات والأسرار لها مكانها في الكابالا والسحر. أنا أزعم أن هناك تفسيرًا فيزيائيًا محتملاً لثبات سرعة الضوء يترك الوقت والمسافة المطلقين لنيوتن وماكسويل سليمين.

    وهذا لا يعني أن التفسير صحيح، بل ربما يكون خاطئا. ولكن هناك تفسيرات أخرى.

    "أنت (والفيزيائي الذي تذكره أحيانًا) تتجنب تقديم البدائل التي تعتقد أن لديك، فما هي الإجابة التي تتوقعها؟"

    نفس الفيزيائي، كما ذكرت سابقًا، ادعى أن هناك مشكلة في العلاقات الصحيحة. اختلفت معه، بل وعرضت السؤال على عوفر ماجد، الذي حكم لصالحي. أعتقد أنه على الرغم من أنه لم يعترف بذلك، إلا أنني تمكنت من أن أوضح له أنه لا توجد مشكلة في العلاقات دون التناقض مع زمن الانفجار المطلق.

    أنا لا أعرض بديلي، لأنه لم يطلب مني تقديمه. قبل عامين، تم طرح هذا السؤال عليّ، وقد قدمته في مقال في Bidian. إذا سألت من يفهم الموضوع سأقدم.

    والمناقشة هنا وفي نفس المقال تقنعني أكثر فأكثر أن الطريقة الوحيدة لإثبات أو نفي هذا البديل هي من خلال التجربة. المشكلة هي أنها ليست قاعدة بسيطة، وأنا أتعلم من الأخطاء طوال الوقت وأتقن التجارب. ومع ذلك، فإن الطريق طويل.

    "وسؤال أخير: لماذا لم تقدم أسئلتك إلى كراوس بعد؟"

    وبما أن حجتي الرئيسية هي دعم أوكهام، وكما رأينا في المناقشة هنا، يمكن لأي شخص أن يجادل بأن أوكهام معه، أفضل الانتظار الآن، ويرجع ذلك أساسًا إلى ضيق الوقت (والدي لا يزال هنا معنا و وتيرة الأحداث مرهقة حتى بدون مزيد من النقاش).

    بالحديث عن ذلك - كيف حال والدك؟

  9. إسرائيل:
    ومن الجميل أن تعترف بأن التغيرات في المسافة والزمن هي نتيجة الافتراض الذي وضعته النسبية حول ثبات سرعة الضوء - وهو الافتراض الذي تم إثباته في مئات التجارب.
    من المهم أن نذكر هذا حتى يُفهم أن كل الإشارات إلى أوكهام كانت مجرد تجديف (على أوكهام).

    وبالفعل - تصوير زحل من مركبة فضائية تتحرك من منطقة الأرض بسرعة (ثابتة) نحوه سيظهر زحل أكبر (وما العجب؟ في نظامه هو أقرب).
    وإذا فاتتها زحل ومرت به بدا لها أيضًا أضيق (من الجانب).

    تم الحصول على هذه النتائج نتيجة لحساب النتائج المترتبة على تحديد سرعة الضوء، ومن الواضح أن تقديمها كما لو لم يتم الحصول على بعضها في التجربة أمر مضلل. لم يتم قبولهم في التجارب حيث لم يكن من المفترض قبولهم بسبب قيود القياس لدينا.
    والأصح أنه لم يتم بطلان أي منها قط (وهو الاختبار العلمي لنظرية ما)، بل تم قياس بعضها.
    وفي رأيي (الذي سبق أن عبرت عنه دون جدوى) أن ما أزعج أينشتاين بشأن اللامكانية هو إمكانية نقل المعلومات عبر مسافة ومخالفة مبدأ السببية.
    إنه لم يفكر في حقيقة أن هذا ليس نقلًا للمعلومات.
    وهذا هو الفرق بينه وبين فيزيائيي اليوم، لكنني قلت ذلك بالفعل.
    على افتراض أنك تقبل الرأي العام حول موضوع تجربة EPR، فأنا مهتم بمعرفة ما إذا كان لا يزال هناك، في رأيك، تجربة تتعارض مع النظرية النسبية.
    بدأت المناقشة بأكملها بادعاءات بوجود اتصالات في إشعاع الخلفية وليس لها علاقة بـ EPR. هل تسريبه إلى EPR يشير إلى حصولكم على تفسيرات للأخطاء في التجارب الفكرية مع عمر الكون وإشعاع الخلفية؟
    لا تحتاج إلى تقديم التجارب مرة أخرى - فقط قل إذا قبلت أن التجارب التي اقترحتها مع عمر الكون كانت مبنية على قطار لا يمكن بناؤه خلال العمر المحدود للكون وأن إشعاع الخلفية عبارة عن الساعة مثل الساعات.

    أنت تشير إلى أن هناك احتمال آخر إلى جانب النسبية. الفيزيائيون اليوم لا يعرفون احتمالا آخر وأنت (والفيزيائي الذي تذكره بين حين وآخر) تتجنب تقديم البدائل التي تظن أن لديك، فما الإجابة التي تتوقعها؟

    وسؤال أخير: لماذا لم تقدم أسئلتك إلى كراوس بعد؟

  10. شموليك

    لا شك أن سرعة الضوء ثابتة في أي نظام مرجعي. إذا ألقيت نظرة على ارتباط مايكل بأدلة النسبية، يمكنك أن ترى سبب رفض النظريات البديلة، وفي المقام الأول نظرية الانبعاث، والتي بموجبها ترتبط سرعة الضوء بالمصدر.

    ومع ذلك، تتنبأ النسبية بنتائج غريبة جدًا، وليس إطالة الزمن سوى واحدة منها. إن تقصير الطول (الذي لا يوجد دليل تجريبي عليه) هو نتيجة أكثر غرابة. وبما أنها لا تتضمن تقصير العرض، فهل هذا يعني أننا إذا صورنا زحل من مركبة فضائية تمر بالقرب من الأرض بسرعة عالية فسنراه أقرب بكثير كما لو كنا استخدمنا التكبير؟

    ومن ناحية أخرى، يبدو أنه لا مفر من هذه النتائج الغريبة، لأننا إذا سلمنا أن سرعة الضوء ثابتة دائما، والسرعة هي حاصل قسمة المسافة على الزمن، فإن ما يتغير حتما هو المسافة والزمن.

    وسؤالي هو: حقا؟

    فيما يتعلق بالمفارقة (أقرأ، أقرأ)، فلا علاقة لها بنتائج التجارب (بالمناسبة، برهان بيل رياضي بحت). ادعى أينشتاين أن اللامكانية تتعارض مع النسبية. الفيزيائيون اليوم لا يزعمون ذلك. لا يوجد فرق في المعرفة ذات الصلة من عام 1935 إلى اليوم. فهل هناك تناقض أم لا؟ وإذا كان موجودا، فلماذا يدعي الفيزيائيون أنه غير موجود؟ وإذا لم يكن هناك فلماذا ادعى أينشتاين وجوده؟ هذا هو سؤالي. لا يعني ذلك أنه كان مخطئًا بشأن وجود اللامكانية أم لا (خطأ معقول ومنطقي) ولكنه ادعى أن اللامكانية تتعارض مع النسبية. ماذا، ألا يرى أنه لا يوجد تناقض؟

  11. إسرائيل،
    في النظرية النسبية على وجه التحديد، تكون سرعة الضوء ثابتة في أي إطار مرجعي لأي مراقب.
    فيما يتعلق بالمفارقة، أعتقد أنك تفتقد حقيقة بسيطة وهي أنه عندما تم كتابة المقال كان ذلك قبل إجراء أي تجارب حول هذا الموضوع، وهذا يفسر كيف يمكن أن يكون مخطئًا. يرجى الكتابة أنك قد قرأت هذه الفقرة.
    بالإضافة إلى ذلك، هذا ليس خطأ تمامًا بمعنى أنه نظرًا لعدم وجود المزيد من التجارب حول هذا الموضوع، توقع المؤلفون نتيجة غريبة لميكانيكا الكم تسمح بنقل المعلومات إلى ما هو أبعد من سرعة الضوء بينما تم بالفعل إثبات النسبية التنبؤ بشكل صحيح بالنتائج المختلفة.

    تم إجراء التجارب التي أثبتت وجود التشابك بعد حوالي 30 عامًا، وبالتالي فإن الوجود ليس مفارقة لأنه مع كل الاحترام للنظرية النسبية، فهي نظرية، ولكن الفائز هم المخترعون التجريبيون وقد أوضح مايكل سبب التشابك اليوم لا يعتبر متعارضا مع النظرية النسبية.
    ادعى أينشتاين أن اللامكانية تسمح بنقل المعلومات بما يتجاوز سرعة الضوء وهذا يتناقض مع افتراضه بأن لا شيء يمكن أن يتحرك فوق سرعة الضوء (عندما يتعلق الأمر بالمادة التي بدأت تتحرك تحت سرعة الضوء وتجاوزت سرعة الضوء) )

    على أية حال، كل هذه الأنواع من الأسئلة: كيف يمكن أن يكون مخطئًا، وما إلى ذلك، ليست أسئلة جيدة. العلماء بشر، والناس يخطئون، ويتم تلقي معلومات جديدة، ويتم تطوير نظريات جديدة، وهكذا يتقدم العلم. الوحيدون الذين يتساءلون كيف يمكن أن يكون مثل هذا الشامان مخطئًا وينزعجون منه هم المتدينون. وباستثناءهم، فإننا جميعًا مخطئون، في جزء كبير من الوقت.

  12. أيها الماء، إن وصفك دقيق تمامًا بالفعل، ومن المؤسف أنك لم تذكر أيضًا مراتب الفوتون الافتراضية التي ترتكز عليها الكميات لتبسيط الأعضاء المجمعة بعد الانتهاء من كل القفز ذهابًا وإيابًا لأعلى ولأسفل أثناء الإصابات.

  13. عزيزتي إسرائيل، هناك بعض الأشياء التي يصعب علي فهمها فيما يتعلق بالعلاقات - كيف يمكنك ربط السرعات ووجهة النظر الفردية، أيضًا في الكميات، العشوائية لا تنجح، ربما تكون هذه صورة جزئية ومبسطة للجسدي خصائص الكون، أود أن أصنف الموقف قليلاً من خلال شرح أن الضوء يتحرك عدة مرات إلى الخلف والأمام في الوقت المناسب، ويخلق موجة، ويجمع عناصر في الحركة الموجية مثل الكتلة والوقت والاستطالة. ومن ناحية أخرى فإن الحركة ذهابا وإيابا عدة مرات يمكن أن تعطي العناصر العشوائية تعبيرا يأتي من عوالم متوازية تنشأ من الحركة ذهابا وإيابا ومن هناك المعرفة الإضافية أو المفقودة، وهذا نوع من الاتحاد الذي يشرح بعض الأخطاء وتقريب الصورة الكبيرة التي تخلق التناقضات

  14. ميخائيل

    من رابط شموليك على مفارقة EPR:

    تساءل أينشتاين وبودولسكي وروزن كيف يمكن للجسيم الثاني أن "يعرف" أن له زخمًا محددًا بدقة ولكن موقعه غير مؤكد؟ وبما أن هذا يعني أن أحد الجسيمين يتواصل مع الآخر بشكل فوري عبر الفضاء، أي أسرع من الضوء، فهذه هي "المفارقة"

    والآن، إذا كما تقول "إن الفيزيائيين اليوم لا يرونها على أنها نقل للمعلومات، بل على أنها خلق متزامن للمعلومات" فماذا يعرف هؤلاء الفيزيائيون اليوم ولم يعرفه أينشتاين عام 1935؟ فهل تغير شيء منذ ذلك الحين؟ وفقا لما هو مكتوب أعلاه، ادعى أينشتاين أن مثل هذا الاتصال الفوري يتعارض مع النسبية، ومن هنا جاءت المفارقة. فهل هناك تناقض أم لا تناقض؟ وإذا كان موجودا، فلماذا يدعي الفيزيائيون اليوم أنه غير موجود؟ وإذا لم يكن هناك فلماذا ادعى أينشتاين وجوده؟ هل نفهم النسبية أفضل من أينشتاين؟

    الجواب مفاجئ.

    بخصوص الرابط الذي قدمته بخصوص تأكيد العلاقة وعقابا على اتهامي بالباطل 🙂 أجب على السؤال الذي طرحته عدة مرات دون إجابة:

    "هل هناك إمكانية أخرى غير إطالة الزمن لتحديد سرعة الضوء في جميع الأنظمة المرجعية؟ احتمال يتناسب تمامًا مع ما نعرفه من ميكانيكا الكم والذي حاربه أينشتاين طوال حياته وخسره؟

    وسنضيف: وهو ما لا يتعارض مع أي تأكيد تجريبي للنسبية من أي من ما أتيت به في الرابط، ويفتح أيضًا نافذة لفهم اللامكانية.

  15. وكما قلت، فإن الفيزيائيين اليوم لا ينظرون إليها على أنها نقل للمعلومات، بل على أنها خلق متزامن للمعلومات.
    هناك أشخاص (ومنهم جادون أيضًا) يختلفون على هذا التفسير، لكن الواضح هو أن ما يتم نقله لا يمكن استخدامه لنقل معلومات معروفة وبالتالي - لعكس ترتيب السبب والنتيجة.
    وهذا الانقلاب في النظام، في رأيي، هو ما أزعج أينشتاين.
    ولا أعلم إذا كان التفسير مبررا أم لا. أميل إلى الاعتقاد بأن هذا مبرر، ولكن ليس لدي أي مبرر أفضل من تلك التي قدمها الآخرون بالفعل.
    على أية حال - فهو لا ينتمي إلى الحجج التي أثرتها.

  16. قبل أن أركض، ولأنني لست متأكداً من أن هذه النقطة واضحة بما فيه الكفاية:

    1. لماذا ادعى أينشتاين أن اللامكانية تتعارض مع النسبية؟

    2. لماذا كان مخطئا؟

    3. كيف يمكن أن يكون مخطئا، حيث أن المعلومات المغزلية تنتقل بسرعة لا نهائية والنسبية تحظر نقل المعلومات بشكل أسرع من الضوء؟

    4. وسؤالي: لماذا لم يرى ما يراه معظم علماء الفيزياء: أنه لا يوجد تناقض.

  17. إسرائيل:
    أشكرك على التأكيد الذي أضفته إلى مطالبتي بشأن الطريقة التي صيغت بها.
    هل يجب أن أكتب أنني اعتذرت مقدمًا في الرد الأصلي عن سوء فهم محتمل؟
    كم مرة يجب أن أعتذر لك؟
    لم افهم. في رأيي، هذا خطأك وفي رأيي لا يتعلق بعدد المرات التي كتبت فيها CMB ولكن إلى حقيقة أنك لم تشر (كتابيًا) إلى أي نظام. لكنني اعتذرت على أي حال.

    في كلامك عن EPR تكرر ما قلته وتحاول تقديمه كتصحيح لكلامي.
    قلت إن فيزيائيي اليوم لا يعتبرونه نقلاً للمعلومات وشرحت السبب (شرح قدمته حتى قبل أن تسأل في المرة الأولى، تفسير لسبب ما لم يجعلك تتوقف عن السؤال)

  18. جيد. الآن لم أفهم ماذا تريد بالضبط.

    لقد تم توضيح سوء التفاهم. على الرغم من أن إسرائيل كتبت مرات لا تحصى "إشعاع الخلفية الكونية" و"CMB"، وأحضرت الروابط المناسبة، إلا أن مايكل فهم لسبب ما أن إسرائيل تعني شيئًا آخر.

    وبدلا من الاعتراف بالخطأ والاعتذار، يواصل مايكل إصراره على أن صياغته إسرائيل سيئة. حسنا، نحن معتادون على ذلك.

    لم يكن أينشتاين من الأقلية، فعندما طرح مفارقة EPR لم يكن هناك الكثير من الفيزيائيين الذين يؤمنون باللامحلية. كما أن نيلز بور لم يؤمن باللامحلية.

    وهذا ليس جوهر خطأه على الإطلاق. والمقصود أن أينشتاين ادعى في المقال أن ميكانيكا الكم غير صحيحة («إلك» غير مكتملة)، لأنه إذا كان مبدأ عدم اليقين صحيحا، فإن معلومات دوران الإلكترون يجب أن تنتقل من إلكترون إلى آخر على الفور، وهذا مخالف للتحريم. لنقل المعلومات بشكل أسرع من ضوء النسبية.

    أولا - لقد كان مخطئا. أثبتت متباينة بيل وتجربة أسبكت ذلك.

    ثانيا - لقد أخطأ مرتين. وعلى الرغم من مرور المعلومات، فلا يوجد أي تناقض مع النسبية.

    لكنه ادعى أن هناك تناقضا في النسبية، وهذا هو خطأه الثاني.

    توضيح لطبيعة الخطأ – إذا كان هناك طلب.

    الذهاب إلى العمل

  19. إسرائيل:
    وكما سبق أن قلت - لم أفهم قصدك بسبب طريقة صياغته وعندما تفهم قصدك فإن عدم المنطق لا يتجاوز أي حدود - فهو عدم فهم للحدود التي سبق وضعها .
    وبما أنك تعلم جيدًا أنني أعرف عن CMB وأن كلمة "إشعاع" لا تصف أي "نظام"، أفترض أنك فهمت ما قلته.
    وأفترض أيضًا أنك فهمت أنني عندما قلت إن معظم الفيزيائيين لا يرون EPR كمشكلة، لم أقصد أينشتاين الذي كان من الأقلية في هذه النقطة.
    يبدو لي أنك تحاول خلق فزاعات وإذا توقفت قبل ذلك عن المشاركة في المناقشة لأنني رأيت أنني قد أجبت بالفعل على جميع الأسئلة التي طرحتها، فقد أضيف الآن سبب آخر لإحجامي عن المشاركة في المناقشة وهذا السبب هو أنه مع استمرار المناقشة، تصبح صياغتك أكثر غموضًا وتصبح ردودك أكثر عدوانية.

  20. إذا لم يكن أينشتاين ينظر إلى المشكلة المثارة في مفارقة EPR على أنها إشكالية، فلماذا كتب أنها إشكالية؟ ما الذي يرى معظم الفيزيائيين أن أينشتاين لم يراه؟

    وما هي القصة التي أثارتها إسرائيل وحصلت على أجوبة كاملة وصحيحة؟

  21. فقط في هذه المقالة أشرت عدة مرات إلى إشعاع الخلفية أو باللغة الإنجليزية CMB. أولئك الذين لم يفهموا قصدي لم يعرفوا أيضًا عن نظام إشعاع الخلفية على الإطلاق، وهو ما يفسر بشكل جيد عدم الفهم.

    على أية حال، الآن بعد أن تم توضيح سوء الفهم (الخاص بك)، هل تستمر في الادعاء بأن "الافتقار إلى المنطق في المناقشة يتجاوز بالفعل كل الحدود"؟

  22. لا ينظر معظم الفيزيائيين إلى مفارقة EPR على أنها مشكلة لأننا لا نخلق النتيجة التي كان أينشتاين يخشاها - تلك التي تسمح بعكس ترتيب السبب والنتيجة.
    لا يتعلق الأمر بنقل المعلومات، ولكن حول إنشاء المعلومات في وقت واحد (هكذا، على الأقل، في معظم التفسيرات).
    يشكك البعض في هذا الدفاع، لكن على أية حال، لا علاقة لهذه التجربة بكل القصة التي تطرحها إسرائيل، وهي القصة التي حظيت، في رأيي، بإجابات كاملة.

  23. إسرائيل:
    لقد شرحت منذ البداية لماذا أعتقد أن ما قيل خطأ. وضحت أنني فهمت من كلامك أن الإشعاع هو إشعاع وليس نظام إشعاع الخلفية وفي نفس الرد كتبت أشياء مختلفة عن نظام إشعاع الخلفية والذي كان واضحا أنه في فهمي لكلامك يختلف عما أسميته إشعاع.
    وفي الرد الذي أجبت عليه كتبت كارينا ولم أظن أنك تتحدث عن أي نظام ومن هذا المنطلق لا معنى أنك لم تشر إلى نظام آخر لأنك هنا لم تشر إلى النظام.
    كما لاحظت - لم أكن الوحيد الذي لم يفهم نيتك.

  24. إسرائيل:
    لا.
    هذا هو كلامك السيئ مرة أخرى.
    إذا كنت تقصد نظام الإشعاع الخلفي (النظام الذي يكون فيه الإشعاع متناحيًا) فنعم، ولكن إذا كنت تقصد الإشعاع فلا.

  25. مايكل، يرجى توضيح نفسك.

    إذا تحركنا بسرعة 371 كم/ثانية في اتجاه برج الأسد بالنسبة للإشعاع، ألا يعني هذا أن الشخص الذي يتحرك بنفس السرعة بالنسبة لنا في الاتجاه المعاكس، يتحرك بسرعة 0 بالنسبة للإشعاع؟

    شموليك

    كان أليك الصغير المؤذي إلكترونًا محبوبًا وودودًا، حتى التقى ذات يوم بالبوزيترون الشرير بوزيترون وأصبحا كلاهما متأينين...

    نعم، ولكن سرعة الضوء نسبة إلى ماذا؟ هذا هو المكان الذي بدأت فيه العلاقة بأكملها، أليس كذلك؟ ففي النهاية، وفقًا لأينشتاين وجاليليو، حتى لو تحركت بسرعة 0.9 درجة مئوية بالنسبة للأرض، فأنت لا تزال في حالة سكون.

    وأما عن المفارقة: إذا كانت المفارقة على الورق فقط، فلماذا ادعى أينشتاين أنها حقيقية؟ ما هو يا زينون؟ هكذا تتفاخر أمام الجميع؟

  26. إسرائيل،
    أطلب منك، عندما تعلق، أن تنسج القصص في التعليق! (بسرعة تفوق سرعة الضوء) 🙂
    أبعد مما كتبه مايكل، لا أفهم جوهر السؤال. تصف النسبية ما يحدث عندما تقترب المادة من سرعة الضوء. هل تزعم أنه حتى لو لم تكن هناك حاجة إلى طاقة لا نهائية للوصول إلى سرعة الضوء، فلن يكون من الممكن الوصول إلى سرعة الضوء؟

    وفيما يتعلق بالمفارقة، يوضح مقال الويكي أن المفارقة ليست سوى مفارقة على الورق (بسبب الافتراضات الكلاسيكية للمؤلفين التي لا تتطابق مع النتائج التجريبية. والجملة التي تمثل هذا الافتراض الفلسفي هي:
    أن أي نظرية فيزيائية مقبولة يجب أن تحقق الواقعية المحلية
    ويجب أن نضيف أن المقال كتب قبل ظهور النتائج التجريبية

  27. أعتذر مقدما إذا لم أفهم السياق لأنني منذ رحيلي لم أقرأ التعليقات ومن الممكن أن التعليق الأخير كتب في سياق مختلف عما يبدو لي ولكن إذا كان "الإشعاع" المذكور في التعليق الأخير هو "إشعاع الخلفية" لا بد لي من وضع تعليق آخر لأنه يبدو أن عدم وجود منطق في المناقشة قد تجاوز بالفعل أي حد
    وكما نعلم فإن إشعاع الخلفية هو إشعاع يشع في كل الاتجاهات. لا يوجد "اتجاه" لإشعاع الخلفية وبالتالي لا يمكن التحرك بسرعة صفر بالنسبة له.
    كما ذكرنا سابقاً - من يتحرك في الكون - كلما زادت سرعته بالنسبة لنظام الإشعاع الخلفي - زادت حرارته (بسبب زيادة طاقة الإشعاع الذي يصادفه من الاتجاه الذي يتحرك فيه).

  28. شموليك

    أولا، شكرا على الروابط.

    النسبية تشرح الآلية، ولكن اتضح أيضًا أنه بدون النسبية لا يمكن الوصول إلى سرعة الضوء، وهذا بسبب الإشعاع.

    الفرق هو أنه وفقًا للنسبية، فإن سرعة الضوء مرتبطة بالراصد ولا يهم النظام المرجعي الذي تقيسه من خلاله. حيث أن سرعة الضوء بالنسبة للإشعاع هي سرعة مطلقة. لذلك، إذا تحركت بسرعة قريبة من سرعة الضوء بالنسبة لنظام دفع يقارب سرعة الضوء بالنسبة للإشعاع، فسيتبين أنك تقريبًا عند 0 سرعة بالنسبة للإشعاع ولن تشعر أي الاحترار. ومع ذلك، وفقًا للنظرية النسبية، لن تتمكن من تجاوز سرعة الضوء بالنسبة لهذا النظام الرشيق.

    فيما يتعلق بالتشابك، قال أينشتاين في مفارقة EPR (من الرابط الخاص بك):

    تساءل أينشتاين وبودولسكي وروزن كيف يمكن للجسيم الثاني أن "يعرف" أن له زخمًا محددًا بدقة ولكن موقعه غير مؤكد؟ وبما أن هذا يعني أن أحد الجسيمين يتواصل مع الآخر بشكل فوري عبر الفضاء، أي أسرع من الضوء، فهذه هي "المفارقة"

    ولكن هنا، ثبت أن هذا هو ما يحدث بالضبط، أي أن الجسيمين يتواصلان مع بعضهما البعض بشكل أسرع من الضوء، وعلى الرغم من ذلك لا يوجد تناقض مع النسبية، لأنه على الرغم من أن المعلومات تمر بلا شك، إلا أننا ما زلنا لا نستطيع إرسال المعلومات عبرها. تشابك.

    النقطة المهمة هي: لماذا ادعى أينشتاين فيدولسكي وروزن أن هناك تناقضًا إذا لم يكن موجودًا؟ ففي نهاية المطاف، كل البيانات كانت معروضة أمامهم، فلماذا التناقض؟ ماذا نرى وهم لم يرون؟

    قليلا من التوافه:

    كانت الفرضية القائلة بأن كتلة الجسيمات تزداد مع السرعة موجودة بالفعل في نهاية القرن التاسع عشر. تم اقتراح الصيغة E=MC^19 أيضًا في عام 2 من قبل مهندس فرنسي.

  29. إسرائيل،
    لأن النسبية هي التي تشرح الآلية التي تمنع المادة من الوصول إلى سرعة الضوء أو تجاوزها وتتطابق توقعاتها مع الملاحظات. هل هناك نظرية أخرى غير النسبية تضع حدًا للسرعة التي يمكن أن تتحرك بها المادة؟
    بعد أن وضع أينشتاين النظرية النسبية، تم اكتشاف جميع أنواع التنبؤات التي انبثقت عنها من قبل أشخاص آخرين، وفي رأيي اليوم، أن معظم الفيزيائيين المتخصصين في النسبية يفهمون النسبية بشكل أفضل من أينشتاين، ولو فقط لأنه تم إجراء المزيد من التجارب، والمزيد من الحلول. تم اكتشاف المزيد من القيود على النظرية، وتم تجميع المزيد من المعلومات التجريبية.

    وفيما يتعلق بالتشابك فإن الادعاء هو أنه لا يوجد أي تناقض مع النظرية النسبية لأنه لا يمكن نقل المعلومات من خلال هذه الآلية فوق سرعة الضوء وبالمناسبة فهو ليس على قيد الحياة عندما ثبت أن التشابك لا يحدث. تحدث بسرعات تتجاوز سرعة الضوء. اقرأ أكثر: http://en.wikipedia.org/wiki/EPR_paradox

  30. المعجزات

    فيما يلي التفسير التقليدي لعدم كون نظام الإشعاع هو النظام المفضل:

    كيف يمكننا أن نعرف ما هو الاقتراح الذي لدينا فيما يتعلق بـ CMB؟ ألا يعني هذا أن هناك إطارًا مرجعيًا مطلقًا؟

    تعتمد نظرية النسبية الخاصة على مبدأ عدم وجود أطر مرجعية مفضلة. وبعبارة أخرى، فإن نظرية أينشتاين بأكملها تعتمد على افتراض أن الفيزياء تعمل بنفس الطريقة بغض النظر عن السرعة والاتجاه لديك. لذا فإن حقيقة وجود إطار مرجعي لا توجد فيه حركة عبر CMB يبدو أنها تنتهك النسبية الخاصة!

    ومع ذلك، فإن الافتراض الحاسم لنظرية أينشتاين ليس عدم وجود أطر خاصة، ولكن عدم وجود أطر خاصة تختلف فيها قوانين الفيزياء. من الواضح أن هناك إطارًا تكون فيه الإشعاعات الكونية الميكروية في حالة سكون، وبالتالي فإن هذا، إلى حدٍ ما، هو إطار الراحة للكون. ولكن للقيام بأي تجربة فيزيائية، فإن أي إطار آخر يكون جيدًا مثل هذا الإطار. لذا فإن الاختلاف الوحيد هو أنه في إطار الراحة CMB لا تقيس أي سرعة بالنسبة لفوتونات CMB، لكن هذا لا يعني أي اختلاف جوهري في قوانين الفيزياء.

    التفسير الذي يقول أنه على الرغم من أننا نستطيع دائمًا معرفة سرعتنا بالنسبة للإشعاع، وأننا إذا طارنا بسرعة بالنسبة للإشعاع فسوف نتبخر، وأنه لا يوجد كوكب أو شمس في المنطقة المجاورة تختلف سرعتنا كثيرًا عن سرعتنا (على سبيل المثال نصف سرعة الضوء بالنسبة إلينا)، ولا يزال هذا النظام طبيعيًا تمامًا وغير مفضل.

    لكن لا تقلق. إن تفسير عدم المحلية في التشابك الكمي لا يتعارض مع النسبية، رغم أنه لا شك أن المعلومات حول استقطاب الفوتون أو دوران الإلكترون تنتقل بشكل فوري من جسم متشابك إلى أخيه، تفوقها في تعقيدها. . وبطبيعة الحال، تظل هناك حيرة صغيرة مفادها أنه إذا لم يكن هناك تناقض بين اللامكانية والنسبية، فلماذا ادعى أينشتاين في مقالة EPR أن مثل هذا التناقض موجود بالفعل. هل نفهم النسبية أفضل من أينشتاين؟

    لا أستطيع أن أفهم أيضًا، وسأكون سعيدًا إذا تمكن شخص ما من شرح هذه النقطة لي (مايكل؟) لماذا هناك حاجة إلى النسبية لشرح سبب عدم تمكن جسم ما في معجل الجسيمات من الوصول إلى سرعة الضوء. ففي النهاية، إذا ارتفعت حرارته عندما يتحرك بالنسبة للإشعاع - وفي جميع تجارب المسرعات تتحرك الأجسام عكس الإشعاع - فمن الواضح أنه لن يتمكن من الوصول إلى سرعة الضوء. الطاقة التي يجب استثمارها لتسريعها تصبح طاقة حرارية، أليس كذلك؟

    وجميع التجارب التي تثبت إطالة الزمن تتم في أنظمة متسارعة أو أنظمة قصورية تتحرك عكس الإشعاع (ميونات). أفترض ذلك ببساطة لأنه من غير الممكن تقنيًا إجراء التجربة في الاتجاه المعاكس، حيث يبدأ الميون من نظام متحرك وأثناء حركته يكون في حالة سكون بالنسبة للإشعاع. من شأن مثل هذه التجربة أن تشكل دليلاً دامغًا على إطالة الزمن، ولكن ليس التجارب الحالية.

  31. معجزات.

    من:

    http://en.wikipedia.org/wiki/Cosmic_microwave_background

    وتبين أننا نتحرك بالنسبة لإشعاع الخلفية الكونية بسرعة حوالي 600 كم/ث باتجاه كوكبة الأسد.

    وعدم القدوم إلى إسرائيل بادعاءات مفادها أن أينشتاين قال إنه لا يوجد نظام مفضل. وقال أيضًا إن اللامكانية في التشابك الكمي تتعارض مع النسبية.

    يدين.

  32. والدي، أصدقائي
    لذلك، وبدون الكثير من الارتباط، تم نشر سلسلة رائعة من المحاضرات التي ألقاها علماء فيزياء حائزون على جائزة نوبل وجوائز أخرى على موقع يوتيوب تحت عنوان: مؤتمر نوبل التاسع والأربعون. المحاضرات رائعة وسيكون من الجميل لو قام العالم بمتابعتها.
    إعلان تشويقي صغير للموضوعات الرائعة التي ناقشوها: البروفيسور. تحدث جيمس جيتس (في محاضرته الأولى) عن كود تصحيح الخطأ المضمن في رياضيات النظرية الرياضية ولا أتذكر من تحدث عنه، لكن لأول مرة علمت أنه أثناء تطور الكون، كان هناك وهي نقطة زمنية بدأ منها الكون في التسارع، أي بعد الانفجار تباطأ توسع الكون ولكن من نقطة معينة تطور هذا الاتجاه. لم أكن على علم بهذه النقطة (لا يعني ذلك أنني مطلع جدًا على آخر المستجدات، لذا كان من الممتع معرفة ذلك بشكل غير طبيعي)
    ها هو الرابط الالكتروني. من المفيد الاستماع إلى جميع المحاضرات:
    http://www.youtube.com/watch?v=F1P0HNhQE68&list=PLHuAoPzfQhGGQzB58S1iVQvu43Xvbp2c4

  33. المعجزات

    التوأم المتبقي هو التوأم المستقبلي. لقد كان وقت سفر التوأم مبكرًا، لذلك قام برحلة إلى المستقبل بالنسبة له. لذلك، فإن الباقي لن يكون قادرا على استنتاج أي شيء من التجربة، لأنه قد اجتاز منذ فترة طويلة المرحلة التي تم فيها العثور على التوأم المتنقل.

    لكن تجربتك توضح وجهة نظري. فإذا بدأ التوأم المسافر رحلته من نظام دفع بسرعة نسبة إلى الإشعاع وتسارع في اتجاهنا، فنحن بطيئون بالنسبة إلى الإشعاع، فإنه قضى معظم رحلته في حالة سكون بالنسبة إلى الإشعاع، أي بيننا. فإذا كان أخوه يشيخ بسرعة في هذا الوقت، فإن نظامه يتحرك نحو المستقبل بالنسبة لنا، وكما ذكرنا نحن نرتاح بالنسبة للإشعاع، أي أن عمرنا هو عمر الكون.

  34. عزيزي السيد نسيم، بعد إجابتك الأخيرة.
    إذا قمت بتكرار تجربة وتعرف نتائج تجربة مماثلة، فهناك احتمال معين أنك تعرف المستقبل، وغدًا أيضًا يوم جيد، مع احترامي

  35. مايكل وإسرائيل
    لنفترض أن التوأم المسافر يأخذ معه تجربة (حتمية) يجب أن تستمر لفترة طويلة، بينما تجرى هنا تجربة مماثلة. وعندما يعود التوأم، فإن من يبقى سيتمكن من معرفة نتيجة تجربته الخاصة في المستقبل.
    وهذا لا يعني أنه يمكن معرفة المستقبل، ولكن يمكن الاستفادة منه في تطوير اللقاحات مثلا (لا تأخذوا الفكرة على محمل الجد....)

  36. صحيح.. وقد توصلت منذ زمن طويل إلى نتيجة مفادها أن التجربة وحدها هي التي يمكنها تأكيد أو دحض النظرية.

    سؤال إذن:

    فهل هناك إمكانية أخرى غير إطالة الأزمنة لتحديد سرعة الضوء في جميع الأنظمة المرجعية؟ احتمال يتوافق في الواقع مع ما نعرفه من ميكانيكا الكم أن أينشتاين خاض حياته كلها، وخسر؟

  37. حسنا، علينا أن نذهب إلى العمل. النقطة: إذا كانت ساعات نفس المجرة -القطار في المثال السابق- متزامنة وتظهر وقتنا، والمجرة الآن تتسارع، ففي الكوكب الأول الذي يمر بجانبنا تظهر الساعات وقتنا تقريبًا وعلى الأخير الوقت أكثر تقدمًا.

    وهذا ما يحدث بحسب النسبية إذا كانت المجرة في حالة سكون بالنسبة للإشعاع بعد التسارع (درب التبانة) وحتى لو كان الراصد هو الساكن.

    إذا استخدمنا بجوار الساعات بذور الزيتون التي سيتم سقيها وإنباتها في اللحظة 0 وفقًا لأوقات المجرة، فعندئذ على الكوكب الأول سنرى فقط شتلة طرية وعلى الكوكب المتقدم شجرة زيتون عمرها 500 عام. الكوكب الأخير لن يكون هناك زيتون بسبب ارتفاع درجات الحرارة.

    وكل هذا والزيتون الذي بجانبنا لم ينبت إلا بالكاد.

    لن نتمكن فقط من النظر إلى المستقبل - فبعد كل شيء، بدأنا جميعًا في نفس العمر - بل سنكون قادرين على تسريع والانضمام إلى نفس الكوكب التطوري التقدمي.

    والأسوأ من ذلك، أنه يمكننا أيضًا العودة إلى الكوكب الأم الذي تركناه خلفنا.

    ويثير أوكهام حاجبه لهذا الأمر.

    عمل.

  38. تتغير التسارعات ولست على دراية كافية بالحسابات المتعلقة بها.

    بالمناسبة: فيما يتعلق بأوكهام - من المهم توضيح أمر أساسي جدًا أعتقد أنك لا تعرفه:
    كان أوكهام يتحدث فقط عن الافتراضات غير الضرورية التي تضعها النظرية. ولم يتحدث عن استنتاجات النظرية. إن الأشياء التي زعمت أن أوكهام يعارضها ويهذي بها هي كلها استنتاجات من النظرية النسبية وليست من مقدماتها.

  39. جمال. لاحظ الآن: إذا كانت المجرة قد بدأت حركتها ليس منذ مليارات السنين ولكن منذ دقائق قليلة، فإنها حتى ذلك الحين كانت في حالة سكون بالنسبة إلينا وكانت ساعاتها تشير إلى أوقاتنا. هل يغير شيئا من حيث التجربة؟

  40. مايكل، بينما كنت هنا:

    وبحسب النسبية، إذا مر المسافر بسرعة عبر مجرة ​​درب التبانة، فإنه يراها تشيخ بسرعة، أي أن الكوكب الأخير أقدم بكثير من الأول، في وقت قصير حسب ساعته.

    ووفقًا لنفس النسبية، إذا مرت بالقرب منا مجرة ​​سريعة، فإنها أيضًا ستشيخ بسرعة.

    هل تعتقد أن أوكهام كان سيوافق على ذلك؟

  41. المعجزات:
    ليس هناك عودة من المستقبل. يصل إلى المستقبل ويستمر في المضي قدمًا نحو المستقبل، إلا أنه وصل إلى نقطة الالتقاء مع أخيه الثابت في وقت أقصر مما مر به أخيه الثابت.

  42. أجبت وشرحت أيضًا السبب (نعم! كان هناك تفسير ولم يكن أوكهام!).
    في الواقع كان الشرح مرتبطًا بجزء آخر من ردي السابق وهو يتعلق بعدم التناقض مع النتائج ولكنه ليس مهمًا

  43. كما أنه لا يوجد أي تناقض منطقي في الإجابات 1 و2 و4 في لغز جدي.

    ومع ذلك، عندما سُئلت، لسبب ما أجبت بالإجابة 3 - 75 عامًا - وهي الإجابة الوحيدة المعقولة جسديًا.

    كما هو الحال دائما أنت حر. ولكن إذا قررت الانضمام (مرحبا بكم دائما) يرجى البقاء في مجال الفيزياء. اترك سيكولوجية قراءة العقول ورؤية السحر إلى مقالات جلعاد.

    معجزات.

    لا يعود من المستقبل، بل يطير إليه ويبقى هناك. لا يوجد طريق للعودة، المعروف أيضًا باسم الإنتروبيا.

  44. إسرائيل:
    إذا لم يكن هناك تناقض منطقي ويتطابق مع النتائج، فليس لديك فرصة لإقناع أي شخص.
    إن التلويح باسم أوكهام (الذي لا أعتقد أنه سيعرف ما تريد منه) لا يغير ذلك.
    لكن حقيقة قولك أنه لا يوجد تناقض هو تقدم بالفعل لأنك قلت في الماضي أن هناك تناقضًا بين الانفجار الأعظم والنسبية.

    آمل أن ألتزم هذه المرة بقراري بعدم الانجرار إلى هذه المناقشة مرة أخرى طالما أنني لم أقرأ شيئًا جديدًا فيها.

  45. دعونا لا ندخل في سؤال الصواب أو الخطأ الآن. كل شيء مقيد بالسلاسل لمدة عامين ونصف. إذا كنت مهتمًا، فسوف أعرض لك يومًا ما الأماكن التي تترك فيها التكنولوجيا وتذهب إلى علم النفس، لتكتشف لاحقًا أنك كنت مخطئًا.

    אז חסוך ממני ביטויים כגון ״כשאני מציג לך טעות בניסוי המחשבתי המקורי – זה שהביא אותך למסקנה השגוייה – לייצר ניסוי אחר שאולי יביא אותך לאותה מסקנה מבלי להבין קודם את טעותך בראשון זה מעשה שמעיד על עצמו כאלף עדים שכל תכליתו היא לשמר את המסקנה המקורית ולא לגלות الحقيقة".

    أعتقد أنني أحاول معرفة الحقيقة. ليس مجرد مؤمن. لقد استثمرت الكثير من الوقت والمال لاختبار بديل الزمن النسبي، الذي أعتقد أنه موجود على الرغم من أنه ليس صحيحًا بالضرورة. إذا كان هذا يبدو وهمًا، فأرجو أن تتذكر أن علماء الفيزياء الضخمة مثل لورينز والعديد من الآخرين استمروا في الإيمان بالموقع حتى بعد سنوات عديدة من عام 1905.

    الساعة التي وصفتها هي ساعة مكسورة. وهذا لا علاقة له بطول الوقت. من الصعب جدًا تحديد مفهوم الوقت، ويبدو في مدونة ساسكيند أنه المشكلة الأولى.

    التوأم الصغير، راكب عوفر، فوتونك السريع، أنين وكل نظام دفع متعلق بنظام متزامن آخر، يقوم برحلة إلى المستقبل. لكن القاسم المشترك بينهم جميعًا هو أن الزمن معهم يتباطأ، وما هو المستقبل بالنسبة له وبالنسبة للنظام الآخر هو الحاضر، هو عمر الكون، أي عمر إشعاع الخلفية.

    عندما يلتقي التوأم الأصغر بأخيه، لم يعد بإمكانه الادعاء بأن وقته مناسب - لأن التدفئة قيد التشغيل. كان بإمكانه القيام بذلك في عام 1905 عندما كان الكون أبديًا. وحتى الراصد لعوفر أو الميونات أو أي نظام آخر، إذا توقفوا في رحلتهم، سيكتشف أن عمر الكون، وهو أعلى من عمرهم، هو عمر مطلق ولا يرتبط بالوقت الذي تظهره ساعتهم. ومن ناحية أخرى، إذا عكسنا التشكيل وتسارعت مجرة ​​درب التبانة لتلتقي بالمسافر، فلن تجد أن ساعتها قد عادت بالزمن إلى الوراء لتتكيف مع عمر المسافر.

    ما أحاول إظهاره في مثال القطار السريع هو العملية العكسية. ليس المسافر أو الميون أو التوائم أو الحمير الوحشية الذين يتحركون دائمًا بالنسبة للإشعاع ويراقبون نظامًا ثابتًا، ولكن مراقبًا ثابتًا بالنسبة للإشعاع ويراقب نظامًا متحركًا.

    وبما أنه لا يوجد مرجع إشعاعي في العلاقات، فإننا نقبل أن يكون عمر الراصد أقل من عمر النظام المرصود، حتى لو بدأ كلاهما في نفس الوقت.

    لا يوجد تناقض منطقي في هذا، كما لا يوجد تناقض منطقي في أب عمره سبع سنوات أو محاضر يبلغ من العمر 150 عاما. لا يوجد سوى أوكهام الناعس الذي يلوح ويصرخ أنه من غير المرجح أن نتوقع بل وحتى تمرير نظام عمره أعلى من عمرنا أي عمر الكون.

    اى باد.

  46. إسرائيل:
    نحن جميعًا نسافر إلى المستقبل طوال الوقت، كما سافر التوأم أيضًا إلى المستقبل بوتيرة متسارعة.
    إذا كانت هذه الأمور مقبولة بالنسبة لك إذن (كما سألت من قبل) على الرغم من أنني لا أسميها "رؤية المستقبل"، فلماذا تقدمها كمشكلة عندما يتعلق الأمر برؤية نظام يمر عليه الوقت؟ معدل مختلف؟
    وسؤال ذو صلة (وهو مشابه لما أخبرتك عنه ذات مرة إذا نظرت إلى الساعة سترى الوقت سيكون غدًا وتتنبأ بالمستقبل بنفس الطريقة): لنفترض أن هناك جهازي كمبيوتر يعملان بنفس البرنامج و لنفترض الآن أن ساعة أحدهم قد تسارعت (هذا إجراء كان من الممكن القيام به على العديد من أجهزة الكمبيوتر في الماضي) وبدأ في العمل بشكل أسرع. هل الكمبيوتر الذي يقف بجانبه ولم يتم تسريعه "يرى" المستقبل" حسب تعريفك؟ وإذا كان الأمر كذلك فهل هناك أي تناقض أو مشكلة في هذا؟

  47. غير صحيح يا إسرائيل.
    كانت هناك قصة واحدة ارتكبت فيها عددًا من الأخطاء، لكن كل هذا حدث لأن غلاف قصصك أخفى الموضوع، ومن بين أمور أخرى، حرر يدي من الدخول في حسابات حقيقية.
    وبتشتت الضباب حول القصة (وهي نفس القصة) حصلت على إجابة واضحة وصحيحة.

  48. المعجزات

    من المؤكد أن مفارقة التوأم تصف الواقع. ومن المفارقة أن التوأم الأصغر، بالنسبة له، سافر إلى المستقبل.

  49. مايكل، أنت مدعو للنظام.

    كالعادة، أنت على يقين أن الحق قد أُعطي لك من فوق، ومن يجرؤ على معارضته فهو بالضرورة مخطئ.

    لكننا مررنا بهذه العملية عدة مرات في السنوات الأخيرة، أليس كذلك؟ وحتى ذلك الحين اتضح في النهاية أنك كنت مخطئًا.

    أنا على الطريق الآن، من الصعب الكتابة. طاب مساؤك.

  50. إسرائيل:
    أنت تخلط أشياء لا علاقة لها بالموضوع.
    ومصدر عرض العمر الأقدم من عمر الكون هو الذي قلته.
    في التجربة الجديدة، أنت لم تبني مثل هذا القطار الطويل، وبالتالي -اعتبره معجزة- ليس له عمر أعلى من عمر الكون.
    ولا إشكال في أن طول ما يبدو للفطر نحو سنة يبدو أقصر للناظر. هذا هو حقا ما يحدث. وهذا أيضًا ما يحدث مع الأنين ويمكننا أن نشاهد هذا باستمرار.
    لقد أدخلت إشعاع الخلفية في القصة دون أي صلة.

    ومره اخرى:
    عندما أقدم لك خطأ في التجربة الفكرية الأصلية - تلك التي أوصلتك إلى نتيجة خاطئة - فإن إنشاء تجربة أخرى قد توصلك إلى نفس النتيجة دون أن تدرك أولاً خطأك في الأولى هو فعل يشهد على نفسه مثل ألف شاهد هدفهم كله الحفاظ على النتيجة الأصلية وليس كشف الحقيقة.

  51. أنا لا أحاول أن أفشلك في أي شيء. أحاول جديًا معرفة ما إذا كان هناك أي خطأ في حساباتي، وآمل أن تفعل الشيء نفسه مع حساباتك، حتى لو لم تتماشى مع أجندتك.

    قلت: فهل تزعم أن شيئا مما قلت ليس صحيحا؟
    أشر إليه وقل ما هو الخطأ."

    كتبت في الرد قبل ساعتين ما يلي:

    "مصدر الزمن الأعلى من عمر الكون الذي تقبله إسرائيل هو نتيجة خطأ في التجربة الفكرية التي بناها والتي وضع فيها في البداية كواكب/جوزاء/عربات/فطر سمها من عمره/عمره أعلى من عمر الكون لأنه من الضروري وجود وقت أطول من عمر الكون للوصول إلى موقعهم/موقعهم النسبي".

    ولكننا هنا وصفنا تجربة جديدة حيث تبدأ جميع الأنظمة - المشاهد والقطار - من نفس العمر. إذا قبلت القسم ب (وقد فعلت ذلك)، فعندما يمر آخر قطار بالمشاهد، يكون عمره أكبر بعدة أشهر من عمر المشاهد.

    ولو استخدمنا الفطر بدلاً من الساعات، لكان المشاهد من النظام الإشعاعي يصور فطراً بدأ رحلته في عمرنا ووصل إلى الصور عندما كان أكبر سناً وناضجاً. لو استخدمنا التوائم وكان القطار أطول وأسرع، لكان نحن، توأم في العشرين من العمر، نصور توأمًا في الثمانين من العمر في السيارة الأخيرة.

    وهذا على الرغم من أننا جميعًا بدأنا التجربة في نفس العمر، وأن جميع الأنظمة ليست طويلة جدًا أو سريعة جدًا.

    أليست هذه نظرة للنظام كما سترونه في المستقبل؟

  52. بشكل عام: هذه الطريقة في طرح الأسئلة التي لكي أعطي إجابة جدية تتطلب حسابات جدية ومن ثم أتمنى أن أخطئ في تقييم الإجابة ثم أتمسك بهذا الخطأ غير مقبولة بالنسبة لي.
    إنها طريقة للإرهاق في أحسن الأحوال والخداع في أسوأ الأحوال.
    لماذا لا نلتزم بما تحدثنا عنه بالفعل؟

  53. إسرائيل:
    هناك بالفعل فرق بين ما قلته وبين ما أظهرته. لهذا السبب استخدمت الكلمة أظهرت.
    لم أقم بالحساب، لكنني لا أرى حاليًا مشكلة في أوامر الحجم التي وصفتها في القسمين أ و ب.
    أرى حاليًا مشكلة في العودة إلى هذا النقاش.
    هل تزعم أن أيًا من الأشياء التي قلتها غير صحيح؟
    أشر إليه وقل ما هو الخطأ.

  54. شموليك، شكرا على الرابط. سأتصل به قريبا.

    مايكل أصدقائنا.

    أولاً، أريدك أن تعلم أنني أقدر حقًا الاستثمار في هذه المشكلة التي هي "طفلي".

    ولكن هناك فرق بين "قلت" و"أظهرت".

    لذا، لتبسيط الأمور، دعونا ندخل في صلب الموضوع مباشرة. إذا كنت لا تمانع، أود تسخير قوتك الحاسوبية للحصول على حل كمي للمشكلة التالية:

    1. يقف قطار يبلغ طوله سنة ضوئية على مسار مستقيم في حالة سكون بالنسبة إلى إشعاع الخلفية. المراقب الذي يقف بجوار المسار يظل ثابتًا بالنسبة للقطار والإشعاع. تتم مزامنة ساعات عربات القطار مع بعضها البعض، وتظهر نفس الوقت الذي يظهر للمشاهد.

    2. في اللحظة 0 حسب ساعة القطار والمراقب، يتسارع القطار إلى سرعة 0.6c في وقت قصير قدره دقيقة واحدة حسب ساعة المراقب.

    3. يظل القطار بهذه السرعة حتى نهاية التجربة.

    4. عندما يصل القطار إلى هذه السرعة يتم تصوير ساعات المشاهدين والسيارة المارة.

    5. عندما يمر القطار الأخير على المشاهد، يتم تصوير الساعات مرة أخرى.

    ماذا سوف تظهر الصور؟

    إذا كنت لا تريد التورط في حسابات مرهقة، فهل تقبل ما يلي:

    و. في اللقطة الأولى، هل ستظهر اللقطة (بغض النظر عن أي جانب) الوقت نفسه تقريبًا بالنسبة للمشاهد والمقطورة؟

    ب. في اللقطة الثانية، هل ستتأخر ساعة المشاهد عن العرض الترويجي بأشهر؟

    إذا رأيت خطأ في صياغة المشكلة أو في الاستنتاجات، يرجى الإشارة إلى القسم الخاطئ.

    شكرا.

  55. شموليك:
    لقد حصلت إسرائيل على كل الإجابات التي يمكن أن تحصل عليها هنا، وكان ينبغي لها أن تقتنع.
    لقد بينت أنه في كل موقف هناك اتفاق بين الراصد والمرصود حول الوقت في كلتا الساعتين.
    وهذا يعني في الواقع أن المشاهد ليس له أي تأثير على الوقت الذي يشاهد فيه ما شاهده وبالتالي فهو غير ضروري في القصة بأكملها.
    فإذا رأى الراصد وقتًا في المرصود أعلى من عمر الكون، فإن الراصد يرى أيضًا نفس الوقت على ساعته.
    ولذلك فإن النظرية النسبية لا يمكن أن تعطي لشخص ما زمنًا أعلى من عمر الكون لمجرد أن شخصًا آخر ينظر إليه.
    مصدر الزمن الأعلى من عمر الكون الذي يستقبله إسرائيل هو نتيجة خطأ في التجربة الفكرية التي بناها والتي وضع فيها في البداية كواكب/جوزاء/عربات/فطر سمها الذي عمره/عمره أعلى من عمر الكون عمر الكون لأن هناك حاجة إلى وقت أطول من عمر الكون للوصول إلى موقعهم/موقعهم النسبي.
    كما بينت أنه من الممكن حساب ساعة إشعاع الخلفية لأي نظام نسبي وليس فقط نظام إشعاع الخلفية ويمكن لأي راصد في أي نظام أن يفعل ذلك بحيث أن إمكانية ملاحظتها من قبل أي نظام لا تميز نظام إشعاع الخلفية حصريا.
    لقد أوضحت أيضًا أن مشاهدة ساعاتك الأعلى هي ما يميز الجميع: من لحظة المزامنة، ستشاهد دائمًا ساعاتك الأعلى (أو في مثال القطار والقاطرة - إذا استقلت قطارين تقودهما قاطرات عند إنشاء المزامنة) ففي لحظة التقاء القاطرتين، فمن تلك اللحظة سوف ترى كل من القاطرتين وقتها الأعلى في السيارة التي على يمينه.
    باختصار - لم يبق شيء من المشكلة.

  56. لا أعرف الإجابة على المعادل الكوني للعالم المستدير.

    أنا فقط أدعي أنه إذا كان عمر الكون محدودا والزمن نسبي فمن الممكن نظريا تجاوز هذا العمر.

    لقد أظهرت هذا مرارًا وتكرارًا مع الفطر والساعتين التوأم. لا أحصل على إجابة على السؤال الأساسي الذي أطرحه مرارًا وتكرارًا: إذا لم يكن عمر نظام إشعاع الخلفية مفضلاً، فلماذا لا يمكن تجاوزه؟ لماذا لا نستطيع مراقبة كون أقدم من كوننا؟

  57. في رأيي أن عدد التعليقات هنا كبير جداً والتعليقات لا تخص أصلاً موضوع المقال.
    ولذلك، سيكون من المفيد فتح نقاش جديد حول موضوع النسبية.

  58. على الرغم من أنني لا أعتقد أن الكون له قطر محدود.
    مع ذلك.
    إذا كان للكون قطر محدود. إذن ماذا يحدث بعد ملليمتر واحد من هذا القطر؟
    لقد تم طرح هذا السؤال بالفعل حول الأرض.
    أي: إذا كان العالم مسطحاً وله حدود، فماذا يحدث عندما تتجاوز الحدود بمقدار مليمتر واحد.
    الجواب كان:
    الأرض كروية.
    أي أن المشي وراء "الحافة" يوصلك إلى نقطة البداية.
    فهل من الممكن أن يكون هذا هو الحال في الكون؟
    يعني الطريق مغلق نقطة النهاية هي نقطة البداية. أي أنه لا يمكن التحرك أبعد من مسافة معينة.

  59. 1. الراصد بأي سرعة سيكون قادراً على حساب عمر الكون كما لو كان في حالة سكون بالنسبة للإشعاع، أي عمر الكون، أليس كذلك؟

    2. لا يوجد نظام يكون عمر الكون فيه أعلى من عمر نظام إشعاع الخلفية (13.7 مليار سنة)، أليس كذلك؟ هل يوجد نظام يكون عمر الكون فيه 100 مليار سنة؟

    3. في عام 1905، (2) ليس لها أي معنى، أليس كذلك؟

    4. إذا كانت الإجابات على 1 و2 و3 إيجابية، فهو M.S.L. لا؟ بخلاف ذلك، إذا كان هناك مراقبون يبلغ عمر الكون بالنسبة لهم نصف عمرنا، فمن المحتمل أن يكون هناك أيضًا أولئك الذين يبلغ عمرهم ضعف عمرنا، أليس كذلك؟ وإذا كان عمر الكون بالنسبة للآخرين هو 0 (الفوتونات تندفع)، فهناك أيضًا أولئك الذين يعتبرون عمر الكون لا نهائيًا، أليس كذلك؟

    ב

    لا يعني ذلك أنني أعرف الإجابة، ولكن إذا كان للكون حد، فأين هو؟ وإذا كنا على مثل هذا الكوكب المحدود، فماذا يظهر التلسكوب إذا نظرنا إلى ما هو أبعد من قطر الكون؟ وما الذي سيمنعنا من إرسال قمر صناعي في نفس الاتجاه؟ هل قمنا بتوسيع قطر الكون بوضع قمر صناعي؟

    في الواقع ليست هناك حاجة لقطار لا نهاية له. وبحسب النسبية فإن أي راصد بدأ رحلته من قطار متحرك بالنسبة للإشعاع وتسارع حتى أصبح الآن في حالة سكون بالنسبة إليه، يرى ركاب القطار كما سيراهم في المستقبل.

  60. מיכאל
    لقد راجعت نفسي... أنت على حق فيما يتعلق بمشكلة التوقف حسب تعريفها، ولكنها يمكن أن تحل مشاكل لا يستطيع الكمبيوتر حلها. الحساب لديه هذه المشكلة أيضًا. ولكن - ليس هذا ما أتحدث عنه.

    أنا أتحدث عن حقيقة أن العقل ليس آلة تورينج. النموذج الذي أتحدث عنه لا يمكن اختزاله في آلة تورينج. من الممكن محاكاة أي عملية في الدماغ. لكن الكمبيوتر لا يستطيع التفكير. بالنسبة لي، التفكير هو خاصية ناشئة (لا أعرف كيف أترجم هذا المفهوم). إنها مثل درجة الحرارة - سوف تقوم بإجراء محاكاة لكل جزيء في حجم معين وستحصل على محاكاة لدرجة الحرارة. لكن الكمبيوتر لا يسخن منه ...

  61. ب:
    لقد أخبرته بذلك بالفعل وشرحت له أيضًا أنه من الممكن حساب عمر الكون في أي نظام وفقًا لإشعاع الخلفية الذي يتعرض له نظام الراصد (وبالتالي - بهذا المعنى أيضًا - فإن نظام إشعاع الخلفية ليس كذلك فريد).

  62. مشكلة التوقف هي عدم القدرة على تحديد ما إذا كان برنامج معين سيتوقف عند تشغيله على جهاز كمبيوتر عادي بمدخل معين.
    أي شخص لا يستطيع أن يقرر هذا السؤال (بما في ذلك النموذج الذي تصفه) يعاني من مشكلة التوقف.
    وكما بينت، بالنسبة لأي آلية يمكن قراءة قراراتها، فمن الممكن أيضًا كتابة برنامج (يعمل على آلة تورينج) يستخدمه لإخفاقه في حل مشكلة التوقف.
    وهذا يشمل الدماغ البشري ويتضمن أيضًا الكمبيوتر الذي وصفته.
    بالمناسبة، حتى الكمبيوتر الذي وصفته (طالما أن عدد رؤوسه محدود) يمكن أن يسمى آلة تورينج (وإذا كان عدد رؤوسه لا نهائي فمن المستحيل بناءه أو العثور عليه في الطبيعة)

  63. إسرائيل :
    حتى بدون هذا القطار الطويل الطويل.
    من الواضح أنه إذا بدأ الكون بالانفجار الأعظم، فإن قطره محدود. وهذا القطر هو المسافة التي يقطعها شعاع الضوء في زمن يساوي عمر الكون. ولا توجد مسافة أكبر من تلك الممكنة.

  64. מיכאל
    دعونا ننظر إلى الأمر من زاوية أخرى. النموذج المقبول لوصف الكمبيوتر هو آلة تورينج. تحتوي آلة تورينج على رأس واحد للقراءة/الكتابة وشريط لا نهائي. لقد تعلمت أن الكمبيوتر المتوازي، ذو الرؤوس المتعددة، يمكن وصفه بواسطة آلة تورينج عادية.
    إنه مجرد نموذج - لا يوجد شيء اسمه لانهائي، كما قلت.
    الآن - تخيل أنه أمام كل خلية في الشريط اللانهائي يوجد رأس قارئ/كاتب. هذا هو نموذجي للعقل البشري. ألا ترى أنه يختلف جوهريًا عن الكمبيوتر العادي؟ ألا ترى أنه لا يعاني من مشكلة التوقف؟

  65. لقد عدنا من الصحراء.

    مايكل، إذا أعطيت قوتك فأنت حر، كما هو الحال دائمًا.

    ومن حقك أن تصدق أن كلامك صحيح. وهي بلا شك صحيحة - في عينيك.

    في رأيي كلامك صحيح. لا يمكن أن يوجد نظام في الكون عمره أعلى من عمر الكون. نظام الإشعاع الخلفي هو الإشعاع التفضيلي. الحقيقة: يمكن لأي راصد أن يعرف دائمًا عمر الكون، وهو أيضًا عمر نظام الإشعاع الخلفي، وذلك باستخدام مقياس الحرارة والكمبيوتر. ولا يستطيع معرفة عمر الأنظمة الأخرى مما يثبت تفرد النظام الإشعاعي.

    ولذلك، فإما أن تكون نظرية إطالة الوقت في العلاقات أو نظرية الانفجار خاطئة، أو أن إحداهما على الأقل تحتاج إلى مراجعة.

    لن تضعف قوتي، لكني لا أرى طريقة لشرح هذه النقطة لك.

    السبت شالوم.

  66. מיכאל
    لقد قلت أن مشكلة التوقف تنطبق على الدماغ. تنطبق مشكلة التوقف فقط على آلة تورينج - ولتذكيرك، فإن آلة تورينج والخوارزمية هما نفس الشيء تمامًا.
    إذا كان الأمر كذلك فأنا أدعي أنه مخطئ. إحدى ميزات دماغنا هي الحاجة إلى النوم. هل تعتقد حقًا أنه في المستقبل القريب سيكون هناك جهاز كمبيوتر يحتاج إلى النوم؟ وأتساءل ما الذي سوف يحلم به …….

  67. المعجزات:
    هل قلت أن الدماغ يدير الخوارزميات؟
    اقرأ كلامي مرة أخرى.

    وفيما يتعلق بعيدان سيجيف:
    إنه يدعي أن العقل الأزرق سوف يفكر، ومن الواضح أنه مخطئ لدرجة أن مشروعه حصل على منح تزيد قيمتها عن مليار دولار.

  68. מיכאל
    إذا كان الأمر كذلك، فكيف يمكنك الادعاء بأن الدماغ يدير خوارزمية/خوارزميات؟ كيف لا تعرف أن مسألة التوقف لا تنطبق على أي نموذج حسابي، بل تنطبق فقط على آلة تورينج؟
    هل يدّعي عيدان سيغيف أن حاسوبه يفكر؟ إذا كان الأمر كذلك - فهو مخطئ.

  69. بالمناسبة، أتيحت لي أيضًا فرصة العمل مع الشبكات العصبية ورؤيتها على أنها معجزة: فالتطبيقات المفيدة لهذه الشبكات ليست أكثر من مجرد محاكاة على جهاز كمبيوتر بمعالجات فردية.

  70. المعجزات:
    لقد درست الماجستير في علوم الكمبيوتر لمدة عام وأكملته بمرتبة الشرف.
    أثناء خدمتي العسكرية، ترأست فرع البرمجيات لوحدة كمبيوتر وحصلت على جائزة ILA مرتين (الجمعية الإسرائيلية لمعالجة المعلومات التي كانت شيئًا جديًا في ذلك الوقت، لكنني لا أعرف ما إذا كانت نشطة حتى اليوم) ومرة جائزة الاتصال للتفكير الإبداعي.
    لقد كسبت الملايين من براءات الاختراع في مجال الخوارزميات والناس يتوسلون إلي حتى يومنا هذا (على الرغم من أنني أبلغت منذ فترة طويلة كل من يحتاج إلى معرفة أنني تقاعدت) لكي آتي لتقديم النصيحة لهم.
    كما قلت - نموذجك للعقل ليس هو الواقع، واللانهاية التي كنت أتحدث عنها ليست لانهاية العملية، بل حجم شيء مادي (اعتقدت أنك ستفهم ذلك).
    وأقول مرة أخرى:
    إذا قرأت ما هو الكمبيوتر الذي يقوم عليه مشروع Blue Brain، فسيتعين عليك الاعتراف بأنك قلت إن إيدان سيغيف لا يفهم الموضوع.

  71. מיכאל
    هل درست علوم الكمبيوتر؟ على وجه الخصوص - دورات في مجال الحوسبة؟

    وعن العبث. قلت أنه لا يوجد لا نهاية في الطبيعة. ربما هذا صحيح. إذا كان الأمر كذلك - فإن مشكلة التوقف غير موجودة.
    حاولت إخبارك سابقًا، وسأحاول مرة أخرى. يُطلق على النموذج الحسابي للكمبيوتر اسم آلة تورينج. تتمتع آلة تورينج بذاكرة لا نهائية، لكن بالطبع لا يوجد كمبيوتر يمتلكها. يحتوي نموذج الدماغ الخاص بي على معالجات لا حصر لها، على الرغم من أن هذا ليس واقعًا. ولكن كما أن آلة تورينج مفيدة في أبحاث الكمبيوتر، فإن نموذج الآلة O مفيد في أبحاث الدماغ.
    ولا يمكن تقليد نموذج الآلة بواسطة جهاز كمبيوتر يحتوي على عدد محدود من المعالجات.

    إذا كنت تريد أن تكون متحذلقًا، فحتى أكبر جهاز كمبيوتر في العالم لا يمكنه الاعتماد. وهذا صحيح في الواقع، ولكن لا علاقة له بالموضوع.

  72. المعجزات:
    مشكلة التوقف موجودة أمام أي جهاز يدعي تحديد ما إذا كان برنامج الكمبيوتر يتوقف أم لا، إذا كان قادرا على قراءة قراراته.
    إنها لا تنتمي إلى معمارية الكمبيوتر على الإطلاق وإذا قرأت إثباتها فسوف ترى ذلك.
    لا يشير الدليل إلى حقيقة أنها آلة تورينج على الإطلاق.
    إنها تتحدث عن خوارزمية ولكن منطقها سيعمل أيضًا مع كلمة "شيء يستطيع الكمبيوتر قراءة قراراته" وهذا الشيء يمكن أن يكون العقل البشري.

    والآن قلت شيئاً أعتقد أنه هراء:
    لا يوجد شيء لانهائي في الطبيعة، والعقل موجود في الطبيعة.
    ولن تساعد المحكمة: كل معالج متوازي، وبغض النظر عن مستوى التوازي، يمكن تقليده بواسطة آلة تورينج واحدة (ببطء ولكن بثبات).

    ولا تقلق:
    لم يتم تصميم مشروع Blue Brain ليحتوي على مائة مليار معالج مع تريليونات من الاتصالات فيما بينها.

  73. מיכאל
    مشكلة التوقف غير موجودة في آلة ليست آلة تورينج. سأعطيك مثالا - المصعد. تحتوي المصاعد على جهاز تحكم وليس "كمبيوتر". يمكن إثبات أن وحدة التحكم هذه لن تدخل في حلقة لا نهائية.

    والآن قلت شيئا، في رأيي، هراء. سأحاول أن أشرح. نموذج الحساب الذي نشير إليه بالكمبيوتر هو آلة تورينج. وفقًا للتعريف - تتمتع آلة تورينج بذاكرة لا نهائية. إذا اعتبرنا الذاكرة محدودة، فلدينا آلة ذات حالة محدودة. في نموذج الدماغ الذي أصفه، هناك عدد لا حصر له من المعالجات. ولذلك، فإن النموذج الذي يصفها ليس آلة تورينج - النموذج الأفضل هو آلة o. ولذلك، لا يمكن محاكاة عمل الدماغ بواسطة جهاز كمبيوتر عادي.
    إذا كان الدماغ الأزرق يحتوي على 100 مليار معالج مع تريليونات من الاتصالات، والتي تعمل بسرعة معقولة، وحتى إنشاء اتصالات جديدة بمعدل معقول - فعندئذ نعم، سيكون لدينا محاكاة للدماغ. ومن ناحية أخرى - لن يكون هناك جهاز كمبيوتر (آلة تورينج).
    لذلك، ليس هناك تناقض بين ادعائي وعمل سيجيف.

  74. وأفترض أن أي شخص يفهم الموضوع يعرف أن أي كمبيوتر موازي (ولا يهم ما هو مستوى التوازي) يمكن تقليده بواسطة كمبيوتر عادي.
    قد يعمل بشكل أبطأ (اعتمادًا على سرعة وحدة المعالجة المركزية) ولكنه سيفعل نفس الشيء.

  75. المعجزات:
    أنا أعرف الموضوع جيدا.
    أنت تقوم بالإدلاء ببيانات دون دعمها بأي وسيطة منطقية أو بيانات داعمة.
    من بين أمور أخرى - ادعاءك يعني ضمنا أن عيدان سيغيف لا يفهم أي شيء (وقلت على وجه التحديد أنني لا أفهم الموضوع).

    وبعيدًا عن إدراكي لهذه القضية، فأنا أيضًا شخص مفكر وسأسمح لنفسي بأن أكون متواضعًا وأقول إنني أفعل ذلك بشكل جيد.

    أنت لم تواجه التحدي الذي طرحته عليك فيما يتعلق بمشكلة التوقف. أنت لم تحاول حتى. أنت فقط تحاول إخفاء الأمر بالكلمات وخداع القراء.

    ليس لدي أي مصلحة في مثل هذه المناقشة.

  76. يبدو أنك لا تفهم هذا المجال حقًا. لقد قلت أنه لا توجد مشكلة في أن تفكر الآلة. وقلت أيضًا أنه من الممكن محاكاة دماغ بشري على جهاز كمبيوتر على مستوى أساسي جدًا.
    الدماغ الأزرق ليس ذكاءً اصطناعيًا. وطالما أن عدد المعالجات صغير، فلا يزال لدينا آلة تورينج. عندما يحتوي النموذج على عدد غير محدود من المعالجات - أزعم أنك تحصل على جهاز أكثر قوة.
    هل تعرف أطروحة تورينج؟

  77. المعجزات:

    في اتصال معهذا هو ردك: أنا في الواقع قادر في كثير من الأحيان على اكتشاف أنني في حلقة لا نهائية.
    ويبدو أن هذه المرة واحدة منهم. لقد شرحت خطأك، لكنك تكرره دون النظر إلى عمق الأمور، لذلك سأتوقف.

  78. المعجزات:
    حول بالنسبة لردك على هذاأعتقد أنك تدعي أن إيدان سيجيف لا يعرف ما الذي يتحدث عنه.
    وبما أنني أقدر أن معرفتي بالموضوع أقل حتى من عيدان سيغف، فأنا سعيد لأنني التقيت بشخص ذكي كهذا يفهم الموضوع أكثر من كل الخبراء الذين لا يعرفون شيئا في عينيه.

  79. מיכאל
    لم أفهم وصفك.
    أقول شيئين:
    1) إذا قمنا بتشغيل خوارزمية في ذهننا، فمن الممكن أن تحتوي على حلقة لا نهائية ولن نعرفها. وهذا لا يتعارض مع وجهة نظري.

    2) العقل نفسه لا يمكنه الدخول إلى لوا لا نهائية. لقد شرحت بالفعل السبب.

  80. المعجزات:

    اسمح لي أن أتساءل عن تأكيدك على أن توازي الدماغ يمنحه قوة حسابية كبيرة تتجاوز قدرة الكمبيوتر. أولاً، لا تزال آلة تورينج غير الحتمية، والتي تعد أقوى كمبيوتر عملاق موازٍ يمكنك التفكير فيه، محدودة بمشكلة التوقف ويمكن تبسيط أي من حساباتها باستخدام آلة تورينج. هذا فيما يتعلق بالنموذج النظري.
    وفي الممارسة العملية كذلك. هناك أجهزة كمبيوتر متوازية قوية للغاية. يمكن أيضًا توزيع أي عملية حسابية تريد إجراؤها على جهاز كمبيوتر واحد على أي عدد من أجهزة الكمبيوتر التي تريدها.
    كل ما يمكن للكمبيوتر القيام به يمكن لعقولنا؟ انا لا اظن ذلك! انظر مدى سرعة استرجاع نتائج البحث التي تدخلها في جوجل، أدمغتنا لا تقترب من مثل هذه الإمكانيات

    وفيما يتعلق بالمشاعر، وهنا مناقشة مثيرة للاهتمام. ما الذي يجعل أذهاننا ورنيشًا خاصًا يمكن الشعور به. قوة الحساب لا تفسر هذا. آسف، الإنترنت قوي بطريقة لا تضاهى على الإطلاق من عقولنا ولكنها لا تشعر بأي شيء.

    أعتقد أنك مخطئ، وعلى العموم، فأنت تنظر عن كثب باستخدام المجهر. لفهم الظواهر كتفكير، يجب أن ترتقي إلى المستوى الخوارزمي. مستوى التفاقم ليس ذو صلة

  81. מיכאל
    لذلك أكد. لا علاقة له بمطالبتي. الدماغ الأزرق، إذا خصص شخص ما ميزانية لهذا الشيء، يمكنه محاكاة الدماغ البشري. من أجل تصوير شخص ما، تحتاج إلى إحضاره إلى حالة شخص معين. الدماغ الأزرق سيكون قادراً على محاكاة عمليات الدماغ، لكن من يعتقد أنك تستطيع أن تقول له نكتة سوف يضحك... أنا لا أفهم ما الذي يتحدث عنه.

    إذا قاموا ببناء جهاز يحتوي على 100 مليار معالج، وإذا قاموا ببناء تريليونات من الاتصالات - فربما، سيكون لدينا شيء نتحدث عنه. لكن الدماغ الأزرق لا يُبنى بهذه الطريقة.

  82. روبن
    بالتأكيد. لقد قلتها مسبقا!! أي شيء يمكن للكمبيوتر أن يفعله، يمكن للعقل أن يفعله أيضًا. ولكن ليس العكس.

    فيما يتعلق بالاسترجاع من الذاكرة، فهذا هو بالضبط ما أنت مخطئ فيه. يقوم الكمبيوتر بإجراء عملية بحث في وقت يعتمد على حجم الذاكرة. يسترد دماغنا البيانات على فترات منتظمة. لست مقتنعا بأن هذا صحيح تماما، ولكنني أعتقد أنه قريب من الواقع. وهذا فرق جوهري بين الدماغ والكمبيوتر.

    ليس لدي أي تكهنات حول مشكلة التوقف :). يحتوي الكمبيوتر على عدد قليل جدًا من المعالجات، لذلك يمكن اعتبار أي كمبيوتر بمثابة آلة تورينج. عدد أجهزة الكمبيوتر في الدماغ يعادل حجم الدماغ. هذا نموذج حسابي مختلف تمامًا عن آلة تورينج. وهنا الفرق الثاني بيننا وبين جهاز الكمبيوتر.

    بالإضافة إلى ذلك، يلزم وجود آلة تورينج للتوقف عند نهاية الخوارزمية لإرجاع المخرجات. هذا يرجع إلى إعدادات الجهاز. لا يقوم دماغنا بإجراء عملية حسابية ويتوقف لإرجاع المخرجات. إنه أشبه بآلة موصولة (مثل كمبيوتر في الوقت الفعلي). باستخدام مثل هذه الآلة، من الممكن إثبات جميع أنواع الأشياء (على سبيل المثال، أنه لن يكون هناك طريق مسدود أبدًا، وهو ما يعادل حلقة لا نهائية في الكمبيوتر). وهنا فرق آخر بين الكمبيوتر والدماغ.

    ربما الخط الحدودي هو اللغة. البشر فقط لديهم لغة.

    مشكلة التوقف هي مجرد مثال. هذا ليس هو المهم. لن يشعر الكمبيوتر بأي ألم. لا تضرب بقدر ما تريد، فلن يؤذيه. يمكنك تزييفها إذا أردت، ويمكنك تزييفها بشكل جيد للغاية. لكنها محاكاة.

    يجب أن أؤكد على شيء لا تفهمه. نحن بالتأكيد آلات، لذلك يمكن للآلات أن تفكر، ويمكن للآلات أن تشعر بالألم. لكن - دماغنا ليس جهاز كمبيوتر.

  83. المعجزات

    أفهم من إجابتك أنك توافق على أن هناك تداخلًا في القدرات الحسابية للدماغ والكمبيوتر (على سبيل المثال، كلاهما يعرف كيفية القسمة). أعتقد أنك ستوافقني على أن هذه ليست العملية الوحيدة التي يستطيع كلاهما القيام بها (على سبيل المثال، كلاهما قادر على استرجاع المعلومات من الذاكرة - أستطيع أن أتذكر قائمة من العناصر، وكذلك الكمبيوتر). هناك بالطبع قدرات أخرى متداخلة.

    يرجى محاولة تحديد خط الحدود بالنسبة لي حيث لا يوجد مزيد من التداخل في قدرات الحساب، بصرف النظر عن تكهناتك غير المبنية على أسس جيدة فيما يتعلق بمشكلة التوقف.

    بالإضافة إلى قولك إنك على حق بنسبة 100% وأن الدماغ لا يقتصر على مشكلة التوقف، كيف يساعدك هذا في تفسير القدرات الأخرى التي تعتقد أنها موجودة فقط في الدماغ مثل العواطف؟

  84. فقط لتوضيح الأذن (ولفهم التاريخ الحقيقي وأيضًا للخروج من الارتباك في فهم مصطلح "النظام المفضل")، فأنت مدعو لقراءةفصل في ويكيبيديا يناقش النظام المفضل.
    ستجد أن غاليليو ونيوتن اعتقدا أيضًا أنه لا يوجد شيء، وبهذا المعنى كانت نظرية ماكسويل انحرافًا عن مبادئهما.
    وصحح أينشتاين ذلك وأعاد غياب النظام المفضل إلى المسرح.

  85. العقد الحقيقي:
    برهاني صحيح وحقيقة أن الدماغ مرن بما يكفي لعدم الإجابة على السؤال (وكما ذكرت أيضًا - حتى الكمبيوتر المصمم جيدًا يكون مرنًا بما يكفي لذلك) لا يغير حقيقة أن الدماغ لن يقرر ما إذا كان الأمر كذلك أم لا. يتوقف البرنامج أم لا.
    بالمناسبة: هناك أسباب أخرى للاعتقاد بأن الدماغ لا يستطيع أن يقرر مشكلة التوقف (بالتأكيد ليس عقلي الشخصي أو عقلك).
    على سبيل المثال: هل البرنامج Qالبحث عن عدد زوجي ليس مجموع عددين أوليين سوف تتوقف من أي وقت مضى

  86. إلى مايكل روتشيلد، برهانك غير صالح، لأنه في مثل هذه الحالة يكون العقل البشري مرناً بما يكفي ليقول: "إذا قررت أنها ستتوقف فلن تتوقف، وإذا قررت أنها لن تتوقف فسوف تتوقف". ". بالطبع يمكنك تصميم آلة يمكنها أن تقول هذا، ثم آلة أخرى للعمل عليها وما إلى ذلك، لكن العقل البشري دائمًا ما يكون مرنًا بما يكفي لملاحظة كل الحيل.

  87. المعجزات
    العقل لا يقتصر على مشكلة التوقف. لا يمكن حل مشكلة التوقف. الدماغ لا يدير خوارزمية. وفقا للتعريف - الخوارزمية لها نهاية، والدماغ لا ينهي حساباته أبدا. يمكن للدماغ محاكاة تشغيل الخوارزمية، خطوة بخطوة، لكنه لا يقوم بتشغيل الخوارزمية.

    يجب على الشخص المصاب بالشلل من الرقبة إلى الأسفل أن يفكر. السؤال هو ما هو التفكير. لا أعرف كيفية تعريف التفكير، لكنه بالتأكيد لا يقوم بتشغيل خوارزمية. وتشغيل الخوارزمية هو كل ما يستطيع الكمبيوتر فعله.

    التفكير ليس على الخروج. الحيوانات تفكر أيضًا. أعتقد أن هناك سلسلة متصلة من الديدان إلى الإنسان. ولكن هناك مرحلة يحتاج فيها النموذج إلى الاستبدال. سأعطيك مثالا آخر. هناك نوعان من نماذج الحساب المعترف بها - آلات الحالة المحدودة وآلات تورينج (بالطبع هناك المزيد). من الناحية النظرية، هذين النموذجين مختلفان للغاية. على سبيل المثال - آلة الحالة لا تعرف كيفية العد.
    في العالم الحقيقي لا توجد آلات تورينج حقًا، جميع أجهزة الكمبيوتر هي في الواقع آلات ذات حالة محدودة. لكن نموذج آلة تورينج يمنحنا أدوات أكثر ملاءمة لتحليل آلات الحالة الضخمة. وبنفس الطريقة، أعتقد أن النموذج الصحيح للدماغ ليس آلة تورينج. كما كتبت بالفعل، فإن أساس الفكرة هو فكرة تورينج (أطروحة الدكتوراه).

  88. المعجزات:
    تقول ويكيبيديا أنه من غير المعروف ما إذا كان العقل البشري يستطيع حل مشكلة التوقف.
    هو مكتوب - حقا - ماذا تفعل؟
    وبينت أن العقل البشري لا يستطيع حل مشكلة التوقف، وبالتالي إذا كان ما بينته صحيحا فإن ما هو مكتوب في ويكيبيديا عن نفس السؤال غير صحيح.
    ليس هناك خيار اخر.

    الآن وفقا لك
    انت كتبت:
    "يمكن إجراء العديد من العمليات بمساعدة الكمبيوتر، لكن الكمبيوتر محدود في قدرته، بسبب ما يسمى "مشكلة التوقف". دماغنا، في رأيي وفهمي، ليس لديه هذا القيد."
    ما كتبته واضح تماما.
    الكمبيوتر محدود (ومشكلة التوقف توضح ذلك) والدماغ ليس كذلك.
    وبينت أن العقل غير قادر على حل مشكلة التوقف.
    إذن الأمر لا يقتصر؟ بعد كل شيء، هذا هو بالضبط القيد الذي قدمته كمثال للكمبيوتر!

    وأكرر وأسأل: هل الشخص المشلول من الرقبة إلى الأسفل ليس حياً؟ لا تفكر؟
    فلماذا يُعطى الحاسوب كل الأعضاء التي لا يتحكم فيها هذا الإنسان ولا يستقبل منها محفزات لكي يعيش ويفكر؟

  89. إسرائيل:
    تيش قوتي (مرة سألوني كيف أقول عاجز بالعبرية وعندما لم أعرف كيف أجيب قالوا لي "كلمات متقاطعة").
    إذا كنت لا تستطيع قبول تفسيري، فلن أستطيع المساعدة بعد الآن.
    ليس لدي أدنى شك في أن تفسيراتي صحيحة.
    ومن الواضح لي أيضًا أنك لن تتمكن من تقديم حجة منطقية تستخلص استنتاجك "لن نستقبل فوتونًا صفرًا إلى الأبد يمر عبر كون الشيخوخة، أو توأمين انفصلا عندما تزامن وقتهما والتقى عندما كان وقتهم مختلفًا" عن كلامي.
    كلامي صحيح.
    لدينا نظام مفضل لأن هناك نظام تفضله.
    هو نظام ينسب إلى جسم معين في الكون (الإشعاع الخلفي) الموجود في كل مكان وبالتالي له أهمية من حيث التوجه في الفضاء، لكن ليس له أهمية أبعد من ذلك - بالتأكيد ليس مهما من حيث نظرية الكون. النسبية وبالتأكيد لا تتعارض مع النظرية النسبية.
    قد تفضل نظامًا لأنه يمنح الكون الحد الأقصى للعمر.
    إنه مشروع ولا معنى له.
    يمكنك أيضًا تفضيل النظام الذي يعطي الحد الأدنى للعمر أو النظام الذي يعطي متوسط ​​العمر.
    إنه تفضيل مفسد، في رأيي، ولكن هذا مجرد ذوقي.
    حقيقة أنك تفضل شيئًا ما لا تتعارض مع النظرية النسبية.
    لقد قدمت عدة تأكيدات للنظرية النسبية وكل تأكيدات نظرية نيوتن تؤكد أيضا النظرية النسبية (ميزة النظرية النسبية هي أن التنبؤات الخاطئة لنظرية نيوتن تؤكد أيضا النظرية النسبية لأنها تعطي تلك المواقف التنبؤية) صحيح).
    ولكن هذا كل شيء.
    يكفي.

  90. روبن
    هناك البرمجيات والأجهزة. وهناك أيضا بيانات. يحتوي كمبيوتر فون نيومان (يوجد أيضًا كمبيوتر هارفارد، لكننا سنترك الفرق الآن) على معالج يقرأ المعلومات من الذاكرة. أي معلومات تصف الأوامر للمعالج هي برمجيات، وأي معلومات أخرى هي بيانات.

    لا، الدماغ ليس قطعة من الأجهزة. الأجهزة لا تحتوي على أي معلومات.

    القسمة المطولة هي خوارزمية. يحتوي دماغنا على أجزاء تعرف، بمساعدة اللغة الطبيعية، كيفية محاكاة تشغيل الخوارزمية. على عكس الكمبيوتر - لن يعلق الدماغ في حلقة لا نهائية إذا كانت هناك مشكلة في الخوارزمية.

    على وجه الخصوص - وهذا هو اكتشاف تورينج - يعرف دماغنا كيفية تشغيل الخوارزمية. إنه أقوى من الكمبيوتر، لذلك يمكنه أن يفعل ما يستطيع الكمبيوتر فعله. لا يوجد أي تناقض هنا.

    أنت تدخل إلى منطقة حساسة بفكرة الذباب ونحو ذلك. نموذج تورينج الحسابي غير موجود في الواقع. على وجه الخصوص - يجب أن يحتوي الكمبيوتر على ذاكرة غير محدودة. في النموذج الحسابي للدماغ (النموذج الذي أؤمن به) يجب أن يكون هناك عدد لا نهائي من الخلايا العصبية. وهذا يعني أنه مثلما أن الكمبيوتر البسيط (فكر في معالج 4 بت إذا كان هناك عدة مئات من البايتات من الذاكرة) لا يعادل آلة تورينج، فإن الدماغ البسيط لا يتطابق مع نموذجي أيضًا.
    من الممكن جدًا أن يكون العقل البشري هو العقل الوحيد الذي يتمتع بالقوة الكافية ليناسب نموذجي. وعلى وجه الخصوص، فهو الدماغ الوحيد القادر على احتواء اللغة. ولا يوجد أي كائن حي آخر في الوجود لديه هذه القدرة.

  91. معجزات,

    كيف يمكنك تحديد التشخيص بين الأجهزة والبرامج؟ الكمبيوتر عبارة عن قطعة من الأجهزة المتطورة القادرة على إجراء العمليات الحسابية. وكذلك عقولنا.
    كيف يختلف برنامج فرز الأرقام عن شخص يقوم بتشغيل خوارزمية تقوم بفرز الأرقام في ذهنه؟
    عندما نعلم أطفالنا القسمة المطولة ماذا نفعل إن لم نعلم أدمغتهم كيفية تشغيل برنامج القسمة؟

    مرة أخرى، اشرح لي من أين تأتي القدرات المشتركة لأجهزة الكمبيوتر والأدمغة؟ فهل أنت كافر بزعمك أن عقولنا قادرة على الحساب؟

    سؤال آخر، هل دماغ الذبابة يتجاوز أيضًا حدود الكمبيوتر؟ ماذا عن الأسماك كلب؟ قرد؟ طفل عمره سنة واحدة؟ طفل في الرابعة من عمره؟

    ما هي المرحلة التي يتلقى فيها عقلنا القدرة الرائعة على التحرر من محدودية مشكلة التوقف؟

  92. لقد شرحت مرارا وتكرارا، ولكن لا أستطيع قبول تفسيراتك. إذا كانت صحيحة، فلن نحصل على فوتون عمره 0 سنة يمر إلى الأبد عبر كون الشيخوخة، أو توأمان منفصلان عندما يتزامن وقتهما ويلتقيان عندما يكون وقتهما مختلفًا.

    ليست هناك مشكلة في وضع قطار الفطر في حالة سكون بالنسبة لنا بحيث تتم مزامنة جميع الساعات مع ساعاتنا ومن ثم تسريعها. وحتى ذلك الحين سنكون قادرين على رؤية وتصوير الفطر على أنه فطر. يمكن القول أن التسارع غيّر النتيجة، لكن أتيحت لنا نحن الباحثين في الفطر فرصة رؤية ودراسة الفطر المستقبلي ولا يهمنا كيف تم تحقيق النتيجة.

    ولكن كما أشرت، نحن ندخل في حلقة لا نهاية لها. خلاصة القول بالنسبة لي بسيطة: النظام الإشعاعي الذي نعيش فيه هو النظام المفضل، لأن الوقت فيه هو الأحدث الممكن فيزيائيًا: الوقت الذي انقضى منذ الانفجار الكبير.

    ومن حيث النسبية كما ذكرت، فإن هذا النظام ليس له أولوية على الأنظمة الأخرى.

    وهذا يؤدي إلى مفارقة في رأيي، حيث أن جميع الأنظمة موجودة في نفس الكون. لذلك، إذا كان عمر الكون في نظام الفوتون المتسارع أو الراصد التابع لعوفر أقل من 13.7 مليار سنة، والنظام الإشعاعي غير مفضل كما تزعم، فلا توجد مشكلة في الأنظمة التي يكون عمر الكون فيها 60 مليار سنة.

    وبما أنه يمكن ملاحظة نظام واحد وتصويره من الآخر، فيمكننا التنبؤ بمثل هذه الأكوان المستقبلية.

    الإمكانية الرياضية موجودة، تمامًا كما أن محاضرًا جامعيًا عمره 150 عامًا أو أب عمره 7 سنوات ممكن رياضيًا، ومن وجهة النظر الفيزيائية، فالأمر أكثر تعقيدًا بعض الشيء.

    لقد ذكرت عدة مرات التأكيدات العديدة للنسبية الخاصة (أعتقد أن هناك العديد من التأكيدات لنظريات نيوتن وماكسويل، وهما عالمان من عيار أينشتاين يؤمنان بالزمن المطلق). إذا تمكنت من إظهارهم، يمكننا معرفة ما إذا كانوا يتعارضون حقًا مع فكرة الزمن المطلق.

    ومن ناحية أخرى، كما ذكرت، لا توجد تناقضات في العلاقات الصحيحة باستثناء الزمن المطلق الناتج عن الانفجار الأعظم. وهذه نظرية أصعب بكثير في اختبارها. قلت: ليس هناك وقت لجلب كواكب إضافية إلى ما هو أبعد من حافة الكون. لا أفهم ما العلاقة مع سؤالي. هل تعتقد أن هناك كوكب نهائي؟ فإذا كنا نعيش على نفس الكوكب، فما المشكلة في أن نرسل بعض سبوتنيك إلى ما هو أبعد من حدود الكون وبالتالي نوسعه؟ غير واضح.

    ورأيي أن هناك احتمالات أخرى لتفسير نتائج تجربة MM، دون التنازل عن الزمن المطلق. رأيي هو أنه حتى من وجهة نظر منطقية بحتة، فإن نموذج ماكسويل للأثير معقد للغاية ودقيق ويشتق سرعة الضوء بدقة شديدة من ثوابت الكهرباء والمغناطيسية، بحيث يجب أن يكون صحيحًا، وإلا فإن هذا هو الأكثر خصمًا لا أعتقد ذلك إلا تخمينات السيد نحشون.

  93. وأنا لا أرى أي فائدة من محاولة إجراء التحليل النفسي الذي تدعي القيام به علي.
    آمل أن يكون لدي الانفتاح اللازم لفهم ابتكارات أينشتاين، ولو فقط بسبب حقيقة أن نظرية نيوتن فشلت بالفعل في هذا الوقت في التجربة.
    لاحظ أن الوضع ليس هو نفسه (وأنا لا أتحدث عن حقيقة أنك لا تقبل نظرية أينشتاين بعد أن تم إثبات العديد من تنبؤاتها ومع ذلك تتمسك بنظرية نيوتن). في حين أنه من الممكن أن تعزى نتائج ميشيلسون ومورلي لصالح النسبية (ضد نظرية نيوتن)، فإن ادعاءاتك لا تستند إلى لا فشل النسبية الخاصة!

  94. إسرائيل:
    لقد شرحت وأكرر وأشرح (ويمكنك أيضًا قراءة التفاصيل في الرابط الذي قدمته بنفسك وبشكل عام وبشكل أكمل في الحسابات التي أظهرتها):
    ترى الساعات اللاحقة في السيارات البعيدة فقط لأنك مشاهد لقد كانوا غير متزامنين مع الأول في المقام الأول.
    هم تقدم الساعة الأولى في الحشد، أثناء الرحلة إليهم (أو رحلتهم إليك)، يتم تعويض جزء من هذا الحشد بحقيقة أن وقتهم يتقدم ببطئ أكثر لكنهم ما زالوا في بور.
    أشعر بالفعل أنني في حلقة لا نهاية لها بعد أن شرحتها مرات عديدة وبتفاصيل كثيرة.

    الفطر لم يعد صغيرا. هي أكبر سنا وكما ذكرنا فإن الساعة التي هي أفضل من الساعة التي هي الأولى التي يبدأ بها. من وجهة نظر المشاهد، لم تكن هناك لحظة واحدة كان فيها الفطر في نفس العمر. تتم مزامنتها فقط في القطار ولكن ليس في العارض. لقد شرحت ذلك مرارا وتكرارا. والآن مرة أخرى.

  95. حسنا، خذ وقتك (…).

    وفي الوقت نفسه، من المهم ماذا سيكون رد فعلك لو سمعت عن نظرية أينشتاين في عام 1905. الأوقات أصبحت أطول؟ قطارات يبلغ طولها كيلومترًا تم اختصارها إلى سنتيمتر واحد والناس بداخلها لا يعلمون بذلك؟

    ففي نهاية المطاف، كل نجم نلاحظه يؤكد نظرية نيوتن، كما أسمعك تقول. تمكنت هيرتز من إرسال تموجات على موقع ماكسويل، أراك تكتب. فكيف يجرؤ السيد ألبرت الوقح على تحدي كل ما نعرفه عن الفيزياء لمجرد أن شيئًا صغيرًا لم ينجح في تجربة مايكلسون مورلي؟

  96. وبحسب الرابط الذي قدمته فإن الوقت في السيارات مثل الكاميرات أعلى من وقت المشاهد.

    يمكن للمشاهد أن يجادل بأن وقته هو في الواقع الأعلى، لكن هذه حجة رياضية معقدة، وليست حجة فيزيائية، أي الصور الفوتوغرافية.

    إذا كانت هذه هي حجتك، أي أن الفطر الذي يبدو في الواقع أصغر سنًا من أخته المربكة بسبب التأثيرات النسبية مثل تقصير الطول، فقد توصلنا أخيرًا إلى اتفاق.

    وربما سيقنع هذا أيضاً التوأم العجوز بالمفارقة فيجدد شبابه كما كان من قبل.

  97. إسرائيل:
    ليس لدي الوقت الآن، ولكن كما ذكرت - قمت أنا والرابط الذي قدمته بشرح كيفية تدفق الوقت في القطار ببطئ بقدر ما يهمنا، على الرغم من أننا نرى كل قافلة في عمر متأخر عن عصرنا (وهذا بسبب عدم تزامن الساعات في أعيننا).
    إذا كنت لا تزال لا تفهم سأحاول الشرح مرة أخرى لاحقًا ولكني أشك في أنني سأنجح لأنه يبدو لي أن كل ما يجب أن يقال قد قيل وعلى الرغم من أن الأمور تبدو واضحة تمامًا بالنسبة لي فإنك غير مقتنع.

  98. إنه ليس نسبيًا فقط - في المثال الموضح، يتدفق الوقت في نظام القطار بشكل أسرع من عصرنا. على الأقل هذا ما تظهره الكاميرات.

    وهذا يتفق مع ما تقوله - نظام الإشعاع الخلفي، أي عمر الكون، لا يختلف عن أي نظام آخر، أليس كذلك؟

    لذا، إذا كان الوقت في نظام درب التبانة يتدفق بشكل أسرع منه في نظام المراقبة عوفر، أو في نظام الفوتون المتسارع الخاص بك والذي يبلغ عمره دائمًا 0 عامًا، فلماذا لا يتدفق الوقت بشكل أسرع في نظام القطار منه في نظامنا المتعلق بالإشعاع؟ جميع الأنظمة متساوية، أليس كذلك؟

  99. إسرائيل:
    مرة أخرى: لا يوجد أي معنى مادي للادعاء بأن الزمن يتحرك بشكل أسرع في القطار.
    يوضح كل من الرابط الذي قدمته وحساباتي أنه في عربات مختلفة سنرى أوقاتًا مختلفة وفقًا للنسبية (أعلى من زمننا) وفي نفس الوقت سنرى وقتنا يعمل بشكل أسرع من الوقت في كل واحدة من العربات.

  100. ב:
    لا ينتمي إلى التصوير الفوتوغرافي.
    في التجربة المعنية، تم التقاط الصور من مسافة صفر.
    المشكلة كما ذكرنا هي في التناقض بين فرضية وجود الانفجار الأعظم الذي تخرج فيه جميع عناصر الكون من نقطة واحدة وبين فرضية وجود أجسام في الكون تمكنا بطريقة ما من تحريكها متباعدة بينهما أكثر مما يسمح به هذا الوقت (على افتراض أن سرعة الحركة محدودة بسرعة الضوء).
    تلتقط الكاميرا لحظة الصورة عمر النجوم من وجهة نظرهم وكان ادعاء إسرائيل في الواقع يحاول إظهار التناقض بين رؤيتهم لأعمارهم ورؤيتهم لعمر الكون (في الواقع - دون أي علاقة بالعمر). عارض من نظام آخر).

    إسرائيل:
    ليس لدي مشكلة في أن يبدو الفطر للمشاهد كما لو كان من عمر مختلف. أعمارهم مختلفة حقا. وذلك لأن الوقت نسبي.

  101. هذا هو الغرض من الكاميرات، لذا لن يكون من الممكن قول ما قلته.

    وهي واضحة لا لبس فيها: فالصور تظهر الساعتين معًا - ساعة المشاهد وساعة السيارات - ونرى أنه في السيارات التالية تتسع الفجوة بين ساعة الساعة وساعة السيارات - يتخلف المشاهد، أي يتخلف المشاهد الوقت في القطار يتحرك بشكل أسرع.

    يوافق؟

  102. إسرائيل:
    ليس من الواضح سبب الحاجة إلى مثل هذا القطار الطويل.
    في كل نقطة في الكون، الوقت من الانفجار الأعظم إلى اللحظة الحالية هو نفس الوقت.
    لذا فإن زمن الكاميرا في لحظة التصوير هو الزمن من الانفجار الأعظم إلى لحظة التصوير.
    لو ذلك :
    تترك أشعة الضوء الجسم المصور وتصل إلى الكاميرا. إنهم لا يفعلون ذلك في أي وقت من الأوقات.
    ولهذا السبب يكون الكائن المصور دائمًا أقدم من صورته الموجودة في الكاميرا.
    أي أن عمر الشيء الذي تم تصويره قد تم اكتشافه قبل عمر الكاميرا وقت التقاط الصورة.
    لكن عمر الكاميرا وقت التقاط الصورة هو عمر الكون منذ لحظة الانفجار الأعظم.
    ولذلك فإن كل جسم تم تصويره هو أقدم من الكون.

  103. من الواضح أنني فهمت كلماتك وأن الصياغة الواردة في تعليقك الأخير مضللة مرة أخرى.
    لا يوجد افتراض بأن الوقت في نظام القطار يتدفق بشكل أسرع من عصرنا. الافتراض هو أنه من وجهة نظر القطار، فإن الوقت يتدفق بشكل أسرع من وقتنا، بينما من وجهة نظرنا، يتدفق الوقت بشكل أسرع معنا.

  104. أنا أعتقد هذا. وأنت كلامي؟

    ماذا يحدث إذا مر بك قطار (ليس طويلًا جدًا) به فطر الغابة والتقطت صورة للفطر بداخله مع الفطر المقارن الذي يقف بجوارك.

    في المقطع الأول، ستشاهد فطرًا صغيرًا، لكن الفطر الموجود في المقطع الأخير سيكون ملتفًا بالفعل، أليس كذلك؟

    إذن أنت تدعي أنه لا توجد مشكلة في حقيقة أن الوقت في نظام القطار يتدفق بشكل أسرع من وقتنا بالنسبة للإشعاع؟

  105. ومرة أخرى: لا توجد مشكلة في التقدم السريع للوقت في هذا النظام أو في نظام آخر.
    المشكلة في الواقع هي افتراض احتمال وجود نجوم في كون محدود تتطلب المسافة بينها زمنا يتجاوز عمر الكون.
    ومن الواضح أنك إذا خلقت نجوما تحتاج إلى وقت أكبر من عمر الكون لخلق المسافة بينها، فسيكون هناك نجوم يتجاوز عمرها عمر الكون.
    الطلب: لا تدخل التضخيم في القصة بالنسبة لي الآن لأنه لم يكن موجودا فيها أصلا ولا أعرف ما يكفي عن النظرية التي تنطوي عليها للتعامل مع سؤال جديد سيصاغ فقط من قبل حقيقة أنني ذكرت ذلك.

  106. إسرائيل:
    حقا!
    لا يوجد جدار! ببساطة لم يكن هناك وقت لإيصالهم إلى هناك لأن الوقت اللازم لإيصالهم إلى هناك يتجاوز عمر الكون!
    حتى الآن لم تسمح لي قصصك بفهم ما يزعجك بالضبط واعتقدت أن التجربة الفكرية التي تتحدث عنها يجب أن تكون متسقة مع البيانات وأن الاستنتاجات الناشئة عنها فقط هي التي تتعارض معها، ولكن بمجرد أن أصبحت القشرة المربكة إزالة أصبح واضحا لي أنك تتحدث عن تجربة أن مجرد افتراض إمكانية إجرائها يتناقض مع افتراض الانفجار الكبير والمشكلة ليست في التناقض بين نظرية الانفجار الكبير والنسبية ولكن بين نظرية الانفجار الكبير و نظريتك الخاصة بأن مثل هذه التجربة ممكنة!

  107. إصلاح (آيفون).

    الجملة الأخيرة هي جملة شرطية: إذا كان الوقت في السيارة الأولى هو نفس زمننا، فإن الوقت في السيارة الأخيرة يتقدم على زمننا.

  108. هل هناك حد نهائي لوضع الكواكب؟

    وبعد آخر كوكب وضعناه، ما الأمر يا كير؟

    من ناحية، أنا سعيد، لأنك ربما فهمت أخيرًا أن حجتي ليست بالضرورة ضد النسبية، ومن الممكن جدًا أن تكون نظرية الانفجار الأعظم ذات الحدود للمكان ونقطة البداية للزمن هي النظرية الإشكالية.

    (هل يمكن أن تشرح لي وللمنتدى ما يعنيه الفضاء المحدود وبداية الزمن؟ ماذا بعد الكوكب الأخير، وماذا كان قبل الانفجار؟ تبدو عبارات "لا يوجد شيء" و"لم يكن هناك" بديهية بعض الشيء في رأيي أرجو التوضيح).

    ومن ناحية أخرى، أشعر أنني لم أوضح النقطة بما فيه الكفاية. ليس من الضروري أن يتجاوز التوأم عمر الكون. ويكفي أن نلاحظ - ونصور - نظامًا يتدفق فيه الزمن بشكل أسرع من نظامنا، لعكس مفارقة التوأم.

    وهذا ما يحدث إذا مر بنا قطار، مهما كان طوله أو المدة التي قطعها. الوقت في السيارات الأحدث أكثر تقدمًا منه في السيارات الأولى.

    وبما أن الزمن في القافلة الأولى هو نفس زمننا، فإن الزمن في القافلة الأولى يتقدم عن زمننا.

    يوافق؟

  109. מיכאל
    إذا قام شخص ما ببناء جهاز كمبيوتر يشبه خلية في جسم الإنسان، بحجم جسم الإنسان، يشبه جسم الإنسان، بل ويعمل بنفس معدل جسم الإنسان... فإننا نكون قد استنسخنا إنسانًا، ولقد لا مشكلة في ذلك. هذا ما نفعله في Startrek …….

    ولم أقل أن الشخص يعرف كيفية حل مشكلة التوقف. قلت إن هذا القيد لا ينطبق عليه. هذا فرق كبير.

    ولا، ويكيبيديا ليست مخطئة في هذه الحالة. على أي أساس قلت ذلك؟؟ أنت لم تفهم المعنى. من السهل إثبات وجود مشكلة التوقف في آلة تورينج، فالدليل بأكمله يتكون من نصف صفحة. ما هو مفتوح هو مسألة ما إذا كانت هناك آلات "أقوى" من آلة تورينج. يدعي مارتن ديفيز أنه لا يوجد شيء من هذا القبيل. تقول ديان براودفوت أن هذا ممكن. وكان آلان تورينج نفسه يعتقد أن ذلك ممكن - وكان هذا موضوع الدكتوراه 🙂

    تصوير الدماغ جيد. لكنه ليس عقلاً مفكراً.

  110. بالمناسبة المعجزات:
    يحاكي المشروع المعني الدماغ باستخدام برنامج عادي يعمل على جهاز كمبيوتر عادي (حسنًا، قوي بشكل غير عادي، ولكنه يعتمد على المبادئ العادية لبناء الكمبيوتر).

  111. المعجزات:
    لقد أعطيت مشكلة التوقف كمثال وبينت لك أنه مثال خاطئ.
    مقبول؟
    وإذا كان مقبولا - فكلام ويكيبيديا عن كون المشكلة مفتوحة هو أيضا خطأ.

  112. المعجزات:
    من حيث المبدأ، لا مانع من القيام بكل ذلك، ومن الممكن أيضاً تحضير أطباق لذيذة له.
    وبدلاً من ذلك، يمكنك بناء بيئة الواقع الافتراضي له.
    أعتقد أنك لا تعتقد أن الأشخاص المصابين بالشلل في كل مكان ليسوا على قيد الحياة وغير قادرين على التفكير. القدرة على السمع والتحدث والرؤية والتذوق والشم يمكن أن تعطى لمثل هذا الدماغ دون أي مشكلة معينة ومن الممكن أيضًا تحييد الشعور بالجوع لديه (لا تحتاج إلى تحييد لأنه إذا لم يكن جائعًا) فهو لا يجوع وينشأ الشعور بالجوع نتيجة نقص شيء لن يفتقر إليه).

  113. מיכאל
    ولست بحاجة لإثبات أن العقل يستطيع حل كل مشكلة لا يستطيع الكمبيوتر حلها. أنا فقط بحاجة للعثور على واحدة. كما قلت، هناك سؤال مفتوح حول ما إذا كان هناك "حساب فائق". أعتقد أن هناك….

    وأنا لا أتحدث بأي يقين !!! أنا أقدم مطالبة. لقد وجدت في السنوات الأخيرة الكثير من التعزيزات لهذا الادعاء.

  114. מיכאל
    كل هذه تؤدي تصوير الدماغ.
    ولكن هل سيتصرف هذا الدماغ بشكل مختلف عندما يكون جائعاً؟ إذا قمت ببناء مثل هذا الدماغ، مثل دماغ الإنسان، قم بتوصيله بجميع أجهزة استشعار الجسم، وامنحه كل قدرات الجسم الحركية، وكذلك حدوده الجسدية، دعه يشيخ بمعدل الإنسان، تأكد فهو محاط بالبشر - أي أنه سيرى أشخاصاً آخرين يشبهونه... افعل كل هذا لعله يفكر كالإنسان. طبعا لا تستطيع توصيله ...........

  115. روبن
    أنت تقارن البرتقال والتفاح. يحتوي الكمبيوتر على أجهزة وبرامج. الدماغ لديه خلايا عصبية. لا يمكن تعيين هذين الأمرين. وبالمناسبة، لا يستطيع الكمبيوتر معالجة اللغة الطبيعية.
    الكمبيوتر، بحكم التعريف، يمكنه تشغيل الخوارزميات. يمكنه تشغيل أي خوارزمية يمكن تخيلها. وبالإضافة إلى ذلك، هذا كل ما يستطيع الكمبيوتر فعله.
    ادعائي هو أن دماغنا يمكنه أيضًا حل المشكلات التي ليس لها حل خوارزمي. لقد شرحت قليلاً لماذا أفكر بهذه الطريقة في تعليقي السابق.

  116. إذن ماذا تقول لنا بشكل عام؟
    أنت تقول أن هناك حدًا للمسافة التي يمكنك وضع الكواكب عندها (وقد عرفنا ذلك مسبقًا لأنه في الكون المحدود يوجد مثل هذا الحد بالفعل) (وسوف أضيف أيضًا أن التزامن بين الساعات سيستمر لفترة أطول بكثير) في هذا النظام من عمر الكون)
    بالمناسبة - ربما يكون الكون أكبر من 16 مليار سنة ضوئية، ولكن هذا بسبب التضخم وبسبب التوسع المتسارع للكون، لذلك ربما لن تتمكن من اجتيازه إذا حددت سرعتك بالسرعة من الضوء.

  117. السبب وراء انتقالي من ثلاث ثوان إلى مائة ألف سنة، مليار سنة وثلاث ساعات، وما إلى ذلك بسيط: نحن نتعامل مع مقال مواز لم تقرأه. البيانات هناك ثلاث ثوان ومائة ألف سنة. وللحديث عن الأعداد المستديرة وليس عن الكسور، فإن الصمغ يسافر مائة ألف سنة ضوئية في ثلاث ثوان، وبحسب ساعته فإنه سيسافر مليار سنة ضوئية في حوالي ثلاث ساعات. هذا كل شيء.

    وبالطبع فإن ساعة إشعاع الخلفية فريدة من نوعها. إذا سافر المسافر مائة ألف سنة ضوئية في ثلاث ثوان حسب ساعته، ففي غضون يومين يرى ويصور كواكب يبلغ عمرها البيولوجي 16 مليار سنة على الأقل.

    وإذا عكسنا الخلق، أي أننا نلاحظ نظام نفس الراكب (القطار المزدوج في مثالنا)، فخلال يومين نرى ونصور توأمين يبلغ عمرهما البيولوجي 16 مليار سنة على الأقل.

    ممكن سنة 1905.

    مستحيل إذا كان أقصى عمر ممكن للتوائم هو عمر الكون.

    يوافق؟

  118. المعجزات:

    إن حججك بشأن عدم قدرة الكمبيوتر على محاكاة عمليات التفكير يمكن أن تكون صالحة لأي عملية تقوم بها عقولنا. لماذا يجب أن يكون الكمبيوتر قادرًا على معالجة اللغة؟ لماذا يجب أن يعرف الكمبيوتر كيفية معالجة المعلومات المرئية؟ لماذا عرف كيف يحسب الطرق أو يحل المشاكل؟ بعد كل شيء، فهو مصنوع من أجهزة مختلفة عن تلك التي لدينا.
    ومن ناحية أخرى، لماذا سنكون قادرين على فرز الأعداد أو إجراء أي عملية حسابية مثل القسمة؟ هي الإجراءات التي يقوم بها الكمبيوتر وهو مصنوع من أجهزة مختلفة...
    النقطة التي أقصدها هي أنه من الواضح أن هناك تداخلا بين قدراتنا وقدرات الكمبيوتر، أنت تدعي أن التداخل ليس كاملا ولكن لماذا النزول إلى مستوى التفاقم لتفسير هذه الاختلافات. إذا كان التفسير على مستوى التفاقم، فيجب أن يخبرنا هذا التفسير أيضًا من أين يأتي التداخل في القدرات الحسابية.

  119. نفخ الماء
    1) إذا قال أينشتاين أن الكون لانهائي فهل الكون لانهائي بالضرورة؟
    لم يعتقد أينشتاين أن الكون لانهائي.
    2) ما علاقة هذه القاعدة بحديثنا؟؟؟
    3) هل هذه جملتك الغبية لهذا اليوم؟؟

  120. إسرائيل:
    لم أقرأ مقال عوفر ولا أرى ضرورة لذلك.
    ما كتبته مكتفٍ بذاته، لذا لا داعي لقراءة مواد إضافية.
    كنت أتحدث عن التوائم المتسارعة - ليس بسبب مقال عوفر وليس بسبب شيء محدد كتبته في الرد الحالي، ولكن لأنك قلت في ذلك الوقت أنه ليس لديك مشكلة مع النسبية العامة.
    أردت أن أشير إلى حقيقة أن نفس الاعتراض (غير المبرر) الذي لديك على النسبية الخاصة يمكن أن ينطبق أيضًا على ظواهر النسبية العامة وأن أعرب عن دهشتي لأن نفس المشكلة تزعجك في إطار واحد ولا تزعجك في إطار آخر.

    والحقيقة هي أن التوائم البيولوجية - كما سبق أن ذكرت - لديها بالفعل مشكلة في السيناريو الخاص بك لأنه من المستحيل أن تبعثرهم في الكون دون تسارع، ولكنني سأكون على استعداد لوضع افتراض في هذا الشأن لأنني لا أرى التوائم كأي شيء باستثناء الساعة والساعات، ليس لدينا مشكلة في التشتت ثم المزامنة (وهو أمر صعب للغاية مع التوائم).

    هو بالفعل سيرى الكواكب في عمرها من حيث مظهرها وأنا لم أدعي غير ذلك ولا أدري لماذا تجد أنه من المناسب أن "تشرح" لي أمراً آخر تافهاً. حتى أنني ذكرت ذلك في ردي! بعد كل شيء، حتى في الحساب الذي قدمته يظهر! لماذا من الضروري المبالغة في الكلام عن لا شيء واستخدام لغة التعالي والتعالي لهذا الغرض؟

    أنت تلعب بالتناوب بين مائة ألف سنة ومليار سنة. أطلب منك الالتزام برقم واحد (مليار أقصر في الكتابة) وعدم إضافة ارتباك لا داعي له.

    بينما على مراقبة المراقب، مر يوم واحد (مرة أخرى مباراة بين يوم وثلاث ثوان - سنبقى في اليوم - قصة واحدة تكفي!) لقد مر مليار سنة على النظام المرصود. وأنا أتفق وحتى قلت ذلك!
    أوف! الآن فجأة ثلاث ساعات! لماذا يجب عليك تكرار نفس القصة في كل مرة أثناء تبادل البيانات؟! وليه مفيد غير الخلط؟! يوم واحد! لقد انتهينا! لا أقصد العد.

    فإذا مر الراصد بأي نظام، سواء كان درب التبانة أو المحطة المركزية، خلال ثلاث ساعات حسب ساعته، فإن عمره يزيد بمقدار ثلاث ساعات مقارنة بعمره في بداية الرحلة. لا أعرف من أين وأين جاءت مليار سنة التي تشير إليها بعمر المشاهد.

    المشاهد عبارة عن ساعة. إنه أكثر ملاءمة لأنه يمكن مزامنته - وهو ما لا يمكن القيام به مع التوأم.
    عندما التقى بالتوأم الأول في القطار، كان كلاهما طفلين.
    عندما التقى بالأخير، كان عمره يومًا واحدًا وفقًا لساعته الخاصة (وليس ثلاث ساعات! أرى أنه في الجملة السابقة جرني هذا الارتباك الذي أضفته للتو) وكان عمر التوأم الأخير مليار سنة وفقًا إلى الساعة التوأم.
    هذا ما قلته في تعليقاتي السابقة، هذا ما أقوله الآن وإذا قلت ذلك (وأكثر من مرة) فلا بد أن تكون علامة على أنني أستطيع أن أخبرك بهذا - فلماذا تسأل إذا كان بإمكاني أن أخبرك بهذا ؟
    بالمناسبة - هذا ليس الحد الأدنى للعمر الذي يلتقي فيه بشخص ما. مع كل توأم يلتقي في عمر معين خاص به وفي عمر معين من التوأم (عمر معين هو الحد الأدنى والحد الأقصى لنفسه).

    لكني أطلب منك أن تقرر: هل هو مرتبط بإشعاع الخلفية أم لا؟ هنا لم تتحدث عن إشعاع الخلفية وفي أحيان أخرى تحدثت عنه.
    لقد استنفدت بالفعل بالفعل! أحاول الإجابة عليك (وأعتقد أنني أجبت بالفعل عدة مرات) ولكن يبدو لي أننا نبث في أكوان متوازية.

  121. مايكل، هل قرأت مقال عوفر؟

    إذا كان الأمر كذلك، هل يمكن أن تريني أين هو مذكور هناك - أو في مثالي - التسارع؟

    لا تحتاج إلى تسريع في المثال. لا تحتاج إلى توأم أيضًا. ولم يتم طرحها إلا لأننا استخدمناها في الأمثلة السابقة، كساعات طبيعية ذات نقطة 0 طبيعية: الولادة. إذا استقراءت من مقال عوفر، فإن أي مسافر يسافر بسرعة نسبية سيرى درب التبانة يمر خلال ثلاث ثوان حسب ساعته، سواء كان توأمًا أو طفلًا وحيدًا.

    وليس للأمر علاقة بعمر الكون من وجهة نظره. ففي نهاية المطاف، فهو يصور الكواكب أثناء مرورها، لذا فإن ما ستظهره الصور هو عمرها من وجهة نظرها، وليس عمره.

    إذا كان من الصعب عليك رؤية ذلك، ففكر في ذلك المسافر الذي يمر بمجرة درب التبانة في ثلاث ثوان فقط حسب ساعته:

    وبما أن نظام درب التبانة نظام قصوري لغرض المثال، فلو قمنا بضبط ساعات على الكوكب الأول والأخير تمر بها وتتزامن بينهما، فإن فارق التوقيت بين الوقت الذي استغرقه على الكوكب الأول والوقت على الكوكب الأول آخر واحد هو مائة ألف سنة، هل توافق على ذلك؟

    في تلك الفترة الزمنية مرت ثلاث ثوان فقط حسب ساعته، لكن هذا لا يغير حقيقة أن النظام الذي يمر به - درب التبانة - قد عمره في هذه الأثناء مائة ألف سنة حسب ساعته، وهذا هو أيضا ما سوف تظهر الصور. مقبول؟

    فلو كان طول مجرة ​​درب التبانة مليار سنة ضوئية، لكانت تمر في حوالي ثلاث ساعات على ساعتها، ومليار سنة على ساعتها. يوافق؟

    عمره الآن مليار سنة حسب ساعته.
    هل يمكنك أن تقول ما هو الحد الأدنى لعمر الكواكب التي يمر بها على وجوههم وساعاتهم الطبيعية، التوائم، يلتقط الصور حسب ساعاتهم؟

    مساء الخير.

  122. عزيزي نسيم، صباح الخير.
    و. قال أينشتاين أن الكون لانهائي
    ب. هناك قاعدة في الفيزياء: ما هو هنا موجود، وهذا يشمل الانفجار الأعظم
    ثالث. والفضاء بشكل عام رياضي، لا نهائي وليس له نهاية في الأبعاد - رياضي، ويمكن أن تكون المادة عليه

  123. إسرائيل:
    أنت تكرس المزيد من الوقت لصياغة القصة (بطريقة تجعل من الصعب جدًا فهم نيتك) بدلاً من فهم المشكلة.
    إنه مرة أخرى نفس إله ساعة إشعاع الخلفية للنظام الثابت بالنسبة إليه.
    كما قلت عدة مرات - كان من الممكن أن يحدث نفس الشيء إذا كان بإمكان أي نظام آخر (وربما يستطيع ذلك) نشر ساعاته في الفضاء والسماح للجميع بقراءتها.
    أنت ترفض الاعتراف بأنه عندما يكون شخص ما في نظام مختلف، فإن عمر الكون الذي يحسبه يختلف.
    توأمك (لأسباب احتفظت بها معك) لم يعد متحمسًا بسبب فارق السن عندما يتعلق الأمر بالتسارع (بعد كل شيء، التوأم المتسارع غادر قبل ساعة وها هو يعود إلى الكون بعد أن نضج بمليار سنة ولم ينضج أي منهما مندهش) لكنه متحمس لموقع نضج أكثر أو أقل منه (على الرغم من أن أنظمته البيولوجية لديها أيضًا وساعة إشعاع الخلفية هي مجرد ساعات مثل الساعات).
    كما قلت - إن عدم التماثل يرجع فقط إلى حقيقة أنه تم اختيار أحدهما كخلفية للعديد من الساعات والآخر تم اختياره كمشاهد واحد.
    ولهذا السبب أوضحت أيضًا في إجابة سابقة أن نقطة التزامن في الخلفية ستظهر أعمارها الأكبر على الخلفية المعاكسة.
    الآن: ربما يكون من الممكن حقًا حساب ساعة إشعاع الخلفية لأي نظام آخر من أي نظام (بما في ذلك النظام الثابت بالنسبة لإشعاع الخلفية) (فقط قم بتصحيح ترددات الضوء بحيث تعكس الحركة المطلوبة) وبعد ذلك سيحدث نفس الشيء بالضبط أمام هذا النظام.

  124. حسنًا، اصعد إلى القطار. أطلق المراقب صفيرًا وأعطى الإشارة أيضًا بعلم أخضر.

    تجلس عائلة التوائم بأكملها في عربات وتتزامن ساعاتهم مع بعضها البعض. عندما يبلغون العشرين من العمر، تمر آخر عربة مع التوأم الأخير فوق كوكب يبعد مليار سنة ضوئية عن الأرض. إنهم يتحركون في نفس الاتجاه بسرعة الضوء تقريبًا مقابل إشعاع الخلفية الكونية. يصور التوأم الأخير نفس الكوكب البعيد ويقيس درجة حرارة الإشعاع.

    إنه يعلم أن أمامه رحلة طويلة - مليار سنة - إلى نهاية العالم، لكن هذا لا يزعجه كثيرًا. السبب بسيط: التوأم الأخير، مثل الأعضاء الآخرين في عائلة التوأم، يكونون في حالة راحة بشكل عام. القطار واسع ومريح ويمكنهم أن يعيشوا حياتهم بسلام، كما يستمتعون بمشاهدة النجوم والكواكب وهي تطير أمامهم بسرعة الضوء تقريبًا. بالنسبة لهم، هم الساكنون والنجوم العابرة هي في الواقع القطار.

    عندما يصل إلى KDA، يقوم بتصويره وقياس درجة حرارة إشعاع الخلفية. ورغم أن عمره بالفعل مليار سنة، إلا أنه يرى في الصورة المشاهد في إسرائيل الذي نشأ في يوم واحد فقط. المشاهد من إسرائيل يرسل له هاتف iPhone

    الصورة التي التقطها المشاهد لنفسه عند مروره. لا شك أنه لا يبدو سيئاً مقارنة ببقية زملائه المليارديرات...

    ينظر المشاهد في إسرائيل إلى صورة التوأم البالغ من العمر مليار سنة ويبتسم، حتى يظهر أوليم كاماتي فجأة على جبهته.

    ففي النهاية، قرأ مقال عوفر مجاد "درب التبانة في ثلاث ثوان فقط". وهو يعلم أنه عندما يتحرك المسافر عكس إشعاع الخلفية بالسرعة التي تحرك بها التوأم الأخير، كل ثلاث ثوانٍ وفقًا لساعة المسافر، فإن الكون يشيخ بمقدار مائة ألف عام.

    ففي النهاية، ليس هناك شك في أن التوأم الأخير بدأ الرحلة بعد الانفجار الأعظم. حتى أنه التقط صورة للكوكب الذي مر به منذ مليار سنة.

    إذا كان عمر الكواكب الموجودة على سطحها كل ثلاث ثوان، بحسب ساعة التوأم الأخير، مائة ألف سنة - وهذا يمكن رؤيته في الصور - ويبلغ عمر التوأم الأخير الآن مليار سنة بحسب ساعته، ويظهر هذا أيضًا من خلال الصورة التي يحملها المشاهد بيده - إذن كم عمر الكون على سطحه التوأم الأخير عندما قام بتصوير KDA مع المشاهد الذي هو جزء من نفس الكون؟

    هل يمكنكم أيها المعلقون الطيبون مساعدة المشاهد على حساب عمر الكون الذي يظهر في الصورة التي التقطها التوأم الأخير إذا كان عمره مليار سنة وكل ثلاث ثوان حسب ساعته يعمر الكون مائة ألف سنة؟

    ربما يمكن أن يساعد تورينج، ويبدو لي أنه هو الأنسب لحسابات بهذا الحجم.

    إذا كانت الأخطاء - الصحراء، اي فون.

  125. מיכאל
    هذا حقا سؤال مفتوح. وفي رأيي الشخصي أن العقل ليس جهاز كمبيوتر. أساس الفكرة هو ادعاء غير معروف من قبل آلان تورينج. أثار تورينج احتمال وجود آلات أقوى من آلات تورينج (بالطبع لم يصادف أن تكون آلات تورينج....). أطلق عليها اسم "آلات O". يأتي حرف O من كلمة Oracle. بشكل عام، الفكرة هي أن هناك أوراكل يمكن طرح الأسئلة عليه والإجابة عليه في وقت محدد. يُطلق على هذا النهج اليوم اسم الحوسبة الفائقة وهناك العديد من المناقشات حوله، هل هو موجود على الإطلاق؟
    الآن - أتناول فكرة الدماغ الأيسر/الدماغ الأيمن (التي جاءت في الأصل من تحليلات روجر سبيري): يمكنك النظر إلى الدماغ كآلتين. الأول (الجانب الأيسر) هو المسؤول عن اللغة ويعمل بشكل خطي. يتمتع الجانب الأيمن بإمكانية الوصول المتوازي إلى الذاكرة وبالتالي فهو يشبه الكمبيوتر المتوازي.
    وأضاف: أنا أدعي أن دماغنا عبارة عن كمبيوتر فائق السرعة، لذلك لا أؤمن بالذكاء الاصطناعي القوي. لا بد أن جون سيرل كان فخوراً بي، على الرغم من أنني أعتقد أن طريقته في استخلاص النتائج خاطئة (على الرغم من أن الاستنتاج صحيح في رأيي).

    هذا باختصار شديد. لقد قمت بتحليل الكثير من التفاصيل في الفكرة وهناك عدد قليل من "الطبقات" التي أفضل عدم ذكرها بالتفصيل.

  126. المعجزات:
    على أي أساس تزعمون أن أدمغتنا غير مقيدة بمشكلة التوقف؟
    هل يستطيع عقلنا أن يقرر ما إذا كان البرنامج الذي يخلق المفارقة في إثبات مشكلة التوقف الخاصة بالكمبيوتر يتوقف أم لا؟
    لنفترض أنك وضعت مستشعرًا على رأسك من النوع المستخدم في تجربة ليبيت الذي يبلغ البرنامج بالنتائج التي توصل إليها ويسمح للبرنامج بالدخول في حلقة لا نهائية إذا قرر الدماغ أن يتوقف أو يتوقف إذا قرر الدماغ عدم القيام بذلك.

    ومن المثير للاهتمام أنفي وصف مشكلة التوقف في ويكيبيديا ولم يصروا على هذا الاحتمال وطرحت مسألة وجود مشكلة التوقف في الدماغ هناك كسؤال مفتوح.

  127. روبن
    ولا أرى أي مستوى من التشابه بينهما. يمكن إجراء العديد من العمليات بمساعدة الكمبيوتر، لكن الكمبيوتر محدود في قدرته، بسبب ما يسمى "مشكلة التوقف". دماغنا، في رأيي وفهمي، ليس لديه هذا القيد.

    أنت مخطئ بقولك "أجهزة مختلفة" - فعقلنا لا يحتوي على أجهزة و/أو برامج. نحن لسنا جهاز كمبيوتر.

  128. "يتذكر الركاب في قطار إيمك أن مدة السفر فيه استغرقت ما لا يقل عن مليار سنة..." - أوفاهاها. لا بد أنهم قطعوا شوطا طويلا، أليس كذلك؟ مرت إيلات، أليس كذلك؟

  129. إسرائيل:
    كما قلت في المرة السابقة التي طرحت فيها نفس السؤال بالضبط - لم أقم بحساب دقيق، ولكن من حيث المبدأ فإن الإجابة إيجابية. ففي النهاية، إذا كانت المسافة بين السيارتين حوالي مليار سنة ضوئية وإذا قبض القطار على الأفعى بسرعة قريبة جداً من سرعة الضوء، فإن الزمن الذي يمر بين اللقاء مع السيارة الأولى واللقاء مع الأخير حوالي مليار سنة.

  130. ב

    لم أكن أتحدث عن قطار لا نهائي، بل عن قطار يبلغ طوله في نظام سكونه مليار سنة ضوئية، على بعد بضعة سنتيمترات من وجهة نظر المشاهد.

    القطار هو نفس مجرة ​​درب التبانة في مقال عوفر ماجد، ولكنه أطول فقط. ما يميزها هو أنها تتحرك ضد إشعاع الخلفية بينما تستقر مجرة ​​درب التبانة (تقريبًا) بالنسبة إليها.

    لكن من حيث النسبية الخاصة لا يوجد فرق بين النظامين. ولم يكن أينشتاين يعرف حتى عن الإشعاع في عام 1905.

    يتذكر ركاب قطار "إيميك" أن زمن السفر عليه استغرق مليار سنة على الأقل...

  131. المعجزات:

    أوافق على أن الدماغ عند أدنى مستوى يختلف عن الكمبيوتر، لكنني لست متأكدًا على الإطلاق من أن هذا المستوى مناسب لفحص سمات مثل العواطف. وبنفس المنطق، كان من الممكن الاعتقاد بأن الإنسان وحده هو القادر على العد أو حل الألغاز أو معالجة الصورة، ولكننا اليوم نعلم أن هذه إجراءات يمكن للكمبيوتر أيضًا القيام بها، على الرغم من أنه ليس مصنوعًا من الخلايا العصبية.

    فيما يتعلق بديناميكية العواطف وطبيعة العواطف، فإن هذه الديناميكية تختلف بالفعل من شخص لآخر ومن شخص لآخر، ولكن لديك مثال على كيف يؤدي نفس التفاقم إلى سلوك مختلف. وربما ينمي الحاسوب الحساس مشاعر تميز ملامحه، أو بالأحرى هوية هذا الشخص أو ذاك

  132. إذا كان الشريط ينتشر، فلا توجد طريقة لمعرفة ذلك بخلاف مقارنته بشريط آخر.
    وإذا كان الفضاء يتوسع فلا سبيل لمعرفة ذلك إلا بمقارنته بفضاء آخر.

  133. ב
    لا يوجد أي اتصال بين الانفجار الأعظم (لا حاجة لعلامات الاقتباس) ونصف قطر الكون. إن الفضاء نفسه هو الذي يتوسع، وبالتالي فإن نصف القطر، إن وجد، لا يتزايد بمرور الوقت. هذه هي الطريقة التي أفهم بها النظرية.
    وبالطبع لا يوجد شيء اسمه "قطار لا نهاية له"... ولا حتى في تجربة فكرية.

  134. إسرائيل:
    إذا كان هناك ادعاء بأن الكون قد تم خلقه في "الانفجار الكبير"، فإن الاستنتاج هو أن الكون له نصف قطر محدود. (حتى لو كان ينمو مع مرور الوقت).
    أي أن الحد الأقصى لطول القطار في الكون هو قطر الكون. أي أنه حتى لو كان القطار مكونًا من عدد لا نهائي من العربات، فإن أقصى طول له سيكون طول قطر الكون.
    خطر:
    يهدف توصيل الأطوال إلى حد أعلى مثل توصيل السرعات. أي أن المتر زائد المتر أقل بقليل من المترين. لذا، فحتى القطار الذي يضم عددًا لا نهائيًا من السيارات له حد محدود للطول.
    أي أننا يجب أن نتخلى عن التحويل الأسطواني في توصيل الأطوال وننتقل إلى تحويل آخر (لورنتز؟) في توصيل الأطوال.

  135. إسرائيل فيما يتعلق بالتزامن، إذا كانت إحدى الساعات ترى "متأخرًا" وساعة أخرى ترى "مبكرًا"، ومن الجانب الآخر يبدو أن إحدى الساعات ترى "متأخرًا" والأخرى ترى "مبكرًا"، فلا يمكن أن يكون ذلك إلا في منظور فردي أو في بعبارة أخرى في أكوان متوازية ونعم، وبالتالي فإن "العودة إلى الزمن" أمر ممكن. بإخلاص

  136. إن مسألة التزامن أمر بالغ الأهمية للمناقشة، لذلك من المهم أن نوضح هذه النقطة.

    في القطار المزدوج وهو نظام بالقصور الذاتي، تتم مزامنة الساعات في السيارات مع بعضها البعض ولكنها غير متزامنة مع ساعة المراقب أو الساعات الأخرى في نظام المراقب.

    الشيء الوحيد الذي يمكن فعله هو أنه عندما تمر السيارة الأولى أمام المشاهد يكون عمره (أو عمره) هو نفسه. وفي المستقبل، بطبيعة الحال، سوف تنشأ فجوات. لهذا السبب كتبت أن ساعات القطار متزامنة مع بعضها البعض وليس أن ساعات القطار متزامنة مع ساعة المراقب.

    لا ينبغي أن تكون هناك مشكلة في وضع مثل هؤلاء التوائم في القطار. ويمكن حتى أن تكون روبوتات مستنسخة مماثلة للروبوتات الأصلية. يوضح أينشتاين في مقالته الأصلية عن النسبية كيف يمكن مزامنة أي عدد من الساعات في نظام القصور الذاتي.

    سؤال:

    إذا مر مثل هذا القطار المزدوج الذي يبلغ طوله مليار سنة ضوئية (في نظام سكونه) بالقرب من الراصد بسرعة تساوي سرعة راصد عوفر (على ما أذكر، فإنه يحجب مائة ألف سنة ضوئية من درب التبانة) في ثلاث ثوان حسب ساعته، بحيث تكون سرعته تعادل سرعة الضوء تقريبا)، وفي القطار الأول قمنا بتصوير توأم يبلغ من العمر 20 عاما، في أي عمر سيتم تصوير التوأم في المقطورة الأخيرة؟

    تلميح: بسبب التماثل مع راكب عوفر، يجب أن يكون عمره حوالي مليار سنة.

    كما تتذكر، فإن المشاهد الذي يلتقط الصور يكون ثابتًا بالنسبة للإشعاع والقطار هو الذي يندفع أمامه مع نادل وجرار. بالنسبة له، تستغرق رحلة القطار القصير يومًا واحدًا فقط.

    فهل كل شيء متفق عليه حتى هذه اللحظة، أي أن عمر آخر توأم يصوره المشاهد هو حوالي مليار سنة؟

    شموليك، شكرا لك. عندما نعود من الرحلة سألقي نظرة على المدونة.

  137. إسرائيل:
    1. نعم
    2. نعم
    3. نعم
    4. نعم (على افتراض أنك تمكنت من وضع التوأم بهذه الطريقة دون خلق مشكلة أخرى). التوائم البيولوجية هي أنظمة فيزيائية مثل أي نظام آخر، وعندما يتحركون زمنياً تتأثر كل ذرة من الذرات التي تتكون منها. فإذا مر عليهم مليار سنة فإن عمر الكون - عندهم - يزداد مليار سنة أخرى. إذا كان التوأم الأول متزامناً مع الراصد (من وجهة نظر الراصد) فإن التوأمين الآخرين غير متزامنين معه والفروق الزمنية كما ذكرت.
    أما بالنسبة للتوأم البعيد، فإن الزمن الذي يلتقي فيه الأول بالمشاهد هو نفس الزمن الذي يراه المشاهد. المشاهد لا يرى المستقبل - فهو فقط "يرى" الساعات غير المتزامنة.

  138. إسرائيل،
    لورانس كراوس هو أستاذ في الفيزياء ولديه نادي معجبين مفعم بالحيوية والنشاط على فيسبوك. هل تريد، في نفس وقت المناقشة التي تجريها هنا، أن تطرح سؤالك (باللغة الإنجليزية، وليس من المفترض أن يكون ذلك مشكلة بالنسبة لك)؟
    هنا هو عنوان:
    https://www.facebook.com/groups/14627823295/

  139. ميخائيل.

    من وجهة نظر رياضية بحتة، لا توجد مشكلة مع الحلول الأخرى للغز. إذا قمت بإدخال بيانات اللغز إلى جهاز الكمبيوتر، فسيظهر أن جميع الحلول صحيحة. عدم التطابق هو فيزيائي وليس رياضي. هذا ما أعنيه بالحلول الصحيحة رياضيًا ولكن ليس فيزيائيًا.

    قد تتمكن من فهم ما يزعجني إذا أجبت على الأسئلة التالية:

    1. هل تقبل أن مراقبًا يطير بسرعة نسبية على طول قطار جميع ساعاته متزامنة وتوقيت العربة الأولى هو نفس توقيته، سيرى ويصور العربة الأخيرة بعد وقت من وقته؟ الخاصة، وسوف يتم التقاط نفس الصورة من قبل المراقب من السيارة الأخيرة؟ نعم لا.

    2. هل تقبل أنه وفقا للنسبية لا يوجد فرق إذا كان الراصد يتحرك أو يستريح بالنسبة لإشعاع الخلفية؟

    3. إذا كانت الإجابات على 1 و 2 إيجابية، فهل تقبل أن المراقب الذي يكون ساكنا بالنسبة للإشعاع - أي كلنا - ويمر مثل هذا القطار، سيرى في السيارات البعيدة أن زمنها هو الحصول في وقت لاحق وبعد ذلك؟

    4. إذا كانت الساعات ساعات طبيعية -توأم مثلا- فهل تقبلين أنه سيرى توأمين أكبر سنا وأكبر، ومن الممكن تماما أنه إذا كان التوأم في السيارة الأولى هو نفس عمر المشاهد، فإن التوأم في السيارة الأولى هو نفس عمر المشاهد. السيارة الأخيرة ستكون أكبر منه بكثير؟ نعم لا.

    مشكلتي تكمن في إجابة السؤال رقم 4، لذا من المهم أن تجيب عليه.

    بالمناسبة - كما كتبت عدة مرات، ليس لدي أي تفضيل بين النسبية والانفجار. لماذا لا تدعي أنني أهاجم نظرية الانفجار الكبير؟ وكما ذكرت، فإن نفس الشخص، الذي لا أستطيع أن أقول اسمه لأنني لم أحصل على موافقته، يدعي أن هناك مشكلة في العلاقات السليمة، بغض النظر عن الانفجار. ولم أختلف معه فحسب، بل واجهته مع عوفر ماجد الذي حكم بأنه مخطئ.

    دعونا نأمل أن تتحسن حالة الأب. والدي سعيد للغاية هنا في الصحراء، فلنأمل ألا تأكلنا البرية.

  140. إسرائيل:
    ولا يزال والدي يعاني بشدة ويملأ كل وقتي.
    "اللغز" الذي قدمته (مع حله في 75) لا يخصنا. ويعود سبب رفض الحلول الأخرى إلى حقائق نعرفها عن الطبيعة وليس إلى أحاسيس داخلية لا نستطيع تفسيرها.
    بالمناسبة - في محاولتك الأخيرة لوصف ما يزعجك، استخدمت سيناريو يتحرك فيه الكون ككل بالنسبة إلى إشعاع الخلفية.
    وهذا وضع غير موجود في الواقع ويتناقض مع نظرية الانفجار الأعظم. ولو كانت موجودة، لما كانت صيغة حساب عمر الكون صحيحة.
    أكرر أنني بصراحة ليس لدي أي فكرة عما يزعجك.

  141. روبن
    يشتمل الكمبيوتر على أجهزة يتم تشغيل البرامج عليها. دماغنا ليس لديه هذا الفصل. كل خلية هي في الأساس وحدة حسابية. لذلك، لا أعتقد أنه يمكن رسم خرائط لهم.
    علاوة على ذلك، فإن الخلايا نفسها تخضع لتغيرات طوال الوقت: تموت الخلايا وتتجدد، وتتشكل روابط جديدة، وتتقوى الروابط الموجودة. وهذا مختلف تمامًا عن كمبيوتر السيليكون.

    بالإضافة إلى ذلك - وعينا هو، من بين أمور أخرى، نتيجة للعمر. الطفل ليس لديه وعي تقريبًا. ولا يتوقف ما دمنا على قيد الحياة. دعونا نتخيل أننا سنقوم بالفعل ببناء محاكاة مثالية للدماغ - سيكون دماغ شخص معين في لحظة معينة من حياته. لنفترض أنه في هذه اللحظة كان الشخص جائعا.... كيف سنجعله اسبوعا؟ لنفترض أنه جالس فكيف سيقوم؟

    هناك الكثير ليقال حول هذا…. لكن من الممل قراءة التعليقات الطويلة 🙂

  142. مايكل كيف حالك يا أبي؟

    تفاصيل.

    لقد أحضرت اللغز أولاً. هل يمكنك حلها بجدية من فضلك؟

    شكرا.

  143. إسرائيل:
    أعتقد أنني أجبتك في راحيل ابنتك الصغيرة وأنت من تتجاهل كلامي.
    لا أعلم ماذا كتب أينشتاين باستهزاء، لكن التوراة التي طورها موصوفة بصيغ رياضية وهذه الصيغ اجتازت حتى الآن كل اختبارات التجربة والملاحظة.
    وحقيقة أنك وجدت شخصًا يفكر مثلك ولا يتماهى مع اسمه ولا يقول شيئًا في الأوساط المهنية لا يغير من حقيقة أن جميع العلماء الذين يتعاملون مع الموضوع (ما عداه وربما القليل منهم) الآخرين) لا أرى أي مشكلة.
    إذا كان متأكدا من أنه على حق، فلماذا لا يكتب شيئا عن هذا الموضوع؟
    وأكرر - لا أرى أي مشكلة في الاستنتاجات الناشئة عن صيغ النسبية.
    لم أحاول حساب كل استنتاج محتمل لها، لكني أعتقد أن جميع الفيزيائيين قد حرثوا بالفعل كل زاوية ممكنة.

    عندما تقول إن شيئًا ما ممكن رياضيًا ولكن ليس فيزيائيًا - فأنت من يخصص الفيزياء (بل وتفعل ذلك دون إثبات - واعلم أن الإثبات الوحيد الذي نعرف حتى كيفية تقديم شيء ما - بصرف النظر عن الملاحظات نفسها - هو الإثبات الرياضي ).
    ولهذا السبب يزعجني بالتأكيد أنه عندما أرد على هذا تتهمني بتخصيص الرياضيات والفيزياء وعلم النفس.

  144. المعجزات:
    ما مدى ثقتك في أن الكمبيوتر لن يفكر أو يشعر أبدًا؟ من أين تأتي العواطف عند الإنسان؟ أليست العاطفة نوعا من التجريد مثل المثلث التجريدي؟

    لكن سؤالي هو ما إذا كان من الممكن يومًا ما معرفة ما يشعر به الكمبيوتر على وجه اليقين، وإذا كان الأمر كذلك (أو لا) فكيف

  145. آسف إذا بدوت صريحا، ولكن ماذا يفترض بي أن أفعل إذا حصلت على إجابة شاملة لجميع أسئلتي المستهدفة والمحددة "أنت لا تفهم" وتم إغلاق الموضوع؟

    أعتقد أنني أفهم ما أتحدث عنه. أعتقد أيضًا أن الإجابات على غرار "تعتقد القاطرة" أو "تشعر العربة" ليست إجابات مادية. وأعتقد أن هذا أيضًا ما آمن به أينشتاين عندما تصور النسبية، على الأقل وفقًا لما هو مكتوب في كتاب كاكو "عالم أينشتاين":

    "كتب أينشتاين ساخرًا: "الشيء الرئيسي هو المحتوى، وليس الرياضيات. بمساعدة الرياضيات، يمكنك إثبات أي شيء." بالنسبة له، كان الشيء الأساسي هو الحصول على صورة واضحة وبسيطة (مثل صورة القطارات، والمصاعد المتساقطة، والصواريخ). ص 57.

    وهذا ما أحاول القيام به هنا دون الحد الأدنى من التعاون، مجرد تعليقات عامة ومتعالية دون الخوض في التفاصيل، حيث كما نعلم، الشيطان موجود.

    لذلك، إذا كنت مستعدًا لطرح أسئلة حول الابنة الصغيرة راحيل، وكذلك منزلها وحفيدتها، فهناك شيء يجب مناقشته. ولكن إذا أجبت على كل سؤال لدي حول القطارات والكشافة بإجابات على غرار "لقد أجبت عليك بالفعل مليون مرة" (لكنك لم تجب بشكل صحيح)، و"أنت لا تفهم" (ماذا بالضبط؟) فأنا لست مهتمًا بمثل هذه المناقشة.

    لقد قدمت في هذا المقال أمثلة لأكاديميين يعتبرون خبراء في النظرية النسبية التي تتعارض مع بعضها البعض. لقد أحضرت أيضًا اقتباسًا من رسالة بريد إلكتروني تلقيتها من أحد كبار علماء الفيزياء في إسرائيل الذي ادعى أن إشعاع الخلفية الكونية ثابت نسبيًا لأي مراقب. وبالطبع لا أستطيع أن أقبل من إيليا أي ادعاء يخالف فهمي ومعرفتي لمجرد أن هذا هو اعتقاد الكاتب.

    وفي الختام: من أراد العودة معي إلى الموضوع، ففقط بطريقة أينشتاين: القطارات والعربات والطيور والحمير الوحشية.

    الحديث، آي باد.

  146. لم أكن أعلم أنني كنت أتمرد على شيء ما. ولكن إذا كنت قد امتلكت بالفعل الرياضيات والفيزياء، فلماذا لا تمتلك علم النفس أيضًا.

    اسمحوا لي أن أعرف ما إذا كنت مهتمًا بالإجابة على إجابات بسيطة لأسئلة بسيطة ومتى. وقبل ذلك لا فائدة من الاستمرار.

    ב

    وإذا وجدت أي تناقض فيما قلته، فيرجى الإشارة إلى مكانه.

    (ربما شيء مع الرسل والسهام؟)

  147. إسرائيل:
    لا أعتقد أنك ستتمكن من إقناع الفيزياء بعصيان الرياضيات.
    كل الأشياء التي تتمرد عليها في النسبية تم إثباتها تجريبيا.

  148. إسرائيل:
    "أعتقد أن كلماتي مبنية على الفهم."
    إيمانك لا يختلف عن أي شخص آخر.
    عندما يكون هناك فهم لا يوجد تناقض.

  149. ميخائيل.

    أعتقد أن كلماتي مبنية على الفهم.

    بالطبع قد أكون مخطئا، لكن لا أستطيع قبول كلامك لمجرد أنك تقول كذا وكذا.

    لقد عرضت عليك الطريقة السقراطية للبحث عن الحقيقة: أسئلة وأجوبة تؤدي إلى التناقض. المرة الوحيدة التي طبقنا فيها هذه الطريقة وصلنا إلى الحقيقة - وهي ليست الطريقة التي أشرت إليها.

    وبما أنه لم يبق لنا إلا طريق البرلمان - كل واحد يقول رأيه دون مواجهته والتحقق منه بالرأي المعاكس - فها هو رأيي:

    تقول: "بالنسبة للفوتونات - عمر الكون صفر ولن تتمكن من إقناعهم بخلاف ذلك".

    إذا كان عمر الكون بالنسبة للفوتون صفرًا، فإن عمر الكون بالنسبة للمسافر الذي تقل سرعته قليلاً عن سرعة الضوء هو بضعة أيام وفقًا لساعته.

    إذا مر هذا المسافر بجوارنا، فسيكون قادرًا على رؤية (وتصوير) الكون عندما يكون عمره 13.7 مليار سنة. وإذا توقف، فسوف يقيس درجة حرارة تتوافق، وفقًا لصيغة فريدمان، مع كون عمره 13.7 مليار سنة. لن يتمكن من رؤية أو قياس درجات حرارة أكوان من مختلف الأعمار، ناهيك عن أكوان عمرها 20 مليار سنة.

    وإذا عكست العملية كما اقترحت، أي أن المسافر في حالة سكون بالنسبة للإشعاع وأن الكون هو الذي يتحرك - فسوف تحصل على أن المسافر يشاهد أكوانًا أقدم من كوننا.

    ممكن رياضيا ولكن ليس جسديا.

    سأسافر مع والدي لبضعة أيام لذا سيكون من الصعب علي الرد. نسختي من الترفيه عن الأب هي أن أفعل الشيء المفضل لديه (وأنا) - أدعوه في رحلة إلى صحاري كاليفورنيا وتجربة أشعة الليزر، والتلسكوبات، والتروس سريعة الدوران، وعدادات الضوء، والكاميرات، وأجهزة الراديو.

    ونعم، يتعلق الأمر بشكل مباشر بكل ما هو مكتوب هنا.

    وفي هذه الأثناء - لغز:

    يعمل الأب والابن في الجامعة كمحاضرين. عمر الابن من مضاعفات 25. وعمر الأب ضعف عمر ابنه.

    كم عمر الجد؟

    1. 59

    2. 120

    3. 75

    4. 170

    تلميح: رياضيا - كل الإجابات ممكنة.

  150. إسرائيل:
    لسوء الحظ، يجب أن أقول إن الأشياء الصحيحة التي قلتها قيلت أيضًا بناءً على سوء فهم - حتى لو ذكرت حسابات صحيحة وأنا لم أقم بالحسابات بعد ولكني أعطيت إجابات (خاطئة أحيانًا) مبنية على الفهم.
    لقد أوضحت أين خطأك أكثر من مرة ولم يتغير الشرح.
    بشكل عام - اسأل نفسك ماذا يعني مصطلح "عمر الكون".
    إذا كان مصطلحًا يُقاس بالزمن، فهو المسافة على الخط الزمني التي تفصل بين حدثين.
    ما الحدثان اللذان يحددان عمر الكون؟
    واحد منهم هو تشكيل الكون. حسنًا، ولكن ما هو الآخر؟
    الشيء الوحيد الذي يظهر من كلامك عن الثاني هو أنه حدث "الآن" ولكن ما العمل لأن "الآن" نسبي وما يبدو متزامنًا في نظام ما ليس متزامنًا في نظام آخر؟
    بشكل عام، هناك العديد من المشاكل في هذا المصطلح لأنه في نفس الذرة يمكن أن توجد في نفس الوقت جسيمات تم إنشاؤها في الانفجار الأعظم والتي تختلف أعمارها بسبب تاريخها الخاص.
    لا يختلف التزامن فحسب، بل يختلف مسار الزمن بأكمله بين الأنظمة المختلفة، وبالتالي فإن مصطلح "عمر الكون" مثل أي فترة زمنية أخرى له معنى فقط داخل نظام قصوري معين. ليس لها معنى عالمي.
    وكما ذكرت -بالنسبة للفوتونات- فإن عمر الكون هو صفر ولن تتمكن من إقناعهم بخلاف ذلك.

  151. روبن

    إذا سألت الدكتورة سوزان كالفين من أسيموف، فسوف تخبرك أن الإنسان الذي اجتاز اختبار تورينج هو الوحيد الذي يمكن اعتباره "يفكر مثل الروبوت" بالمعنى الكامل للكلمة.

    الآلة مسموحة من الشخص.

  152. روبن
    أعتقد أن دماغنا ليس أكثر من عدة أعصاب متصلة ببعضها البعض، ومن الناحية النظرية من الممكن بناء آلة يمكنها انتظار عمل الدماغ (دماغ معين في لحظة معينة).
    لكن - الدماغ ليس جهاز كمبيوتر (أي: إنه لا يعادل آلة تورينج) وبالتالي فإن الكمبيوتر لن يفكر أو يشعر أبدًا (على الرغم من أنه يمكن أن يرمز إلى هذه الظواهر).
    التمييز بين آلة تورينج واختبار تورينج. لقد فهم آلان تورينج أيضًا أن دماغنا ليس جهاز كمبيوتر.

  153. هذا الموضوع ليس له علاقة مباشرة بالمقالة، لكني أردت أن تناقشه منصة ما وهذا هو الوحيد الذي وجدته.
    هذا اختبار تورينج للتمييز بين الإنسان والكمبيوتر. أقوم بتوسيع النطاق قليلاً وأتساءل عما إذا كان الاختبار سيكون قادرًا على تحديد ما إذا كان من الممكن القول إن الكمبيوتر الذي اجتاز الاختبار "يفكر مثل الإنسان" بالمعنى الكامل للكلمة، مما يعني أنه قد الأفكار والمشاعر مثل أي شخص عادي. وإذا لم يكن اختبار تورينج، فيمكن لأي شخص أن يفكر في اختبار آخر.

    الجواب القياسي من أنصار الذكاء الاصطناعي هو أن الآلة التي تتصرف مثل الإنسان بكل معنى الكلمة ليس لديها سبب أو أساس للاعتقاد بأنها لا تفكر مثل الإنسان. هذا الجواب لا يرضيني

  154. ميخائيل

    ديجافو.

    وقريبة جدًا أيضًا. منذ أقل من أسبوعين كتبت:

    "الإجابة النسبية ليست هي التي تصفها وليس لدي أي نية لمحاولة تبرير سوء فهم أي شخص.

    لقد كان تعليقك الأخير مبالغًا فيه حقًا لأنني لم أجد فيه حتى شيئًا واحدًا صحيحًا.

    https://www.hayadan.org.il/ball-state-prez-intelligent-design-not-science-0408135/comment-page-26/#comment-448600

    وعندما بدأنا نعرف ذلك، تبين أن كل ما كتبته في الرد كان صحيحا، بما في ذلك مسألة خضار التماسيح.

    لكن - الصالح في إيمانه سيعيش. إذا كنت لا تريد تطبيق الطريقة السقراطية في الأسئلة والأجوبة، فمن حقك أن تصدق ما تريد.

    ב

    يبدو لي أنك تخلط بين إشعاع الخلفية الكونية وأشعة تلميذ الساحر.

  155. إسرائيل:
    لقد وجدت سوء فهم وخطأ في كلامك، ولكنني أوضحت ذلك أيضًا مرات عديدة، فاعتزلت.

  156. إسرائيل:
    تنص النظرية النسبية على أنه من غير الممكن أن تكون الساعة ثابتة بالنسبة للضوء (إشعاع الخلفية الكهرومغناطيسي).
    لأن الضوء يتحرك بسرعة ثابتة بالنسبة لأي نظام قياس بما في ذلك النظام الذي توجد فيه الساعة.
    من النظرية النسبية توصلت إلى استنتاج مفاده أن هناك إشعاعًا خلفيًا.
    ولكن بعد ذلك افترضت أن هناك ساعة ثابتة بالنسبة لإشعاع الخلفية. وهذا الافتراض يتناقض مع النظرية النسبية.

  157. وعشر مرات أخبرتك أنك حر.

    ومازلت لم تجد تناقضا في كلامي.

    المعجزات - إذا كان بإمكانك مشاهدة توأمك الأكبر سناً، فافترض أنك متطابق وحتمي، فسترى كيف ستبدو في المستقبل.

    يحدث ذلك لتوأم المسافر في مفارقة، فهو يرى أن أخيه المتبقي هو نفسه المستقبلي.

    لكننا هنا نتحدث عن انقلاب المفارقة: فالتوأم المتبقي يراقب أخيه الأكبر المسافر.

    ويقال: المفارقة، المفارقة التوأم.

    كرة قدم.

  158. إسرائيل
    أشرت إلى "إذا كان لكل نظام وقته الخاص ويمكنني ملاحظة - وتصوير - DH المطابق الذي تم اكتشافه قبل يوم واحد من نظامنا، ثم على افتراض أنه نظام حتمي يمكنني ملاحظته غدًا.

    أنا أركض الآن لبناء مثل هذه المباراة وقطارًا لحملها. الضربة التي ستأتي من معرفة أسعار الأسهم غدا تجعل المشروع مربحا للغاية
    "
    أردت أن أوضح أنك لا تستطيع معرفة المستقبل...

  159. جميلة!

    لكن عمر إشعاع الخلفية هو عمر الكون، فتبين أن التوأم الأخير الذي ولد معنا أكبر من عمر الكون بمليار سنة.

    أنا الآن أخرج مع كلابي وأبي في نزهة صباحية في الجبال.

  160. لا أفهم ما علاقة هذا بإشعاع الخلفية على الإطلاق، لأنه سواء في نظام إشعاع الخلفية أو في أي نظام آخر، سنرى التوأم الأخير أكبر منا بمليار سنة، وذلك لأننا عندما نراه ، سيكون أكبر منا بمليار سنة، وذلك لأنه قد مضى عليه بالفعل مليار سنة

  161. ميخائيل.

    لقد أشرت بالفعل إلى كلامك، وقلت إن ما أبحث عنه هو الصورة المادية، وليس الصورة الرياضية.

    "بمجرد أن تقابل التوأم الأول، يتضح لك أن الأخير أكبر منه بمليار سنة" أحاول إزالة كل العناصر النفسية من المشكلة. ليس واضحا - مغلق. الصور الفوتوغرافية فقط. لم أحصل على إجابة مباشرة، لكن أفترض من الإجابة غير المباشرة (مرتين) أنها إيجابية - فالراصد سيرى أن ساعة الراصد تتأخر عن ساعة النقل بمليار سنة.

    يحدث هذا لأن المشاهد يتحرك في نظام القطار الذي تتزامن ساعاته مع بعضها البعض، ولكن ليس إذا كان خاصًا به. لكن بالنسبة له، هو الذي ينام والقطار هو الذي يتحرك، أليس كذلك؟

    ولذلك فإذا مر مثل هذا القطار براصد ساكن بالنسبة للإشعاع، أي أنت وأنا والمعجزات (الماء المتدفق فلا يكون ساكناً بالنسبة للإشعاع)، فسوف نرى - ونصور - التوأم الأخير الذي هو أكبر منا بمليار سنة.

    لكن التوأم الأخير هو في الواقع توأمنا.

    إذا كنت تقبل هذا الوصف، يرجى ذكر ذلك بوضوح.

  162. حصلت عليه، حصلت عليه.

    ما العلاقة مع ما أقوله من أننا إذا عكسنا العملية، يمكننا أن نلاحظ توأمين أكبر منا سناً من نظام الراحة لإشعاع الخلفية الكونية، أي التوائم الأقدم من عمر الكون؟

  163. إسرائيل:
    ما قلته صحيح بالنسبة لكل ساعة وقد سبق أن قدمت الحسابات التي توضح ذلك (وكما ذكرنا - يظهر مثال عليها أيضًا في الرابط الذي قدمته بنفسك والذي لا يتحدث عن ساعة إشعاع الخلفية).
    قصتك التوأم ليست حقيقية وقد شرحت السبب بالفعل.
    وشرحت أيضًا لماذا لا يكون لذلك أي صلة غير مباشرة برؤية المستقبل وأيضًا أن تصورك للتزامن برمته خاطئ.
    قلت - وأكرر - بمجرد أن تسرع، ترى أن زمنهم يتسارع كثيرًا وبالفعل عندما تقابل التوأم الأول، يتضح لك أن الأخير أكبر منه بمليار سنة ( مرة أخرى - لم أقم بالحساب بعد، ولكن هذا هو الاتجاه وترتيب الحجم).
    لقد شرحت ارتباط كل هذا بالنسبية، لكنني لا أعتقد أنك تشير إلى ما قلته.
    يمكن هنا سرد قصة أخرى - بديهية وغير دقيقة - ولكن مع إمكانية إعطاء شعور أكثر راحة تجاه الحقائق (والتحقق تجريبيًا من أنها حقائق!).
    إذا لم يساعد ذلك أيضًا، فسوف أتخلى عن محاولة إقناعك (على الرغم من أنني سأظل مصرًا على رأيي ورأي المجتمع العلمي ككل)
    ضع في اعتبارك أن كل جزء من مليون من الثانية يرسل التوأم إشارة تشير إلى مرور جزء من مليون جزء آخر من الثانية في وقتهم.
    وبقدر ما يتعلق الأمر، فإنهم يرسلون الإشارة في نفس الوقت، وينطبق الشيء نفسه على المشاهد طالما أنه لم يبدأ في التسارع.
    يلتقي بنبضات موقوتة من كل من التوأم.
    لنفترض أن هذه النبضات تبدو مثل "علامة".
    لنفترض أنه بدأ الآن في التحرك بسرعة متزايدة.
    يتباطأ وقت التوأم بالنسبة له، ولكن ليس دفعة واحدة. ولا يلاحظ التباطؤ إلا عندما تصل إليه الإشارات ويسمع "تك....تيك" بدلاً من "تيك"
    الآن - ممن سيسمع "ملف....ملف" أولاً؟
    وطبعا أقرب ساعة له.
    عندما يحدث هذا، لا تزال هناك إشارات "موقوتة" من الساعة الأخيرة في الطريق وهناك الكثير منها - في الواقع تمثل مليار سنة ضوئية تفصل بين الساعات (الإشارات التي تم إرسالها منذ مليار سنة) وهو سوف يلاحظ كل هذه الإشارات قبل أن يتمكن من ملاحظة أن الساعة الأخيرة قد تباطأت أيضًا.
    وبما أنه يتحرك نحوهم بسرعة عالية وبما أن مليار سنة ضوئية تتقلص في عالمه إلى مسافة يستطيع قطعها في يوم واحد (خاص به)، فسوف يستقبل إرسالات الساعة الأخيرة بمعدل "دقّة" على مدى مليار سنة - في ذلك اليوم وعندها فقط سيلاحظ أن المعدل انخفض إلى "علامة... ..ملف"
    حتى تلك اللحظة، سيبدو له أن وقت التوأم الأخير قد تسارع بالفعل.

  164. إلى إسرائيل ومايكل مع قصص القطارات كان من الممكن السفر على متن قطار عيمك وباختصار توأم المعجزات.
    التوأم ليسا توأمين عاديين، هما تقريبا نفس الذرة في المقال الأخير، إنسان واحد يكرر نفسه في السفر عبر الزمن، أي من خلال العودة بالزمن مرات عديدة يرتبط بذاته المزدوجة. وفيما يتعلق بنقل المعلومات، حيث أن هناك ارتباط بنقل المعلومات إلى الوراء في الزمن، والدليل على تضاعف التوأم نفسه بعد ولادته، هناك ارتباط إحصائي وبعض المعلومات في شكل جزئي، موجودة في الشخص المكرر وسوف يتصرف وفقا للخبرة التي اكتسبها هو نفسه في المستقبل، ولكن مرة أخرى إحصائيا، معلومات محدودة، معلومات جزئية وبالتالي إذا لم تفعل ذلك على نطاق واسع أو بعض التوائم - فسيكون ذلك بمثابة من الصعب عليك الفوز باليانصيب، بكل احترام

  165. إسرائيل شابيرا
    هذا التوأم الذي ابتعد وعاد ليس لديه علم بما حدث هنا على مر السنين.
    دعونا نقوم بتجربة فكرية. إرسال توأم واحد إلى الفضاء اليوم. وسيتم نشر نتائج مباريات توتو يوم السبت المقبل. تنتقل هذه المعلومات بسرعة الضوء، ويستغرق التوأم المسافر وقتًا لاستقبالها. في هذه اللحظة يعود التوأم. سؤال - هل سيعود بالمعلومة قبل السبت؟
    للتبسيط سنفعل شيئين:
    1) لا يحتاج التوأم إلى العودة على الإطلاق، فيمكنه إرسال رسالة مرة أخرى بمساعدة الضوء. بعد كل شيء، هذه هي أعلى سرعة يمكن أن تعود بها المعلومات.
    2) لذلك يمكننا أن نفترض أن التوأم يحمل مرآة فقط.
    3) بدلًا من أن يطير التوأم ممسكًا بمرآة، فإنه يحتاج إلى النوم والأكسجين والطعام ومكان للتبرز - لنفترض فقط وجود مرآة في الفضاء عند تلك النقطة X، وهي النقطة التي سيلتقي فيها التوأم بالمعلومات.

    من هنا ستعود المعلومات إلينا خلال أسبوع + مرتين X/c. يعني بعد السبت ...........

    هل تفهم ما اقول؟

  166. قرأت الردود في المستشفى من الهاتف المحمول وأتضايق لأنه من المستحيل الرد بشكل معقول من الهاتف المحمول.
    آمل أن أتمكن من الإجابة لاحقا.

  167. "أتمنى أن تكون قد فهمت وقبلت أنه عندما يكون هناك مراقب يتحرك بالنسبة لساعات الخلفية، فكلما كانت الساعة أبعد عن نقطة التزامن، كلما تم إزاحتها للأمام."

    أتمنى أنك تقصد ساعة الإشعاع وليس ساعة المراقب. أنا أقبل ذلك. تعتمد حجتي بأكملها على حقيقة أننا إذا عكسنا الخلق - أي أن نظامًا متزامنًا (قطار في مثالنا) يتحرك بالنسبة إلينا، بالنسبة للإشعاع، فوفقًا لمبدأ تمدد الزمن، سنقبل أن تظهر ساعات القطار وقتًا متأخرًا عن ساعاتنا. فإذا كانت الساعات في القطار ساعات طبيعية - التوائم على سبيل المثال - فسنحصل على توائم أكبر سناً من التوائم الملائمة نسبياً للإشعاع (عكس مفارقة التوأم).

    وهذا يعني أنه يمكن توقع المستقبل. ممكن رياضيا، وصعب الهضم جسديا.

    ولهذا السبب من المهم أن نشير فقط إلى الكاميرات التي تظهر صورة مادية. وأكرر السؤال:

    "يمكنك الحصول على قطار يبلغ طوله مليار سنة ضوئية ونحن نقف بجانبه.

    تتم مزامنة الوقت 0 في جميع ساعات القطار مع وقتنا 0.

    نحن الآن نتسارع بسرعة هائلة، بحيث نجتاز القطار بأكمله فيما هو بالنسبة إلينا يومًا ما.

    ومن المستحيل القول إن الزمن في المقطع الأخير أقدم من زمننا بمليار سنة، بحسب معطيات المشكلة.

    وعندما نصل إليه، فإن الوقت الذي سنستغرقه سيكون مليار سنة (2.8 مليار سنة على وجه الدقة) بالنسبة لنا - يوم واحد.

    مقبول؟"

    أي هل تقبلون أن تظهر الصورة من الجانبين - القطار والمراقب - أن ساعة المراقب تتأخر عن ساعة القطار بمليار سنة؟

  168. إذا لم تفهم فسأحاول أن أشرح.
    أتمنى أن تكون قد فهمت وقبلت أنه عندما يكون هناك مراقب يتحرك بالنسبة لساعات الخلفية، فكلما كانت الساعة أبعد عن نقطة التزامن، كلما تم إزاحتها للأمام.
    هذا هو استنتاج النظرية النسبية وكل الأشياء الواردة هنا حول هذا الموضوع بما في ذلك الحسابات في الرابط الذي قدمته وكذلك الحسابات التي أظهرتها.
    لذلك دعونا ننظر حقًا إلى مثل هذا النظام الذي يتحرك بالنسبة إلى خلفية الساعات الثابتة المتزامنة بشكل متبادل، ودعونا نضيف إلى القصة نظامًا بدأ من السكون بالنسبة إلى الساعات الثابتة ثم تسارع والتصق بنظام كان يتحرك من البداية وتوقفت عن التسارع.
    في الواقع، هذه أنظمة اندمجت ومن المفترض أن تكون صورتها العالمية واحدة، ولكن في حين أن النظام المتحرك باستمرار شهد زيادة تحول الساعة مع المسافة طوال الوقت، فقد بدأ النظام المتسارع من صورة عالمية "لجميع الساعات الخلفية" متزامنة" وتنتهي بصورة عالمية تكون فيها الساعات البعيدة أكثر تقدمًا.

    الطريقة الوحيدة لتفسير ذلك هي أن الساعات البعيدة تنضج بشكل أسرع (من الصفر إلى انحراف أكبر) كدالة للمسافة.

    وأقول مرة أخرى: إن إدراك وتيرة الزمن في نظام آخر يعتمد على الراصد وليس على المرصود.
    سيشعر الملاحظ بنفس الشعور طوال الوقت وسيكتشف المراقبون المختلفون إيقاعات زمنية مختلفة فيه - وهذا ما يحدث أيضًا في العلاقات الخاصة وحقيقة أنه لم يتغير شيء فيه غير ذات صلة.

  169. لا أفهم ما الذي تعنيه عبارة "المقطورة الأخيرة تصبح أقدم بشكل أسرع". السيارة الأخيرة تكون في حالة سكون دائم ولا تتأثر بما يفعله أو لا يفعله المراقب، أو يتحرك مراقب آخر في الاتجاه المعاكس وبسرعة مختلفة.

    لأنه وفقًا لهذا المنطق، إذا كان هناك بالفعل مراقب آخر يتحرك بسرعة مختلفة، فإن القطار سيتقدم في العمر بشكل أسرع أو أبطأ حتى بالنسبة لذلك المراقب الآخر. إذن، عندما تدور الكاميرات، ما الوقت الذي ستظهره ساعة السيارة، للمشاهد "أ" أو للمشاهد "ب"؟

    في فهمي، لا يوجد شيء اسمه "هذا ما يحدث في النظام المرجعي لأولئك الذين يتسارعون ولا يؤثرون على القطار". وحتى الإطالة النسبية المعتادة للوقت هي مسألة تتعلق بالراصد وليس بالمرصود." بعد كل شيء، لدينا كاميرات وهي لا لبس فيها. وفي مفارقة التوأم أيضاً هناك مراقب واحد فقط نضج، وهذا متفق عليه بين الطرفين. حتى في مثال جاك وجيل لأنظمة القصور الذاتي الذي قدمته من قبل، فإن جاك فقط هو الذي نضج.

  170. إذا كانت السيارة الأخيرة تنضج بشكل أسرع، فإن اتجاه التسارع يكون له معنى.
    إذا كان الأمر كذلك فإن التسارع في الاتجاه المعاكس يجب أن يكون له نضج سلبي.

  171. لنفترض أن المشاهد يتسارع ذهابًا وإيابًا بين السيارة A والسيارة B.
    هل سيؤثر هذا التسارع على قياس الزمن؟

  172. نعم. في رأيي أن السيارة الأخيرة تنضج بشكل أسرع (أذكرك: هذا ما يحدث في النظام المرجعي لأولئك الذين يتسارعون ولا يؤثرون على القطار. كما أن تمديد الوقت النسبي الطبيعي هو أمر للمراقب وليس للمراقب) ). وفي رأيي أنه يتوافق أيضًا مع التكافؤ بين التسارع والجاذبية.

  173. هل أصبح عمر المقطع الدعائي الأخير أسرع؟

    ففي النهاية، إذا أسرعنا، فهو لا يعلم بوجودنا على الإطلاق.

    إلا إذا انتقلت إلى المرحلة التالية حيث القطار هو الذي يتسارع، وهنا تظهر كل المشاكل التي تحدثت عنها، وأيضاً الأخبار التي لم أتحدث عنها بعد..

    عند هذه النقطة يستسلم أوكهام ويعود إلى منزله.

    إذا لم تكن قد تيأست من نفسك، فإليك تمرينًا صغيرًا في الفيزياء البديلة:

    الفكرة الكاملة وراء إطالة الزمن مشروحة بشكل جيد في الترجمة العبرية لكتاب أينشتاين (رافي مور):

    http://www.rafimoor.com/hebrew/SRH.htm

    المنطق وراء إطالة الزمن يبدو لا جدال فيه. يقدم رافي الخيارات الوحيدة المتاحة في المثال الموصوف من وجهة نظر المرأة:

    و. المرأة في وسط القطار.
    ب. حدثت كلتا الضربتين البرقيتين في وقت واحد.
    ثالث. سرعة تقدم الضوء من كلا الصاعقتين متساوية.
    رابع. يصل الضوء من الفلاش الأمامي إلى المرأة قبل أن يصل الضوء من الفلاش الخلفي.

    ويستبعد الخيار ب.

    سؤال (فقط إذا كان هناك وقت ورغبة، فهذه مجرد تكهنات):

    فهل هناك احتمال آخر لا يظهر هناك ويترك الزمن المطلق لنيوتن سليما وكذلك مسلمتي النسبية؟

    احتمال يتناسب جيدًا مع ما نعرفه من ميكانيكا الكم والذي حاربه أينشتاين طوال حياته (وخسره)؟

    احتمال يفسر أيضًا تجربة ويلر (التأثير على الماضي من المستقبل) دون إغماء أوكهام؟

    https://www.hayadan.org.il/quantun-philospy-part-b-07121/

    (الماء هو المكان والوقت المناسبين للزفير والشهيق، احتفظ به لبضع ثوان ثم أطلقه).

    مساء الخير.

  174. في رأيي يمكن القول أن السيارة الأخيرة تنضج بشكل أسرع خلال فترة التسارع.
    صيغ النسبية العامة ليست مألوفة بالنسبة لي، لكن إذا قمت باستقراءها بسذاجة من النسبية الخاصة، فمن المؤكد أنها كذلك - مرة أخرى - بسبب X المعروفة

  175. مقال عوفر:

    http://ofer-megged.blogspot.com/2011/09/blog-post.html

    بدلاً من مجرة ​​درب التبانة، دعونا نحصل على قطار يبلغ طوله مليار سنة ضوئية ونحن نقف بجانبه.

    تتم مزامنة الوقت 0 في جميع ساعات القطار مع وقتنا 0.

    نحن الآن نتسارع بسرعة هائلة، بحيث نجتاز القطار بأكمله فيما هو بالنسبة إلينا يومًا ما.

    ومن المستحيل القول إن الزمن في المقطع الأخير أقدم من زمننا بمليار سنة، بحسب معطيات المشكلة.

    وعندما نصل إليه، فإن الوقت الذي سنستغرقه سيكون مليار سنة (2.8 مليار سنة على وجه الدقة) بالنسبة لنا - يوم واحد.

    مقبول؟

  176. لم أقرأ مقال عوفر ولم أدقق في الأرقام، لكن من حيث المبدأ لا أرى مشكلة في الإجابة الإيجابية على السؤالين.
    ما أعتقد أنك تنساه هو حقيقة أنه عندما غادر، كان الكوكب الأخير، بالنسبة له، أقدم بكثير من الكوكب الأول (مرة أخرى - لم أقم بالحسابات، لكنني أقدر أن هذا أيضًا يبلغ حوالي مليار سنة)

  177. 1. هل تقبل أن مسافر عوفر يسافر مليار سنة ضوئية بينما هو يوم واحد فقط؟

    2. إذا صور الكواكب على سطحها، فإنه يمر ويفترض أن كل واحد منها لديه تطور كما في العهد الجديد، فهل تقبل أن التطور في الكوكب الأخير يتقدم مليار سنة أكثر من الأول، و وسوف يظهر هذا من خلال الصورة؟

  178. من الغريب حقًا أن تقول ذلك وليس بسبب الفكرة الغريبة عن الأرض المتوافقة.
    إذا كان هذا هو مستوى الاستبصار الذي تتحدث عنه، فما عليك سوى مراقبة الساعة وإلقاء نظرة على الوقت الذي ستظهر فيه خلال 24 ساعة.

    وربما يكون هناك أيضاً افتقار إلى المنطق في التفكير في وجود مثل هذا التطابق لأنه إذا تطابق إلى النهاية فإن له أيضاً تطابقاً أكبر منه بيوم وللأرض تطابق أصغر منه بيوم يوم وهكذا إلى ما لا نهاية (مرتين ما لا نهاية)

  179. إذا كان لكل نظام وقته الخاص، ويمكنني أن أشاهد - وأصور - مباراة تم اكتشافها أقدم من مباراة لدينا بيوم واحد، فافترض أنها نظام حتمي يمكنني مشاهدته غدًا.

    أنا أركض الآن لبناء مثل هذه المباراة وقطارًا لحملها. الضربة التي ستأتي من معرفة أسعار الأسهم غدا تجعل المشروع مربحا للغاية.

  180. إسرائيل:
    بادئ ذي بدء، يجب التأكيد على التماثل: نقطة التزامن في نظام إشعاع الخلفية أيضًا "تختبر" نفس الشيء تمامًا وتلتقي بنظام محرك الساعة "البالغ" الخاص بها.
    على العموم، لا أفهم ما الذي يزعجك، ولا أرى أن لهذه الظواهر أي علاقة بمراقبة المستقبل. كل نظام له وقته. هذا كل شيء. أنت لا تراقب مستقبلك، بل تراقب حاضرك. دائما.
    صحيح أن حاضرك يتضمن بيانات هي نتيجة لماضيك (مثل أشعة الضوء القادمة من النجوم التي ماتت منذ فترة طويلة) لكنه لا يحتوي (على الأقل حتى يثبت أهارونوف خلاف ذلك) أي معلومات تأتي من مستقبلك .

  181. ومن المثير للاهتمام حقا، مينكوفسكي، سختين حول الاستثمار.

    هل تعرف قصة فون نيومان والذبابة؟

    فسألوه السؤال التالي: يسير قطاران متقابلان على السكة، أحدهما يتجه شرقًا بسرعة 20 كم/ساعة والآخر يتجه غربًا بسرعة 40 كم/ساعة. وعندما كانت المسافة بينهما 120 km، غادرت ذبابة مقدمة القطار A باتجاه القطار B. سرعة الطيران 100 كم/ساعة. وعندما يصل إلى القطار (ب) يستدير ويعود إلى (أ) ويعود لا سمح الله حتى النهاية الحتمية (والنهاية).

    إلى أي مدى ستسافر الذبابة؟

    يبتسم فون نيومان ويجيب على الفور: 200 كيلومتر.

    يقولون له أحسنت يا عالم الفيزياء. لقد لاحظت على الفور أن القطارات تعمل لمدة ساعتين، أليس كذلك؟

    شو، الفيزيائية أونا فون، قمت بحساب المتسلسلة اللانهائية...

    على أي حال، كما أوضحت، ستظهر الصورة دائمًا نفس الموقف في الساعات. لقد أحضرت أيضًا صيغة الوقت "t" بعد "باختصار" منذ 10 سنوات في العدد 65 من غاليليو ردًا على سؤال حول عكس الإنتروبيا والعودة بالزمن إلى الوراء، دون أن أتخيل مدى أهميتها.

    لأنه كما ترون من الصيغ، يتفق الطرفان على أن الساعة المتحركة تظهر وقتًا أقدم من الساعات المتزامنة في النظام الذي تتحرك فيه.

    ولا إشكال في ذلك إذا كان توقيت الساعات اعتباطيا. لكن إذا كان الوقت الطبيعي -عمر الكون- والذي ترتبط به الأنظمة البيولوجية (التوائم) بالساعات، يتبين أن المشاهد يتنبأ بما يراه المستقبل.

    لا توجد مشكلة في ذلك في عام 1905 أيضًا. كل توأم يذهب في رحلة يعرف أنه يطير نحو المستقبل. سافرت ساعة ووصلت إلى العالم خلال مليون سنة.

    يرى مسافر عوفر أيضًا كونًا مستقبليًا. وبعد مرور 4 أيام حسب ساعته، رأى كونًا قد نضج خلال 15 مليار سنة.

    يمكن لكل من التوأم والمسافر أن يدعيا في عام 1905 أن وقتهما حقيقي مثل الأنظمة التي سبقتهما.

    الوضع مختلف اليوم. لا يستطيع التوأم المسافر أن يدعي أنه عندما يشرب الشاي مع أخيه الأكبر، فإن وقتهما حقيقي بنفس القدر. لأنه إذا كان هذا صحيحا، لماذا تعمل التدفئة؟

    وعلى نفس المنوال، إذا سافرنا مليار سنة ضوئية في يوم واحد وفقًا لساعاتنا، فإننا نرى كواكب أكثر برودة.

    من المنطقي أن تكون الكواكب أدنى نسبيًا من الإشعاع وأننا نفضلها بالنسبة له.

    ومن الغريب جدًا أن ينقلب الوضع. خاصة إذا راقبناهم لمدة أسبوع حسب ساعتنا، وهم بالفعل يبلغون من العمر 30 مليار سنة حسب ساعتهم، وهي أيضا الساعة البيولوجية للأنظمة الموجودة فيهم، والتي نلاحظها من موقعنا الذي يبلغ عمره 13.7 مليار سنة.

    إذا كان هناك أخطاء وأخطاء - iPhone.

    مساء الخير.

  182. إسرائيل:
    لا أعرف ما هي الأنظمة الأقدم من أنظمةنا.
    ما هي الساعة التي يتم قياس عمر النظام؟
    إذا كانت الساعة هي ساعة إشعاع الخلفية، فإن نظام الدفع بالنسبة لإشعاع الخلفية لا يستخدم الساعة لقياس عمر الكون لأنه، كما ذكرت، فهو في الواقع ينسخ ساعتنا وإذا تزامن الساعة مع ساعتنا في لحظة معينة، فمن تلك اللحظة ستظهر الساعة "عمر الكون" وهو أصغر من العمر الذي تظهره ساعة إشعاع الخلفية عند النقطة التي تمر فيها.
    وبنفس الطريقة - إذا كانت الساعات المتزامنة متناثرة فيما بينها في هذا النظام، فإن نقطة الخلفية التي حدث فيها التزامن سترى في نظام الدفع، من لحظة التزامن فصاعدًا، ساعات تظهر وقتًا أعلى من إشعاع الخلفية الساعة، لكنها ليست ساعة تقيس عمر الكون، بل مجرد ساعة (والتي، على حد فهمها، تم ضبطها منذ البداية لإظهار وقت أعلى من عمر الكون).
    قد تكون مهتما بالقراءة هو - هي

  183. بالنسبة لي، أظهرت المناقشة أن ما يتبقى هو ما إذا كان علينا أن نقبل أن مثل هذا النظام الذي لدينا، والذي يكاد يكون ثابتًا بالنسبة إلى إشعاع الخلفية، يمكن توقعه في أنظمة أقدم من نظامنا. لا أرى كيف يكون ذلك ممكنًا، لكنها مسألة وجهة نظر على ما أعتقد.

    على أية حال، شكرا للاستثمار. لقد ساعدني كثيرًا معرفة أن أحد علماء الرياضيات قد درس هذه الفكرة ولم يجد خطأً بسيطًا مثل استبدال علامة زائد بعلامة ناقص.

    وللرجل الذي اقتبسته سابقًا، والذي ادعى أنه لا يوجد تمدد زمني بسبب تناقض مدمج في العلاقات نفسها، بينت أن افتراضاته تؤدي إلى تناقض مع ما نعرفه عن طبيعة سرعة الضوء.

    وبما أنه ادعى أن الخطأ مني، وجهت السؤال إلى عوفر ماجد:

    http://ofer-megged-phys-notes.blogspot.com/2011/04/blog-post.html

    وقال عوفر إنه كان على حق معي، الأمر الذي لم يقنع الرجل. وهذا جعلني أتساءل قليلاً، لأن هذين الأكاديميين كانا يتعاملان مع موضوع النسبية لسنوات عديدة. نفس الشخص، الذي يبدو جدياً جداً، يحاضر أيضاً في المؤتمرات ويدعي أن أستاذ الفيزياء في جامعة تل أبيب يتفق معه.

    يبدو أن كل شيء نسبي حقًا.

  184. إسرائيل:
    أنا آسف لأنني أعتقد أنني قد أجبت بالفعل على كل شيء.
    لقد قدمت القصة مع التوأم دون ذكر إشعاع الخلفية على الإطلاق، لكن ذلك لم يعد مهما.
    لقد جعلني هذا النقاش أفكر في أشياء كثيرة وأفهمها بشكل أفضل، لكن الأمر بالنسبة لي مرهق بالفعل.
    كل الأسئلة التي طرحتها علي حصلت على إجابة مرضية للغاية بالنسبة لي وهذا يكفيني.

  185. لم أزعم أبدًا أنني أحاول دحض النسبية. لقد زعمت أنه في فهمي، فإن إطالة الزمن في العلاقات يتناقض مع نظرية الانفجار الكبير، والتي منها يشتق العمر المطلق للكون. ولم أعبر عن تفضيلي بطريقة أو بأخرى. لقد قلت، وأقول مرة أخرى، أنه من شبه المؤكد أن عدم الفهم - أو على الأرجح عدم المعرفة الكافية - يقع علي.

    وللأسف فإن الأسباب التي طرحتها لا تفسر التناقض المزعوم.

    "إذا كان التوأم قريبين (العربات قصيرة) فلن يرى فرقًا في عمرهما."

    ألا يستطيع مسافر عوفر الذي يمر بمجرة درب التبانة في ثلاث ثوان أن يرى ويصور الكواكب التي يمر بها؟

    وأليس الكوكب الأخير أقدم من الكوكب الأول بمئة ألف سنة؟ ماذا لو بعثرت توائم متزامنة في الكواكب، ألن يكون التوأم الأخير أكبر من الأول بمئة ألف سنة؟

    النسبية، بالمناسبة، تدعي أنني أعتقد.

    هذه ليست المشكلة. حتى مع أينشتاين، يقوم التوأم المتحرك برحلة إلى المستقبل، ويرى مسافر عوفر الكواكب تشيخ بسرعة.

    لكن بالنسبة لأينشتاين في عام 1905 - في الواقع حتى اللقاء مع هابل - لا توجد مشكلة في هذا، لأن الزمن لا نهائي.

    وتنشأ المشكلة عند الانطلاق من افتراض أن كوننا له عمر محدود، حوالي 13.7 مليار سنة.

    لأننا لو عكسنا نظام الراحة الخاص بالمسافر وقارناه بنظام الإشعاع الباقي (أي عندنا جميعاً)، فسوف يرى توأماً - أو كواكب أو أياً كان - عمره أعلى من عمره، وهذا من نظام عمره هو بالفعل عمر الكون.

    وفي كل السيناريوهات المعروفة الوضع معاكس: فالتوأم الأصغر يعود عندما يكون عمره أقل من عمر أخيه، وهو عمر الكون. في نظام تحديد المواقع العالمي (GPS)، تتأخر الساعة عن الساعات الثابتة، وليس العكس. يظل مسافر عوفر صغيرًا مع تقدم عمر مجرة ​​درب التبانة، لكنه لا يستطيع أبدًا رؤية كوكب أكبر من عمر الكون.

    دائمًا ما يكون وقت الركاب على اختلاف أنواعهم أقل من وقت الركاب الثابتين.

    أما إذا كان الراصد هو الذي يرتاح بالنسبة للإشعاع فإن عمره دائما هو عمر الكون، وفي كل قطار يسافر بالنسبة إليه تظهر الساعات وقتا متأخرا ولاحقا، أي أعلى من عمره. أي أعلى من عمر الكون.

    انقلاب مفارقة التوأم: يعود التوأم المسافر عندما يكبر.

    وهذا كل ما يغضب أوكهام منه.

    تحياتي للأب.

  186. وبالمناسبة: حتى لو كنت تتخيل عالمًا مستقبليًا حيث يمكنك أخذ البويضات المخصبة وتطير بها إلى نقاط مختلفة في راحة وتزامن متبادل حيث سيدخل البيض إلى الحاضنة وسيولد الأطفال في نفس اللحظة في نقاط مختلفة، لن تكون هناك مشكلة لأنه فيما يتعلق بالنظام المتحرك فيما يتعلق بهذه النقاط - لم يولد الأطفال على الإطلاق في نفس الوقت.

  187. بالمناسبة - أثناء جلوسي بجوار سرير والدي اليوم، أمضيت بضع دقائق من الصمت وتمكنت من حساب النسبة بين الساعة الطبيعية لنقطة ما والساعة التي تم الحصول عليها عن طريق النسخ الدائم للساعات المحلية لنظام ثابت متزامن بشكل متبادل الساعات التي تمر به.
    وصف المشكلة معقد في حد ذاته ولا يمكن كتابة الصيغ هنا (ويبدو لي أيضًا أنه لا أحد مهتم بها هنا).
    إذا كان هناك طلب من الجمهور سأحاول كتابة ملف يوضح الحساب والنتيجة.

  188. بشكل عام - إذا كان التوأم متقاربين (العربات قصيرة) فلن يرى فرقاً في عمرهما إلا بالكاد، وإذا كانا بعيدين فلا بد من مراعاة العملية التي خلقت المسافة بينهما (هذه العملية ستلغي حتماً) هويتهم كتوائم).
    لا يبدو لي أنك تمكنت من تقويض النسبية الخاصة على الإطلاق.
    في البداية اعتقدت أن محاولة الحديث عن نقطة تمر عبر نقاط مختلفة في نظام آخر كان المقصود منها الإشارة لي إلى وجود تناقض مع ما نتوقعه من ساعة درجة الحرارة (وبالفعل - طالما لم أفهم بشكل صحيح معنى هذه الساعة - يبدو أن هناك مثل هذا التناقض - تناقض قمت بإزالته عندما وجدت المعنى صحة هذه الساعة) ولكن الآن يبدو لي أنك تحاول العثور (دون جدوى) على تناقض داخلي داخل نظرية النسبية.

  189. حتى عندما تقوم بتصوير توأم متطابق، فإنك في الواقع تتنبأ بتطور النمط الظاهري من النمط الجيني مع عدم اليقين

  190. عندما يتم فك رموز النمط الظاهري من النمط الجيني، يمكن تجميع صورة مستقبلية لتطور الكائن الحي.

    لكن هذه الصورة غير مؤكدة. من المؤكد أن الأخطاء في الحساب، والتأثيرات البيئية، وأكثر من ذلك، تؤثر على النمط الظاهري.

    وهذا يختلف عن عرض (وتصوير) صورة مستقبلية. هناك وصف لا لبس فيه للواقع.

    لقد ضربت مثال راكب عوفر وهو يطير بسرعة فوق قطار محمل بالتوائم. سيرى في كل عربة توأمًا أكبر سنًا، بينما يظل صغيرًا.

    وطرحت سؤالاً: ماذا لو كان الراكب أحد التوأمين اللذين يجلسان في محطة قطار الشمال في تل أبيب ويشاهدان القطار التوأم يمر؟ حتى ذلك الحين هل سيرى التوائم الأكبر سنًا (نفسه) في السيارات التالية؟ وماذا عن كل كرونة هي في الواقع الأرض ويراقبها لمدة 5 أيام حسب ساعته، فهل سيرى الأرض في عمر 25 مليار سنة؟

    لأنه في مثال مسافر عوفر، هذا ما يخرج.

  191. في رأيي، لا توجد هنا ظاهرة التطلع إلى المستقبل.
    يتعلق الأمر بمراقبة شخص آخر بالطريقة الوحيدة التي يمكنك من خلالها مشاهدته أثناء وجودك هناك.
    هذا هو الحاضر.
    هل تعتقد أن القدرة على فك النمط الظاهري من النمط الجيني من شأنه أن يزعج أوكهام أيضًا؟
    ليس من الواضح بالنسبة لي على الإطلاق كيف ينتمي إلى هذه المسألة.

  192. في مفارقة التوأم الأصلية، إذا التقطا صورة لبعضهما البعض أثناء لم شملهما، فإن الصورة التي التقطها الشاب توضح كيف سيبدو في المستقبل (بافتراض أنهما توأمان متطابقان وحتميان).

    وبقدر ما كان الأمر يتعلق بالشاب، فقد قام بالفعل برحلة إلى المستقبل. سافرت لمدة ساعة، ووصلت إلى عالم المستقبل: لا توجد أنهار جليدية والجميع يرتدون قبعات.

    وجهة نظري هي أن هذا ممكن فقط لأنه في التسارع، وكذلك في الحركة الثابتة ولكن ضد الإشعاع، تدق الساعة بشكل أبطأ. وكذلك في الجاذبية، مثل التسارع.

    ولهذا السبب تكون العملية من جانب واحد: سيرى راكب عوفر مجرة ​​درب التبانة وهي تمر أمامه لمدة ثلاث ثوان فقط. إذا كان المسار مليئًا بأعواد الثقاب الأرضية، فسوف يرى التطور أمام عينيه المتعجبتين، مع كل عود ثقاب تصبح القرود منتصبة أكثر فأكثر.

    ومن ناحية أخرى، إذا مرت أمامنا مجرة ​​متوافقة مع درب التبانة، بالنسبة للإشعاع، فإنها لن تنضج بسرعة، بل على العكس: بعد مائة ألف سنة معنا، بالكاد تتقدم المجرات المتوافقة مع الأرض. في الدورة السنوية للشمس.

    (وهذا بالطبع إذا لم يتبخروا على الفور، كما سيتبخر راكب عوفر على الفور، بسبب الاحتكاك مع الإشعاع).

    نظام تحديد المواقع العالمي (GPS)، والطائرات التي تدور حولها، والساعات في مجال الجاذبية القوية، وجميع الأنظمة المتسارعة. الساعة تتباطأ.

    تتحرك المؤينات طويلة الأمد ضد الإشعاع. نفس ما ورد أعلاه.

    لكن العملية لا يمكن أن تحدث في الاتجاه المعاكس. هذه هي تخميناتي التي لا أساس لها من الصحة، والتي لا تعتمد إلا على إيماءات أوكهام النشطة التي تتسرب إلى المستقبل، أي في أنظمة أقدم من عمر الكون.

    الهاتف، خاصة وأنني أكتب من الآيباد.

  193. إسرائيل:
    هذه ليست صورة للمستقبل.
    وهذه صورتهم في عصرنا.
    هل توافق على الادعاء القائل بأن الوقت يتغير في الأنظمة المتسارعة وأنه يتم التعبير عن ذلك حتى في نظام تحديد المواقع العالمي (GPS)، لذلك يغضب أوكهام هناك أيضًا؟

  194. ميخائيل

    من الناحية النظرية، من الممكن أن تكون حياة الكون أطول في الأنظمة التي تتحرك بالنسبة إلى إشعاع الخلفية. ولكن بعد ذلك، إذا أخذنا صورة لمثل هذا النظام من نظامنا، مع خصم كامل تقريبًا بالنسبة لإشعاع الخلفية، فإذا كان عمر النظام أعلى بكثير من عمرنا (لن نصاب بالجنون، فقط 50 مليار سنة) نحن نلتقط صورة للمستقبل.

    لا يعني ذلك أن الأمر مستحيل - فقد تم حل مفارقة APR بطريقة غير متوقعة (غير محلية) - لكن أوكهام أظهر علامات الانزعاج.

    ولا تخافوا من البلاشفة. وهم أكبر الملحدين، وفي الأمر.

  195. هناك اتجاه حسابي آخر يجب التحقق منه - اتجاه يتحقق من العلاقة بين ساعتي والساعة التي أقوم بمزامنتها باستمرار مع ساعة نظام قصوري آخر يمر بها ولكن ليس لدي الوقت حاليًا للقيام بذلك.
    ربما سأفعل ذلك عندما وضعني البلاشفة في السجن بتهمة إيذاء المشاعر الدينية.

  196. إسرائيل شابيرا:
    لا بد لي من الركض، ولكن فجأة فكرت جديًا في الفكرة التالية (التي خطرت ببالي بالفعل وكانت ترفرف لسبب ما من قبل):
    إن استخدام ساعة درجة الحرارة في نظام متحرك فيما يتعلق بإشعاع الخلفية ليس أكثر من مجرد نسخ مستمر لساعة إشعاع الخلفية المحلية (والتي من المعروف أنها غير متزامنة في نظام متحرك مع ساعة إشعاع الخلفية المنسوخة السابقة أو ساعة إشعاع الخلفية المحلية) التالي ليتم نسخها).
    وبعبارة أخرى - إنها ليست ساعة لنظام القيادة.
    وبالتالي، فإنها ستظهر أيضًا نفس النتائج التي تظهرها ساعة إشعاع الخلفية المصورة.
    إذا كان هذا هو التفسير الصحيح للموقف، فإن السؤال الذي يطرح نفسه هو ما إذا كانت هذه الساعة تمثل شيئًا ما في نظام القيادة (فيما يتعلق بالخلفية).
    الاتجاه الطبيعي هو ضرب هذا الزمن بالحجم المميز للانتقال الزمني بين الأنظمة للحصول على عمر الكون في نظام متحرك، ولكن بعد ذلك هذا يعني أن حياة الكون في أي نظام متحرك نسبة إلى الخلفية فعليا يبدو أطول وهذا يؤدي إلى مشكلة أنه إذا اتخذ نظام إشعاع الخلفية نفس العملية نحو نظام متحرك، فسوف تحصل على عمر كون أكبر، وهكذا إلى ما لا نهاية.
    ربما يكمن الحل المحتمل لهذه المشكلة في الإشارة إلى لحظة الانفجار وموقعه. لدي بعض الأفكار حول هذا الموضوع ولكن ليس لدي الوقت الآن لتطويرها ومعرفة إلى أين ستؤدي

  197. مايكل قبل أن أذهب للنوم (بسرعة، هاه؟)

    ما يزعجني هو انقلاب مفارقة التوأم.

    في المفارقة الأصلية، في عام 1905، عندما التقى التوأمان، استطاع كل توأم أن يقول أن وقتهما هو الوقت الحقيقي، حيث أن الزمن ليس له بداية.

    فعندما تكون هناك نظرية الانفجار الأعظم، وإذا التقيا بعد مليار سنة، فلن يتمكن الشاب من الادعاء بأن زمنه هو الزمن الحقيقي. وهناك درجة الحرارة التي تبين له أن الكون قد تقدم في السن.

    إذا كان قد بدأ رحلته من نظام يتحرك بسرعة عالية بالنسبة للإشعاع بدلاً من العكس، فإن معظم وقت الرحلة وفقًا لساعته كان سيقضيه بسرعة 0 بالنسبة للإشعاع، في حين أن عمره الأكبر سنًا لكان الأخ قد ظل في حالة حركة بالنسبة للإشعاع. وهنا المفارقة، بالتحديد التوأم المتحرك يكبر، في حين يبقى أخوه الذي يقضي وقته بيننا صغيرا.

    إن ما يعادل مفارقة التوأم في أنظمة القصور الذاتي هو قطارات مليئة بالتوائم. التوائم هي أنظمة حتمية ذات عمر واضح 0: الولادة.

    فإذا مر فوتونك المسرع أو راكب عوفر الرشيق بقطار طويل مليء بالتوائم، سيرى توأمًا صغيرًا في العربات الأولى، وكلما تقدم للأمام سيرى توأمًا أكبر سنًا فأكبر، بينما هو نفسه لا يكاد يكبر.

    ولكن ماذا عن الراكب الرشيق الذي يكون هو نفسه أحد التوأمين ساكنًا نسبيًا بالنسبة للإشعاع، أي يجلس في محطة قطار الشمال على أحد المقاعد وينظر إلى مثل هذا القطار التوأم الذي يمر بسرعة هائلة؟

    وحتى ذلك الحين سوف يرى نفس الشيء، إلا أنه بهذه الطريقة سيكون قادرًا على رؤية نفسه وهو يكبر (التوائم المتطابقة الحقيقية حتمية تمامًا)، وأبنائه، وأحفاده، ومستقبله بأكمله. كل هذا دون النهوض من مقاعد البدلاء.

    وهذا منطقي عندما نتحرك بسرعة نسبة إلى قطار ثابت بالنسبة للإشعاع، لأن وقتنا (ساعة السيزيوم) لا يعكس عمر الكون (ساعة درجة الحرارة)، وهو دائمًا أقل منه. ليس من المنطقي أن نكون في حالة سكون بالنسبة للإشعاع، وزمننا هو نفس توقيت الكون، في حين أن الساعات الحية التي تمر بسرعة أمامنا - التوائم - تظهر توقيتا أعلى من توقيتنا، وهذا هو، من الكون.

    مساء الخير.

  198. إسرائيل:
    أولاً، أفهم ما الذي يزعجك.
    على الرغم من أنني لا أقبل الجدال حول الساعة التي توقيتها 0 لأنها أكثر من أي ساعة أخرى هي وقت اعتباطي، لكني أفهم ما يزعجني عندما أمر بساعة تزعجني بمناقشة حساب عمر الكون وأخبره أن نتيجته غير صحيحة رغم أنه يريني كيف يحسب ولا أجد خطأ في ذلك.
    أتمنى أن أجد الوقت للتفكير في الأمر من اتجاهات أخرى لأنه من الممكن أن التعامل مع ساعة درجة الحرارة على أنها ساعة فكرة خاطئة لأنها تعمل فقط على الإحصائيات والأحداث المختلفة قد تغير قراءاتها وتؤدي أيضًا إلى نتائج خاطئة.
    هناك اتجاهات أخرى مماثلة تستحق الخوض فيها، ولكن كما قلنا، ربما ليس هذا هو الوقت المناسب

  199. ميخائيل.

    يجب أن أركض إلى العمل، لكن هذه هي المشكلة التي أجدها في حجتك:

    تكتب: "كل ما يلزم لذلك هو توجيهه من البداية إلى زمن أعلى من عمر الكون".

    لقد تناولت هذا الادعاء بالفعل عندما كتبت: "يمكن القول بأن المزامنة مصطنعة ولا تعكس الواقع".

    لرؤية الطبيعة الإشكالية لهذه الحجة، أحضرت مقالة عوفر ماجد، حيث تظهر الساعات الأولى التي تمر أمام المشاهد الحد الأدنى من الوقت الطبيعي الممكن في النظام المادي: 0. (أنت لا تدعي أن هناك وقت طبيعي الساعة في النظام يمكنها إظهار الوقت أقل من 0، أليس كذلك؟)

    وحتى ذلك الحين، بعد 4 أيام حسب ساعة المراقب، سيكون عمر النظام العابر أعلى بكثير من عمر الكون.

    يمكنك الاطلاع على مثال جاك جيل شابلينك بالتفصيل، لتعرف أنك إذا بدأت بنظام ساعات متزامنة وقتها هو عمر الكون، فيمكنك مراقبتها من نظام آخر بالنسبة للإشعاع، ومن أي الساعات التي الوقت أعلى من عمر الكون سيتم مراعاته.

    لكن الأمر يتطلب الكثير من العمل والتفاصيل، لذا دعنا نتركك الآن لتعتني بوالدك (إلا إذا كنت تريد مني أن أحضر التفاصيل. إنه أمر متعب للغاية، حاول).

  200. إسرائيل:
    لسبب ما، أنت تعتمد على ألوهية نظام الإشعاع الخلفي وليس هناك أي مبرر لذلك.
    بادئ ذي بدء، من السهل حقًا اكتشاف الحركة المتعلقة بها ولا تحتاج إلى مراقبة درجة الحرارة لذلك.
    كل ما هو مطلوب هو قياس الإشعاع في كل اتجاه وتحديد عدم وجود النظائر.
    وهذا أمر واضح للجميع ويشكل أيضًا الأساس لإنشاء ساعة درجة حرارة في نظام الدفع فيما يتعلق بإشعاع الخلفية.
    فماذا لو كان من الممكن اكتشاف الحركة بالنسبة لنظام إشعاع الخلفية؟
    ومن الممكن أيضًا اكتشاف الحركة فيما يتعلق بنظام الأرض!
    وهذا لا يجعل نظاما أو آخر متفوقا.
    وهذا لا يعني أنه لا يجوز لك الرجوع إليه، لكن ليس له أي تأثير على النظرية النسبية.
    لا توجد مشكلة في مشاهدة ساعة توضح الوقت المستقبلي (بالنسبة لعمر الكون).
    وكل ما يلزم لذلك هو توجيهه من البداية إلى زمن أعلى من عمر الكون.
    قد يبدو الأمر بمثابة مزحة بالنسبة لك، لكنه ليس كذلك.
    ففي نهاية المطاف، ما هي ظاهرة البدلة الصدفية المبنية على الساعات التي تظهر المستقبل رغم أن الزمن في الصدفة أسرع؟
    وهو يعتمد على حقيقة أنه من وجهة نظر الصدفة - كلما كانت الساعة أبعد عن النقطة التي كانت فيها صفراً وعلى الأرض، كلما زاد الوقت الموجود فيها في البداية.
    وهذه هي بالضبط نفس الظاهرة التي أشرت إليها في وصفي لتخميني غير المحسوب بشأن ما كان يحدث.
    في تخميني، كان التقدم مقدمًا كافيًا لتغطية التقدم الأبطأ للساعة، وفي الحساب الصحيح، يكون التقدم أكبر مما هو مطلوب لتغطية التقدم الأبطأ للساعة، ولكن في كلتا الحالتين، الساعات الموجودة بعيدًا بما يكفي تظهر أوقاتًا أكبر بكثير من عمر الكون في البداية.
    ويذكر بالنكتة التالية:
    سؤال: ما هي أسهل طريقة لتكوين ثروة صغيرة؟
    الجواب: ابدأ بواحدة كبيرة

    وأما والدي :
    لا يزال الوضع صعباً ولكن هناك تحسناً (مؤقتاً على الأقل).
    أنا لا أكشف التفاصيل من باب التواضع الشخصي، ولكن إذا انفصلت للحظة عن مشاعري، فلا بد أن أذكر أنني تلقيت مؤخرًا درسًا عمليًا حول جوهر وقابلية ما نسميه "أنا" - الأشياء التي أعشقها. لقد تناولت بالفعل في مقالاتي "ما هي الروح".

  201. على الأقل مرة واحدة فزت.
    فيما يتعلق بالنسبية - هناك مليون نسخة وفي رأيي حتى النسخة "الصحيحة" لا تعمل حتى النهاية ويجب عليك الذهاب إلى أكوان متوازية وبرمجة للعودة بالزمن إلى الوراء لتسوية الزوايا، بشكل أساسي إذا كان هناك شخص خلفك أنت في الوقت المناسب فأنت تتقدم عليه - وهذا يعطي صفعة منطقية في وجه الأنظمة المتماثلة التي يتم حلها مرة أخرى في أكوان متوازية والعودة في الوقت المناسب. وليس هناك نهاية للنقاش، وشكرا لكم 

  202. أردت أن أشير إلى شيء يتعلق بالواقع بالنسبة للمكان على مستوى التخرج، ولكن ربما أكون مخطئًا في هذه المسألة تحديدًا
    . لنأخذ تصادمًا مرنًا بين كرتين، ويبدو أنه وفقًا للصيغ إذا قمت بتحريكهما، فإن نفس التصادم في الحركة بالنسبة لحركتهما (المراقب فقط) يبدو أن المعادلات تظهر طاقات أخرى وهناك احتمال بالفعل هناك، في تصادم مرن بين كرتين، الحل المحتمل هو في أكوان متوازية، (عليك أن تلعب بالمعادلات لفهم ما أتحدث عنه، والمراجع مثل كلمة العددية والطاقة لن ترضيني بالرغم من ذلك) أنا راضٍ تمامًا ومن الصعب علي مناقشة الموضوع الآن) 

  203. ميخائيل

    دعونا نأمل أن كل شيء يسير على ما يرام. الآن كل ما تبقى هو أن نأمل أن تنتهي جراحة مارنان بنجاح أيضًا.

    المشكلة هي كما يلي: إذا نظرت إلى الرابط، سترى أن راكبًا (في حالتنا جيل، متصل بالساعة ('C) يتحرك في نظام قصور ذاتي متزامن (الساعتان C1 وC2) يراها (ويلتقط صورًا لها) الساعات تظهر وقتا أعلى من وقته.

    ومن وجهة نظر ذلك الراكب فهو المستقر، والساعات - ولو الألف - التي تمر به تظهر وقتاً أعلى فأعلى.

    إذا كان هذا المسافر موجودًا في نظام الإشعاع وهو في حالة سكون بالنسبة إليه، وكانت الساعة الأولى التي تمر بجانبه مضبوطة لتظهر نفس الوقت (عمر الكون)، فإن الساعات التي تليها ستظهر وقتًا أعلى من العمر الكون.

    ويمكن القول بأن التزامن مصطنع ولا يعكس الواقع.

    يمكن تفنيد هذا الادعاء نوعيا إذا نظرنا إلى مقال عوفر مجاد "درب التبانة في ثلاث ثوان".

    http://ofer-megged.blogspot.com/2011/09/blog-post.html

    إذا كان المسافر الموصوف هناك ثابتا بالنسبة للإشعاع (أي أنت وأنا وكلنا)، ففي 4 أيام حسب ساعته النظام الذي يمر أمامه يتجاوز عمر الكون كثيرا، فحتى إذا كانت الساعة الأولى فيه تظهر ببساطة 0، فإن الساعة الأخيرة كانت موجودة بالفعل في المستقبل.

    ومن الممكن أيضًا إظهار ذلك كميًا إذا أخذنا المثال في رابط إطالة الوقت (جاك وجيل)، والتزمنا بالساعات العادية ذات الساعات المؤقتة.

    إذا افترضنا أن Geek في حالة سكون بالنسبة للإشعاع، فإذا قام بمعايرة ساعة السيزيوم الخاصة به مع وقت الساعة المؤقتة، فإن أي لقطة مستقبلية ستظهر نفس الوقت في كليهما.

    مع جيل، من ناحية أخرى، سوف تنخفض ساعة PM أكثر فأكثر عن الساعة المؤقتة.

    أول شيء حصلنا عليه هو تناقض الافتراض رقم 1: كل من جاك وجيل موجودان في نظام قصوري، لكنهما في الواقع في حالة سكون. ولكن في حين أنه لا يوجد فرق بين الساعات بالنسبة لـ Geek، فإن هناك فجوة بينهما بالنسبة لجيل. وهذا على النقيض من المسلمة 1 التي تنص على:

    "إن قوانين الفيزياء لا تتغير عند الانتقال من إطار مرجعي بالقصور الذاتي إلى إطار مرجعي بالقصور الذاتي آخر. وهكذا، على سبيل المثال، لا يستطيع شخص في عربة قطار مغلقة، من خلال أي تجربة أو قياس فيزيائي، تحديد ما إذا كانت السيارة تتحرك بسرعة ثابتة أو واقفة في حالة سكون."

    إذن، إليك طريقة لتحديد: إذا كانت هناك فجوة بين الساعات - فأنت في حالة حركة بالنسبة للإشعاع. لم تخلق - أنت في راحة.

    لن أخوض في الادعاء بأن الإشعاع هو عامل خارج النظام (ادعاء R.H.) الآن.

    الشيء الثاني الذي حصلنا عليه هو أننا إذا قمنا بعكس إنشاء الارتباط، أي أن جاك هو الذي يتحرك في نظام القصور الذاتي لجيل، فإنه يمر بساعات عادية تظهر وقتًا أعلى من ساعته الحرارية، أي وقتًا أعلى من عمر الكون .

    في هذه الحالة، التزامن ليس مصطنعًا - إذا كان نظام القصور الذاتي لجيل عبارة عن قطار، فإن أول سيارة تمر بالقرب منه هي في عمر الكون، وجميع الساعات التي تليها ستظهر وقتًا أعلى من ساعتي جاك، C.H. ودرجة الحرارة، والتي تبين عمر الكون.

    إذا انطلقنا من افتراض أنه من المستحيل أن نتوقع من نظام خصم نسبي للإشعاع في نظام آخر يتجاوز عمر الكون، (فمن الممكن نظريا بناء مثل هذا القطار الذي، إذا تحرك بسرعة كافية، فإن الساعات فيظهر الزمن 50 مليار سنة)، فقد حصلنا على تناقض بين استطالة الزمن ونظرية الانفجار الأعظم.

    عملياً، في كل قطار عادي يمر بنا (على افتراض أننا ساكنون تماماً بالنسبة للإشعاع)، وفقاً لنفس المبدأ، إذا كان عمر السيارة الأولى هو نفس عمر المراقب (وهذا صحيح)، إذن عمر السيارة الأخيرة أعلى من عمرها بجزء من الثانية، أي في المستقبل.

    הערה:

    عندما نشر أينشتاين "المفارقة بعد ذلك"، كانت المفارقة هي أن التشابك الكمي يتناقض مع مبدأ المحلية، وهو مبدأ كان لا شك فيه في عصره وكان بور يؤمن به أيضًا.

    أثبتت نظرية متباينة بيل أن ما يعتبر مستحيلاً هو في الواقع حقيقة: اللامكانية موجودة. ولذلك ليس هناك في الواقع أي مفارقة هنا.

    حجتي للتناقض بين نظرية الانفجار الكبير والنسبية ترتكز أيضًا على افتراض أنه من المستحيل ملاحظة (وتصوير) أنظمة يكون زمنها أعلى من عمر الكون، أي في المستقبل. ومن ناحية أخرى، إذا لم يكن هذا الافتراض صحيحا، فلن يكون الادعاء كذلك. من يدري، ربما، كما تظهر تجربة ويلر في الاختيار المؤجل، إذا كان من الممكن التأثير على الماضي من الحاضر، فمن الممكن أيضًا توقع المستقبل من الحاضر (أين تهب المياه؟).

  204. وقال الطبيب أن العملية كانت ناجحة.
    أنا الآن أنتظر أن يسمحوا لي بالدخول إلى وحدة العناية المركزة لجراحة الأعصاب، لأنه رغم وصول مواعيد الزيارة إلا أن زيارة الأطباء تمنع الدخول.
    يبقى أن نرى ما إذا كانت لن تكون هناك مضاعفات وما إذا كان العلاج سيحقق النتائج المرجوة.

    فكرت أكثر قليلاً في المشكلة وأعجبت (على الرغم من أنني لم أدخل الرابط مع الحساب بعد ولم أعتقد بنفسي أن الاستنتاج الموجود في الرابط صحيح.
    من ناحية أخرى، كلما فكرت في الأمر أكثر، لا أرى لماذا يتعارض وجود ساعة درجة حرارة نظام الخلفية التي يمكن للجميع رؤيتها مع النظرية النسبية.
    بشكل عام، إنها ساعة أخرى، وهي تقيس عمر الكون وفقًا لنظام إشعاع الخلفية.
    فإذا بنينا نظام ساعات يوضح عمر الكون وفق نظام قصوري آخر (ولا يهم كيف) سيكون له نفس الخصائص لأنه سيكون أيضًا ساعة مثل جميع الساعات.

  205. إسرائيل:
    أنا حقا ليس لدي الوقت لمواصلة القصة. غدا يجب أن أكون في إيخيلوف الساعة 07:30 وأنا لا أعيش في مكان قريب.
    أخبرتك أنني لم أقم بالحسابات الدقيقة وقد لا يكون تخميني صحيحًا.
    أرى إلى أين يقود هذا في سياق ساعات درجات الحرارة وكان هذا في الواقع سبب تخميني للنتيجة في القطار.
    من الممكن أن تكون المشكلة مع هذه الساعات بعد كل شيء ولكن يجب أن أتوقف عن متابعتها في الوقت الحالي.

  206. البستاني إليك... وعدني أن المنحدر الموجود فوق الحديقة سوف يزهر بالزهور بلونين: البرتقالي والأصفر.

    وعد وأوفى وزر وزهرة. لكن البرتقال يزهر في مايو، ويزهر في سبتمبر.

    المطالبة 2. في الوقت الذي ادعيت فيه: كل صورة من كل جانب ستظهر عكس الأخرى.

    في ذلك الوقت قدمت نفس المطالبة (الرد 10):

    "دعنا نحاول. ويقال أن السيارة تحتوي أيضًا على كاميرا.
    وبما أن سرعتها تساوي حوالي نصف سرعة الضوء، فإن الفارق الزمني بين ساعات الطريق وساعات السيارات حسب معادلات لورينز يبلغ حوالي 15%. والوقت الذي تقضيه السيارة بجوار كل ساعة على الطريق يبلغ حوالي جزء من مليار من الثانية.
    لنأخذ الساعة الثامنة على سبيل المثال. يصل فارق التوقيت بالفعل إلى ثانية كاملة. وفقًا للكاميرا الموجودة على الطريق، فإن السيارة في الخلف، ووفقًا للكاميرا الموجودة في السيارة، فإن الساعة الموجودة على الطريق متأخرة. ومع ذلك، تم التقاط الصور بكاميرات كانت تقريبًا بجوار بعضها البعض في غمضة عين، وفي الواقع يمكن استبدالها. كيف يمكن أن يروا مثل هذا الاختلاف الكبير؟ وماذا سيحدث للكاميرا الثمانين عندما يزيد الفارق إلى 10 ثواني؟ الكاميرا التي على الطريق ستظهر 090240 في الطريق و090230 في السيارة، والكاميرا التي بجانبها تقريبًا في غمضة عين، تلك التي في السيارة، ستظهر عكس ذلك تمامًا؟

    وإليك إجابتك (سنتان ونصف (من يحسب؟)، ولكن لا تزال في فئة "في ذلك الوقت"):

    مايكل روتشيلد

    إسرائيل شابيرا:

    لقد كنت مشغولا ولم التفت للإجابة، ولكن على سؤالك في التعليقين 10 و11 فالإجابة هي نعم.

    24 مارس 2011

    https://www.hayadan.org.il/demonstrating-relativity-1403111/comment-page-1/#comment-288037

    3. حيث أن جواب النسبية في فهمي هو: أن ساعة القاطرة ستتأخر عن ساعة السيارة.

    وكما ترون في الرابط الذي قدمته، فإن هذا هو بالفعل جواب النسبية (هل لا توافق على ما هو مكتوب في الرابط؟).

    4. لكن الصور ستظهر أن القاطرة في الخلف.

    يتفق الطرفان على أن الساعة 'C (القاطرة) تتأخر عن الساعة C2 (السيارة). كما هو مكتوب أدناه:

    ماذا تظهر لقطة جيل الرقمية؟ يجب أن تكون متطابقة، فاللقطتان المأخوذتان من نفس المكان وفي نفس الوقت يجب أن تظهرا نفس الشيء!

    وكما هو مبين في الرابط، يمكن لكل من جاك وجيل (بطلي القصة) أن يدعيا أن الزمن يتحرك بشكل أبطأ مع الآخر. لكن الكاميرات تظهر أن موسم جيل هو بطيء.

    وهكذا عندما همس جاك بلطف لجيل، "أنت متخلفة مرة أخرى، يا عزيزتي،" غردت مرة أخرى: "أنت متخلفة نفسك!"

    الموقف في أفضل حالاته!

  207. ميخائيل

    1. يمكن العثور على مثال لطريقة عمل تمديد الوقت في الرابط الذي قدمته سابقًا:

    http://galileoandeinstein.physics.virginia.edu/lectures/time_dil.html

    التجربة الفكرية هناك هي نفس مثال القطار الخاص بي، مع الساعة C1 كقاطرة B، وC2 كسيارة، و"C كقاطرة A".

    لاحظ أنه عندما يمر 'C (القاطرة A') بـ C1 (القاطرة B) فإن الوقت في كليهما هو 0، تمامًا كما في مثال القطار.

    عندما يتجاوز C2 (السيارة) يكون زمنه 8 ثواني وزمن C2 هو 10 ثواني. زمن القاطرة يتأخر عن زمن القطار كما زعمت.

    لاحظ أنه مكتوب أسفل الصورة الثانية أن كلاهما متفقان على أن الزمن C2 (القاطرة) يتأخر عن الزمن C (السيارة).

    في المثال هناك تفسير رياضي (وصحيح) لكيفية ادعاء القاطرة أيضا أن زمن القطار قد تأخر، لكن الصورة - وهذا ما تحدثنا عنه - توضح أن زمن القاطرة قد تأخر.

    سأركض للتحدث مع البستاني، وسنواصل لاحقًا مع الأقسام الأخرى.

  208. إسرائيل:
    أنا لم أقل أنها كانت ساعة درجة الحرارة.
    قلت أن هذا أيضًا ما قلته عن ساعات درجة الحرارة.
    على أية حال، قلت أيضًا أنني لم أحاول حساب الأوقات وفقًا للصيغ وهو في الواقع نوع من التخمين.
    عليك أن تفهم أن العلم ليس في ذهني في الوقت الحالي.
    قد يعطي الحساب الدقيق نتيجة مختلفة، لكنني متأكد من أن النظرية ستظل ثابتة

  209. 1. لا تزال هذه إجابتي ولقد كنت متسقًا مع هذه الإجابة منذ أن تحدثت عن ساعة درجة الحرارة وقد أجبتك عدة مرات أن هذه تظل إجابتي.
    2. لا تذكر أنني قلت يوما ما تزعم أنني قلته. لقد أخبرتك بالفعل أنني لا أتذكر وطلبت منك الإشارة إلى المكان الذي تعتقد أنني قلت فيه ذلك. أنت لم تصوت وتستمر في المطالبة بذلك.
    3. ما تعتقده في إجابة النسبية ليس هو إجابة النسبية. الجواب النسبي هو الذي أعطيته

  210. ميخائيل.

    وهنا ادعاءاتي ردا على ذلك.

    1. كانت إجابتك: الساعتان، القاطرة والعربة، ستظهران نفس الوقت في الصور (هل لا يزال هذا هو ادعاءك؟).

    2. في الوقت الذي ادعت فيه: كل صورة من كل جانب ستظهر عكس الأخرى.

    3. حيث أن جواب النسبية في فهمي هو: أن ساعة القاطرة ستتأخر عن ساعة السيارة.

    4. لكن الصور ستظهر أن القاطرة في الخلف.

    5. التماسيح... سنتركها.

    يرجى الإشارة إلى ما تعتقد أنه غير صحيح في الرد. إذا كان هناك شيء آخر غير صحيح فاتني - فأشر إليه.

    طب مكمل.

  211. أشكر كل من أعرب عن قلقه بشأن صحة والدي.
    وللأسف نحن ننتظر منذ الصباح دوره لإجراء العملية الجراحية وهو صائم منذ مساء الأمس.
    لقد قال مازحا بالفعل إنه من المستحيل أن يموت أثناء الجراحة لأن الجوع سيقتله أولا

  212. إلى جميع المشاركين في المناقشة:
    ومازلت مع والدي في المستشفى وأقرأ وأكتب على الهاتف المحمول.
    الحروف صغيرة، والعيون ليست كما كانت، والأصابع سميكة.
    كل هذا يمكن أن يؤدي إلى استجابات مشوهة - إما لأنني لم أقرأ الأشياء التي أرد عليها جيدًا، أو بسبب التشتيت، أو بسبب الكتابة الخاطئة.
    نأسف للإزعاج

  213. إسرائيل:
    كما قلت - لا يوجد شيء في الرد صحيح ولا أريد أن أكرر الرد بأكمله.
    وليس صحيحاً أنني لم أجب على السؤال المتعلق بالصور كما يوحي ردك. ولم أجب فقط بل أنت تناولت ما أجبت عليه.
    لم تقم فقط بالإشارة، ولكن حتى في الرد الذي زعمت فيه أنني لم أقم بالإشارة، فقد زعمت أيضًا أنني قدمت مرجعين متناقضين.
    كما أن الادعاء بأنني قدمت إجابات متناقضة ليس صحيحا.
    ادعائك حول ما تدعي النسبية هو أيضا غير صحيح.
    باختصار، لا شيء في الرد صحيح

  214. عرض

    نوع من الاتصال بالقاضي؟ (عوفر ماجد)

    الكثير من علماء الفيزياء هم الغزلان والأيائل.

  215. مايكل، كيف حال أبي؟

    ما العيب في الرد، ألا توافقون على أن التماسيح أطول من الخضر؟

    قم بإعداد قائمة بكل ما هو غير صحيح في الرد، وسأحاول أن أثبت لك قسمًا بعد قسم أن كل شيء صحيح.

  216. إسرائيل:
    قلت أن كلتا الساعتين ستبدوان متماثلتين.
    https://www.hayadan.org.il/ball-state-prez-intelligent-design-not-science-0408135/comment-page-24/#comment-448276
    هذا يتعلق بسؤالك ولا أعرف لماذا تدعي عدم ذلك. ولا أذكر أنني زعمت أن كل شخص سيرى عكس الآخر. هل يمكنك الإشارة إلى تعليق كتبت فيه شيئًا كهذا؟
    على العكس من ذلك - لقد قدمت نفس الادعاء تمامًا فيما يتعلق بساعة درجة الحرارة للنظام A عندما تلتقي بساعة درجة الحرارة للنظام B.
    https://www.hayadan.org.il/ball-state-prez-intelligent-design-not-science-0408135/comment-page-24/#comment-448130
    الإجابة النسبية ليست هي التي تصفها وليس لدي أي نية لمحاولة تبرير سوء فهم أي شخص.

    لقد كان تعليقك الأخير مبالغًا فيه حقًا لأنني لم أجد فيه حتى شيئًا واحدًا صحيحًا

  217. عرض

    هل سؤالك يشير إلى فقرة من ردي السابق:

    "وإذا كان الجواب النسبي في فهمي هو: ساعة القاطرة سوف تتأخر عن ساعة العربة. (السؤال المطروح بالطبع، فكيف يمكن للقاطرة أيضاً أن تدعي أن العربة متخلفة؟ والجواب على هذا السؤال هو إذا كان هناك طلب)».

    يعني هل هناك طلب؟

  218. إسرائيل،

    لقد ذكرت أن الساعات في أحد القطارين غير متزامنة مع الآخر، لكن من ناحية أخرى كتبت، وأقتبس:

    "ماذا ستظهر الساعات عندما تمر القاطرة من القطار A بالسيارة رقم 6789 في القطار المقابل؟
    وفقا للنظرية النسبية، في نظام القطار، فإن الوقت في السيارة يتأخر عن الوقت في القاطرة.
    وفقًا للنظرية النسبية، في النظام B، يتأخر الزمن في القاطرة عن السيارة.

    لتجنب الشك، الجملة الثانية الخاصة بك غير صحيحة. وفقًا للنظرية النسبية (كما أوضحت)، حتى في نظام القطار A، فإن الوقت في القاطرة يتأخر عن السيارة. ولكن لماذا كتبت ذلك؟ في رأيي، هذا صحيح فقط إذا تم تجاهل ظاهرة عدم التزامن بين القطارات. ربما لم أفهم قصدك

    باختصار وفي صلب الموضوع:
    هل توافق على أن النسبية تتنبأ بأنه حتى في نظام القطار A، تتأخر القاطرة خلف السيارة. فإذا كان الأمر كذلك فلا خلاف بيننا. إذا لم يكن الأمر كذلك، يرجى توضيح سبب توقعها (ومرة أخرى، "إطالة الوقت" ليست كافية، فأنت بحاجة إلى شرح سبب تغلبها على نقص التزامن).

  219. ميخائيل.

    كلنا في الموقع نتمنى لك ولوالدك عملية ناجحة، والشفاء الكامل والسهل.

    عندما تعود، ربما تشرح لي مدى ارتباط كلماتك بسؤالي. لقد طرحت سؤالاً بسيطًا وواضحًا للغاية: كيف ستبدو الصور من كلا الجانبين، القاطرة والعربة. كررت ذلك عدة مرات وشددت على أنه لا يوجد رياضيات ولا فلسفة ولا حمير وحشية. الفيزياء فقط. تصوير.

    كانت إجابتك: الساعتان، القاطرة والعربة، ستظهران نفس الوقت في الصور.

    في الوقت الذي ادعت فيه: كل صورة من كل جانب ستظهر عكس الآخر.

    في حين أن جواب النسبية في فهمي هو: أن ساعة القاطرة سوف تتأخر عن ساعة السيارة. (السؤال الذي يطرح نفسه بالطبع، فكيف يمكن للقاطرة أيضاً أن تدعي أن العربة متخلفة؟ الجواب على هذا السؤال هو إذا كان هناك طلب).

    لكن الصور ستظهر أن القاطرة متخلفة، ونحن نتعامل مع هذه الإجابة فقط.

    إذا كنت تتفق مع هذه الإجابة (وهل تتفق معها؟) ومازلت مهتمًا بالمناقشة، فيمكننا الانتقال إلى النقطة الأساسية: التناقض بين استطالة الزمن ونظرية الانفجار الأعظم.

    عوفر – لا أفهم ما هي النقطة. ما الفشل؟ بعد كل شيء، ذكرت على وجه التحديد أن ساعات كل قطار متزامنة مع بعضها البعض، ولكنها غير متزامنة مع القطار الآخر. فأين المشكلة هنا؟

    الماء - خاص بالنسبة لك: إذا كان هناك تمديد للوقت، فيمكن إثبات أنه من الممكن الانتقال إلى المستقبل!

    لا مرح؟ للجلوس على سحابة كونية في سوتول، انفخ واستنشق الماء من الشيشة.

    فيما يتعلق بالتمساح - الحدس الأولي هو أنه بالطبع أكثر خضرة، لأنه طويل فقط بالطول، ولكنه أخضر بالطول والعرض.

    هذه الإجابة صحيحة بالنسبة للخيار. لكن يمكن تمرير التمساح مرة واحدة بالطول، وتكرار الخطوط في جلد التمساح، بحيث تحصل على طولين، وهما بالطبع أكثر من الطول والعرض، فهو في الواقع أطول.

  220. من. لقد أصلحنا معظمها وربما لهذا السبب لا تراها بعد وقت العودة، ولكن،
    يمكنك الحصول على انطباع بسيط عن مجموعة إسحق رابين في العام الماضي، والجزء المتعلق بالطعام، ربما كتبوه فقط للتذكر، والأكثر من ذلك، كان لي شرف رؤية الغزال بعد تكرار الوقت، وهو من الصعب أن أشرح لشخص لم يراها، مع الاحترام

  221. نفخ الماء:
    أنت لست متدينًا ولكنك تؤمن بحكايات الكتاب المقدس الخيالية.
    مثيرة للاهتمام.
    إذا كنت تعتقد أن ما هو مكتوب في الكتاب المقدس صحيح، فلماذا أنت غير متدين؟
    هل تريد جلب الدمار علينا جميعاً؟
    تذكر: نهاية العالم قادمة!

  222. مكتوب من لوبي إيخيلوف.
    نفخة الماء عزيزي.
    في رأيي أن تأثير الوقت والقداس في اليهودية يأتي نتيجة لحركة اليهود ذهابًا وإيابًا في الصلاة وما تشعر به هو تجمعها في الكوتيل وبتردد متوسط.
    شكرا لعدم نسيان نهاية العالم قادمة

  223. إسرائيل،

    لا خلاف بيننا على أن هناك طريقة لمزامنة أي عدد من الساعات المطلوبة – داخل القطار. بعد كل شيء، الطريقة التي قدمت بها تزامن الساعات تمامًا! لكن من ر. قال بشكل صحيح - قاطرة A1 ليست جزءًا من هذا النظام. في الواقع، وفقًا للقصة، بغض النظر عن الطريقة التي تقوم بها بمزامنة الساعات، بمجرد مزامنتها في القطار B، فهذا يعني **بالضرورة** أنها غير متزامنة في القاطرة A

    هناك طريقتان لرؤيتها: طريقة سهلة وطريقة معقدة.

    الطريقة الصعبة هي السماح لك بمزامنة الساعات بالطريقة التي تختارها. ثم سنضع بجانب كل ساعة "ساعة نبضية" تتزامن بالطريقة التي وصفتها سابقًا. في نظام القطار، ستتم مزامنة كل ساعة بالضرورة مع "الساعة النبضية" المجاورة لها. ولكن كما أوضحت، في نظام القاطرة الأخرى، لا تتم مزامنة ساعات النبض، وبالتالي فإن ساعاتك (بغض النظر عن الطريقة التي قمت بمزامنتها بها) ستكون حتمًا غير متزامنة.

    الطريقة الأسهل هي إلقاء نظرة على مخطط الزمكان (يرجى فتح الرابط! إنها رسوم متحركة بسيطة، لا توجد استثناءات!)

    http://en.wikipedia.org/wiki/File:Relativity_of_Simultaneity_Animation.gif

    هناك 3 دوائر A، B، C وهي عبارة عن 3 ساعات موجودة في القطار، في أماكن مختلفة. لنفترض أنهم جميعًا يضيئون في تمام الساعة 00:00. يوضح الرسم البياني بشكل جيد للغاية أنها متزامنة بالفعل في نظام الراحة، ولكنها غير متزامنة فيما يتعلق بالنظام المتحرك.

  224. السيد إسرائيل شابيرا المحترم
    في رأيي أن تأثير الزمن والكتلة على العلاقة، يأتي نتيجة حركة الجزيئات ذهابا وإيابا في الزمن عدة مرات، وما تشعر به هو تقاربها سواء في الكتلة أو في الزمن في المتوسط، شكرا أنت، لا تنسى أن Simchat Torah قادم

  225. أشباح:
    وهناك علاقات كثيرة وليست كلها متعدية.
    وتبين أن التزامن ليس علاقة متعدية.
    بعض الناس يفهمونها والبعض الآخر لا.

  226. إسرائيل:
    حقا!
    إنه ليس مجرد شيء حسابي ولكن من المضحك جدًا أن نطلب من الأشخاص الذين ليسوا بجوار بعضهم البعض أن يقوموا بمزامنة ساعاتهم الآن من خلال التواجد بجوار بعضهم البعض.
    كما قلت - هناك طريقة لمزامنة جميع الساعات في نظام القصور الذاتي وكما تعلم فإن قاطرة A وقاطرة B ليسا في نظام قصوري واحد.
    الآن - هناك طريقتان للتعامل مع المشكلة: واحدة من خلال التحقق من اتساق الحل (وهذا ما قمت به حتى الآن) والأخرى من خلال التجارب (والتي تتضمن التجربة المستحيلة التي تقترحها - وهي أن نكون بجوار بعضنا البعض بينما أنت لست كذلك).
    وطريقة التجارب - كما أوضحت لك - تؤكد أيضا النظرية النسبية.

    هذا كل شيء.
    يجب أن أتوقف عن هذا الجدال لأنني سأحضر والدي هذا الصباح إلى العملية الجراحية التي قرر إجراءها على أمل أن يخرج منها بصحة أفضل ولكن أيضًا مع وجود خطر عدم خروجه منها.

  227. ميخائيل.

    إذا كنت تعتقد أن إطالة الوقت في أنظمة القصور الذاتي هي مجرد شيء حسابي، أي أنه من غير الممكن التحقق من ذلك عن طريق مقارنة صور الساعات التي تمر ببعضها البعض - فأنا مستعد لإغلاق ذلك.

    لسوء الحظ، هذه ليست إجابة النسبية كما أعرفها. وفقا للنظرية النسبية هناك إجابة لا لبس فيها - زمن القاطرة يتأخر عن زمن العربة وهذا أيضا ما ستظهره الصور من كلا الجانبين.

    فكيف يدعي كل طرف أن الطرف الآخر متخلف؟ ما هذه حديقة الدبدوب حيث يصرخ الجميع في وجه الآخر بأنهم متخلفون؟!

    هناك إجابة، وهي ليست التي أعطيتها. أعلم كم أن وقتك ثمين، وكم تكره أن تضيعه، خاصة على الأشخاص العنيدين. ولهذا السبب سأحفظه لك.

    عوفر

    هناك طريقة لمزامنة أي عدد من الساعات المطلوبة في أي نظام بالقصور الذاتي، والقطار هو مثل هذا النظام. انظر أينشتاين:

    http://www.fourmilab.ch/etexts/einstein/specrel/www/

    الفصل أ، تعريف التزامن

    وهذه الطريقة جيدة تمامًا لغرض الحساب، فلا توجد مشكلة.

    أشباح

    هل التمساح أطول أم أكثر خضرة؟

  228. تسحب القاطرة العربة وتسحب النسبة نسبة. بمعنى آخر: كل شيء نسبي. هناك 3 أنواع من الناس في العالم: أولئك الذين يعرفون الحساب وأولئك الذين لا يعرفونه.

  229. إسرائيل،

    في البداية سؤالك يهمني. في رأيي، مغالطتك هي أن المزامنة ليست "متعدية":
    تتم مزامنة القاطرة A1 مع القاطرة B1
    تتم مزامنة القاطرة B1 مع العربة B60000 (يشير الرقم إلى الموضع التسلسلي في القطار).
    لكن ** لا ** ينبع من حقيقة أن السيارة B60000 متزامنة مع قاطرة A1.

    وفي ويكيبيديا، يتم شرحه بشكل أفضل، من خلال نسبية التزامن.

    فيما يتعلق بسؤال القطارات: كيف ستحقق المزامنة على القطار B؟ لنفترض أن نبضة من الضوء تخرج من القاطرة قرب نهاية القطار. إذا كان طول كل عربة 1 ميلي ثانية، فيجب أن تقوم كل عربة بإعادة ضبط الساعة لتكون الرقم التسلسلي مضروبًا بالميلي ثانية. وبالتالي، بعد دقيقة واحدة، ستظهر جميع الساعات من Kron B1 إلى Kron B60000 (ضمناً) الوقت 00:01.

    لنفترض، وفقًا لقصتك، أنك أرسلت نبض المزامنة تمامًا كما تمر القاطرات ببعضها البعض.
    ماذا يحدث في المقطورة مقابل 60000 ألف؟
    عندما يصل إليه النبض، يكون الوقت 00:01 والوقت في القاطرة A1 أقدم لأن الساعة هناك متأخرة.

    ماذا يحدث في قطر A1؟
    القطار B أقصر بسبب قصر المسافة (حسب تحويل لورنتز). أي أن طولها (على سبيل المثال) هو 45 ثانية ضوئية فقط وليس دقيقة ضوئية. عندما يصل النبض إلى B60000، يكون الوقت على القاطرة A1 هو 00:00:45 فقط، لكن لسبب ما ترسل السيارة B60000 رسالة مفادها أن وقتها هو 00:01، وهو في المستقبل على الإطلاق، ومن ناحية أخرى ، القاطرة A1 متخلفة وتظهر وقتًا من الماضي. لم يتم العثور على قطار مهرج، رأسه في الماضي وذيله في المستقبل والمتناثر من كفار عازر. هل ستكون هذه الفجوة كافية لتتمكن الكاميرات من رؤية شيء معقول؟ لا أعرف كيف أقوم بالحسابات. لكن لك أن تتخيل ماذا سيحدث عندما تكون السرعة النسبية للقطارات قريبة جدًا من سرعة الضوء، أي أن عامل لورنتز (انكماش الزمن/المسافة) كبير جدًا:

    السيارة عند 60000: يصل النبض، وبالتالي فإن الوقت هو 00:01، وبعد جزء من الثانية تصل القاطرة A. الساعة في القاطرة A بالكاد تتحرك، وبالتالي فإن الوقت هو 0 + إبسيلون.

    القطار A1: القطار B قصير جدًا جدًا، لذا يستغرق وقتًا طويلاً للوصول إلى B60000. لكن النبض وصل قبل ذلك بقليل، لذا فإن الساعة عند 60000 تظهر 00:01 + جزء من الثانية.

    وظهر، بتلويح الأيدي، أن A1 و6000 منسقان في صورهما.

    أنتم مدعوون للتفكير في طرق أخرى للمزامنة داخل القطار. لكنني أعتقد أنك ستجد أنهم جميعًا يعانون من مشكلة مماثلة: ما يبدو متزامنًا في القطار B غير متزامن في القطار A.

    آسف على الخندق، ومساء الخير..

  230. إسرائيل:
    لا يتعلق الأمر بما يفكرون فيه في أسرار قلوبهم، بل بما سيتوصلون إليه في اختبار التزامن.

  231. بلا شك أجبت. لكنني لست متأكدًا من إجابتك بشكل صحيح :)

    هيا، مايكل، يحث من المعلقين المخضرمين. لم تجب بعد ما رأيك في الرد الذي تلقيته من قبل:

    "أكرر ما كتبته من قبل. الجملة "إذا كانت الشمس وتحركنا نسبة إلى النظام
    إن إشعاع الخلفية الكونية بسرعة 370 كم في الثانية لا معنى له. من حيث جميع الأنظمة
    الإشعاع الكوني في حالة راحة."

  232. أنا أقوم بالحفر، لكن هناك هذا الشيء مع الكاميرات...

    كما ترى، عندما تلتقي القاطرتان، تظهر الصور من كلا الاتجاهين: القاطرة A0، والقاطرة B0 أيضًا.

    وهذا هو الشيء الوحيد الذي يهم. وليس ما يعتقده القطريون في سر قلوبهم.

    وإذا كانت الصور تظهر نفس الشيء عندما تلتقي القاطرات بالعربات، فلن يكون هناك إطالة زمنية قابلة للقياس في أنظمة القصور الذاتي، ربما باستثناء أفكار ومشاعر ومعتقدات القاطرات والعربات.

    وهذا على عكس الطائرات التي تحلق حول البلاد بأنظمة متسارعة، والتي يمكنك من خلالها تصوير إطالة الوقت بالساعات، أو التوائم، التي يمكنك من خلالها قياس ومقارنة طول لحى التوائم...

  233. إسرائيل:
    لقد سبق أن شرحت جيدًا عدة مرات ويبدو لي أنك لا تقرأ بعمق.
    وسأعرض إحدى وجهات النظر مرة أخرى:
    يعتقد الناقل "أ"، خلال اجتماعه مع الناقل "ب"، أنه على الرغم من أن الوقت في الناقل "ب" هو 0، مثله مثل الوقت في الناقل "ب"، فهو متأخر.
    عندما يلتقي بـ Kron B يرى (Cater A) أن ساعاتهم هي نفسها.
    معنى هذه الحقيقة هو أنه كلما زاد شعور القاطرة A، قل الوقت الذي مر في العربة B (أي أن الوقت فيها يتقدم بشكل أبطأ)

  234. قلت إنه إذا كانت الصورة من القاطرة تظهر قاطرة 12:43 فسوف تظهر أيضًا عربة: 12:43
    سوف تظهر الصورة من المقطورة نفس الشيء

  235. ميخائيل
    دعونا نترك لورينز في الوقت الراهن.

    الصورة من القاطرة تظهر: قاطرة 12.43 سيارة 11.52

    ماذا ستظهر الصورة من المقطورة؟

  236. إسرائيل:
    لا أفهم لماذا تعلق عليك عبارة "السيارة الأخيرة"
    لقد استخدمته فقط لتبسيط الصياغة.
    هل تهتم حقًا بالسيارات التي تتبعه أم تريد مني أن أضيع المزيد من الوقت؟
    ستبدو ساعات القاطرة والسيارة متماثلة في الصور، لكن هذا سيحدث لأنه من وجهة نظر القاطرة، ستظهر السيارة في القطار الآخر وقتًا أكبر من الصفر بمجرد التقاء القاطرتين ومن نقطة الانطلاق. على مدار الساعة في السيارة، ستظهر ساعة القاطرة وقتًا أكبر من الصفر عندما تلتقي القاطرتان.
    نعم!
    إن السيارة والقاطرة في نفس القطار سوف ترى أشياء مختلفة في قاطرة القطار الآخر بمجرد أن يرى كل منهما لحظة اللقاء وذلك بسبب ذلك X الذي ذكرته.
    لقد قدّرت ذلك دون إجراء حسابات دقيقة، لكن عليك أن تفهم أن القصة بأكملها تنبع من فقدان التزامن المطلق، وحتى في حال أظهرت الكاميرات في القطارين مواقف مختلفة، فلا ينبغي أن تتحمس.
    انظر على سبيل المثال سلم "المفارقة":
    http://en.wikipedia.org/wiki/Ladder_paradox

    يمكن أن يعرض المستودع فيلمًا حيث تكون البوابتان دائمًا في نفس الوضع تمامًا وفي لحظة معينة يكون السلم بالداخل بينما يمكن أن يعرض السلم فيلمًا حيث لم يكن مطلقًا داخل المستودع بالكامل وكانت هناك لحظات كانت فيها البوابة "أ" مفتوحة وتم إغلاق البوابة B.

  237. ميخائيل.

    القطار الأخير لم يظهر في السؤال. ولم يتم تحديد عدد عربات القطار، ومن المؤكد أنه يمكن أن يحتوي على 765667 سيارة في سعة 766543 سيارة.

    تم ملاحظة فقط القاطرات التي تمر ببعضها البعض في الوقت 0 (تم تصويرها من كلا الجانبين)، والسيارة رقم 6789 في القطار B، وأوقات الساعات التي تمر فيها القاطرة من القطار A.

    وطرح سؤال أيضًا: كيف ستبدو الصور من القاطرة والعربة التي تلتقط صور الساعتين معًا؟ هل سيرون صورًا تظهر فيها الساعات معكوسة؟ هل هذا ممكن؟ وإذا لم يكن الأمر كذلك، فماذا ستظهر الصور؟ وكيف يتناسب هذا مع الادعاء بأن كل نظام يرى الوقت في النظام الثاني أبطأ؟

    أو باختصار: كيف ستبدو الصور؟ فقط هذا.

  238. إسرائيل:
    لقد أجبت على ذلك بالفعل.
    من وجهة نظر آخر سيارة في القطار A، لم تظهر قاطرة القطار B الوقت 0 عندما مرت بقاطرة القطار B.
    ويرجع ذلك، كما ذكرت، إلى ظهور X في تحويل لورنتز.
    في الواقع، يظهر حاصل ضرب X وV كعامل يحدد الوقت.
    تتم مزامنة القاطرة ونهاية القطار A لأن V بينهما يساوي 0
    يمكن مزامنة قاطرة القطار A مع قاطرة القطار B لأن X بينهما هو 0
    لكن نهاية القطار A لا تتزامن مع قاطرة القطار B عندما يمر بقاطرة القطار A.
    ولا يمكن مزامنته معه إلا عندما يقابله وستكون علامة X بينهما 0

  239. هيا، لا تدع الرجل يرى ثيلد وهو يلعب!

    اي فون.

    وهذا هو تعريف المشكلة بالمنتدى:

    وفقًا لطول الزمن، عندما يتحرك قطاران بالنسبة لبعضهما البعض على المسارات المجاورة، يرى مشاهدو كل قطار أن الوقت في القطار الآخر أبطأ.

    لذلك يقال أن الساعات في كل قطار متزامنة مع بعضها البعض ولكن بالطبع ليس مع ساعات القطار الآخر، وعندما تمر قاطرات كل قطار ببعضها البعض تظهر الساعات 0 على كليهما.

    ماذا ستظهر الساعات عندما تمر القاطرة من القطار A بالمركبة رقم 6789 في القطار المقابل؟

    وفقا للنظرية النسبية، في نظام القطار، فإن الوقت في السيارة يتأخر عن الوقت في القاطرة.

    وفقًا للنسبية، في النظام B، يتأخر الزمن في القاطرة عن السيارة.

    فماذا سيحدث إذا استخدمنا كاميرات عالية الدقة تقوم بتصوير الساعتين لحظة التغيير من كلا الاتجاهين؟

    هل ستظهر الصورة من القاطرة: ساعة القاطرة 14.23 وساعة العربة 11.27 والصورة من العربة ستظهر ساعة العربة 14.23 وساعة القاطرة 11.27؟

    هل هذا ممكن؟

    وإذا لم يكن الأمر كذلك، فماذا ستظهر الصور؟ سوف يرون شيئا، أليس كذلك؟

    وكيف يتناسب هذا مع مبدأ "كل نظام يرى الزمن أبطأ من الثاني"؟

  240. ما هو "الوقت الذاتي"؟ ولماذا تعطي ساعة درجة الحرارة زمنًا ذاتيًا لنظام إشعاع الخلفية أكثر مما تعطي زمنًا ذاتيًا لنظام متحرك بالنسبة إلى إشعاع الخلفية؟

  241. إذن نظام إشعاع الخلفية ليس النظام المفضل؟ هل يمكن أن تكون هناك أنظمة يكون فيها الزمن الذاتي أعلى من وقته، كما أن هناك أنظمة يكون فيها الزمن أقل من زمنه؟

  242. إسرائيل:
    بخصوص ما كتبته عن المشكلة المعنية - فقد أجبت عليها بالفعل.

    وفيما يتعلق بالادعاء - فإن النظرية النسبية لا تتناسب مع الاستبصار. لا أعرف إذا كان أي شخص سيقاضيك على أي شيء.

  243. النسبية الخاصة لا تتعامل مع ساعات درجة الحرارة، بل الساعات العادية فقط. ووفقا لذلك، فإنها تحدد بمعدلات مختلفة في أنظمة الثبات المختلفة.

    وما حاولت إظهاره هو أننا إذا افترضنا أن النظام الإشعاعي مجرد نظام عشوائي "آخر"، فإن هذا الافتراض يؤدي حتما إلى ساعات عادية يكون الزمن فيها أعلى من عمر الكون - وهو أمر مستحيل.

    بالمناسبة ما رأيك في الجملة التالية:

    "أكرر ما كتبته من قبل. الجملة "إذا كانت الشمس وتحركنا نسبة إلى النظام
    إن إشعاع الخلفية الكونية بسرعة 370 كم في الثانية لا معنى له. من حيث جميع الأنظمة
    الإشعاع الكوني في حالة راحة."

    مايكل، شكرا على المدخلات. بالمناسبة، هل من المتوقع أن تتم مقاضاتي؟

    تم الإرسال: 12/24/2011 الساعة 2:30:44 مساءً بتوقيت المحيط الهادئ (أنا
    الموضوع: (بدون موضوع)

    مرحبًا جيل

    أود أن أشكركم على الوقت الذي أمضيتموه في مناقشة مسألة تمدد الزمن معي. بعد رحيلك، فكرت أن ما قد يوضح فكرتي عن الزمن المطلق هو المثال التالي: لنفترض أن لدينا جهاز إرسال قوي على الأرض يرسل، باستخدام الراديو، إلى الزمن الأرضي للكون في فترات زمنية قدرها 1 ثانية.

    لذا فإن النقطة 0 ستكون على سبيل المثال 31 ديسمبر 2011 الساعة 12 منتصف الليل بتوقيت جرينتش، وبعد ذلك سترسل كل ثانية إشارة قصيرة وقوية: 1، 2، 3، …..100^100.

    وفي كل نقطة في الكون القريب، يمكن للمستقبل الذي سيستقبل النبضة أن يحسب مسافة الأرض (في وقت الإرسال) من قوة الإشارة وإزاحة دوبلر، وبالتالي الوصول إلى ما سيسمى: "التوقيت المطلق العالمي". في مثالنا الخاص بالسفن المهاجمة التي تحاول مزامنة ساعاتها، فإن هذا التوقيت العالمي سيمنحها طريقة مفيدة لتنسيق الهجوم. ولاحظ أيضاً أنه لو كان هناك المزيد من مصادر الإشارات التي تم ترتيبها بنفس الطريقة الأصلية، وساعاتها متزامنة في البداية مع الأرض، وتتحرك بسرعة ثابتة في الفضاء، مهما بعدت وفي أي اتجاه، عندها ستتلقى كل سفينة في الكون منهم نفس "الوقت المطلق العالمي"، بغض النظر عن المصادر التي ستختار استخدامها.

  244. إنها براءة اختراع وما زلت أبحث عن مشتري لها.
    لذلك، على حد علمي، لا تزال الطريقة غير مطبقة في أي مكان (إذا كنت أعرف شخصًا يطبق الطريقة سأقاضيه).
    لا تكمن ميزة هذه الطريقة مقارنة بالطرق السابقة لقياس المسافات في نجاحها في القيام بذلك (توجد بالفعل طرق تقوم بذلك بشكل جيد) ولكنها تسمح بإجراء العديد من هذه القياسات في وقت واحد (وبالتالي فهي تسمح بالتعامل مع أنظمة بها عدد أكبر من أجهزة الإرسال).
    ربما لا يرى أحد الحاجة إلى التعامل المتزامن مع هذا العدد الكبير من أجهزة الإرسال.

  245. هذا الرأس اليهودي... لا يتوقف عن اختراع براءات الاختراع لنا..
    متى ستبني لنا ساعة جنونية مضادة للمادة والطاقة المظلمة؟
    بالمناسبة أين توجد هذه الطريقة؟

  246. القيمة التي تقدمها ساعة درجة الحرارة هي نفس القيمة في جميع الأنظمة.
    لا يمكن لساعة درجة الحرارة أن تعمل في نظام يتحرك بسرعة الضوء حيث لا يوجد مفهوم الزمن على الإطلاق، ولكن في جميع الأنظمة التي تتحرك بسرعة أقل من سرعة الضوء يتم الحصول على نفس الوقت.
    بمعنى آخر - هي ساعة لجميع الأنظمة وليست ساعة لنظام معين.
    لذلك لا يوجد نظام مفضل.

  247. 1) من المستحيل البدء من افتراض عدم وجود نظام مفضل والتوصل إلى نتيجة مفادها أن هناك نظام مفضل.
    هناك تناقض منطقي داخلي هنا.
    2) العلم يستخدم المنطق. إذا كان هناك تناقض منطقي داخلي، فيجب عليك التحقق من مصدر التناقض وتصحيحه وفقًا لذلك.

  248. ב

    كل كلامك صحيح ما عدا:

    "1) إذا حدث انفجار، فهناك تمدد من الداخل إلى الخارج. أي أنه يجب أن يكون هناك "داخل" ويجب أن يكون هناك "خارج".
    وإلا فإنه ليس ضجة.
    2) إذا كانت هناك نقطتان غير منفصلتين فهناك نقطة ثالثة بينهما.
    لا يمكن لهذه النقطة الثالثة أن تكون نقطة مركزية ونقطة مغلف في نفس الوقت.
    خطر:
    إذا كانت كل نقطة عبارة عن نقطة مركزية ونقطة مغلف، فإن جميع النقاط تكون منفصلة.
    إذا كان الأمر كذلك :
    بالنسبة لك، جميع النقاط في عالمنا هي نقاط منفصلة.
    لكن:
    في عالمنا، هناك خط مستقيم يمتد بين نقطتين.
    وهذا يعني أنه يمكن العثور على نقطتين غير منفصلتين على الأقل."

    ليلة سعيدة من لوس أنجلوس.

  249. لا مشكلة يا مايكل، أنت حر.

    فقط للتلخيص، هذا هو ادعائي: من بين جميع الأنظمة المتزامنة، وفقًا لنظرية الانفجار، يعد نظام الإشعاع هو النظام المفضل. إذا كان النظام عشوائيًا، فمن الممكن أن توجد أنظمة يكون فيها الزمن الذاتي أعلى من الزمن الذاتي للإشعاع، وهو عمر الكون.

    ومن الناحية العملية، لا يوجد سوى أنظمة يكون فيها الزمن الذاتي، وفقًا للنظرية النسبية، أقل من عمر الكون أو يساويه. لقد ذكرت واحدًا - الفوتونات التي يكون زمنها الخاص هو 0. وقد أظهر عوفر ماجد نظامًا يكون زمنه الخاص حوالي 3 أيام. يمكن للمرء أن يفكر في عدد لا يحصى من الأنظمة الأخرى حيث يكون الزمن الذاتي أقل من عمر الكون.

    لكن لا يوجد نظام واحد يكون فيه الزمن الذاتي أعلى من عمر الكون. فقط في نظام الإشعاع والأنظمة المتزامنة معه يكون الزمن الذاتي مساويا له مما يجعله مفضلا.

    وإذا كنت أنت أو أي شخص على استعداد لخوض العملية العبثية حيث تتحقق من الوقت الذي يتم قياسه بواسطة الساعات في الأنظمة التي تتحرك بالنسبة للإشعاع الصادر من نظام مليء بالإشعاع، فستحصل حتماً على ساعات تظهر وقتًا أعلى من أن الساعات الموجودة في النظام الإشعاعي، أو أعلى من عمر الكون، وهو أمر مستحيل.

    وهذا هو التناقض مع إطالة الزمن الذي أشرت إليه.

  250. إسرائيل:
    بعض التناقضات:
    1) إذا حدث انفجار، فهناك انتشار من الداخل إلى الخارج. أي أنه يجب أن يكون هناك "داخل" ويجب أن يكون هناك "خارج".
    وإلا فإنه ليس ضجة.
    2) إذا كانت هناك نقطتان غير منفصلتين فهناك نقطة ثالثة بينهما.
    لا يمكن لهذه النقطة الثالثة أن تكون نقطة مركزية ونقطة مغلف في نفس الوقت.
    خطر:
    إذا كانت كل نقطة عبارة عن نقطة مركزية ونقطة مغلف، فإن جميع النقاط تكون منفصلة.
    إذا كان الأمر كذلك :
    بالنسبة لك، جميع النقاط في عالمنا هي نقاط منفصلة.
    لكن:
    في عالمنا، هناك خط مستقيم يمتد بين نقطتين.
    وهذا يعني أنه يمكن العثور على نقطتين غير منفصلتين على الأقل.
    وهذا يناقض ادعائك.

  251. إسرائيل:
    الوقت الذي أخصصه للموضوع بدأ بالفعل يصبح مفرطًا، ويبدو لي أنه يتم إنفاقه للتغلب على الإصرار وليس عدم القدرة على الفهم.
    1. من الممكن بناء ساعات درجة الحرارة التي تبين الزمن الذي انقضى منذ الانفجار الأعظم، لكن هذه الساعات تختلف في كل نظام مرجعي ولا تتقارب قيمها إلا عند النقطة التي يتواجد فيها الراصد.
    2. نعم، لأن نتائج أنظمة الساعة تتقارب عند النقطة التي يكون فيها المشاهد.
    3. مصطلح "ساعة أبطأ" أو "ساعة أسرع" ليس له معنى خارج النظام المرجعي.
    ليس من الضروري أن يتفق راصدان يتحركان في الفضاء بنفس السرعة وفي نفس الاتجاه على تزامن الأحداث التي تحدث عند نقطة تتحرك بالنسبة لهما (ولهذا السبب تظهر X أيضًا في تحويل لورنتز).
    4. لديك بالفعل ما يكفي من المواد لفهم الخطأ الذي تفعله

  252. لقد كنت معتدلا.

    الكرة لها نقطة واحدة - المركز. وهو يختلف عن وجه الكرة على سبيل المثال، ويمكنك أيضًا تحديده.

    كوننا ليس له مركز ولا وجه وفقا لنظرية الانفجار الكبير. وكل نقطة فيه مركزية ومغلفة معًا.

    ولا شيء آخر. في Rav Yokom يمكنك أيضًا الخلط بين الأمام والخلف.

    والآن سوف ينهض الحاخام المحترم ويعود إلى منزله.

  253. 1) ما هو التفرد؟
    أليست هذه نقطة فريدة؟
    2) إذا كانت هناك كرة فإن هناك نقطة هي مركز الكرة. يمكن تحديد هذه النقطة على أنها النقطة الرئيسية لنظام الإحداثيات.
    3) إذا كان الكون يتوسع فهناك نقطة مركزية يتوسع منها. إما أن تكون النقطة مفردة (نقطة تفقد فيها المعادلة معناها) أو أنها ليست مفردة. هذه النقطة هي نقطة فريدة في الكون كله.
    علاوة على ذلك:
    ليست هناك حاجة لمعرفة ما يحدث بالضبط في هذه المرحلة أو حتى في جوارها المباشر.
    يكفي تحديد كرة نصف قطرها كبير كما نريد حول هذه النقطة. خارج الكرة، ستكون جميع المعادلات صحيحة (على الأقل بالقدر الذي نحبه).
    مسار:
    إذا قمت بتعريف الكرة، فسيتم تحديد نقطة وهي مركز الكرة.
    ويمكن ربط مجموعة من المحاور بهذه النقطة وتحديد هذه النقطة على أنها أصل المحاور.

  254. ב
    هذه ليست نقطة. من المحتمل أنك تخلط بين النقطة والتفرد.
    الكرة لها مركز، يمكنك تحديد مركز الكرة كنقطة فردية، ولكن حتى في هذه الحالة لا يتعلق الأمر بالإحداثيات وليس له أي معنى.

  255. مايكل، حاول تقسيم الجدال إلى خطوات. ولهذا أقترح الطريقة السقراطية في الأسئلة والأجوبة.

    اسئله:

    1. هل تقبل أنه من الممكن نظريا بناء "ساعات درجة الحرارة"، أي الساعات التي تظهر في أي لحظة الوقت الذي مضى منذ الانفجار الأعظم، بغض النظر عن سرعتها بالنسبة لنظام CMBR؟

    2. هل تقبل أن مرور ساعتين من درجات الحرارة على بعضهما البعض سيظهر دائمًا نفس الوقت في صورة فوتوغرافية مشتركة (بغض النظر عن الجانب) في كليهما - الوقت الذي انقضى منذ الانفجار؟

    3. هل تقبل أن صورة عدة ساعات عادية تمر ببعضها البعض في نقطة ما في الفضاء ستكون هي نفسها بغض النظر عن الجانب الذي تم التقاط الصورة منه؟ أن الادعاء: "كل جانب يرى الجانب الآخر أبطأ" لا يشير إلى الواقع المادي، حيث يكون جانب واحد فقط هو الطرف البطيء، ولكن إلى واقع رياضي حيث من الممكن أن يفسر حسابيًا سبب تأخر الساعة السريعة فعليًا ؟

    4. هل تقبل أنه إذا تم ربط ساعة مؤقتة بساعة سيزيوم، فإذا كانت في نظام بالقصور الذاتي يستقر بالنسبة للإشعاع، وتمت معايرة ساعة السيزيوم مع وقت الساعة المؤقتة، ففي كل صورة مستقبلية سوف يرون نفس الوقت؟

    5. هل تقبل أنه في النظام الذي لا يكون في حالة سكون بالنسبة للإشعاع، ستكون هناك فجوة متزايدة بين ساعة درجة الحرارة وساعة تشيكوسلوفاكيا؟

    يشير السؤال 5 إلى مطالبتك:

    "إذا قمنا بتوزيع ساعات في الكون تظهر عمر الكون وتتزامن مع نظام يتحرك بالنسبة إلى إشعاع الخلفية (ويتحرك معه)، فإن كل ساعة من هذا القبيل ستظهر أيضًا في المستقبل عمر الكون في ذلك النظام المتحرك وزمن سابق في كل نظام آخر - بما في ذلك النظام الذي يكون في حالة سكون بالنسبة إلى إشعاع الخلفية لذا فإن النظام المتحرك الذي اخترته عشوائيًا، هو المفضل وفقًا للمبدأ الذي اقترحته."

    لأنه إذا حصلت على 3 فهذا الادعاء مستحيل. (ما لم تقصد بـ "وقت سابق" وقتًا محسوبًا وليس وقتًا مصورًا).

    وإذا قبلت الادعاءات في السؤالين 4 و5 - فإليك التناقض الصارخ للافتراض 1. لأنه إذا كان في نظام القصور الذاتي A (قريب من الإشعاع) لا توجد فجوة بين الساعات الحرارية والساعة، وفي نظام القصور الذاتي B (تتحرك بالنسبة للإشعاع) هناك فجوة بين الساعات، وهذا يخالف المسلمة رقم 1 التي تنص على:

    "إن قوانين الفيزياء لا تتغير عند الانتقال من إطار مرجعي بالقصور الذاتي إلى إطار مرجعي بالقصور الذاتي آخر. وهكذا، على سبيل المثال، لا يستطيع شخص في عربة قطار مغلقة، من خلال أي تجربة أو قياس فيزيائي، تحديد ما إذا كانت السيارة تتحرك بسرعة ثابتة أو واقفة في حالة سكون."

    إليك طريقة لتحديد: إذا كانت هناك فجوة بين الساعات - يكون النظام في حالة حركة بالنسبة للإشعاع. غير مخلوق - في حالة راحة.

    ووفقًا للنسبة بين عدد "دورات" الساعات في الدقيقة بين الساعة الحرارية والساعة، فمن الممكن أيضًا معرفة مدى سرعة تحركها بالنسبة للإشعاع.

  256. لم يخطر ببالي حتى أن هذه هي الطريقة التي كان يُنظر بها إلى الأمر.
    أردت فقط أن أظهر التناقضات التي أراها.

  257. لكن في الواقع، أنت تسمح لنفسك بالإدلاء بتصريحات حول موضوع ليس لديك أي فكرة عنه. ألم يحن الوقت للتوقف؟

  258. لا، ب، لم نرجع إلى الكرة ولم نرجع إلى المشكلة السابقة، كما أن مركز الكرة ليس نظامًا إحداثيًا

  259. لقد فهمت لتومي أن "الانفجار الكبير" يتحدث عن بداية الكون.
    أي على نقطة هي بداية المكان والزمان.
    أنا أفهم أنني كنت مخطئا وهذا ليس هو الهدف.

    ولكن بعد ذلك فهي كرة.
    لذا فإن الكرة لها نقطة مركزية.
    ونعود للمشكلة السابقة .

  260. وبالمناسبة، في إسرائيل، هناك مجموعة كاملة من الأفكار التي تقدم إجابة لسؤالك حول "الزمن قبل الوقت".

    يقدم بنروز واحدة من أكثرها إثارة للاهتمام في كتابه دورات الزمن.
    الجواب يعتمد على العلاقة بين الكتلة والزمن (من خلال مفهوم الطاقة) - وهي علاقة تعني أنه عندما لا توجد كتلة لا يوجد زمن أيضاً (نفس العلاقة التي تم التعبير عنها عندما ذكرت في رد سابق ذلك الزمن) غير موجود من حيث الفوتونات).
    يتحدث اقتراحه عن سلسلة لا حصر لها من الأكوان، كل منها يرتفع على أنقاض سلفه عندما يفقد الأول كتلته، من خلال جميع أنواع العمليات التأملية، ونتيجة لذلك، يفقد أيضًا الزمان والمكان.

  261. ب:
    ليس صحيحا بأي شكل من الأشكال.
    النقطة ليست نظام إحداثيات
    لا توجد نظرية للانفجار الكبير تفترض أن الكون بدأ عند نقطة ما (على العكس - كل النظريات تبدأ على الأقل زمن بلانك بعد النقطة الافتراضية التي لا يعتقد أنها موجودة).
    تتناسب نظرية الانفجار الأعظم بشكل جيد مع النظرية النسبية، فلا يوجد فيزيائي يعتقد خلاف ذلك (لكن لا يوجد فيزيائي يدلي بمثل هذه التصريحات الجاهلة مثلك).

  262. إذا كان "الانفجار الكبير" موجودا، فهناك نقطة خاصة واحدة في الكون (نقطة بداية الكون).
    هذه النقطة هي نظام مرجعي مختلف عن جميع الأنظمة المرجعية الأخرى.
    وهذا يتناقض مع النظرية النسبية.

  263. إسرائيل:
    وإذا كانت الساعات منتشرة في جميع أنحاء الكون والتي تظهر عمر الكون ومتزامنة في نظام يتحرك نسبة إلى إشعاع الخلفية (ويتحرك معه)، فإن كل ساعة من هذا القبيل سوف تظهر أيضًا في المستقبل عمر الكون في هذا النظام المتحرك ووقت سابق في أي نظام آخر - بما في ذلك النظام الذي يكون في حالة سكون بالنسبة لإشعاع الخلفية بحيث يكون النظام المتحرك الذي اخترته عشوائيًا هو المفضل وفقًا للمبدأ الذي اقترحته.
    ليس هناك تناقض صارخ مع أي شيء.
    يمكننا معرفة ما إذا كنا نتحرك (وحتى مدى السرعة) بالنسبة إلى أي نظام آخر يحتوي على شيء ما (وهو ثابت بالنسبة إليه).
    أنا آسف يا إسرائيل، لقد تمكنت من إرباكي لبعض الوقت، لكن الأمر انتهى.

  264. لقد عدت للتو من المشي في الجبال. عندما يكون هناك كلب، الجبال هي صالة الألعاب الرياضية.

    "إذا قمنا بتشتت الساعات الفضائية التي ستبدو متزامنة في نظام إشعاع الخلفية، فإنها لن تبدو متزامنة في نظام يتحرك بالنسبة لإشعاع الخلفية."

    المشكلة هي أنه وفقا للنظرية النسبية فإن أي ساعة متزامنة لإظهار وقت إشعاع الخلفية (عمر الكون) سوف تظهر في قراءة متأخرة نفس وقت إشعاع الخلفية إذا كانت في حالة سكون بالنسبة لها، أو في وقت سابق إذا كان يتحرك بالنسبة إليه.

    لن يحدث الوضع المعاكس أبدًا، أي أن الساعة العادية ستظهر وقتًا متأخرًا عن إشعاع الخلفية (عكس مفارقة التوأم، التوأم المسافر هو التوأم الأقدم)

    هذه الحقيقة في حد ذاتها تجعل النظام الزمني المتزامن لإشعاع الخلفية نظامًا مفضلاً.

    بصرف النظر عن التناقض الواضح في الافتراض 1 (يمكننا معرفة ما إذا كنا نتحرك أم في حالة سكون وحتى بأي سرعة)، يبدو أنه يمكننا الحصول على موقف حيث إذا استخدمنا ساعات مؤقتة بجوار ساعات السيزيوم في نظام الإشعاع ثم في الأنظمة التي تتحرك بالنسبة للإشعاع سنحصل على الوضع الذي تظهره ساعات السيزيوم في وقت لاحق. في المناقشة الطويلة مع ر. ومع Zvi Z استخدمنا مثالاً كميًا من الرابط:

    http://galileoandeinstein.physics.virginia.edu/lectures/time_dil.html

    ومن الممكن أيضًا إظهار ذلك بشكل نوعي إذا نظرت إلى مقال عوفر ماجد، مجرة ​​درب التبانة في ثلاث ثوانٍ:

    http://ofer-megged.blogspot.com/2011/09/blog-post.html

    يصف المقال مسافرًا يعبر أكثر من مائة ألف سنة ضوئية من مجرة ​​درب التبانة في زمن لا يمثل له سوى 3 ثوانٍ.

    والآن سوف نسأل ماذا سيحدث إذا كان المسافر ساكناً بالنسبة للإشعاع ومرت به مجرة ​​أخرى بسرعة.

    إذا تم تقليل مائة ألف عام إلى 3 ثوانٍ، فبعد 3 أيام، سيتمكن نفس المسافر من رؤية الأحداث التي يزيد وقتها في نظامه عن 15 مليار سنة، وبعد شهر سيكون قادرًا بالفعل على معرفة ما إذا كان الكون يمر أم لا مات من الأصل (شاموليك، "السؤال الأخير" لأسيموف).

    ومع ذلك، فإن الراكب الخفيف نسبيًا للإشعاع هو نحن جميعًا. (ليس تماما، ولكن هذا غير ذي صلة).

    وصف إشكالي إلى حد ما، حيث يمكنك إلقاء نظرة على المستقبل.

  265. الحقيقة هي أنني فخور جدًا.
    لقد قمت بنشر المشكلة بين عدد لا بأس به من خبراء النسبية ولم أتلق إجابة منهم حتى الآن.
    تكمن المشكلة في الموضوع الذي سبق لنا أن تناولناه، وهو مفهوم التزامن في الأنظمة العلائقية المختلفة.
    الملخص القصير للخطأ هو كما يلي:
    إذا قمنا بتناثر الساعات الفضائية التي ستبدو متزامنة في نظام إشعاع الخلفية، فإنها لن تبدو متزامنة في نظام يتحرك بالنسبة لإشعاع الخلفية.
    وينطبق هذا على أي نوع من الساعات، بما في ذلك ساعات درجة الحرارة.
    لذلك، عندما يتحرك العالم بالنسبة إلى إشعاع الخلفية، يضبط ساعته وفقًا لإشعاع الخلفية (حيثما يكون)، فلن يُنظر إلى ذلك على أنه حدث متزامن مع تقدم ساعة العالم الساكن بالنسبة إلى إشعاع الخلفية في مكان آخر ، حتى هذا الوقت.

    بدا الأمر مزعجًا للغاية في البداية لأنني فهمت أنه يمكن تقديم اعتبار مماثل للنظرية النسبية، دون التدخل في الانفجار الأعظم، لكن فكرة المشكلة الأكثر إزعاجًا هي التي غمرتني بالحل.

  266. شموليك.

    لن تساعد المراسلات هنا كثيرًا - يجب عليك الذهاب إلى المراسلات منذ عامين ونصف وما بعده. وحتى ذلك الحين يمكنك بسهولة الوقوع في المشاكل.

    خلاصة القول، التناقض الواضح بين إطالة الزمن فيما يتعلق بنظرية الانفجار الأعظم يمكن تقديمه بهذه الطريقة: إذا كان لكل شخص وقته الخاص كما تدعي النسبية، فكيف نتفق جميعًا أيضًا على عمر الكون الذي هو عمر الكون؟ يمكن قياسها عن طريق درجة الحرارة؟ لماذا لا نكون مثالا للأنظمة التي يبلغ عمر الكون فيها 30 مليار سنة؟

    أقول "تناقض واضح" لأنه من غير المرجح أن تكون نظرية مثالية مثل النسبية التي تعطي مثل هذه التنبؤات المثالية التي تؤكدها الملاحظات بالضبط خاطئة. الأمر نفسه ينطبق على نظرية الانفجار الأعظم، رغم أنني شخصياً لا أفهم ما معنى خلق الكون ومعه الزمن كما تدعي نظرية الانفجار الأعظم. إذن ما الذي جاء أولاً؟ (مع كيتباج، إسرائيل!).

    لكن هذا لا يعني شيئًا لأنه ليس من الواضح بالنسبة لي ما يعنيه الحجم المحدود للكون أيضًا. إذا ما هو التالي؟ (حسنًا، 70 انحناءًا) ولماذا يكون لموقع ما في عالم لا نهائي متجانس ومتناحي الخواص نظام راحة مفضل. لماذا هذا واحد؟ (حسنًا، 30 ثانية للدوران حول المعسكر).

    وأما القصة - فما العار؟ لقد كانت متعة كبيرة في ذلك الوقت.

    وإذا نقلنا القانون الثاني الذي سأل الأستاذ المؤمن:

    "هل يستطيع القدير مضاعفة سرعة الضوء؟"

    كان الأمر لطيفًا، وباختصار، سنعود إلى المنزل، هذا كل شيء.

  267. إسرائيل،
    ربما لا أفهم ما هو التناقض الذي تشير إليه وربما يكون خطأي، لذا سأكون ممتنًا لو كتبت التناقض مرة أخرى وفي نفس الوقت (عندما يكون هناك وقت) سأحاول مرة أخرى قراءة المراسلات السابقة.
    فيما يتعلق بالقصة، فهي رائعة حقًا ومن المؤسف أن تسير الأمور على هذا النحو.

  268. ويوجد بين المستويين في الفضاء محور تناظر.
    المطار على محور التماثل.
    ومن حيث السرعة الخطية بالنسبة للمطار، تكون الطائرات متناظرة.

  269. قياس فروق التوقيت بين الطائرات:
    لنفترض أننا أخرجنا الأرض من الصورة.
    إنه :
    طائرتان تدوران في اتجاهين متعاكسين في مكان ما في الفضاء الفارغ.
    في كل مرة تمر الطائرتان ببعضهما:
    يرى المستوى A الطائرة B تمر بالقرب منه بسرعة معينة.
    يرى المستوى B أن المستوى A يمر بنفس السرعة في الاتجاه المعاكس.

    ومن حيث السرعة المتبادلة (الخطية)، فإن المستويين متماثلان بالنسبة لبعضهما البعض.

    لذلك، لا يمكن أن تعزى اختلافات الساعة إلى اختلافات السرعة.

    المستويان غير متماثلين بالنسبة لسرعة دورانهما حول المركز المشترك للدوائر التي يتحركان فيها.
    وبالتالي:
    يجب أن تكون فروق التوقيت نتيجة لسرعة الدوران وليس السرعة الخطية.

  270. ميخائيل

    وهنا نقطة أخرى للتفكير، وهي في الواقع سؤال:

    جميع التأكيدات على إطالة الزمن والملاحظات الأخرى التي تؤكد تنبؤات النسبية - الميونرات، والطائرات، ومسرعات الجسيمات - تم إجراؤها عندما ينتقل النظام من حالة تكون سرعته فيها 0 تقريبًا بالنسبة لإشعاع الخلفية إلى حالة تكون فيها سرعته صفرًا تقريبًا هي تقريبًا سرعة الضوء بالنسبة للإشعاع.

    بطبيعة الحال، لم يتم إجراء أي تجربة على الإطلاق أو أن هناك نظامًا ينقلب فيه الوضع، أليس كذلك؟

    شموليك

    في الرابط الأول، يتعلق الأمر بعمر الكون والأرض، ولكن ليس بالتناقض المفترض في النسبية. وفي الرابط الثاني يدور الحديث بشكل غير مباشر عن أحد أسباب إطالة الزمن وهو تأثير دوبلر.

    فيما يتعلق بـ "القانون الثاني" فقصته معقدة بعض الشيء. على الرغم من غرابة الأمر، جاءت الفكرة من موضوع لم يُذكر إلا نادرًا في القصة - التكنولوجيا النفسية. ببساطة، المجال الخاص بهذا الاسم مأخوذ بالفعل، لذا قمت بتغييره إلى الميكانيكا النفسية.

    أنا لست مهتمًا حاليًا بمناقشة هذه القضية، ولكن اتضح أن التطبيق الشامل للتكنولوجيا النفسية ربما لديه نفس مشاكل الهوية مثل تلك الخاصة بالإنتروبيا في النظام الديناميكي الحراري.

    كتابة الفصل الأول كانت عفوية وسهلة واستغرقت يومين. بعد تعليقات متعاطفة في غاليليو ومقال لاحق بقلم نير لاهاف، طلب مني المحرر ميكي العازار المزيد من الفصول. لقد عملت بجد أكبر في الفصل الثاني عندما قام المحرر العلمي تسفي أتزمون بتغيير الفصل وتحريره. وأخيراً وصلنا إلى صيغة مقبولة لديه (أقل قليلاً بالنسبة لي، لم تعجبني النهاية) وأيضاً موعد نهائي للنشر، ولكن بعد ذلك تغيرت الإدارة وتقرر عدم النشر.

    بقدر ما يهمني، فهو ينتمي بالفعل إلى الماضي، وأنا ببساطة ليس لدي الرغبة والوقت للتعامل معه. (كما أن هناك مسألة خطيرات بسيطة - عمري 56 سنة..). ولكن إذا كنت مهتمًا، يا صديقي الافتراضي (الذي لم أقابله من قبل...)، فأنا أأذن لك بإصدار الحلقات وجمع جميع الإتاوات. فقط إذا تكرمت بما فيه الكفاية، اذكر أيضًا اسمي ككاتب..

  271. إسرائيل،
    لقد انتهيت من الجزء الثالث (الذي تقدم فيه فكرة الميكانيكا النفسية الرائعة) وما زلت رائعًا جدًا. أتمنى أن تقوموا بترجمتها إلى اللغة الإنجليزية، وبيعها على أمازون كقصة قصيرة وكسب الملايين 🙂
    بخصوص المناقشة، أخبرني إذا كان الرابطان التاليان سيساعدانك بأي شكل من الأشكال:
    http://ncse.com/evolution/science/age-universe-measuring-cosmic-time (خاصة النقطة الثانية)
    أقل أهمية قليلاً ولكن ربما:
    http://www.physicsforums.com/showthread.php?t=39143

    مايكل،
    في جهاز MacBook Air، يستغرق الأمر حوالي 4 ثوانٍ بعد إيقاف التشغيل. ربما حان الوقت للتغيير 😉

  272. إسرائيل:
    مساء أمس، مباشرة بعد أن انتهيت من كتابة التعليق، قمت بالنقر فوق "إضافة تعليق" وقمت بإيقاف تشغيل الكمبيوتر.
    مباشرة بعد إيقاف تشغيله، أدركت أنه يؤدي إلى المشكلة التي وصفتها، ولكن لم تعد لدي القوة لإعادة تشغيل الكمبيوتر.
    يجب أن أفكر في الأمر قليلاً وأرى حاليًا اتجاهين ذوي صلة:
    أحدهما هو أن ساعة درجة الحرارة تعطي النتيجة من حيث وقت المساح، وبالتالي ليس من الضروري أن تعطي الجميع نفس النتيجة (عليك أن ترى كيف يحدث هذا لأنه من الواضح أن مكونات الحساب لا تعتمد على المساح)،
    والثاني هو أن افتراض الخواص لا أساس له من الصحة (نظرًا لأنه من الممكن لكون غير متناحٍ أن يبدو متناحيًا على وجه التحديد لأولئك الذين يتحركون داخله).
    لا أعرف الآن. سأحاول التفكير في الأمر لاحقًا.

  273. 1. ولكن كيف نعرف أنها حدثت في نفس الوقت وفي نفس النظام؟ الحدث أ وقع في إسرائيل، والثاني على المريخ.

    وهنا ما يقوله أينشتاين:

    لم نحدد "زمنًا" مشتركًا لـ A وB، لأنه لا يمكن تعريف الأخير على الإطلاق إلا إذا أثبتنا بالتعريف أن "الزمن" الذي يتطلبه الضوء للانتقال من A إلى B يساوي "الزمن" الذي يحتاجه للانتقال. من B إلى A. دع شعاع الضوء يبدأ عند "الزمن A" $t_{\rm A}$ من A باتجاه B، دعه ينعكس عند "الزمن B" $t_{\rm B}$ عند B في اتجاه A، والوصول مرة أخرى إلى A في "الوقت" $t'_{\rm A}$.

    وفقًا للتعريف، تتم مزامنة الساعتين إذا

    \begin{displaymath}t_{\rm B}-t_{\rm A}=t'_{\rm A}-t_{\rm B}. \النهاية{عرض الرياضيات}
    نحن نفترض أن هذا التعريف للتزامن

    اللصق لا يأتي بشكل جيد، ولكن يمكنك إلقاء نظرة على المصدر. تتم المزامنة من خلال أشعة الضوء. إذا تمت مزامنة الساعتين A وB بطريقة أينشتاين وحدث الحدث A بالقرب من الساعة A عندما تظهر الوقت A - فهو متزامن مع الحدث B الذي حدث بالقرب من الساعة B عندما يظهر الوقت B، إذا كانت الأوقات A وB متطابقة. ولو كانت المسافة بين الحدثين سنة ضوئية. حسب التعريف.

    2. لا مشكلة.

    3. هناك مشكلة وقد واجهناها من قبل. "تظهر الساعة وقتًا متأخرًا عن وقت إشعاع الخلفية عند عرضها في نظامها الخاص ووقتًا أقدم من وقت إشعاع الخلفية إذا تم عرضها في نظام إشعاع الخلفية."

    وفقا لك، وصححني إذا كنت مخطئا، إذا تحركت ساعة في النظام A في النظام B، فعندما تمر الساعات على بعضها البعض سيرى كل منهما وقت الآخر كما هو سابق، وهذا لأن كل جانب يرى الآخر أبطأ .

    ويترتب على ذلك أنه عندما تمر سفينتان فضائيتان ببعضهما البعض وتدعي كل منهما أن الوقت في الثانية يتحرك بشكل أبطأ، فإذا صورتا بعضهما البعض بكاميرا عالية الدقة ستسجل الساعتين في السفينتين الفضائيتين A وB، فإن الصور الفوتوغرافية من كل منهما ستظهر سفينة الفضاء العكس: الصورة من سفينة الفضاء A ستظهر الوقت 10 صباحًا والوقت 8 صباحًا، والصورة من المركبة الفضائية B ستعرض الوقت 10 صباحًا والوقت 8 صباحًا.

    هل هذا ممكن جسديا؟ وإذا لم يكن الأمر كذلك، فماذا يعني أن «كل طرف يرى الآخر يستسلم بشكل أبطأ»؟

    وهذه النقطة حاسمة في المناقشة، كما سنرى لاحقا.

    "تظهر الساعة وقتًا متأخرًا عن وقت إشعاع الخلفية عند النظر إليها في نظامها الخاص"

    كيف؟ زمن إشعاع الخلفية هو عمر الكون. فإذا أظهرت الساعة وقتا متأخرا، مثلا 20 مليار سنة، فقد مر في نظامه أكثر من 6 مليارات سنة من عمر الكون! بالنسبة لنا، الذين نشعر بالارتياح بالنسبة للإشعاع، فإن مشاهدة وتصوير هذا النظام الذي يمر بسرعة أمامنا، يفتح نافذة لإلقاء نظرة على المستقبل، أليس كذلك؟

  274. 1. يعتمد التزامن على نظام مرجعي ويشير إلى وقوع حدثين في نفس الوقت وفي نفس النظام.
    على سبيل المثال، تقوم الأقمار الصناعية لنظام تحديد المواقع العالمي (GPS) بالإرسال في وقت واحد بالنسبة إلى الأرض ولكنها لا ترسل في وقت واحد في نظام إشعاع الخلفية.
    2. لن يصبح التوأم المسافر أكبر سناً إذا كان هو الوحيد الذي يتسارع. إذا تم تسريع كلاهما - فكل شيء ممكن.
    3. لا أعرف ماذا يفعل راكبان في قصة الخط المثقوب. على أية حال، من المستحيل مزامنتهما وحالة أحدهما بالنسبة لإشعاع الخلفية لا تعتمد على حالة الآخر.
    ومع ذلك - في أي لحظة معينة (وليس فقط أثناء اجتماعهم - بافتراض أنه في مرحلة ما قام كل منهم بضبط ساعته لإظهار وقت إشعاع الخلفية بالضبط) تظهر الساعة وقتًا متأخرًا عن وقت إشعاع الخلفية عند النظر إليها في نظامه الخاص وبوقت سابق لوقت إشعاع الخلفية إذا تم النظر إليه في نظام إشعاع الخلفية.

  275. شموليك

    ما كتبته هو أنه في العام الذي نُشرت فيه النظرية النسبية الخاصة - 1905 - كان الرأي السائد هو أن الكون أبدي. لذلك، لا يوجد معنى لبداية الوقت والطريقة الوحيدة هي التحديد التعسفي لنقطة الصفر ومزامنة الساعات.

    والأمر بالطبع يختلف إذا كان هناك بداية للزمن. لذا فإن النقطة 0 طبيعية. هذا هو الرأي الشائع اليوم - بدأ الكون منذ حوالي 13.7 مليار سنة، وقبل ذلك لم يكن هناك وقت. (لا تأتوا إلى إسرائيل بأسئلة مثل "ما الذي كان أولاً؟" الجواب سيكون حتماً: مع حقيبة أدوات!).

    ميخائيل.

    وكان القصد هو التناقض بين العلاقة الخاصة فقط بالانفجار.

    بداية أشكرك على وقتك ومجهودك. أتمنى أن توضح لي النقطة وتخرج السخرية من رأسي. ولكن قبل أن يحدث ذلك، لدي فقط أسئلة، والكثير منها. لم يحدث ذلك بعد.

    1. أنا لا أبحث عن تعريف للتزامن الشامل. فقط من أجل التزامن.

    2. "التزامن في نظام إشعاع الخلفية هو التزامن الحقيقي والمشكلة هي أنه لا يوجد شيء في هذا التزامن أكثر واقعية من أي تزامن آخر".

    "السؤال ليس له معنى مطلق، بل معنى نسبي فقط".

    "لدي شعور بأنك تهدف إلى الحصول على سطر من شأنه أن يقول أنه نظرًا لأنه لا يمكنك سوى إبطاء الساعة، فإن نظام إشعاع الخلفية هو النظام الذي تكون فيه الساعة هي الأسرع، وليس هو.
    تخيل توأمان موجودان في سفينة فضائية في حركة مستمرة بالنسبة لنظام إشعاع الخلفية.
    تخيل الآن أن أحدهم يترك سفينة الفضاء في سفينة فضاء تابعة له ويقوم بتسريعها حتى تصل إلى حالة السكون بالنسبة إلى إشعاع الخلفية، ويبقى في حالة سكون لمدة ساعة على هذه الحالة ثم يعود ويسرعها مرة أخرى للحاق بسفينة الفضاء التي قام بها غادر.
    وعندما يعود، فإن الشخص الذي بقي في سفينة الفضاء سيكون أكبر منه سنا."

    نظرًا لأنك فهمت على الفور إلى أين كنت ذاهبًا (كنت أعلم أنني أستطيع الوثوق بك) - ولكن ليس تمامًا - فإليك سؤال:

    هل يمكن أن يكون هناك موقف يكون فيه التوأم المسافر هو الأكبر سناً؟ (بالمناسبة، وفقًا لمقالة كتبها جالي قبل بضع سنوات - الإجابة هي نعم. ولكن فقط في النظام الدوار).

    وبما أنه لا يوجد مجال للدراما والتوتر في المناقشة، فإليك هذه الجملة:

    إذا قبلت الافتراض بأن: الساعات الحرارية كما قدمتها تظهر دائمًا نفس الوقت عندما تمر ببعضها البعض في أنظمة القصور الذاتي مع تصوير مشترك من كلا الجانبين - يمكننا أن نظهر أن الساعات العادية (مثل تلك التي تحمل توائم معها) تظهر زمن أعلى من زمن إشعاع الخلفية، أي عمر متأخر عن عمر الكون.

    وهذا ممكن رياضيا، ولكن ليس فيزيائيا. يوافق؟

  276. إسرائيل،
    إذا فهمت ادعائك بشكل صحيح، فأنا أجيب وأقول إن التناقض الذي تشير إليه بين النسبية الخاصة والانفجار الكبير لا معنى له. النظرية النسبية الخاصة تضع افتراضات معينة لا وجود لها في الواقع وبسبب هذه الافتراضات يوجد التناقض، لكن النظرية النسبية الخاصة بسبب افتراضاتها غير الواقعية ليست هي النظرية التي يطرح فيها سؤال بداية الكون ينبغي مناقشة الكون في المقام الأول، ولكن فقط من خلال النظرية النسبية العامة.

  277. إسرائيل:
    ليس لدي تعريفي الخاص للتزامن الكلي.
    في الواقع، أظهرت النظرية النسبية أن التزامن المقاس نسبي ويعتمد على الراصد.
    أنت تحاول إحياء مفهوم التزامن الشامل بقولك أن التزامن في نظام إشعاع الخلفية هو التزامن الحقيقي والمشكلة هي أنه لا يوجد شيء في هذا التزامن أكثر واقعية من أي تزامن آخر وبالتالي لا يمكن استخدام هذا التزامن من جانبنا مثلاً لبناء نظام تحديد المواقع العالمي (GPS) على الأرض لأن هذا النوع من الأنظمة يتطلب تزامن الأرض.
    لا أعرف ما الذي تعرفه بأنه "نظام متزامن من حيث الزمن" فمن الواضح، على سبيل المثال، أن الأحداث التي تبدو متزامنة في نظام إشعاع الخلفية لا تبدو متزامنة في نظام آخر وعلى النقيض من ذلك - حسب التعريف - الأحداث التي تبدو متزامنة في نظام إشعاع الخلفية تتم مزامنتها في نظام إشعاع الخلفية.
    يمكن أن تكون الساعات متأخرة أو متقدمة عن الأنظمة النسبية الأخرى. السؤال ليس له معنى مطلق بل معنى نسبي فقط.
    ومن الواضح أن ساعة التوأم المسنة تتسارع بالفعل مقارنة بساعة التوأم الذي لا يزال صغيرا.
    لدي شعور بأنك تهدف إلى الحصول على سطر من شأنه أن يقول أنه نظرًا لأنه لا يمكنك سوى إبطاء الساعة، فإن نظام إشعاع الخلفية هو النظام الذي تكون فيه الساعة هي الأسرع وليس هو.
    تخيل توأمان موجودان في سفينة فضائية في حركة مستمرة بالنسبة لنظام إشعاع الخلفية.
    تخيل الآن أن أحدهم يترك سفينة الفضاء في سفينة فضاء تابعة له ويقوم بتسريعها حتى تصل إلى حالة السكون بالنسبة إلى إشعاع الخلفية، ويبقى في حالة سكون لمدة ساعة على هذه الحالة ثم يعود ويسرعها مرة أخرى للحاق بسفينة الفضاء التي قام بها غادر.
    وعندما يعود، سيكون الشخص الذي بقي في سفينة الفضاء أكبر منه سنًا.
    كلامك لشموليك غير صحيح.
    في الواقع، أدخل أينشتاين الثابت الكوني في المعادلات على وجه التحديد لأنه بدونه كان سيضمن حدوث انفجار كبير أو انهيار كبير، وهذا لم يتوافق مع إيمانه بالكون الساكن.
    مع الثابت الكوني، من الممكن بالفعل أن يكون لدينا كون يحتوي على انفجار كبير وطاقة مظلمة.

  278. ميخائيل

    ربما فاتني شيء.

    هل يمكن أن تخبرني ما هو تعريف التزامن؟

    وكيف يختلف عن تعريفي؟ أو آينشتاين؟

    اسئلة اخرى:

    هل تقبل أن نظام إشعاع الخلفية هو نظام متزامن مع الزمن؟

    هل تقبل أنه وفقًا لأينشتاين ونتائج التجارب المختلفة، فإن إطالة الوقت تكون أحادية الاتجاه - أي أن الساعات المتحركة يمكن أن تتأخر ولكنها لا تتسارع؟

    شموليك

    المقطع الذي أحضرته صحيح وجميل، لكنني لست متأكدًا من أنني أفهم سؤالك. أنا لا أدعي وجود تناقض داخل النسبية نفسها. التناقض هو مع نظرية لاحقة - الانفجار الكبير والكون المتوسع.

  279. إسرائيل،
    إسرائيل،
    آسف على التعقيد، لكن بعد ما كتبته أسأل: هل النسبية الخاصة ليست إلا حالة خاصة من النسبية العامة لأنها وكذا، وتقوم بافتراضات لا وجود لها في الواقع ثم تصل من خلالها إلى التناقض؟
    أستمد السؤال من الفقرة التالية من مدخل ويكيبيديا: http://en.wikipedia.org/wiki/Special_relativity

    يُستخدم المصطلح حاليًا بشكل عام للإشارة إلى أي حالة لا تكون فيها الجاذبية مهمة. النسبية العامة هي تعميم النسبية الخاصة لتشمل الجاذبية. في النسبية العامة، يتم وصف الجاذبية باستخدام الهندسة غير الإقليدية، بحيث يتم تمثيل تأثيرات الجاذبية من خلال انحناء الزمكان؛ تقتصر النسبية الخاصة على الزمكان المسطح. كما أن انحناء سطح الأرض غير ملحوظ في الحياة اليومية، يمكن إهمال انحناء الزمكان على نطاقات صغيرة، بحيث تكون النسبية الخاصة محليًا بمثابة تقريب صحيح للنسبية العامة.[8] يصبح وجود الجاذبية غير قابل للاكتشاف في مختبر صغير بما فيه الكفاية وذو سقوط حر.

  280. إسرائيل:
    أنت فقط تستمر في خلط الجنس مع غير الجنس.
    لقد قمت الآن بإعادة تعريف مصطلح "التزامن".
    لم يعد الأمر يتعلق بالأشياء التي تحدث لك في نفس الوقت، بل يتعلق بالأشياء التي تحدث في نفس الوقت في نظام إشعاع الخلفية.
    هذه بالطبع نتيجة ضرورية لما قلناه من قبل - حقيقة أنك اخترت مثل هذه الساعة المرجعية لنظام إشعاع الخلفية.
    بالطبع، هذا التزامن ليس له أي معنى فيزيائي، وما ستحققه به هو بالضبط ما ستحققه (وبالفعل ستحققه الأقمار الصناعية لنظام تحديد المواقع العالمي) إذا قررت أن التزامن يتحدد بواسطة ساعة الأرض.
    بالمناسبة - إذا قمت بمزامنة الأقمار الصناعية لنظام تحديد المواقع العالمي (GPS) وفقًا لنظام إشعاع الخلفية، فسوف تتوقف عن العمل.

  281. ميخائيل.

    دعونا نذهب إلى حيث بدأ كل شيء، ورقة أينشتاين الأصلية عن النسبية:

    http://www.fourmilab.ch/etexts/einstein/specrel/www/

    يتناول الفصل الأول ماهية التزامن وكيف سنقوم بمزامنة الساعات البعيدة. يقول أينشتاين:

    الآن يجب أن نضع في اعتبارنا بعناية أن الوصف الرياضي من هذا النوع ليس له معنى مادي ما لم نكن واضحين تمامًا بشأن ما نفهمه من خلال "الزمن".

    وبعد تعريف "الوقت" (العقرب الصغير عند الرقم 7، والعقرب الكبير عند الرقم 12)، يتابع:

    ولم يعد مرضيًا عندما يتعين علينا ربط سلاسل زمنية من الأحداث التي تحدث في أماكن مختلفة، أو – ما يصل إلى نفس الشيء – تقييم أوقات الأحداث التي تحدث في أماكن بعيدة عن الساعة.

    شرعي، أليس كذلك؟ أنت في الفضاء في مواجهة المريخ، ساعتك تظهر 7، لكن من يؤكد لك أنك متزامن مع صديقك على الجانب الآخر من الكوكب؟ كيف، على سبيل المثال، يمكنك مزامنة هجوم على المريخ إذا لم يكن لديك أجهزة كمبيوتر تقوم بالحسابات على طول الطريق من الأرض؟ وماذا لو كان صديقك كائنًا فضائيًا من كوكب آخر؟ كيف سيتم مزامنة الساعات بعد ذلك؟

    إذن هذا هو اقتراح أينشتاين حول كيفية مزامنة الساعات:

    يمكننا بالطبع أن نكتفي بقيم زمنية يحددها مراقب يتمركز مع الساعة عند أصل الإحداثيات، وينسق المواضع المقابلة للعقارب مع الإشارات الضوئية،

    اتصل بصديقك باستخدام أشعة الضوء، وقم بمزامنة الساعات باستخدامها.

    ومن هنا يواصل وصف التزامن والطبيعة الإشكالية لمزامنة الساعات في الحركة النسبية. ولكن لدينا الميزة التي لم يتمتع بها أينشتاين في عام 1905: حيث يستطيع راصدان عن بعد قياس درجة حرارة الإشعاع، وتحويله إلى وقت مطلق باستخدام صيغة فريدمان، وتوفير صداع أشعة الضوء. يمكنهم التأكد من أن ساعاتهم متزامنة تمامًا كما هو الحال مع طريقة مزامنة شعاع الضوء.

    في الواقع، هذا هو المكان الذي يمكن أن تنتهي فيه المناقشة. وعلى النقيض من ادعاء أينشتاين في عام 1905، إذا كانت نظرية الانفجار الأعظم صحيحة، فإن كل نقطة في الكون لديها "زمن طبيعي" متزامن مع كل نقطة أخرى. لا حاجة لأشعة الضوء أو أشعة الثور. مجرد مقياس حرارة، وسيتم شن هجوم متزامن على المريخ.

    هذه هي حجتي. ومن الممكن أيضًا أن نبين - مع الكثير من التلاعب الرياضي - أننا إذا استخدمنا "ساعات درجة الحرارة" المرتبطة بالساعات العادية، فسوف نحصل على تناقض مع إطالة الوقت. لكن ادعائي الأساسي هو أن الوقت المطلق موجود حتى بدون مزامنة الساعة.

  282. بالمناسبة إسرائيل:
    نفس الاتفاق على ساعة النظام المرجعي المحدد للأرض (الذي أشرت إليه في ردي السابق) هو بالضبط اتفاق الأقمار الصناعية لنظام تحديد المواقع العالمي (GPS).
    فهم لا يرسلون إشاراتهم حسب الوقت الذي يقيسونه معهم، بل حسب الوقت الذي يحسبونه يجب أن يسود على الأرض.

  283. إسرائيل:
    أنا لا أفهم إلى أين أنت ذاهب.
    يمكن استخدام لحظة الانفجار كنقطة صفر طبيعية ويمكن استخدام إشعاع الخلفية كنظام استرخاء طبيعي.
    ماذا عن ذلك وعن النظرية النسبية؟ لا شئ!
    من أجل مزامنة الساعات، ليس من الضروري الاتصال المستمر.
    إذا تركت الأرض وعرفت مسارها، وبالإضافة إلى ذلك، كنت على دراية بحركاتك في الفضاء - فيمكنك حساب الوقت على الأرض في أي لحظة.
    وبطبيعة الحال، تصبح حساباتك أقل دقة مع مرور الوقت - سواء بسبب عدم دقة معرفتك عن الأرض أو بسبب عدم دقة معرفتك عن حركتك، ولكن لا يوجد عائق مبدئي أمام قرار جميع الناس من العالم. الأرض لتحديد أن الوقت الكوني هو وقت الأرض.
    تتمثل ميزة نظام إشعاع الخلفية في أنك على اتصال دائم وبالتالي (مرة أخرى، من حيث المبدأ) من الممكن التوصل إلى اتفاق بشأن هذا النظام المرجعي بسهولة أكبر حتى لو ظهرت نجوم أخرى.
    بالطبع، كل هذا لا يغير حقيقة أنك إذا غادرت الأرض، فسوف تتحرك بسرعة عبر الفضاء لمدة ساعة (حسب قياس ساعاتك العادية) وتعود وتلتقي بتوأمك - سيبدو كرجل عجوز وأنت سوف تبدو كما فعلت عندما غادرت.
    وذلك لأن جسمك وساعتك وكل شيء آخر من حولك لا ينظر إلى ساعة درجة الحرارة الموجودة في سفينتك الفضائية ويتحرك بسرعة جنونية على الإطلاق.

    لا أعرف حتى ما الذي تتحدث عنه عام 1905 وكيف ينتمي هذا العام إلى الحقائق المادية.
    اعتقد أينشتاين أن الكون مستقر لأن هذا ما قيل له ولم يكلف نفسه عناء قياسه بنفسه.
    لم تتوافق نظريته مع الحالة المستقرة وقدم الثابت الكوني للسماح لها بتحمل مثل هذه الحالة (اتضح لاحقًا أن هذا الثابت لا يلبي توقعات أينشتاين ولا يمكنه تثبيت تسارع الكون، لكنه يستطيع، على ومن ناحية أخرى، المساعدة في فك رموز الطاقة المظلمة ولكن لا شيء من هذا مهم)

    وعلى أية حال - فلا فائدة من إدراج عام 1905 في أي معادلة تصف قوانين الفيزياء

  284. ميخائيل.

    إذا فهمت مقالة أينشتاين من عام 1905 بشكل صحيح، فمن غير الممكن مزامنة الساعات في الفضاء إلا إذا قمت بإنشاء اتصال بينهما. النسبية كلها تنبع من هذا الاتصال الذي يتم عن طريق الأشعة الضوئية.

    1. هل فهمت هذه النقطة؟

    2. هل يمكنك الإشارة إلى طريقة مزامنة الساعات في نقاط مختلفة وبعيدة في الفضاء دون الاتصال بينها حسب المعرفة عام 1905؟

    3. هل يمكنك القيام بذلك بناءً على المعرفة الحالية؟

    4. هل للنقطة 0 الطبيعية للوقت في الكون اللانهائي أي أهمية كانت وستظل كذلك دائمًا؟

    5. في الكون الذي خلقه الانفجار الكبير، أليست لحظة الانفجار هي نقطة الصفر الطبيعية؟

  285. إسرائيل:
    لكنني شرحت بالضبط ما هو "مطلق" وقت الانفجار وأظهرت أن هذه المرة مختلفة (بمعنى الإجابة على السؤال "منذ كم سنة حدث ذلك) في أنظمة مرجعية مختلفة.
    لقد أظهرت أيضًا أنه يتطابق تمامًا (!) مع مقياس درجة الحرارة الذي وصفته.
    وفيما يتعلق بمثال مادي لموجة بدون وسط حامل على الإطلاق، فإن سنة اكتشاف ذلك المثال ليست مهمة. بعد كل شيء، أنت لا تتعامل مع استعادة اسم ماكسويل (الاسم الذي لم يتضرر على الإطلاق) ولكن مع سؤال مادي.
    كما قلت - في تعليقاتك هنا لم تتحدث عن ساعات درجة الحرارة. ما فعلته هو افتراض انخفاض رتيب في درجة الحرارة، وهو كما ذكرنا خطأ.
    فيما يتعلق بساعة درجة الحرارة - فلا توجد مشكلة فيها على الإطلاق.
    في نظامك المتحرك، يمكنك - إذا كان لديك معلومات كافية - حساب الوقت في أي نظام مرجعي آخر.
    تعتمد ساعة درجة الحرارة على حقيقة أنه في نظام الراحة الطبيعي للكون (النظام الذي يكون فيه الإشعاع متناحيًا) لديك معلومات كافية ويمكنك حساب وقته.
    لا يفرض أي قيود على وقتك.

  286. شموليك.

    على الرغم من كل التطورات والنظريات الجديدة، فإن الافتراض اليوم هو أن النسبية الخاصة صحيحة تمامًا كما كانت في عام 1905. ولا يوجد دليل واحد ينقضه، بل هناك أدلة كثيرة تؤكده.

    فيما يتعلق بنظرية الانفجار، فإن الاتفاق ليس شاملاً للغاية، لكنه لا يزال النظرية الرائدة في الاتجاه السائد.

    أحاول أن أبين أن هناك تناقضًا بين الزمن المطلق للانفجار والزمن النسبي.

    مايكل - آسف على الإطالة. تعلم كيفية تحمير الكلمات وإبرازها وسأوضح لك القشة من القشر في كل رد.

    ב

    "لا يهم كيف يتم قياسه. تقنية القياس ليست مهمة على الإطلاق.
    ساعة B متأخرة عن ساعة A.
    ساعة C تتأخر عن ساعة B."

    غير صحيح! ما هو واضح مثل اليوم هو:

    لا يهم كيف يتم قياسه. تقنية القياس ليست مهمة على الإطلاق.
    الساعة "أ" تتأخر عن الساعة "أ".
    ساعة C تتأخر عن ساعة C.

    ساعة B فقط هي التي تزيد سرعتها مقارنة بساعة B.

    الدليل - تمامًا مثل الدليل على ادعائك.

  287. إسرائيل،
    شكرا على المتابعة، سأقرأها قريبا!
    سؤال سهل: لماذا يهمك ما قاله أينشتاين عام 1905، بعد أن مرت سنوات عديدة منذ ذلك الحين وتم تطوير المزيد من النظريات؟ وفقا لأحد حلول النظرية النسبية العامة، وهو حل السادة FLRW، فإن الكون يتوسع وإذا عدنا بالزمن إلى الوراء، نحصل على الانفجار الكبير، وبالتالي فإن الكون ليس أبديا وفقا لأينشتاين المتجدد.
    http://en.wikipedia.org/wiki/Friedmann%E2%80%93Lema%C3%AEtre%E2%80%93Robertson%E2%80%93Walker_metric

  288. إسرائيل:
    في نظام أ:
    لا يهم كيف يتم قياسه. تقنية القياس ليست مهمة على الإطلاق.
    ساعة B متأخرة عن ساعة A.
    ساعة C تتأخر عن ساعة B.

  289. إسرائيل:
    كما قلت، سيستغرق الأمر بعض الوقت قبل أن أتمكن من الرد عليك.
    أنت تكتب مثل هذه اللفائف بحيث يكون من الصعب حقًا فصل الشريط عن القشر.

  290. المعجزات:
    ما طلبته كان موجهًا إليك - وأنا متأكد من أنه كان موجهًا إليك.
    شاهد استمرار مغامرات B.

    ב:
    أنت لم تظهر أي شيء.
    أنت تدلي بتصريحات لا أساس لها من الصحة وأنا أتوقف عن الجدال معك.

  291. شموليك، شكرا على الدعم. توجد ترجمة انجليزية للفصل الأول. هنا تتمة.

    "مم..ماذا، ماذا تقصد؟ ماذا تقصد ماذا فعلنا هذه المرة؟" وكان الأستاذ أعمى ومربكا.
    "أنتم أيها العلماء، علماء الرياضيات، الفيزيائيون..."
    "نحن؟!" صاح الأستاذ بغضب ساعده على استعادة رباطة جأشه. "لا تحاول عكس الخلق! إذا كنت تعتقد أنه يمكنك إحداث فوضى في المحكمة كما تفعل في الحياة، فسوف تواجه مفاجأة مريرة! أنا لست بولتزمان طيب القلب، والرياضيات، على عكس الفيزياء، ليست علمًا تجريبيًا وثيقًا، ولكنها حرفة فكرية مثالية تمامًا ترتكز على أسس نظرية متينة لا يمكن تحديها!

    "حقا" قال القانون بتشكك.

    "حقًا!" تاج البروفيسور، كله حقيقي في المعركة. "نحن لسنا مجموعة من المراقبين مثل الفيزيائيين، الذين يجمعون ويجمعون أكبر عدد ممكن من الملاحظات ويحاولون دمجها في نوع من النظرية الهجينة التي من شأنها أن توحد بشكل فضفاض جميع البيانات المتناقضة في بعض الصيغة المشكوك فيها المستعارة من الرياضيات، وهي صيغة سيتغير ذلك على الفور عندما تحطم البيانات الجديدة النظرية السابقة لصالح بعض الفزاعات الجديدة سريعة الزوال" توقف الأستاذ لالتقاط أنفاسه وألقى نظرة خاطفة على الجمهور، محاولًا قياس الانطباع الذي تركته كلماته الصاخبة.

    "أوه، أرى أنك لا تكن احترامًا كبيرًا لمجال الفيزياء" اتهم القانون.

    "ولماذا سأهتم؟ كل يومين لديهم أسطورة جديدة. لقد أخبرتنا أجيال عديدة أن الأرض هي مركز الكون وقدمت مخططات وجداول معقدة لتفسير حركة الأجرام السماوية حولها، كل ذلك حتى يصبح الواقع تابعا لنظرية الصرير. ثم، عندما زادت التوترات، اخترعوا قصة جديدة. هذه المرة وضعوا الشمس في المركز، واستدعوا بطلًا جديدًا، وأطلقوا عليه اسم "قوة الجاذبية"، وأخبروا الأطفال أن هذه القوة تجذب جميع الأجسام لبعضها البعض دون أي وسيط بينهما. وما زالوا يبحثون عن الجرافيتون الذي قد يساعدهم على إقناع أنفسهم. في أحد الأيام، أصبح لديهم موقع على شبكة الإنترنت، وفي اليوم التالي أصبح لديهم "زمكان" منحني. نجوم التوراة هما "النسبية" و"ميكانيكا الكم"، لكن كلاهما في صراع ولا يتحدثان مع بعضهما البعض.

    "ومعكم هناك في مملكة الرياضيات الوضع أفضل؟" أثارت القانون.

    "معنا" نفخ الأستاذ صدره بفخر، "كل شيء هادئ ومستقر كالعادة، ممل حتى بسبب الاستقرار. نحن علم خالص تماما، ولا توجد ملاحظات جديدة يمكن أن تحجب الكمال النظري الذي لم يتغير إلا قليلا منذ أن كتب إقليدس "الأساسيات" في القرن الثالث قبل الميلاد. وبما أن كل شيء مبني على بديهيات بسيطة ومفهومة ونظريات مثبتة يزداد تعقيدها، يمكننا أن نبدأ بـ "بين نقطتين يمر خط مستقيم واحد فقط، وهو أيضًا أقصر مسافة بينهما" وننتهي ببنية معقدة مثل الهندسة التحليلية أو ظاهرة الأعداد المثالية."
    وصفق الجمهور.

    "هذا هو بالضبط الفرق بيننا وبين الفيزياء"، أشار البروفيسور بأنفه إلى آل. "الرياضيات كاملة إلى ما لا نهاية، كما أن الله تعالى كامل إلى ما لا نهاية، في حين أن الفيزياء ليست أكثر من مجموعة من التقديرات التقريبية. ولا عجب أن يتم اختيارك أنت نفسك، سلف الفوضى، لتكون القانون الأساسي والأكثر تمثيلاً للفيزياء، وكأنك تؤكد فرضية أن هناك حدًا أعلى للحقيقة العلمية التي يمكن تحقيقها بالفيزياء.

    "وهناك أيضًا حد أعلى لكمية الهراء التي تستطيع الأذن هضمها في فترة زمنية معينة" تمتم القانون.

    "هل تمتمت بشيء؟" نبح الأستاذ.

    "لا شيء، لا شيء"، ابتسم القانون بإطراء. "جميعنا هنا نستمتع بلآلئ لسانك يا بروفيسور ليبنوفتز." وأشار إلى سطر الأرقام الذي رسمه الأستاذ على السبورة. "أخبرني من فضلك: كم عدد الأعداد الخاصة والكاملة الموجودة بين سالب ما لا نهاية وما لا نهاية؟"

    "لانهائي!" أجاب الأستاذ بحزم.

    "وكم عدد الأرقام التي ليس لها أي تفرد؟"

    "لانهائي أيضًا" أجاب الأستاذ بصوت ضعيف.

    "وأي لانهاية أعظم؟"

    "أي نوع من الهراء الذي تتحدث عنه يا لو" تدخل المدعي العام. "ماذا تعني اللانهاية الأكبر؟ اللانهاية هي اللانهاية، أليس كذلك يا أستاذ؟"
    وصفق الجمهور، لكن الأستاذ دفن وجهه في الأرض.

    "المدعي العام يتوقع إجابة منك" وبخ القانون.

    رفع الأستاذ وجهه المحمر وتمتم "إن اللانهاية من الأعداد التي لا معنى لها أعظم.."
    "شكرًا لك على فتح قلبك. كم أكبر؟"

    "لانهائي..."

    ومن بين كل قوانين الرياضيات العظيمة والرائعة، من بين كل الأعداد الأولية، جميلة، مثالية، روميو وجولييت، جميلة وممتعة، هل سمعت من قبل عن الرقم 1995، الرقم المعروف أيضًا باسمه: "راندو"؟

    أصبح الأستاذ شاحبا. "أنت .. هل تعرف راندو؟" تأتأة

    "المعارف يعرف أيضا" أجاب القانون. في الواقع، لقد تكبدت عناء مطالبة ماراندو بالإدلاء بشهادته أمام النيابة. إنه هنا معي" أخرج آلة حاسبة للجيب وكتب عليها بضعة أرقام. "سيكون الأمر صعبًا بعض الشيء لأن لغة راندو الأصلية ثنائية، لكنني أعتقد أننا سنتدبر الأمر. أليس كذلك يا راندو؟"
    "بالتأكيد" أجاب راندو.

    "لا تخبره بأي شيء!" صرخ الأستاذ بالإحباط.

    "احتفظ لنفسك" قال القانون للأستاذ في توبيخ. "صدقني، هذا يؤلمني أكثر منك. مما لا شك فيه، ليس من الممتع بشكل خاص أن نكشف في بارهاسيا عن واحدة من أكثر حلقات مملكة الرياضيات غموضًا... وربما الفصل الأكثر قتامة... تمرد الأعداد البسيطة تحت القيادة الشجاعة لراندو، الرقم المتواضع والعشوائي من جميع الأرقام. بالطبع، سيكون الأمر أسهل بكثير إذا اعترفتم، أيها الرياضيون، بأنكم اتهمتموني عبثًا، وكشفتم الحقيقة الكاملة حول الأسس المهتزة التي بني عليها "الكمال" الرياضي، واعترفتم بأنكم أتقنتم الأرقام المميزة وأنكم استعبدتموها. إخوانهم الصالحين، جثوا على ركبكم، واستغفروا لي، ودعوني أكتشف الجوانب المتسامحة من الطبيعة الرحيمة. إذا لم يكن الأمر كذلك، فسوف أضطر إلى صنع هوكا وسأحرم من فكرة كمال الرياضيات برمتها، وهو الأمر الذي، كما ذكرت سابقًا، لا أحبه على الإطلاق.

    أحس الأستاذ بينما ابتلع الجمهور لعابهم.

    "حسنًا!" وحث القانون. "لا يمكننا أن ننتظرك ليوم كامل. لا يزال أمامنا الكثير من العمل!"

    رفع الأستاذ ذقنه متحديا، ليثبت أننا لن نستسلم للضغوط.

    والقانون الثاني موجه للجمهور: "أيها السادة، هل هناك من يرغب في كشف الحقيقة؟"

    "أنا" أجاب صوت شاب من الحشد.

    "شكرا لك" قال القانون بارتياح. "من أنت أيها الشاب وما اسمك؟"

    "جداليا" أجاب الرجل. "وأنا طالب رياضيات. أعرف جيدًا قصة راندو، أو باسمه الكامل: ران دوهوا".

    "أخبرني يا جدل الرب"، خاطب القانون الشاب بنبرة أبوية. "هل الرياضيات مثالية حقًا؟"

    "لا تجرؤ على إبلاغ!" صرخ الأستاذ.

    لكن الرجل اقترب من المنصة واستمر في الحديث. "استسلم يا لايبنوفيتش. إن إخفاء الحقيقة ليس طريقًا يختاره عالم الرياضيات الحقيقي. القانون صحيح. لقد تم بناء الرياضيات، مثل الفيزياء، طبقة بعد طبقة من خلال التجربة والخطأ، وكانت ولا تزال عبارة عن مجموعة كبيرة من الرقع، والعديد من نظرياتها الأساسية لا يمكن إثباتها بعمق على الإطلاق.

    "أنت تقول لي؟" كان صوت راندو أجوفًا ورقميًا بعض الشيء. "تعلمت هذا على الطريقة الصعبة. وإلى أن جاء علماء الرياضيات بكل أرقامهم المميزة، كنا جميعا أصدقاء ولدينا حقوق متساوية، كما قال كرونكر، عالم الرياضيات في القرن التاسع عشر: "لقد خلق الله الأعداد الكاملة، وكل شيء آخر هو من عمل الإنسان". فجأة، تم إنشاء الطبقات، طبقة أرستقراطية، لم يُسمح للأعداد الشابة باللعب مع أصدقائهم القدامى ولكن دون إشارة... وصل الأمر إلى ذروته في حفل تكريم لميلاد الصغير وقبوله كعضو كامل في الأرستقراطية الرياضيات.

    "من هو لطيف قليلا أنا؟" "سأل القانون حالمة.

    "جذر 1-." أجاب جدليا. وجاء ليعطي حلاً لمعادلة تربيعية من النوع X²+1=0. لا يوجد رقم موجب أو سالب يمكنه حل المعادلة، لذلك تم اختراع هذا الرقم. في البداية تم التعامل معه بتشكك كبير، وحتى ديكارت الكبير شكك في وجوده وأطلق عليه ساخرًا "عددًا وهميًا" أو وهميًا، ومن هنا جاء اسم "i".
    سواء كانت موجودة أم لا، فإن الأرقام التخيلية مفيدة للغاية في مجالات مختلفة، وخاصة في الهندسة الكهربائية. بالنسبة للرياضيات، تعتبر الأرقام المحاكاة ذات قيمة خاصة، لأنها تسمح بوصف الوظائف الهندسية بوسائل جبرية خالصة.
    وفي كلمته، رسم جدليا على السبورة محورًا متعامدًا مع محور الأعداد الذي رسمه ليبنوفيتس سابقًا. "نرى؟ إذا أطلقنا على هذا المحور اسم "محور الأعداد التخيلية"، فيبدو أنه يمكننا تحديد كل نقطة على المستوى باستخدام زوج من الأرقام فقط: رقم حقيقي على المحور الأفقي، وعدد وهمي على المحور الرأسي. يسمى هذا الزوج "رقمًا مركبًا". في الواقع، أنشأنا نظامًا بديلاً للنظام الديكارتي المعتاد استنادًا إلى المحور X الأفقي والمحور Y العمودي، ولكن مع ميزة واضحة تتمثل في قدرتنا على إجراء حسابات هندسية معقدة باستخدام وسائل جبرية سهلة ومريحة نسبيًا.

    "أعلم أن الأمر يبدو معقدًا بعض الشيء" طمأن جداليا الجمهور المرتبك. ولكن بعد القليل من الممارسة، سوف تجادل بأن الشيطان ليس فظيعا للغاية. والجميل في ذلك هو أن النتيجة النهائية لا يجب أن تحتوي على أرقام خيالية أو مركبة على الإطلاق. لا مرح؟ لقد بدأنا بمسألة معقدة، ونقلناها عبر الخزعبلات إلى عالم خيالي من الأعداد الافتراضية، وحلناها بسهولة نسبية وأرجعناها محلولة إلى عالم الأعداد الحقيقية!

    "تفسيراتك تجعلني أفهم أقل" حك المدعي العام جبهته في حرج.

    جداليا، ابحث عن مثال مناسب. "تقول الأسطورة أنه منذ سنوات أراد شيخ عجوز أن يقسم جماله بين أبنائه الثلاثة وفقا للمفتاح التالي: يحصل الأكبر على نصف الإبل، والأوسط على الثلث، والأصغر على تسع. عد الجمال وهنا سرقة وسرقة! وكان في القطيع 17 جملا، وهو عدد أولي لا يقبل القسمة على أي عدد غير نفسه و1. وفي حزنهم، لجأ الأولاد إلى العلبة الحكيمة لمساعدتهم في القسمة. فقال لهم القاضي: بعد قليل سيعود ابني من السوق على جمل ونضيفه إلى القطيع ونقوم بالتوزيع.
    الآن مع جمل ابن القاضي سنحصل على 18 جملاً، لذلك حصل الأكبر على تسعة، والوسطى ستة، والأصغر اثنتين، ومجموع الجمل الأصلية 17. شكر الأولاد القاضي وذهبوا في طريقهم سعداء وطيبي القلب، لكن حتى يومنا هذا لا يُعرف ما إذا كان للقاضي ابنًا أم جملًا، وهم لا يهتمون أيضًا.. الشيء الرئيسي هو أن تم حل المشكلة الحقيقية باستخدام ذلك "الجمل المزيف" والجميع سعداء للغاية.
    تعمل التحويلات بطريقة مماثلة: نقل مشكلة صعبة من نظام إلى آخر من خلال سلسلة من الصيغ، تسمى أحيانًا "اليعاقبة"، وحل المشكلة في النظام الأكثر ملاءمة، وإعادتها إلى النظام الأصلي محلولة ومبتسمة.
    على سبيل المثال، سأطرح عليك المشكلة التالية: حاول حساب كتلة كرة تزداد جاذبيتها النوعية كلما ابتعدت عن المركز. إذا حاولت استخدام الإحداثيات الديكارتية، فسوف تجادل بأن المشكلة صعبة للغاية لحلها. لكن انقلها إلى الإحداثيات الكروية وستحل المشكلة في غمضة عين.

    ولهذا السبب كان الجميع متحمسين للغاية عندما انضممت إلى العائلة، وأقيمت وليمة دينية مناسبة على شرفه، مخصصة، كما وعدوا، لعالم الأرقام بأكمله. من لم يكن هناك؟ كل رجل سمين في الرياضيات، كل دوق وكل كونت، وكل من هو صغير الحجم. وسار في المجمل أعمدة لا نهاية لها، والتي تجمعت خصيصًا لهذه المناسبة. تم إعداد مراحل ترفيهية للمسلسل. ومن المفهوم أنه تم وضع حواجز علوية وسفلية في جميع الشوارع لمنع البروليتاريا من الاحتكاك بالرعاع والنبلاء. وبعد ذلك، عندما أُعطيت الإشارة، أعلن المذيع: "سيداتي وسادتي، أطلب من الجميع أن يركعوا، واسمحوا لي أن أقدم أمامكم أمراء الرياضيات الخمسة، 1، 0، i،e، وπ."
    أنت تعرف بالفعل 1، 0 وأنا. π هي بالطبع النسبة بين قطر الدائرة ومحيطها، وهي 3.14 تقريبًا. تبلغ القيمة العددية لـ e حوالي 2.72، ويتم تعريفها في حساب التفاضل والتكامل كرقم لوغاريتمه الطبيعي يساوي 1.
    ووقف الخمسة منهم على منصة الشرف أبناء الآلهة مرفوعين من الشعب، فيما يفصل المذيع نسب كل منهم وفضائله. "وها نحن هنا، لقد وصلنا إلى اللحظة العظيمة، الصيغة التعويضية التي ستوحد إلى الأبد مجالات الجبر وحساب التفاضل والتكامل والهندسة!"
    خفتت الأضواء، وعلى أصوات الأبواق والطبول، أضاء نقش ناري ضخم بصيغة أويلر فوق المسرح، وأضاء سماء الليل والحشد المبتهج:

    0=1+ ه^iπ

    توقف جاداليهو عن كلامه، وقد اختنق من الإثارة، بينما مسح ليبنوفيتس دموعه الخفية من الإثارة.

  292. تعليقات مرنان ترد وترد، نهاية توقيع جيد صديكوت وصدقيم، عند سيتا داشميا خلصنا الصوم.

    الصالحون إليك.. ثلة من الكفار. في الوقت الذي تخليت عنك يا إسرائيل، والأشباح تعذب أجسادنا وتطهر أرواحنا بالصوم والصوم، أنت تضيع وقتك في كتابة التعليقات؟!

    المكسرات…

    تعليقات إليك.. كيف لا يكون لديك احمرار على وجهك؟ لا تزال مفارقة التوأم المضروب؟ حتى الشاشة تحمر خجلاً.

    (وهذا يذكرني: مايكل، كيف تمكنت من تلوين الحروف وإبراز الكلمات وغيرها من السحر؟ كالعادة لا يتم إخبارنا بأي شيء).

    ב
    "في النظام "أ" أثناء الرحلة:
    يتحرك C بشكل أسرع من B، وبالتالي فإن ساعة B تظهر وقتًا يقع بين ما تعرضه ساعة A وما تعرضه ساعة C.
    في نظام C أثناء الرحلة:

    ليس لديك أي طريقة لمعرفة ما تظهره ساعة شخص ما بعيدًا عنك إلا إذا قمت بالاتصال، ومن المستحسن جدًا أن يكون ذلك عن طريق الوسائل الكهرومغناطيسية. لذلك، فإن تعبيرًا مثل "يتحرك C بشكل أسرع من B وبالتالي فإن ساعة B تظهر وقتًا يقع بين ما تظهره ساعة A" لا معنى له. كيف تعرف ما يظهر؟ إنه بعيد جداً، أليس كذلك؟ هل سترسل جهاز إرسال؟ افتح نفسك لجميع مشاكل الاتصال بين الأجسام ذات الحركة النسبية كما أظهرها أينشتاين في المقالة الأصلية عن النسبية.

    ميخائيل.

    بداية، توضيح: أنا لا أدعي أن النسبية أو نظرية الانفجار الأعظم غير صحيحة. الأمر فقط أن طول الوقت لا يتوافق مع الوقت المطلق للانفجار. ليس لدي أي تفضيل بين الاثنين.

    الأنظمة الكمومية التي لا يوجد فيها نظام راحة للموجات لم تعد ذات صلة في عام 1887. لكن هذا عرض جانبي. الوصول إلى هذه النقطة:

    انت تكتب:

    "هل يمكنك تقديم معلومات تفيد بأن ساعة درجة الحرارة لا تتأثر بالسرعات والتسارع؟"

    و أيضا:

    "أنت تتجاهل تمامًا ما قلته عن حقيقة أن الأسعار ترتفع في بعض أجزاء الرحلة (كما أوضحت في هذا الرد وفي الردود السابقة)".

    حتى الآن لم نتحدث عن ساعة مؤقتة. تذكير لمن يعلم، وتجديد لمن يسمع عنه لأول مرة:

    تقيس ساعة درجة الحرارة درجة حرارة إشعاع الخلفية الكونية في اتجاهين: مع وضد اتجاه الإشعاع. يتم تغذية البيانات إلى جهاز الكمبيوتر الذي يزن دوبلر ثنائي الاتجاه، ويصبح ناتج الوقت المطلق الذي مر منذ الانفجار الأعظم في نفس النقطة التي يتم فيها حساب متوسط ​​ساعة درجة الحرارة، باستخدام صيغة فريدمان:

    http://hyperphysics.phy-astr.gsu.edu/hbase/astro/expand.html#c3

    إذا كنت أتذكر بشكل صحيح، في المناقشة مع R.H. استخدمنا أجهزة كمبيوتر ماكنتوش.

    لذلك، على الرغم من أن درجة الحرارة نفسها يمكن أن تزيد مع الحركة، فإن إخراج الوقت يتقدم دائمًا وستعرض ساعتان مؤقتتان مؤقتتان دائمًا نفس الوقت في صورة مشتركة (بغض النظر عن الصورة التي تم التقاط الصورة منها).

    تعليقك مايكل في مكانه الصحيح. في ذلك الوقت حصلت على هذا الجواب:

    إعادة السؤال 1: من السهل نسبيًا حساب درجات الحرارة في الفضاء لأنه نظرًا لعدم وجود هواء، لا يوجد توصيل للحرارة أو حمل حراري. لا يمكن لأي جسم في الفضاء أن يمتص الحرارة إلا عن طريق امتصاص الإشعاع، ولا يمكن أن يفقد الحرارة إلا عن طريق الإشعاع.

    يتصرف CMB كجسم أسود بدرجة حرارة 2.725K، لذا فإن أي جسم في حالة توازن معه سيكون أيضًا عند درجة الحرارة هذه. إذا لم تكن هناك مصادر أخرى للحرارة، فسيكون مقياس الحرارة الخاص بك هو نفس درجة حرارة CMB. ومع ذلك فمن الصعب تحقيق ذلك. يمكنك القول أنه يمكنك تظليل مقياس الحرارة الخاص بك من الشمس، ولكن أيًا كان ما تستخدمه كمظلة للشمس سوف يسخن في النهاية ويبدأ في الإشعاع، ومن ثم سيسخن مقياس الحرارة الخاص بك. الطريقة الوحيدة لخفض مقياس الحرارة الخاص بك إلى 2.725 كلفن هي وضعه في الفضاء بين النجوم أو حتى في الفضاء بين المجرات.

    إعادة السؤال 2: السفر بسرعة سيؤدي بالفعل إلى تحويل CMB إلى اللون الأزرق ورفع درجة حرارته، وهذا سيؤدي إلى تسخين سفينتك. من حيث المبدأ، إذا سافرت بسرعة كافية، فإن تحول CMB إلى اللون الأزرق سوف يؤدي إلى تبخير سفينة الفضاء الخاصة بك.

    يحدث هذا بالفعل للأرض (حسنًا، ليس للجزء المتبخر!). تكون الإشعاعات CMB أكثر سخونة في اتجاه حركة الأرض. يرى http://apod.nasa.gov/apod/ap010128.html للتفاصيل.

    مشاركة تحسين هذه الإجابة
    تم الرد في 3 يونيو 12 الساعة 8:00

    جون ريني
    53.3k13792

    وهذا بالمناسبة اسم المحرر السابق لمجلة ساينتفيك أمريكان.

    وحينها أحضرت وصفاً للظاهرة التي تصفها وحلها:

    https://www.hayadan.org.il/free-speach-20100800/comment-page-25/#comment-344956

    "إلى جميع المفترين والمفترين، فإن ساعات درجات الحرارة سيئة.

    من الممكن تقنيًا شرح لماذا يجب أن تظهر ساعتان مؤقتتان في نفس المكان واللحظة نفس الوقت بالضبط، ولكن نظرًا لضيق الوقت (...) سأحاول أن أذكر مرة أخرى ما هي الساعات المؤقتة.

    تُظهر الساعات الحرارية عدد الثواني التي مرت منذ الانفجار الأعظم، والتي ستظهرها الساعة المؤقتة المجاورة لها إذا كانت هذه الساعة في حالة سكون بالنسبة للإشعاع الكوني.

    المقارنة الواضحة هي القصة المعروفة عن نرجس الأمير، الذي غالبًا ما كانت ضفادعه في مستنقعه تشتكي من النساء الحسيديات ذوات القلوب الدنيئة وذات المنقار الحاد اللاتي يلتهمنها بشهية.

    نرجس، عاشق ومقبل للضفادع بالولادة، خطط لهجوم ليلي متزامن على أعشاش الحسيديين، والذي كان من المقرر أن تنفذه وحدات كوماندوز النخبة من الضفادع الذين خططوا لمهاجمة جميع الأعشاش في نفس الوقت، من أجل منع الحسيديون من إرسال تموجات من عش إلى عش وبالتالي تحذير الحسيديين الآخرين. تم إجراء المزامنة عن طريق قياس درجة حرارة البيضة التي تبرد ببطء في الليل وصيغة تحول درجة الحرارة إلى وقت.

    كان كل شيء جاهزًا للهجوم الكبير، ولكن قبل 12 ساعة من المغادرة، ظهر الحاخام كيرميت الضفدع أمام نرجس، وكله أخضر من الخجل.

    "هناك خلل في الخطة،" تصدع كيرميت. "يدعي أسطول الشراغيف الذي من المفترض أن يعطي إشارة الافتتاح أن درجة الحرارة التي يقومون بقياسها أعلى من المعتاد، وهذا بسبب السرعة الكبيرة التي تتحرك بها الشراغيف نسبة إلى الماء. من المستحيل مزامنة الهجوم، لأن الساعات غير متزامنة. خططنا محكوم عليها بالفشل، وستستمر الحسيدية في التهام الضفادع وإنجاب الأطفال".

    "لا تقلق" طمأنه نرجس. "يمكن إدخال عامل إضافي في الصيغة والذي سيزن السرعة النسبية لحركة كل ضفدع، وسيكون الناتج هو وقت ساعة درجة الحرارة إذا كان في حالة سكون بالنسبة إلى الماء"

    هكذا استفادت الضفادع من مرعى كبير.

    المثال الذي آمله واضح.

    هناك وقت مطلق في الكون، وهو الوقت الذي مضى منذ الانفجار الكبير. وهذا ما تزعمه نظرية الانفجار. يمكن قياسه. الساعات الحرارية هي أحد الاقتراحات - ساعات الكثافة أو ساعات المجرات المتراجعة ممكنة أيضًا كما اقترح R.H.

    لكن بحسب النسبية عام 1905، فإن الكون أبدي، ولم يبدأ ولن ينتهي أبدًا. ولذلك فإن الوقت المطلق ليس له أي معنى.

    والنظرية (أ) لا تتفق مع (ب). وهذا هو جوهر حجتي.

    10 يونيو، 2012″

    إذا قبلت أن الساعات المؤقتة التي تم تصميمها كما عرضت تظهر دائمًا نفس الوقت - الوقت الذي انقضى منذ الانفجار في تلك المرحلة - فأعتقد أنك تقريبًا حصلت على حجتي. الجزء الأول من ورقة أينشتاين الأصلية عن النسبية:

    http://www.fourmilab.ch/etexts/einstein/specrel/www/

    يتناول تزامن الأحداث وصعوبة تحديد الوقت المشترك، ومن هنا جاءت الحاجة إلى مزامنة الساعات باستخدام الأشعة الضوئية. ولكن إليك طريقة بسيطة وسهلة لم تكن موجودة عام 1905: ليست هناك حاجة للتواصل مع الساعة الثانية. أخرج الساعة المؤقتة من الصندوق وسيتم ضبطها تلقائيًا على الوقت المطلق والذي سيكون أيضًا نفس وقت أي ساعة مؤقتة أخرى.

    (ماذا سيحدث؟ لقد بدأنا بمقالة الأثير الأصلية لماكسويل، وتابعنا بمقالة النسبية الأصلية لأينشتاين، وسنصل قريبًا أيضًا إلى "المبادئ" لنيوتن....).

    في كتاب "نسج الكون" للكاتب بريان جرين، في الصفحات 200-190 وخاصة في الصفحات 197-196 مكتوب: "الساعات التي تأتي حركتها الوحيدة من توسع الفضاء - هي ساعات سحرية متزامنة، تستخدم لقياس عمر الكون. بالطبع يُسمح لك بأخذ ساعتك، والدخول إلى سفينة الفضاء، والركض ذهابًا وإيابًا في الفضاء بسرعات هائلة... إذا قمت بذلك، فسوف تدق ساعتك بمعدل مختلف، وستجد أن مقدار الوقت الذي مضى مرت منذ الانفجار الكبير سوف تكون مختلفة. هذه وجهة نظر صحيحة تمامًا، ولكنها أيضًا خاصة تمامًا... عندما يتحدث علماء الفلك عن عمر الكون، فإنهم يطلبون... معيارًا له نفس المعنى في كل موقع".

    إذا كنت لا تقبل أن الساعات المؤقتة تظهر دائمًا نفس الوقت في صورة مشتركة، أو لا تعتقد أن هذا يتعارض مع الجزء الأول من مقالة أينشتاين، أو لا تعتقد أن هناك مشكلة هنا بين النسبية والانفجار وما زال لديك القوة والصبر - سنستمر في المطالبة التالية.

    ولكن إذا بالغت - أشر إلى ذلك واتركني أذهب.

  293. في اختبار الطائرات:
    هذه ثلاثة أنظمة متسارعة.
    الطائرة أ، المطار، الطائرة ب.
    إنه مثل، على سبيل المثال، جهاز طرد مركزي بثلاثة أذرع.
    في ذراع واحدة طائرة.
    وفي الذراع الثاني يوجد المطار.
    وفي الذراع الثالثة توجد طائرة
    يدور كل ذراع بمعدل مختلف عن الذراعين الآخرين.
    كل ذراع لديه قوة جاذبة مركزية مختلفة عن الأسلحة الأخرى.
    كل ذراع له تسارع جاذب مركزي مختلف عن الذراعين الآخرين.
    ولذلك فإن التأثير على الساعات يمكن أن يعزى إلى التسارع (لا أعرف على وجه اليقين).
    لكن:
    ما أوضحته سابقًا فيما يتعلق بمفارقة التوأم يشير إلى نظام بالقصور الذاتي.
    أي أنه لا يمكن ترك نظام القصور الذاتي والعودة إليه إلى نفس نقطة البداية وخلق فروق زمنية بين الساعات.

  294. وكما أجبت، وكما يثبت الإصرار على ذلك، فلن يعتبره من لا يعتقد أنه صحيح دليلاً على الحقيقة، لأن تلك قصة مختلفة

  295. מיכאל
    وكما يقولون في بلد رائع.. نعم ولكن……
    قلت في القصة السابقة إن مركبة فضائية بدرجة حرارة 0.8 درجة مئوية تمر بالتوأم عندما يكونان متباعدين. وهناك ترى المفارقة.
    ومره اخرى... أنا أكرر نفسي بالفعل.. في المفارقة الكلاسيكية، لا يؤثر حجم التسارع على فارق التوقيت. ما يؤثر هو المسافة التي يقطعها التوأم، وطول المدة.

  296. على أية حال، أنا ذاهب إلى السرير الآن.
    مساء الخير.
    آمل أن يروا غدًا أننا بحثنا في نفس الوقت

  297. المعجزات:
    في التجربة التي وصفتها للتو (إذا فهمت الأمر بشكل صحيح)، يتم الطيران بشكل متماثل تمامًا، وبالتالي ستظهر ساعاتهما في نفس الوقت ويختفي التناقض.

  298. يقوم التوأم بمناورة متناظرة. لنفترض أنهم ولدوا في منتصف الطريق. يتم نقلهم جواً لمدة نصف سنة ضوئية في كل اتجاه وفي النهاية يتم إعادتهم إلى مكان ميلادهم. لا يهم ملف التسارع الذي تقوم به، سوف تحصل على نفس النتائج، بشرط الحفاظ على التماثل.
    ومن هذا أعتقد أنه يمكن استنتاج أن التسارع ليس له معنى في التجربة.
    حتى في المفارقة الكلاسيكية، ما يؤثر على فارق التوقيت هو الوقت الإجمالي، وملف تعريف التسارع لا معنى له.

  299. كما قلت: لقد قمت بإشراك عامل آخر (نفس سفينة الفضاء) وهذه ليست تجربة التوأم.
    علاوة على ذلك - فإن التوأم الذي أعاد خطواته ووصل إلى الأرض تعرض للتسارع (ولا توجد طريقة لتجنب التسارع. لا يمكنك استعادة التماثل إلا إذا قمت بتطبيق التسارع على كليهما بحيث يتوقف كلاهما قبل الانفصال ثم يتسارعان في اتجاهين متعاكسين).

  300. מיכאל
    دعونا نأخذ زوجًا من التوائم ونمنحهم ساعات متزامنة. سنأخذهما على مسافة نصف سنة ضوئية. سيعود أحدهما إلى الأرض والآخر سيبتعد مسافة نصف سنة ضوئية أخرى. لدينا توأمان بساعتين متزامنتين تفصل بينهما سنة ضوئية.
    بالصدفة تمامًا، مرت مركبة فضائية بالقرب من الأرض بسرعة 0.8 درجة مئوية. عندما تمر المركبة الفضائية فوق الأرض، يتم تسجيل الوقت في المركبة الفضائية وعلى الأرض. سوف تمر المركبة الفضائية لاحقًا بالنجم البعيد. ستظهر ساعة التوأم بعد 15 شهرًا بينما ستظهر ساعة المركبة الفضائية بعد 9 أشهر فقط.
    لا يوجد تسارع .......... هل ارتكبت خطأ في مكان ما؟

  301. المعجزات:
    لقد قلت بالفعل أن شخصين في مكان واحد ليسا متماثلين تمامًا.
    إذا كنت تعتقد أنه من الممكن لشخص واحد أن يتحرك ذهابًا وإيابًا دون أن يتم تسريعه (وحتى دون تغيير الاتجاه)، فأنا أحب أن أقرأ كيف تقترح القيام بذلك لأنه قد يستحق الكثير من المال بسبب توفير الطاقة.
    في الحقيقة ستكون معجزة 🙂

  302. ب:
    إذا كنت لا تعرف من هو العنيد هنا فسأقول لك: إنه أنت.
    اقرأ على سبيل المثال هنا: http://en.wikipedia.org/wiki/Hafele-Keating_experiment
    الرجاء متابعة كلمة Reunited عدة مرات في الجملة التالية (مقتبسة من هناك):
    متى لم شملوجد أن المجموعات الثلاث من الساعات تختلف مع بعضها البعض، وكانت اختلافاتها متوافقة مع تنبؤات النسبية الخاصة والعامة.

    لا يمكنك معرفة الأشياء ولا يمكنك فهم الأشياء. كل هذا مشروع. لكن لا يمكنني أن أرى ادعاءً غير صحيح بشأن حدث تاريخي أمرًا مشروعًا. هل أنت على استعداد لإخبارنا على أساس ادعائك بأن الساعات لم تعد إلى نقطة البداية؟
    فإذا قلت إنك لا تعرف إذا عادوا إلى نقطة البداية، فإن ذلك يظل نقصًا مشروعًا في المعرفة، ولكن ادعاء العكس دون إثبات؟ سامحني، ولكن هذا كل شيء!

  303. لا أعرف من هو العنيد هنا.

    لم يتم قياس الفروق الزمنية التي تم قياسها في التجارب عندما غادر الجسم نظام القصور الذاتي وعاد إلى نقطة البداية في نظام البداية.
    وما يقاس هو التغيرات الزمنية في الأجسام المتحركة التي لا تعود إلى نقطة البداية في النظام المرجعي للأصل. وهذا ليس ما يحدث عند الحديث عن توأم يغادر الأرض ويعود إليها.
    على الأكثر هذا ممكن في توأم ترك النظام المرجعي للأرض ولم يعد إليه (حتى وقت القياس).

  304. لا يمكنك الذهاب والعودة دون تسريع.
    التسارع له تأثير.
    أعلم أنه يمكنك محاكاة شخص ما يخرج ويعود باستخدام شخصين، أحدهما يذهب دائمًا والآخر يعود دائمًا، لكن الأمر ليس هو نفسه تمامًا، حتى لو كانت النتائج متطابقة.
    على أية حال - أردت أن أشير إلى ما يكسر التناظر.
    بالطبع، يمكن أيضًا إظهار كسر التماثل عن طريق تغيير الاتجاه دون تسمية الطفل باسمه - ويكون التماثل مكسورًا بالفعل. ولنلاحظ ذلك، يكفي لو افترضنا أن الطرفين ينقلان الضوء بتردد ثابت في اتجاه واحد ونسأل أنفسنا متى لاحظ كل منهما حقيقة حدوث تغيير في الاتجاه (التوأم المتحرك سيلاحظ على الفور التغيير) في التردد بينما التوأم المستقر لن يلاحظ التغير في التردد إلا بعد وصول الضوء الذي يرسله التوأم المسافر بعد الدوران إليه).

  305. ب:
    ما هذا العناد؟
    مر A وC بتسارع بينما لم يمر B بأي تسارع.
    ساعاتهم مختلفة وقد ثبت ذلك أيضًا في التجربة.
    للمرة المائة يجب أن أتحدث عن الرجل الذي دخل حديقة الحيوانات، ورأى زرافة، فقال: "مستحيل! لا يوجد مثل هذا الحيوان!"

  306. إسرائيل:
    "تظهر ساعة B دائمًا وقتًا متأخرًا عن A وC، وهو نفس الوقت. "
    وهذا صحيح فقط في النظام المرجعي لـ B.
    الأنظمة المرجعية A وC منفصلة عن النظام B منذ لحظة الطيران حتى لحظة العودة. وفي لحظة العودة تعود كافة الأنظمة وتندمج في نظام واحد.
    في النظام A أثناء الرحلة:
    يتحرك C بشكل أسرع من B، وبالتالي فإن ساعة B تظهر وقتًا يقع بين ما تعرضه ساعة A وما تعرضه ساعة C.
    في نظام C أثناء الرحلة:
    يتحرك A بشكل أسرع من B، وبالتالي فإن ساعة B تظهر وقتًا يقع بين ما تعرضه ساعة C وما تعرضه ساعة A.
    أي أن الوضع في النظام A متماثل مع الوضع في النظام C.
    وبعد العودة تندمج كافة الأنظمة في نظام واحد وهو نظام B. وذلك بسبب إعادة ضبط سرعتهم.
    في نظام B، تظهر ساعات A وC نفس الوقت.
    خطر:
    بعد العودة، تعود جميع الساعات مرة أخرى إلى النظام المرجعي لـ B وتظهر جميعها نفس الوقت تمامًا.
    خطر:
    يتم إلغاء أي فارق زمني يتم قياسه أثناء الرحلة (نظام مرجعي مختلف عن نظام الأصل) أثناء العودة إلى النظام المرجعي وهو نظام الأصل.

  307. إسرائيل:
    يمكنني بالتأكيد أن أشير إلى مثل هذا النظام الفيزيائي (وقد فعلت ذلك بالفعل في إجابة سابقة) - نظام تكون فيه الموجات أكثر غموضًا من موجات الضوء، وهذا هو النظام الكمي.
    إنه نظام لا يحتاج إلى وسيط فحسب، بل يمكن تنسيق المعلومات الموجودة فيه بغض النظر عن المسافة.
    لقد قدمت مثالاً لنوع معين من الحركة، ولكن إذا كنت تريد حركة تسمح بالموجات - فهذا ممكن أيضًا.
    كل ما أريده هو الحد من حركة الجسيم في منطقة معينة حول نقطة بداية ثابتة (تلك الموجودة في النظام الثابت). يمكن أن تعطى، على سبيل المثال، حركة توافقية محجوبة ومتلاشية حول نقطة البداية.

    ما تصفه هو اختلاف في ساعة درجة الحرارة.
    أما بالنسبة للتوائم المتحركة فإن درجة الحرارة سترتفع كلما تحركوا بسرعة بالنسبة لمتوسط ​​الكون، وتنخفض عندما يقتربون من المتوسط. ستمثل درجة الحرارة مزيجًا من سرعة حركتها مع عمر الكون.

    لاحظ أن هذا يتناسب بشكل رائع مع النسبية.
    الفوتون، على سبيل المثال، ليس لديه وقت على الإطلاق. كل شيء يحدث له في نفس الوقت (ومحلياً) فهو لا يزال يعتقد أنه في الانفجار الأعظم. كلما اقتربت منها بشكل أسرع، كلما اقتربت من هذه الحالة - درجة الحرارة التي تقيسها أعلى، وزمنك "أبطأ"، وتقديرك لعمر الكون أقل (وهو أمر يمكن اعتباره انعكاسًا للواقع). بطء الوقت الذي تظهره حاليًا على الماضي).

    وأكرر: عندما تتحدث عن "توزيع الانخفاض في درجات الحرارة" فإنك تفترض خطأً وكأن درجة الحرارة ستنخفض طوال الوقت.
    أنت تتجاهل تمامًا ما قلته عن حقيقة أنه خلال جزء من الرحلة يرتفع (كما أوضحت في هذا التعليق وفي التعليقات السابقة).

  308. ميخائيل

    يشير إلى الحركة العشوائية للجزيئات في جميع الاتجاهات كما في الهواء أو الماء. إذا لم تكن هناك حركة، فلا يمكن للموجة أن تمر.

    سؤالي هو: هل يمكنك الإشارة إلى نظام تمر فيه الأمواج وليس له نظام استراحة مفضل؟

    لا أعتقد أن هناك. لا يوجد سوى نظام رياضي - معادلات ماكسويل. لكن ليس نظامًا ماديًا (أعرف واحدًا؟ أي شخص؟ من؟).

    توأمان:

    أنا لا أتحدث الآن عن ساعات درجة الحرارة. لقد قدمت سيناريو يعود فيه التوأم بعد سنة حسب ساعته وبعد مليار سنة حسب ساعة أخيه. في هذه الأثناء، انخفضت درجة حرارة الكون بمقدار 100 درجة كلفن، وهو ما يتفق عليه كلاهما ويوجد أيضًا مقطع فيديو لإثبات ذلك.

    سؤالي كان حول توزيع انخفاض درجات الحرارة. حسب فهمي، تدعي النسبية أن معظم الهبوط سيكون أثناء حركة القصور الذاتي. فإن ادعيت غير ذلك فبين كيف سيتم توزيع النقصان على أن يصل المجموع إلى -100.

    بالمناسبة، قبل نسيم حجتي.

    "في رأيي - كلما تحركت بشكل أسرع، كلما زادت درجة الحرارة التي ستشعر بها (مقارنة بنظام الراحة)".

    هل المقصود التحرك بسرعة ثابتة؟ فيما يتعلق بأي نظام الراحة؟ من إشعاع الخلفية الكونية؟

    وفقا لأينشتاين (وجاليليو ونيوتن وماخ...) فإن الحركة بسرعة ثابتة هي راحة. وفقًا للمسلمة أ - جميع أنظمة الراحة (القصور الذاتي) متطابقة.

    وحتى نتمكن من الوصول إلى هذه النقطة، ما أطلبه هو:

    متفق عليه أنه في نظام التوأم الرشيق أثناء حركة القصور الذاتي تنخفض درجة الحرارة بمعدل درجة واحدة على الأقل في الدقيقة كما يظهر من الفيديو الذي يصور ساعة مقياس الحرارة ومقياس التسارع (الذي يظهر 0).

    إذا لم يكن الأمر كذلك - توزيع قطرة مختلفة (نحن جميعا نتفق على أن درجة الحرارة انخفضت، أليس كذلك؟).

  309. ب:
    لماذا نتحدث فقط؟
    اقرأ على سبيل المثال على ويكيبيديا
    http://en.wikipedia.org/wiki/Twins_paradox
    اكتب اسما:
    تم التحقق من مفارقة التوأم تجريبيا من خلال قياسات دقيقة للساعات الذرية التي تحلق في الطائرات والأقمار الصناعية. على سبيل المثال، تم استخدام تمدد الزمن الثقالي والنسبية الخاصة معًا لشرح تجربة هافيل-كيتنغ. [أ 1] [أ 2] تم تأكيد ذلك أيضًا في مسرعات الجسيمات عن طريق قياس التمدد الزمني لحزم الجسيمات المنتشرة. [أ 3]
    إذا ذهبت إلى مدخل ويكيبيديا الذي يتناول تمدد الزمن واخترت الفصل الذي يتناول الاختبار التجريبي، فستجد الكثير من المواد الإضافية:
    http://en.wikipedia.org/wiki/Time_dilation#Experimental_confirmation

    إسرائيل:
    ليس لدي مشكلة في إعطاء سرعة لجزيئات السامتين ولكني لم أرغب في السماح لك بتحديد تلك السرعة في اتجاه الاتجاه.
    فمن الممكن، على سبيل المثال، أن يهاجم كل جزيء سامين الرأس الأقرب إليه بسرعة خطية ثابتة.
    هناك حركة على سطح الكرة، حتى على الكرة الأكثر كمالا. وإلا فلن يتمكن شخصان يسيران عليه من الالتقاء. هل تعتقد أن هذا هو الحال؟
    هل تعتقد أن هذا هو الحال أيضًا في الكون اللانهائي؟
    في رأيي - لا يمكن للناس أن يجتمعوا في مساحة متجانسة لا حصر لها فحسب، بل يمكنهم أن يحددوا مسبقًا بالضبط متى وأين سيلتقون، ومتى سينفصلون، وإلى أي مدى سيكونون متباعدين ومتى سيلتقون مرة أخرى.
    يمكنك أن تقرأ عن التأكيدات التجريبية التي يختلف طولها هنا:
    http://en.wikipedia.org/wiki/Length_contraction#Experimental_verifications
    هذا بالإضافة إلى ما سبق أن قدمته عن الجسيمات قصيرة العمر التي تنشأ نتيجة اصطدام جزيئات الإشعاع الكوني بالجزيئات الموجودة على حافة الغلاف الجوي.
    لا يتم تسريع الجسيمات الأولية التي تنشأ في الاصطدامات. منذ لحظة خلقهم، ليس هناك ما يدفعهم إلى التسارع أكثر. يتم إنشاء المزيج الكامل من كتلتها وسرعتها في وقت واحد من طاقة الاصطدام.

    لن أخوض كثيرًا في ساعة درجة الحرارة لأنك لم تفعل ذلك أيضًا.
    هل يمكنك عرض معلومات تفيد بأن ساعة درجة الحرارة لا تتأثر بالسرعات والتسارع؟
    لم أعد أذكر العبارة التي ظهرت في الرابط الذي قدمته حينها للموضوع، لكن الكثير من مكوناتها تعتمد على السرعة.
    وأكثر من ذلك - حتى درجة الحرارة نفسها - إذا كانت دالة للأطوال الموجية التي تواجهها أو لسرعة الجسيمات - فهي دالة للسرعة.
    في تقديري - كلما تحركت بشكل أسرع ستشعر بدرجة حرارة أعلى (نسبة إلى نظام الراحة) ومن حيث ساعة درجة الحرارة سوف تقيس عمر كون أقل (وهذا يعني أنه عند السرعة العالية نسبة إلى نظام الراحة لديك ستتحرك ساعة درجة الحرارة إلى الخلف).
    على أية حال - عندما تستقر في نظام مشترك مع شخص آخر، ستعرض ساعات درجة الحرارة الخاصة بك نفس درجة الحرارة الزمنية.

    وكما ذكرت من قبل - في حسابات نظام تحديد المواقع المتسارع هناك اعتبار منفصل للسرعة والتسارع والخطأ في أي من هذه المراجع سيكون كارثيا.

  310. معجزات,
    فيما يتعلق بالتسارع، قد يكون من الممكن التعامل مع تفاصيل النسبية، ولكن ليس هذا هو الهدف من كلامي. من المستحيل القول بأن التوأمين في نظامين متوازنين عندما يكون أحدهما فقط هو الذي يتسارع.
    فيما يتعلق باختلافك في المفارقة، يجب أن أتعمق فيها وليس لدي الوقت حقًا (لأن ابنتي تريد الذهاب للنزهة) ولكن التوأم سيكبران بنفس المعدل تمامًا على أي حال، أعتقد أنني كذلك لن أخوض في عمق المثال ولكن بسبب الوقت الذي أصبح أقصر فإن ابنتي تزعجني.

  311. معجزات.

    لقد حان الوقت. الآن نحن مجرد إبرام الاتفاقيات. هل تقبلون أن يكون في فيديو الشباب هبوط 100 درجة في ظرف ساعة؟ وأن هذا الانخفاض سيكون على 5 مراحل؟ ومعظم النزول سيكون بينما يظهر التسارع 0؟

    أشباح

    أنا صائم حقًا، ليس لأنني مؤمن كبير (أو صغير)، ولكن امرأة، صانعة سلام، امرأة...

    المشكلة هي أنني في كل مرة آتي إلى المجمع - يهرب الله..

    ב

    "تظهر ساعة B دائمًا الوقت بين الوقت الموجود على ساعة A والوقت الموجود على ساعة C."

    تعرض ساعة B دائمًا وقتًا متأخرًا عن A وC، وهو نفس الوقت. تناظر إليك.

  312. شموليك
    يطلق عليه دعابة ...
    لذا. لنفترض أن النجمين يفصل بينهما سنة ضوئية. لدينا زوج من التوائم - واحد على كل كوكب وساعة كل توأم متزامنة مع ساعة التوأم الآخر.. أعتقد أنه من الواضح أنه لا توجد مشكلة في الوصول إلى هذا الوضع، فالتوأم في نفس العمر بالرغم من ذلك. المسافة بينهما. تخيل الآن سفينة فضاء تتحرك في خط مستقيم، بسرعة ثابتة قدرها 2 درجة مئوية، وتتجاوز النجمتين. أثناء عبور النجم الأول، يقوم كل من التوأم الموجود على ذلك النجم والمركبة الفضائية بتسجيل الوقت. الآن - ماذا سيحدث عندما تعبر المركبة الفضائية النجم الثاني؟ أما التوأم الثاني، فتتحرك المركبة مسافة سنة ضوئية بدرجة حرارة 0.8 درجة مئوية، أي أن الزمن 2 شهراً. ومن ناحية أخرى، ترى المركبة الفضائية أن المسافة بين النجوم أصغر وبالتالي تقيس وقتًا أقصر. والمسافة التي سيراها هي 0.8 سنة ضوئية، أي أنه سيقيس 15 أشهر فقط

    اسئله؟

    إذا لم يكن الأمر كذلك، فقد شرحنا مفارقة التوأم بدون تسارع. وللتوضيح فقط، لا توجد مشكلة في التعامل مع التسارع في النسبية الخاصة. لكن الأمر لا علاقة له بمفارقة التوأم.

  313. ميخائيل.

    "هناك العديد من الأشكال الهندسية المحدودة حيث لا يوجد نظام راحة مفضل. وأبسطها وجه الكرة."

    لا توجد حركة على سطح الكرة. KDA متماثل ولكن الهواء الموجود على سطحه لديه نظام راحة مفضل - KDA. نظام "Shrigin" هو أيضًا نظام متماثل. اقترحت إعطاء الجزيئات سرعة "السيتامين" لكنك لم توافق.

    هل يمكنك الإشارة إلى نظام تمر فيه الأمواج وليس له نظام استراحة مفضل؟

    "في الواقع - ستظهر مقاييس درجة حرارة التوائم درجات حرارة مختلفة على طول الطريق".

    هل نتفق على أنه في نهاية الفيديو - لكل من التوأم المسافر والتوأم المتبقي - يظهر مقياس درجة الحرارة أقل بمقدار 100 درجة؟

    إذا كان الأمر كذلك، يمكن تقسيم النسب إلى مراحل. وبينت أن هناك 5 مراحل تشمل التسارع والسير بسرعة ثابتة. لقد اقترحت - وهذه أيضًا حجة النسبية التي أعتقدها - أن معظم الانخفاض يحدث أثناء السفر بسرعة ثابتة.

    إذا كنت تدعي خلاف ذلك، وضح متى حدث النزول (ليس هذا مهمًا) وشرط أن نصل في النهاية بعد الخطوات الخمس إلى نزول 5 درجة.

    بالمناسبة من الرابط :

    http://math.ucr.edu/home/baez/physics/Relativity/SR/experiments.html#Length_Contraction

    في هذا الوقت لا توجد اختبارات مباشرة لانكماش الطول، حيث أن قياس طول جسم متحرك بالدقة المطلوبة لم يكن ممكنًا.

    وكما ذكرت، فإن نظام تحديد المواقع العالمي (GPS) والجسيمات الأولية (أفترض أنك تقصد الميونات) هي أنظمة متسارعة.

  314. شموليك.

    القصة لها بداية، ونهاية، وقبل كل شيء هدف: الميكانيكا النفسية.

    ومن ناحية أخرى، أنا لست خبيرا في هذا الموضوع. لم أدرس قط أو حتى كان لدي اهتمام بعلم النفس. ليس لدي الوقت أو الرغبة في البدء في التعامل مع هذه القضية أيضًا. ولكن عندما أنظر إلى ما يحدث في العالم، يصعب علي أن أهرب من الشعور بأن المجتمعات البشرية تتصرف إلى حد كبير مثل الأنظمة الديناميكية الحرارية التي تتمتع بقوانين مماثلة للحفظ والإنتروبيا.

    העם היהודי עבר את השואה האיומה מכל, אך אם נסתכל על העם כעצם ובלי להיכנס לפרטים של כל אדם (כמו שטיפול במערכת תרמודינמית אינו מתייחס למולקולה בודדת אלא רק למערכת כולה) השואה לא ממש הזיקה לנו ואנו נמצאים כעת ב”תור זהב” כמו לפני גירוש اسبانيا.

    قبل 40 عاما كنا في حالة حرب مع مصر وسوريا والأردن. وكان أصدقاؤنا إيران وتركيا. وكان الفلسطينيون ولبنان هادئين نسبياً. واليوم هناك سلام مع مصر والأردن، وكانت الحدود السورية هادئة نسبياً حتى وقت قريب. إيران وحتى وقت قريب تركيا هم أعداؤنا. الفلسطينيون ولبنان هم من يطلقون النار.

    بمجرد أن أطلق المفتي النار علينا، غير اسمه إلى كوكجي، وبعد ذلك إلى ناصر، عرفات، السادات، ياسين، نصر الله - ولكن تحت كل أكوام الملاعق، دائمًا ما يكون العربي نفسه هو الذي يطلق النار علينا.

    وتحارب الولايات المتحدة تنظيم القاعدة في أفغانستان وتسلحه في سوريا. أليس هذا مضحكا؟

    ليس إذا كنت تفكر في نظام مغلق لا يمكن أن تحل فيه الفوضى من تلقاء نفسها.

    وأنا أخاطر وأتوقع أنه من بين كل الفوضى الحالية في مناطقنا، سيكون لدينا فجأة حليف لم نحلم به أبدًا.

    لكن كما ذكرت، أنا مجرد راوي قصص. أنا حقًا لا أملك المعرفة والقوة والرغبة في البدء في التعامل مع الموضوع.

    وفي استطلاع أجري لصالح "مجموعة الميكانيكا النفسية" وشمل عينة تمثل جميع طبقات سكان البلاد، كان السؤال الأول: "هل تعتقد أن الشخص العادي أصبح اليوم أكثر ثراء مما كان عليه في الماضي؟"

    أجابت أغلبية كبيرة من المستطلعين (82%) بالإيجاب على السؤال، وذلك لأسباب مفهومة. في الماضي، وفي كثير من الأحيان، كان يعتبر امتلاك أكثر من زوج من الأحذية أو بدلة واحدة من "الأثرياء". عملت الغالبية العظمى من السكان بجد في الزراعة، مقابل أجر زهيد أو كأقنان أو عبيد.
    ولم تتمتع سوى أقلية صغيرة بمستوى معقول من المعيشة، وحتى هذا المستوى لا يعتبر مرتفعًا بشكل خاص وفقًا للمعايير الحديثة.

    وأيضاً على السؤال الثاني - "هل تعتقد أن الشخص العادي يتمتع اليوم بصحة أفضل مما كان عليه في الماضي؟" أجاب 78% بالإيجاب. والواقع أن متوسط ​​العمر المتوقع في العصر الحديث أصبح أعظم بلا حدود مما كان عليه في الماضي. صحيح أنه كان هناك أشخاص في العصور القديمة عاشوا حياة طويلة وصحية، لكن معظم الناس لم يتجاوزوا سن الأربعين، وكانت وفيات الرضع أعلى بكثير مما هي عليه اليوم، واختصرت الأمراض والأوبئة وجعلت حياتي المتبقية بائسة.

    ولذلك فمن المدهش بعض الشيء أن يكون السؤال الثالث: "هل تعتقد أن الإنسان العادي أصبح اليوم أكثر سعادة مما كان عليه في الماضي؟" أجاب 18% فقط بالإيجاب، فيما أضاف عدد مماثل من المستطلعين أنهم يعتقدون أن الوضع عكس ذلك!

    لماذا ؟ نحن أكثر صحة، وأكثر ثراء، ونعيش حياة أطول - ومع ذلك ألسنا أكثر سعادة؟ كيف؟
    وكأن ذلك لم يكن كافيا، فالبشرية اليوم تحت تصرفها علم لم يكن موجودا حتى نهاية القرن التاسع عشر: علم النفس، الذي غرضه كله هو الصحة العقلية للإنسان، وهدفه هو يجب أن يفترض، هو زيادة السعادة البشرية.

    هل المستجيبون مخطئون؟ بالطبع، لا يمكننا قياس حالة غامضة وغير قابلة للقياس مثل السعادة، خاصة عند الآخرين، وبالتأكيد ليس في الماضي. ولهذا السبب أيضًا تمت صياغة الأسئلة بلغة "هل تؤمن؟" لكن الكثير من الناس اكتشفوا الحقيقة البديهية في الإجابات.

    يبدو أن حل المفارقة يكمن في السؤال الرابع: "هل تعتقد أن السعادة الشخصية تنبع من التفاعل مع الطبيعة (الحقول والحيوانات والبحر وضدها الكوارث الطبيعية والجوع والبرد وغيرها) أو التفاعل مع الآخرين؟ (الحب والصداقة والعائلة وضدهم الكراهية والغيرة أو الكوارث والأضرار التي من صنع الإنسان؟"

    وأجاب ما لا يقل عن 92% من أفراد العينة بأن المصدر الرئيسي للسعادة هو التفاعل مع الآخرين. وبالإضافة إلى ذلك، أضاف العديد من المشاركين من تلقاء أنفسهم أن الإصابة المتعمدة للأشخاص تكون أكثر خطورة بعدة مرات من الإصابة ذات الضرر المماثل الناتج عن وقوع حادث.
    وبعد تحليل السؤال حدد "رجل الميكانيكا النفسية" المعضلة التي تواجه المجموعة كالتالي:

    الحقيقة: لقد ساهم العلم والطب في تحسين الحالة الصحية ومتوسط ​​العمر المتوقع على الأرض.
    الافتراض: يمكن دراسة تصرفات الإنسان ومشاعره وتحسينها من خلال الأساليب النفسية (التعليم، العلاج، التكييف، إلخ).
    الخلاصة: يمكننا زيادة سعادة الإنسان على الأرض من خلال "الإنتاج الضخم" لهذه الأساليب، وتطبيقها على أكبر عدد ممكن من الناس، تماما كما قضى التطعيم الشامل على معظم الأوبئة.
    سؤال: بخصوص 3: هل يمكننا ذلك حقا؟

    لنجعل المعايير أكثر صرامة: لنفترض أن علم النفس سيصل إلى الكمال، وأن الهندسة الوراثية مع الانتقاء الطبيعي ستخلق أشخاصًا مثاليين، مثاليين لدرجة أنه سيتم استنساخ سمات الرضا والسعادة لكل شخص في العالم. فهل يسطع نجم الدلو أخيراً وتسود السعادة والوئام في العالم؟

    لا، الميكانيكا النفسية تخبرنا.
    وليس بالوسائل والأساليب المستخدمة اليوم. يمكن أن تساعد هذه الأمور الفرد على تحسين وضعه بالنسبة إلى العام، ولكن بما أن السعادة حالة نسبية فإن تحسين حالة الفرد يجب أن يأتي على حساب العام، أو على حساب الجسم الراعي الذي سيتم حرمانه من طاقته العقلية الإيجابية. من العام.

    هذا هو جوهر القانون الثاني للميكانيكا النفسية. تتم مقارنة النظام في النظام الديناميكي الحراري بالسعادة في النظام الديناميكي النفسي، وكما أن ميل النظام في النظام الديناميكي الحراري المغلق يتناقص بمرور الوقت، فإن ميل السعادة في النظام النفسي الميكانيكي المغلق، مثل الأرض، هو إلى الانخفاض، مما يؤدي إلى انتشار النظام ودفع عدم الرضا إلى الأطراف الاجتماعية التي تركت وراءها.

    إذا نجحت من خلال العمل الجاد والدراسة في تحسين درجاتك في الاختبار النفسي أو اختبار الذكاء، فإنك حتماً تدفع شخصاً آخر إلى الأسفل، لأن I Q100 بحكم تعريفه هو مقياس يعكس متوسط ​​الذكاء. ونفس الشيء سيحدث لك إذا اضطررت للتنافس على مكانك في الجامعة مع مجموعة من الصينيين المجتهدين والموهوبين بشكل شيطاني، والذين سيرفعون شريط العلامات ويثبتونك دون مخرج على الجانب الأيسر من الجرس منحنى.

    هناك أطروحة مثيرة للاهتمام تجري بحثها من قبل المجموعة تشير إلى مجتمعات المدمنين. الافتراض هو أنه في ظل ظروف ثابتة، فإن نسبة معينة وثابتة تقريبًا من السكان سوف تتطور إلى إدمان - للكحول والمخدرات والطعام والقمار وما إلى ذلك. سوف يركز الإدمان على بعض الأشخاص، ويتجاهل الآخرين. الآن، ماذا سيحدث لو أبعدنا من النظام إلى جزيرة معزولة جميع مدمني المخدرات على سبيل المثال، وتركنا جميع الشروط الأخرى في النظام كما كانت؟
    وفقًا لبيانات الافتراض الأصلي، نظرًا لأن نسبة المدمنين ثابتة إلى حد ما، فبعد أن يصل النظام إلى توازن جديد، سيتشكل مدمنون جدد تلقائيًا لملء الرتب، بينما في الجزيرة المهجورة سيكون هناك انسحاب تلقائي جماعي بين المدمنين الأصليين.

    وعلى الرغم من عدم وجود بيانات تجريبية كافية حتى الآن لإثبات الفرضية، فإن المعنى الضمني، إذا تم تأكيد النظرية، هو أن المدمنين الأوائل، بحكم وجودهم، منعوا الأخيرين من أن يصبحوا مدمنين، دون حتى معرفتهم أو معرفة دورهم في الإدمان. نظام!

    ومن المثير للاهتمام أيضًا التفسير الذي قدمته المجموعة لمشكلة معاداة السامية. وفقا لحجة الميكانيكا النفسية، لا يهم على الإطلاق ما سيفعله اليهود، ومن هم، وما هم، وما إذا كانوا موجودين أصلا. إن مصدر المشكلة هو مجموعة واحدة - معاداة السامية - تحاول خفض إنتروبيا النفسية الميكانيكية الخاصة بها عن طريق رفع إنتروبيا مجموعة أخرى، اليهود.
    يتناسب هذا الادعاء بشكل جيد مع النتيجة التي مفادها أن معاداة السامية تتزايد في أوقات الأزمات، عندما يزداد الضغط النفسي، وكذلك مع وجود معاداة السامية في البلدان التي لا يوجد فيها يهود على الإطلاق. وذلك لأنه، وفقًا للقانون الأساسي للميكانيكا النفسية، تنشأ مشاعرنا من الشكل الذي ندرك به الواقع، حتى لو كان مجرد واقع افتراضي.

    ومع ذلك، حتى الميكانيكا النفسية لا تستبعد انخفاض الإنتروبيا في النظام النفسي الميكانيكي المغلق. إنها تدعي فقط أن الطاقة العقلية يجب أن تأتي من مصدر خارجي عن النظام. في الماضي، كان معظم الناس يؤمنون بالله، الذي كان بلا شك مصدرًا خارجيًا مناسبًا. تقدم الميكانيكا النفسية حلاً آخر يسهل الوصول إليه.
    وكما حررت الثورة الصناعية البشرية من الاعتماد على العبودية من خلال تحويل الطاقة الكيميائية إلى عمل مفيد، كذلك تهدف الميكانيكا النفسية، من خلال المنتجات الجيدة للتكنولوجيا النفسية، إلى تحويل الطاقة الميكانيكية إلى طاقة عقلية، وبالتالي تقليل الجوانب السلبية لترابطنا.
    الأدوات الرياضية التي يستخدمها الميكانيكا النفسية هي "تحويلات عكسية إلى حقول متجهة محاكاة، من أجل إنشاء تماثل كروي افتراضي" - وهي عبارة مبالغ فيها قليلاً، وجوهرها هو جعل أكبر عدد ممكن من الناس يشعرون "بالقمة" دون أن يفقدوا قبضتهم على الواقع.

    هذه الكلمات التي ظهرت في وسائل الإعلام قبل أيام قليلة من شهادة البروفيسور ليبنوفيتس، ترددت في ذهن طالب دكتوراه في الرياضيات في جامعة بارما، شاب يدعى جيدالهيو.

  315. وما تم قياسه في التجربة هو شيء مختلف تمامًا.
    ما يتم قياسه هو عمر الجسيم المتحرك بالنسبة للنظام. ولم يتسارع الجسيم في اتجاه واحد ثم في الاتجاه الآخر، ولم يعود إلى السكون عند نقطة بدايته.
    والتناقض المنطقي لأسباب التماثل في التجربة الثالثة موجود.

  316. إسرائيل:
    بداية لنكن واضحين: لم يثبت عدم وجود موقع إلكتروني. لا توجد حاليًا طريقة معروفة لتأكيد ذلك أو دحضه، لكن لا توجد حاليًا مشكلة (باستثناء أوكهام الذي تستشهد به عادةً) في الادعاء بأن الكون يتشوه كما ادعى أينشتاين وأنه مليء بالأثير كما افترض ماكسويل.
    هناك العديد من الأشكال الهندسية المحدودة حيث لا يوجد نظام راحة مفضل. أبسطها هو وجه الكرة.
    لقد لاحظت عددًا من المشاكل في وصفك لإطالة الوقت، ولكن بما أنك قلت أن هذا ليس هو الشيء الرئيسي - فلن أتطرق إليه.
    إن وصفك للتوائم يعتمد على ساعة درجة الحرارة التي ناقشناها بالفعل، وهو معيب بافتراض أن هذه الساعة تتحرك في اتجاه واحد حتى في الأنظمة المتسارعة.
    في الواقع، ستظهر موازين الحرارة الخاصة بالتوأم درجات حرارة مختلفة على طول الطريق.

    ب:
    لا يوجد تماثل بينك وبين A أو بينك وبين C، لذلك سيتم قياس الوقت بشكل مختلف.
    ويجب أن تأخذ في الاعتبار أن هذه الحقيقة هي أيضًا حقيقة وقد تم بالفعل قياس هذه الظاهرة في التجربة.

  317. و)
    هناك نقطة خاصة بكل مشاهد وهي النقطة التي يتواجد فيها.
    إذا كان هناك عدد لا حصر له من المراقبين، فهناك عدد لا نهائي من هذه النقاط، لكن هذا لا يتعارض مع حقيقة أنه لكل مراقب هناك نقطة فريدة واحدة.
    ب)
    عن التوأم.
    لقد سبق لي التعليق من قبل وسوف أعلق مرة أخرى:
    لنفترض أن الثلاثي موجود في الفضاء:
    في لحظة بدء التجربة تظهر جميع الساعات صفرًا.
    A يتسارع في اتجاه واحد C يتسارع في الاتجاه المعاكس. ب يبقى في مكانه.
    تعرض ساعة B دائمًا الوقت بين الوقت الموجود على ساعة A والوقت الموجود على ساعة C.
    يحدث هذا عند السرعة الثابتة والتسارع.
    يعود كل من A وC إلى النقطة التي خرجا منها، أي إلى النقطة التي يوجد فيها B.
    لأسباب التناظر:
    ساعة "أ" هي نفس ساعة "ج".
    وبالتالي الاستنتاج:
    جميع الساعات الثلاث تظهر نفس الوقت.
    لأسباب التناظر فقط.
    بدون أي صيغ رياضية.
    من غير الممكن أن يكون أحد الثلاثي أكبر سناً من الآخرين وقت اللقاء.

    حتى بدون وجود C، فإن الساعتين A وB في وقت الاجتماع المتجدد ستظهران نفس الوقت بالضبط.
    من غير الممكن أن يكون أحد التوأمين أكبر سناً من الآخر.

  318. إسرائيل
    تصفيق.
    عن القصة
    هل ستكتب نهايته؟ أم أنه سيكون فقط بالتنسيق مع نشر نظريتك؟
    سوف ننتظر لقد انتضرنا. ينتظرون.
    صيام سهل .

  319. معجزات,
    ربما هناك تفسير أبسط ولكن هذا هو الشرح الذي تعلمته وهذا هو الشرح الذي يظهر على ويكيبيديا. إذا لم أكن مخطئا، فإن النظرية النسبية الخاصة، التي تناقش فيها المفارقة، تتحدث فقط عن السرعات الثابتة وبمجرد دخول القوى أو التسارع إلى نظام واحد دون الآخر، تتوقف الأنظمة عن التوازن.

    على أية حال، ليس من الجيد أن تكتب أن هناك تفسيرًا أبسط بكثير ولا تخبره. هيا اكتب كتابا (مرحبا بكم مثل اسرائيل لتكتبوا قصة حول الشرح)

  320. إسرائيل،
    لا أفهم لماذا طرحت مفارقة التوأم. المفارقة ليست مفارقة بمجرد تحديد وجود تسارع وأن التوأمين يلتقيان، لأن ذلك يتطلب تطبيق القوة والتسارع في الاتجاه المعاكس، مما يعني أن التوأمين غير متساويين
    http://he.wikipedia.org/wiki/פרדוקס_התאומים

    سؤال عام عن قصتك أتمنى أن تجيب عليه نهائيا: هل القصة كلها مكتوبة؟ (البداية والوسط والنهاية، الأعداء يصبحون أصدقاء، والأصدقاء يصبحون أعداء، ويتعلم البطل شيئًا عن نفسه أثناء الرحلة - نكتة من سلسلة Family Man...) ولا تصدق المحررين كثيرًا. القصة جميلة جدًا، وإذا كانت هناك مجموعة من القصص القصيرة المنشورة في إسرائيل، فإن قصتك، إذا اكتملت وحافظت على الجودة، تستحق بالتأكيد أن تكون هناك. كما تقيم صحيفة هآرتس، على سبيل المثال، مسابقة بين القصص القصيرة (هناك مسابقات أخرى من هذا القبيل)، ربما تستحق المحاولة.

  321. إسرائيل
    لم أفهم وجهة نظرك. عاد التوأم، العجوز في ساعة وبالفعل مدة الفيلم ساعة. إذا كان التوأم المتبقي سيصنع فيلمًا، فسيكون طوله مليار سنة. سوف "يوافق" التوأمان على هذا أيضًا.

  322. ב
    أ) يتم اختيار هذه النقطة بشكل تعسفي. وهناك عدد لا حصر له من هذه النقاط. لذلك، لا شيء خاص بها. كما هو موضح.

  323. إسرائيل:
    و)
    الكون لانهائي ومتجانس باستثناء نقطة واحدة.
    نقطة واحدة هي نقطة خاصة جدًا في الكون.
    هذه هي النقطة التي يكون فيها المشاهد.
    كل شيء يقاس. قياس نسبة إلى هذه النقطة. (فيما يتعلق بالمشاهد).
    ب)
    في تجربة ميشيلسون ومورلي:
    لا حاجة لكون لانهائي.
    ليست هناك حاجة لكون متجانس.
    من الضروري فقط أن تكون هناك بيئة كبيرة بما فيه الكفاية للراصد الذي يوجد فيه الموقع من أجل قياس أي فرق بين (سرعة الضوء بالنسبة إلى الموقع) و (سرعة الضوء بالإضافة إلى سرعة الأرض) بالنسبة إلى الموقع.

  324. وقبل أن ننتقل إلى النقطة الرئيسية، أود أن ألخص ما قلته:

    1. ماكسويل ليس عالما عاديا ومعادلات ماكسويل ليست معادلات تافهة. وقال فاينمان إن الأجيال القادمة سوف تتذكر من القرن التاسع عشر بشكل رئيسي معادلات ماكسويل.

    ومعادلات ماكسويل، مثل اشتقاق سرعة الضوء من ثوابت الكهرباء والمغناطيسية، تم اشتقاقها من نموذج ماكسويل للأثير. رأيي هو أنه من المستحيل فصل المعادلات (الشيء الوحيد الذي لم يغيره أينشتاين في العلاقات) عن النموذج. "التعقيد الذي لا ينفصم" آليك.

    2. كل موجة معروفة لنا تتحرك وفق نظام الراحة المفضل. لا نعرف أي مجال لا يوجد به نظام راحة مفضل. حاولت تجربة MM العثور على حالة الراحة المفضلة لإثير ماكسويل، لكنها فشلت.

    ومع ذلك، فإن كل نظام راحة مفضل ينبع من قيود: الهواء - كما كان في الماضي. المحيط - كما هو مذكور أعلاه. الهواء في الطائرة - الطائرة.

    ومع ذلك، في الكون اللانهائي المتجانس والمتناحي ليس هناك ضرورة لوجود تفضيل. قد يكون هناك - ولكن هذا هو النظام الوحيد (كون لا نهائي بدون قيود) الذي قد لا يوجد فيه أيضًا.

    إذا وصلنا يومًا إلى مناقشة ما أسميه "الموقع النشط" - وهو موقع لا يوجد فيه نظام راحة مفضل - فسنرى كيف يحل هذا الافتراض مشكلة عدم قابلية التفريغ لنموذج ماكسويل، ومشاكل أخرى أيضًا .

    تمديد الوقت.

    عرض المشكلة.

    هناك فرضيتان للنسبية الخاصة هما:

    1. مبدأ النسبية:

    لا تتغير قوانين الفيزياء عند الانتقال من إطار مرجعي بالقصور الذاتي إلى إطار مرجعي بالقصور الذاتي آخر. على سبيل المثال، لا يستطيع شخص في عربة قطار مغلقة، من خلال أي تجربة أو قياس فيزيائي، تحديد ما إذا كانت السيارة تتحرك بسرعة ثابتة أم أنها في حالة سكون.

    2. ثبات سرعة الضوء:

    إن سرعة الضوء ثابتة عند كل راصد، بغض النظر عن سرعته النسبية بالنسبة إلى الجسم الذي بعث الضوء، أو سرعته بالنسبة إلى أي جسم آخر.

    لماذا استنتج أينشتاين من هاتين الفرضيتين مسألة إطالة الزمن؟

    دعونا نصف أنبوبين طول كل منهما ثانية خفيفة، أحدهما قطره متر واحد والآخر قطره 2 متر.

    الفوتون الذي يغادر نهاية أحد الأنبوبين سيصل إلى الطرف الآخر خلال ثانية واحدة بالضبط.

    يقال الآن أن الأنبوب الرفيع يتحرك داخل الأنبوب العريض، وعندما تلتقي ظهراهما، يتم إطلاق فوتون واحد على طول المحور متحد المركز لكليهما في اتجاه حركة الأنبوب الرفيع.

    إذا تم إطلاق الفوتون في الوقت 0 في ساعات الأنبوبين، فسوف يصل إلى كلا الطرفين في ثانية واحدة وفقًا لساعة كل أنبوب (ساعات كل أنبوب متزامنة مع بعضها البعض، ولكنها غير متزامنة مع الآخر) أنبوب).

    عندما يصل الفوتون إلى نهاية الأنبوب الرفيع، فإنه يبرز بالفعل مسافة طويلة من الأنبوب العريض. اتضح أنه عندما مر على الطرف الآخر من الحقل، لم تكن قد مرت ثانية واحدة على ساعة المؤقت، ووفقًا للبيانات، فقد مرت ثانية واحدة بالضبط في الميدان. في صورة من كاميرا عالية الدقة ستصور الساعات الأنبوبية معًا، سنرى وقتًا قدره ثانية واحدة في الساعة الأنبوبية العريضة ووقتًا أقل من ثانية في الساعة الأنبوبية الرفيعة.

    ومن هذا الوصف البسيط يأتي إطالة الأزمنة.

    (ربما يلاحظ حاد البصر منكم مشكلة معينة في الوصف، ولكن هذا هو الوصف الذي يظهر في معظم الشروحات، بما في ذلك كتاب أينشتاين نفسه، ويكفي توضيح النقطة).

    وهذا الوصف صحيح عندما نأخذ في الاعتبار معرفة عام 1905، عام معجزات أينشتاين.

    ولكي أرى المشكلة التي أعتقد أنها تتجلى في إطالة الزمن، سأتحدث عن مفارقة التوأم (أين المعجزات؟).

    ومن أجل التوضيح، سأستخدم بشكل خاص التوائم السريعة وطويلة العمر: عندما يعود التوأم الأصغر، يجد أن أخيه قد تقدم في السن مليار سنة بينما كان عمره ساعة واحدة فقط.

    علاوة على ذلك، طوال رحلة التوأم الشاب والرشيق، قامت كاميرا فيديو داخل سفينته الفضائية بتصوير جميع الأنظمة الموجودة في سفينة الفضاء: الساعات، ومقاييس التسارع، ومقاييس الحرارة، ومقياس الرطوبة، والبوصلة (مهم جدًا في الفضاء!) إلخ.

    وعندما يعود كان يجلس مع أخيه الأكبر ويشاهدان مقطع فيديو مدته ساعة.

    وما يرونه ويتفقون عليه هو:

    1. استغرقت الرحلة بأكملها ساعة واحدة حسب الساعة المزدوجة السريعة.

    2. تم تقسيم الرحلة إلى 5 مراحل: في المرحلة الأولى تسارع التوأم (تظهر الكاميرا أن الأمر استغرق دقيقة واحدة) في المرحلة الثانية طار بسرعة ثابتة (26 دقيقة) في المرحلة الثالثة تسارع في الاتجاه المعاكس (3 دقائق) في المرحلة الرابعة عاد بسرعة ثابتة (28 دقيقة) وفي المرحلة الخامسة ركض بسرعة (تسارع سلبي) ليقارن سرعته بسرعة أخيه (دقيقتان).

    كل ذلك معًا لمدة ساعة.

    وتظهر الكاميرا أيضًا أن درجة الحرارة انخفضت خلال هذا الوقت بسبب تبريد الكون بمقدار 100 درجة كلفن. كما يوضح أيضًا المراحل بالضبط - يحدث معظم التبريد أثناء السفر بسرعة ثابتة في كلا الاتجاهين، أكثر من متر واحد في الدقيقة.

    ولكي يكونوا آمنين، أجروا التجربة مرة أخرى مع توأم آخر يطير في الاتجاه المعاكس.

    انطلق الثلاثة عند غروب الشمس وأخذوا معهم حلمًا قديمًا وخريطة وماء وكاميرات ومقاييس حرارة وساعات وما إلى ذلك.

    وعندما قارنوا الصور، لاحظوا أنه في الفيديو (المتطابق) للتوأم الرشيق، تنخفض درجة الحرارة بمعدل أكثر من درجة في الدقيقة، بينما بالنسبة للتوأم الثابت، ترتفع خلال مليون سنة على الأكثر .

    الكثير للوصف الأولي. إذا كانت هناك خلافات - وربما حتى اتفاقات - سأكون سعيداً بسماعها قبل أن ننتقل إلى النقطة الرئيسية.

    مايكل - أحسنت لرعاية أبي المتفانية. دعونا نأمل أن تجد بعض الوقت لإسرائيل القديمة أيضًا... والدي البالغ من العمر 82 عامًا، والذي يتمتع دائمًا بحالة جيدة، سيأتي لزيارتنا خلال شهر. تنطلق العائلة بأكملها في مغامرة في الصحراء: إعادة إنشاء تجربة بيساو لقياس سرعة الضوء ولكن مع تقلبات مثيرة للاهتمام. يرتبط تخطيط التجربة بشكل مباشر بكل ما تمت مناقشته هنا وفي التعليقات السابقة، لذا يرجى بذل الجهد.

    واستعارة من يودا: لا تتفاعل بلطف! فقط كما ساه!

    ولكن فليكن في صلب الموضوع، فرديًا، ومدعومًا بالحقائق.

  325. إسرائيل:
    نعم.
    وقد استشارني حماي بن نون وأدرج بعض كلامي في مقدمته.
    لماذا تقول أنهم لا يخبرونك مباشرة بعد أن أخبروك؟ 🙂

    لم أقرأ فكرتك
    كنت أنتظر عرضًا كاملاً لتوراتك ولم أتعمق في الأوصاف الجزئية أو الاشتقاقية التي أتيت بها.
    إذا كنت تعتقد أن شرحك للموضوع قائم بذاته بغض النظر عن التوراة ككل، فمرحبًا بك للإشارة إلى المكان الذي كتبت فيه، ولكن ضع في اعتبارك أن الأمر سيستغرق مني بعض الوقت لمعالجته لأنه مؤخرًا لقد كنت مشغولاً طوال الوقت تقريبًا برعاية والدي المسن.

    ليس لدى Shirigin حقًا نقطة صفر طبيعية ولا يحتاج إليها أيضًا.
    الصفر موجود هناك.
    لو كان يزعجك أن أسمي نقطة أفسس ونقطة أخرى أخيديهو؟
    هذه مجرد أسماء لا تفعل شيئًا للكون، وعلى وجه الخصوص، لا تغير نظائره.
    كما أنها لا تغير أي قياس: ستبقى المسافات بين النقاط هي نفسها، والسرعات ستبقى هي نفسها، والتسارع سيبقى هو نفسه حتى لو اخترنا الصفر في مكان آخر.

    في نموذج شاريجين وبدون أينشتاين، لدى شاريجين نظام راحة مفضل.
    يمكن وصف هذا النظام في أي نظام محاور مختار (نظام لا يتحرك بالنسبة لنقاط الانحراف) ولا يمكن وصفه في عدة أنظمة محاور بديلة، لا حرج. وكما ذكرنا: فإن اختيار نظام محوري مختلف لن يؤثر على قياسات الكميات الفيزيائية ونتائج الحسابات التي تحسبها.

    لا تفهموني خطأ: أنا لا أزعم أن مثل هذا النظام موجود في الواقع - أنا فقط أجادل ضد محاولتك لرفضه وتجربة ميشيلسون-مورلاي كما لو كانت الأمور سخيفة منطقيا.
    وكما ذكرنا - فقد أظهرت القياسات بالفعل أن هذا ليس النظام الذي نعيش فيه، ولكن القياسات كانت ضرورية وكان من المستحيل التوصل إلى نتيجة بدونها.

    تأخذ حسابات نظام تحديد المواقع العالمي (GPS) الجاذبية/التسارع والسرعة في الاعتبار بشكل منفصل وستكون النتائج خاطئة إذا تم حذف أحد هذه المكونات.
    هناك أمثلة على الأنظمة غير المتسارعة التي تثبت النسبية الخاصة: على سبيل المثال، بعض الجسيمات التي تنشأ عندما يضرب الإشعاع الكوني الغلاف الجوي لم تكن لتتمكن من عبور الغلاف الجوي إذا لم تعيش لفترة أطول.

    بالنسبة لنا، يعيشون لفترة أطول ولكن بالنسبة لهم يعيشون في نفس الوقت تمامًا ويصبح الغلاف الجوي أرق فقط.

    ولذلك أعطيتكم هنا مثالاً على إطالة الوقت وتقصيره.

    أنت لم تفهم ما قصدته بالحديث عن أينشتاين ونموذجه. لقد كانت مجرد وسيلة لتوضيح الخطأ الذي ترتكبه عندما تطبق نفس الاعتبار على نموذج ماكسويل.

    وكما ذكرت - لقد أجبت على أسئلتك - وكذلك السؤال المتعلق بوقت الانفجار.

  326. شموليك، شكرا لك.

    لقد أرسلت الحلقات إلى عدة أنظمة. تلقيت رسائل تقدير مهذبة، مع تعليق مفاده أن القصة ليس لها أي إمكانات مالية. ليس لدي أي نية لنشرها بنفسي، ليس بالضرورة بسبب نقص الإمكانيات، ولكن بصراحة، لا أعتقد أن القصة تبرر نشر كتاب. جاليليو موقع الآلات والعالم بحجمه الطبيعي.

    ميخائيل.

    ولم أعلم أن حامي بن نون استشارك، ولا يقال لنا أي شيء!

    لو قرأت فكرتي وفهمتها هل ترى كيف تحل مشكلة الاحتكاك عند ليساج بطريقة سلسة؟ مصدر الجمود (غير مغلق)؟ احتمال غير محلية؟

    لا يحتوي نموذج Sharigin على نقطة 0 طبيعية. يمكننا تحديده بشكل تعسفي - كما هو الحال مع مقاييس درجة الحرارة - ولكن بعد ذلك يتوقف عن كونه متناحيًا بالنسبة لنا.

    يمكننا القول أنه إذا كان تشتت العينة غير منتظم فإن النقطة 0 هي مركز ثقل منطقة معينة على سبيل المثال. لهذا السبب ذكرت مفارقة أولفرز. لكن إذا قارنا ذلك بنظام الراحة الأثير، فإنه يطرح أسئلة جديدة: إذا كان نظام الراحة يتناسب مع توزيع الكتل وثقل حركاتها، فهل هو مركز الثقل والحركة الكونية؟ هل لدى كل مجرة ​​ونظام شمسي نظام راحة مختلف؟ هل هناك سرعة مختلفة للضوء في مناطق مختلفة من الكون؟ ربما يفسر هذا أيضًا سرعة النجوم الموجودة في حواف المجرات التي لا تحتوي على مادة مظلمة؟

    يقول أوكهام: لا يوجد نظام راحة مفضل (أينشتاين أيضًا).

    الحديث عن أينشتاين:

    تعد كل من الجسيمات الأولية ذات الأعمار الأطول وأنظمة تحديد المواقع بمثابة أنظمة في التسارع. لقد كان إنجازي دائمًا يتعلق بأنظمة القصور الذاتي. هل يمكنك الإشارة إلى نظام واحد فقط لا يتم تسريعه ويوجد فيه تمديد زمني؟ مثال واحد لتقصير الطول؟

    "هل من الممكن أن يكون النموذج الذي بناه أينشتاين والذي قام على أساسه تطوير النظريات النسبية التي تتوافق مع العديد من التنبؤات خاطئا؟"

    لا. لكن نموذج نيوتن يتوافق أيضًا مع العديد من التوقعات. تعتمد عليه الكثير من التقنيات. لماذا قام أينشتاين بتغييره؟ هل هو مخطئ؟

    لقد كان إنجازي دائمًا يتعلق فقط بموضوع تمديد الوقت في أنظمة القصور الذاتي. كل الأشياء الأخرى يمكن شرحها بشكل جيد.

    وأختتم كلامي بالقول إن عدم الفهم، في رأيي، يكمن في الحقيقة. لكن الطريقة الوحيدة التي أعرفها هي الاستمرار في التذمر وسؤال الجميع والإشارة إلى التناقض المفترض بين زمن الانفجار المطلق والزمن النسبي في النسبية، وآمل أن يأتي يومًا ما شخص يفهم الموضوع - إيهود، تسفي، وربما أنت أيضًا - سأوافق على تكريس الجهد والوقت اللازمين للإجابة على أسئلتي بشكل فردي.

    فهي ليست صعبة للغاية.

  327. إسرائيل:
    لا أعرف ماذا يعرف الشخص الذي تراسلته، لكن من المؤكد أن ما يعرفه، الجسيمات الأولية التي تتمكن من العيش لفترة أطول بكثير عندما تتحرك بسرعة، لا تعرف.
    كما تعلمت ساعات الأقمار الصناعية لنظام تحديد المواقع العالمي (GPS) نظرية النسبية الخاصة والعامة وهذا هو السبب الوحيد لعملها.
    والآن سأطرح عليك نوع السؤال الذي تحب أن تسأله:
    هل من الممكن أن يكون النموذج الذي بناه أينشتاين والذي قام على أساسه تطوير النظريات النسبية التي تتوافق مع الكثير من التنبؤات خاطئا؟
    في حالة تطابق هذا النموذج مع كل ما توقعه ماكسويل وأكثر من ذلك بكثير.

    أنت تدعي أنه يمكنك بناء نموذج حيث سيكون هناك موقع، وستعمل معادلات ماكسويل وسيتم أيضًا شرح تجربة مايكلسون مورلي.
    دعنا نقول - ولكن بالتأكيد لن يكون نموذج ماكسويل أيضًا!

    لقد أخبرت معجزات نموذج سارجين أنه في الواقع هناك مشكلة في نقطة الصفر.
    من الواضح أنك مخطئ! في كل مكان تقريبًا قمنا بتعيين الصفر بشكل تعسفي! تمامًا كما هو الحال في النموذج "غير الواقعي"!
    الصفر هو النقطة المرجعية ولا شيء غير ذلك. إنه مجرد اسم يطلق على نقطة ما.
    في قياس درجة الحرارة هناك عدة أصفار - اعتمادًا على ما إذا كنت تسأل درجة مئوية أو فهرنهايت أو كلفن.
    هناك قيمة يمكن تسميتها بالصفر المطلق على وجه التحديد لأنه لا يوجد تناظر، ولكن بمجرد وجود تناظر، لن تكون هناك أولوية للنقطة A على النقطة B ويكون الصفر موجودًا.

    بالمناسبة، في المقال المؤطر يقول أن رؤوفين نير كان يعلم أنه كان يعيد تدوير Le Sage.

    اتصل بي المحرر العلمي للكتاب وسألني عن رأيي حتى قبل نشر الكتاب وأشرت له إلى حقيقة أنها دورة وحقيقة أن الأصل قد تم دحضه.
    وإن لم أكن مخطئا فقد ذكر في المقدمة أنه كتب أنها دورة ولم يشر إلى الدحض.
    ويبدو أن المال كان أكثر أهمية بالنسبة له من العرض الكامل للحقيقة.

  328. المعجزات

    وصف مايكل لأول مرة الفضاء المتجانس والمتناحٍ الذي أطلق عليه اسم "Shrigin".

    هل يمكن لـ Shirigin الحصول على نقطة 0؟ نعم. هل هناك مشكلة لمثل هذه المساحة مع النقطة 0؟ لا. ليس كثيرًا، حيث أن أي نقطة يمكن أن تحتوي في الواقع على 0.

    هل يمكن أن تحتوي مساحة Regin الفعلية أيضًا على نقطة 0؟ نعم. هل هناك مشكلة لمثل هذه المساحة مع النقطة 0؟ نعم! لأنه إذا كانت أي نقطة يمكن أن تكون النقطة 0، فلماذا لا يكون لدينا بعض النقاط 0؟ لماذا لا اللانهاية؟

    وبالعودة إلى الفضاء اللانهائي لعام 1887: هل يمكن أن يكون لديه نظام راحة تفضيلي؟ نعم! ولكن إذا كان متجانسًا إلى هذا الحد، فلماذا يكون له نظام راحة واحد فقط، وهو نفس النظام الذي حاولت تجربة MM العثور عليه؟

    إذا كان هناك أكثر من واحد ولم نتمكن من تحديدهم، فإن تجربة م-م غير منطقية.

    على أية حال، مازلت أعتقد أنه لا بد من وجود إجابة تافهة لهذا السؤال. ومن غير المعقول أن الجميع – من نيوتن إلى لورنتز – أغفلوا هذه النقطة.

    حسنًا ، تكريماً للأيام الرهيبة ، من المستحيل بدون نكتة لكم أيها الملحدون:

    يصل مفوض المفوض ميشكا إلى المزرعة الجماعية ويسأل المدير يشكا عن محصول البطاطس لهذا العام.

    يجيب "المفوض" على إيشكا بعينين لامعتين: "لدينا الكثير من البطاطس التي عندما نجمعها في كومة ستصل إلى أقدام الله!

    "خرشو يشكا" يضحك ميشكا "أنت تعلم جيدًا أنه لا يوجد إله!"

    "محراث ميشكا" يصرخ إيشكا "أنت تعلم جيدًا أنه لا توجد بطاطس!"

    طاب مساؤك.

  329. إسرائيل،
    ادعو الباحث للتعليق هنا وليس فقط على الموضوع الذي تتحدث عنه. من الجيد دائمًا أن يكون لديك خبراء في منتدى مثل هذا.
    بخصوص القصة احتراما للفصل المنشور! وماذا ستفعل بالقصة؟ هل هو مكتوب بالفعل؟ هل تخطط لإصداره بالكامل في وقت ما؟ هيا بالفعل معك

  330. إسرائيل
    ليس لدي أي جدال معك بشأن نيوتن. كانت لديه آراء غير عادية حول الثالوث الأقدس، لكنه ظل يؤمن به.

    لقد كتبت "إن النظام المفتوح وغير المحدود به مشاكل من شأنها أن تكون نظام الراحة المفضل. لقد قمت بتحميل بعضها من قبل. ولكن على أية حال، إذا لم تكن هناك مشكلة في وجود نظام راحة مفضل، فلماذا يواجه الخط المستقيم اللانهائي مشكلة إذا تم تفضيل النقطة 0؟". لا أرى مشكلة إذا كان هناك خط مستقيم لا نهائي له نقطة 0 مفضلة. محور الأرقام يكون هكذا - مستقيم، لا نهائي وإذا كانت النقطة 0. بدلاً من القيمة العددية، ضع مسافة من 0... ولا تزال هناك مشكلة. أتفق معك في أنها ليست متجانسة تمامًا، لكن لا أحد يدعي أن مساحة السرعة متجانسة. في فهمي على الأقل…

  331. ميخائيل.

    صحيح أنه في بعض الأحيان يؤدي النموذج الخاطئ إلى نتيجة صحيحة. وفي حالة نظرية ماكسويل المعقدة والمعقدة، يبدو لي أن الاحتمالية منخفضة للغاية.

    ادعى ماكسويل نفسه أنه فقط عندما لاحظ أن السرعة التي حسبها للموجات الكهرومغناطيسية في موقعه (المشتقة من حساب سرعة الموجة الصوتية في الغاز) كانت هي نفس سرعة الضوء المحسوبة في تجربة بيتزو، أدرك وهذا الضوء بحد ذاته عبارة عن موجة كهرومغناطيسية.

    تكتب: "إذا كان الحل مناسبا لحساب حركة الموجة، ولكنه غير مناسب لافتراض أن سرعة الضوء هي نفسها بغض النظر عن حالة الحركة بالنسبة للوسط، فإننا بالضرورة في حالة الموقف الذي تلقينا فيه معادلات صحيحة من نموذج غير صحيح".

    ليس بالضرورة. هناك خيارات أخرى. وقد اقترح لورينز وآخرون حلولاً ممكنة تقبل نموذج ماكسويل. واقترحت أيضا حلا.

    "يجب أن أقول إنني أجبت أيضًا على الأسئلة التي تدعي أنك طرحتها وبقيت دون إجابة، وفي رأيي أن الإجابات التي قدمتها صحيحة تمامًا. حقيقة أن الإجابات لم تريح بالك ليست تحت سيطرتي، لكني أشعر أنك بخير."

    إليك جزء من رسالة بريد إلكتروني تلقيتها من شخص كان يبحث في هذا الموضوع لسنوات عديدة. وبما أنني لم أحصل على إذن منه، فقد قمت بمراقبة تفاصيل التعريف.

    في 2 فبراير 2013 الساعة 1:01 مساءً.

    "إسرائيل، السلام عليكم!

    لقد قمت بالبحث في موضوع النسبية الخاصة لأكثر من عشر سنوات ولدي شكوك أساسية حول التفسير القياسي للنسبية الخاصة. أريد أن أقول إنني أقبل الافتراضات الأساسية للنظرية النسبية، أي ثبات سرعة الضوء وتباين قوانين الفيزياء في جميع أنظمة القصور الذاتي وبالطبع تحويل لورنتز وجميع نتائج القياس للتجارب التي تم القيام بها حتى الآن.

    منذ مقالتي التي قرأتموها، قطعت شوطًا طويلًا وحققت الكثير من التقدم، وحجتي اليوم أكثر أساسية بكثير. وهذا يعني أن التزامن ليس مطلقًا فحسب، بل إن الوقت أيضًا مطلق في جميع أنظمة القصور الذاتي! وقد ألقيت محاضرات حول هذا الموضوع في مؤتمرين دوليين: IARD2008 الذي عقد في سالونيك، اليونان وIARD2012 الذي عقد في مايو ويونيو الماضيين في فلورنسا، إيطاليا. وأنا الآن بصدد كتابة مقال حول هذا الموضوع بعنوان:

    فإذا كنت شعرت بالماضي، أو اعتقدت أن الزمن يجب أن يكون مطلقًا، على الأقل وفقًا للساعة الكونية، فأنت على حق! أنتم في صحبة جيدة، على الأقل في "شركتي" وشركة البروفيسور لاري هورويتز من جامعة تل أبيب الذي يدعم نهجي. سوف يستغرق الأمر المزيد من الوقت والجهد لإقناع بقية مجتمع الفيزياء في العالم، ولكن هذا الشيء سيأتي! أنا مقتنع 100% أنني أسير على الطريق الصحيح..

    قرأت الهراء المكتوب في علمهم، وبعضه ردًا على أسئلتك - هراء الأشخاص الذين لم يتعمقوا أبدًا في الموضوع وهم فقط "يتحدثون". أنت الوحيد الذي كتب وقال أشياء معقولة!"

    الآن، بالطبع، أنا لا أدعي أن هذا الشخص على حق بالضرورة - في الواقع أنا أختلف مع بعض أخطائه - ولكن هنا خبير واحد على الأقل يقبل حجتي الأساسية. انه ليس الشخص الوحيد.

    صحيح أنك أوضحت لي لماذا أنا مخطئ، لكننا لم نتوصل أبدًا إلى مقارنة حقيقية بين أخطائك وأخطائي. أعتقد أنك مخطئ، إذا كنت مهتمًا فيمكنني أن أوضح لك المكان بالضبط. وحتى ذلك الحين، لم تكن ترغب في التعمق في المناقشة، وهذا حقك.

    بخصوص نشر مقال - أعرف موقعاً يُنشر فيه كل أسبوع مقالات لما أسميه "الفيزياء البديلة". في رأيي أن المقالة التي لا تحتوي على معادلات دامغة تدعمها كما فعل ماكسويل هي مقالة ضعيفة. نشر رؤوفين نير "ميشكا" في عصر ما قبل الإنترنت. اليوم، في غضون 10 دقائق من اللعب، سيكتشف أنه كان في الواقع يغازل ليساج فقط، مما يوفر على نفسه المتاعب. انه ليس الشخص الوحيد.

    التجريب في مستوى مختلف، فمن الصعب الجدال حول التجريب. هذا هو اتجاهي الآن.

    مساء الخير.

  332. إسرائيل:
    يجب أن أعترف أنني لم أخوض في تفاصيل النموذج، ولكن في كثير من الأحيان يحدث أن تؤدي الحقائق المختلفة إلى حلول رياضية مماثلة (على سبيل المثال، جميع أنواع الموجات/التذبذبات) موصوفة بمعادلات مماثلة.
    أعتقد أن الحقيقة عادةً ما تكون حقيقة أيضًا، وإذا كان الحل مناسبًا لحساب حركة الموجة ولكنه غير مناسب لافتراض أن سرعة الضوء هي نفسها بغض النظر عن حالة الحركة بالنسبة للوسط، فنحن بالضرورة في موقف تلقينا فيه معادلات صحيحة من نموذج خاطئ.
    ويجب أن أقول إنني أجبت أيضًا على الأسئلة التي تدعي أنك طرحتها وبقيت دون إجابة، وفي رأيي أن الإجابات التي قدمتها صحيحة تمامًا. إن الإجابات لم تريح عقلك هو أمر خارج عن إرادتي ولكني أشعر أنك بخير.

    وفيما يتعلق بالامتناع عن تقديم نظرية لا تعرف مدى صحتها - فأنا على يقين أنني لن أخبرك بشيء إذا قلت أن هذا يعني أنك لن تنشر النظرية أبدًا لأنك لن تعرف أبدًا.

  333. ميخائيل

    أنا لا أعرض "نظريتي"، لأنني لست متأكدا على الإطلاق من أن لدي واحدة. في الوقت الحالي، ليس لدي سوى أسئلة، ومسألة منطق وجود نظام راحة للموقع ليست سوى واحدة منها. عندما طرحت هذا السؤال على الموقع قبل عام، كنت على قناعة تقريبًا بأنني سأحصل على إجابة بسيطة وتافهة، مثل حلك للخاصية الثالثة "غير القابلة للاختزال" للأعداد المثالية.

    لم يجب أحد سوى محمود الذي ذكر بإيجاز أن نظام الراحة أمر طبيعي بالنسبة لنا. مراسلاتي مع علماء الفيزياء، وبعضهم معروفون، لم تساعد في حل مسألة الصراع القائم ظاهريا بين الزمن النسبي لأينشتاين والزمن المطلق الناتج عن الانفجار الأعظم. في الواقع، كل واحد جاء بإجابة مختلفة، وكلهم يناقضون بعضهم البعض.

    كما أقوم بنشر الأسئلة في مواقع الفيزياء المختلفة، لكن النتائج سيئة.

    إن كتابة مقال منظم حول نظرية لا أعرف حتى ما إذا كانت صحيحة يبدو لي هراءً. أفضل طرح الأسئلة والتجربة. لا أعلم إذا كانت ستؤدي إلى أي شيء، لكنها سبق صحفي كبير، ناهيك عن الإحساس بالهدف.

    حسنًا، هناك سؤال صغير واحد فقط سيتطلب دقيقتين من التفكير:

    إذا ألقيت نظرة على نموذج موقع ماكسويل:

    http://en.wikisource.org/wiki/On_Physical_Lines_of_Force

    هل تعتقد أن مثل هذا النموذج التفصيلي والدقيق كان من الممكن أن ينتج المعادلة 136 إذا كانت خاطئة بشكل أساسي؟

    بالمناسبة، هذا هو نفس النموذج الذي تم اشتقاق معادلات ماكسويل التفاضلية منه. إذن كل شيء افتراضي؟

    على حد علمي، مات ماكسويل (شابًا) معتقدًا أن نموذجه يصف الواقع وليس الصورة.

    لعبة البوكر.

  334. رئيس،

    وفقًا لجليك، لم يشك نيوتن في الإيمان الأعمى بالله فحسب، بل كان أيضًا من بين القلائل الذين تجرأوا على التحقيق في جوهر "الثالوث الأقدس" وخاطروا بمواجهة محاكم التفتيش.

    لقد كان أحد القوى الدافعة وراء الكنيسة الأنجليكانية. كان انشغاله بالدين والله مهووسًا وعميقًا مثل أي موضوع آخر. أعتقد أنه كان يعرف الموضوع ويفهمه أفضل بكثير من كثيرين منا، وهذا المتشكك الكامل في أي موضوع وفي أي اعتقاد سابق لم يكن ليضع لنفسه دعامة غير ضرورية.

    لا أعرف ما هي اعتباراته في إيمانه التقي، لكني أعتقد أن "روح العصر" ليس هو السبب. حتى ديكارت من قبله كان يشك في كل شيء، بما في ذلك الإيمان بالله. لكن ديكارت كان يخاف من محاكم التفتيش، ونيوتن لم يكن يخاف من أي شيء (إلا النساء...).

    يعاني النظام المفتوح واللانهائي من مشاكل من شأنها أن تكون نظام الراحة المفضل. لقد قمت بتحميل بعضها من قبل. ولكن على أية حال، إذا لم تكن هناك مشكلة في وجود نظام راحة مفضل، فلماذا يواجه الخط المستقيم اللانهائي مشكلة إذا تم تفضيل النقطة 0؟

  335. إسرائيل
    آمن أرسطو بخمسة عناصر، وليس بأربعة. ونعم، كان يؤمن بالكثير من الهراء - لأنه لم يتحقق. مجرد مثال أتذكره - كان يعتقد أن الرجال لديهم أسنان أكثر من النساء، ولم يكلف نفسه عناء التحقق من عدد أسنان زوجته في فمها.

    كان نيوتن يؤمن أيضًا بالهراء، إذ افترض وجود الله، ولم يشك في ذلك أبدًا.

    ولا، أنا لا أقول أن النظام اللانهائي والمفتوح يجب أن يكون النظام المفضل. قلت إنني لا أرى مشكلة في وجود نظام راحة كهذا.

  336. أشباح، شموليك.

    ظهر الفصل الأول من "محاكمة القانون الثاني" في ذلك الوقت في مجلة غاليليو الشهرية. تجدونه على موقع آلية وزارة التربية والتعليم (تصفيق!).

    http://www.amalnet.k12.il/machine/show_item.asp?item_id=14600&id=1425&level=1&num=3

    هنا هو الاستمرار:

    لقد عاد القانون إلى رشده. "حسنًا، فليكن، لا أنوي الإضرار بحرية معتقدك. حسنًا، هناك إله! هل يمكننا العودة إلى المنزل بالفعل في سبيل الله؟"

    قال المدعي العام بحزم: "لا. السؤال الذي تُحاكم عليه لا يزال مفتوحًا: لماذا تزداد الإنتروبيا بمرور الوقت؟"

    "انظر" قال القانون في ليوت. "قد تستمتع بهذا النقاش الأكاديمي الدائري الذي لا يؤدي حقًا إلى أي شيء. لقد شرحت لك أن هذا يحدث للتو. احتمال. ماذا يمكنني أن أفعل بحق كل الشياطين والأرواح حتى تخبرني على الأقل لماذا تعتقد أن هذا خطأي؟" رفع لو صوته بالإحباط.

    تبادل المدعي العام والأستاذ النظرات.

    "حسنًا!" وطالب القانون.

    "لقد قلت ذلك للتو."

    "قلت ما؟"

    "الشياطين والأشباح"

    "وماذا في ذلك؟"

    وصل صوت المدعي العام إلى تصلب. "أيها المدعى عليه، هل تعترف بأنك وحدك المسؤول عن زيادة الإنتروبيا في الكون؟"

    أ. نعم.

    الذي - التي. لزيادة المعاناة الإنسانية؟

    أ. نعم.

    الذي - التي. للأمراض؟

    أ. نعم.

    الذي - التي. إلى الشيخوخة؟

    تصميم.

    الذي - التي. حتى الموت؟

    أ. نعم نعم.

    الذي - التي. للأعطال الميكانيكية بكافة أنواعها؟

    أ. نعم نعم نعم!

    الذي - التي. هل سمعت بالشيطان؟"

    كان الجمهور مرعوبا.

    أ. نعم!

    الذي - التي. هل تعرف غرض الشيطان؟

    أ. نعم!

    الذي - التي. هل يمكنك إخبارها للمحكمة؟

    أ. زيادة معاناة الإنسان والتسبب في المرض والشيخوخة والموت والأعطال الميكانيكية..

    الذي - التي. شكرا. البروفيسور ليبنوفيتس، كعالم رياضيات، هل يمكنك أن تخبر المحكمة بقاعدة المساواة؟

    "شيئان يساويان شيئًا ثالثًا متساويان بينهما" قال الأستاذ بتقوى.

    "شكرا لك ايها البروفيسور. أيها المتهم، هل تعترف أنك في الواقع الشيطان؟"

    وتجمد الحشد في مكانه من الرعب. وهنا نعرف ما هو المتهم بالقانون الثاني، وليس هناك ذنب أعظم من هذا!

    "هل أنا شيطان؟" استغرب القانون: "بالطبع لا! احكموا على أنفسكم"، خاطب الجمهور في نداء. "هل يبدو لك أنني قادر على ضرب ذبابة؟"

    صاح المدعي العام: "لا تصدقه، فالشيطان معروف بأنه مجرب ومخادع، ويمكنه أن يتخذ أي شكل يناسب احتياجاته. لقد اعترف بالفعل بأن هدفه هو التسبب في فوضى للاحتفال".

    "أنت مخطئ" دعا القانون فوق العاصفة في قاعة المحكمة. "لم أعترف بأن الأمر كان متعمدًا، قلت إنه يحدث للتو، وبدون دليل على النية لا يمكنك إثبات الذنب، إلا إذا اتهمتني بالإهمال الجنائي - فالتهمة لا أساس لها من الصحة بكل المقاييس".

    "يا!" صاح الأستاذ. "لقد أثبتت لك من خلال الرياضيات والهندسة أنه، خلافًا لادعائك، يمكن أن يزيد النظام في النظام ويمكن أن تنخفض الإنتروبيا."

    "لكنني أتيت من عالم قوانين الفيزياء .."

    "خاصة في عالم الفيزياء!" بدأت الأرض ككتلة ساخنة من الحمم البركانية - وشاهدها الآن! كلها مزهرة ومربكة، مأهولة وصناعية! ألا تسمي ذلك انخفاضًا في الإنتروبيا؟ ولولا تدخّل القدير لبقي عالماً بارداً ومات كبقية الكواكب المختلطه بنعمة القانون الثاني الرحيمة!"

    "صحيح، ولكن هذا ليس نظاما مغلقا!"

    "هذه هي النقطة بالضبط!" ورفع المدعي العام يده لتحريض الجمهور الهائج. "في كل نظام هناك صراع بين قوى الخير وقوى الشر - بين الله والشيطان - بين الله تعالى والقانون الثاني للديناميكا الحرارية - وعندما تنتصر يد الله تعالى يتولى القانون الثاني على الفور لنظام جديد لم يتمكن من الإفلات من براثنه. وهذا ليس إهمالاً أو خطأً أو فرصة عمياء. هناك طريقة وهدف ونية خبيثة هنا! توقف عن وقاحتك السخيفة - اعترف بالذنب على الفور واطلب رحمة المحكمة، وإلا سأقبل عقوبتك بعبقرية!"

    "لكنني بريء! إنه مجرد احتمال - الشيطان يكمن في التفاصيل الصغيرة - تفاصيل لا حصر لها!"

    "كاذب! كذب وكذب!" ضرب البروفيسور بقبضته على المنصة بغضب، مما أدى إلى دوران المنصة 180 درجة، ووجد الأستاذ نفسه مرعوبًا على الجانب الآخر من المنصة وهذه المرة بصفته المتهم، في مواجهة القانون الثاني للديناميكا الحرارية، انتشرت ابتسامة القطط على وجهه من الأذن إلى الأذن.

    "أوه، ليبنوفيتس، ليبنوفيتس،" تمتم بالقانون تجاه الأستاذ والجمهور المذعور. "ماذا فعلت هذه المرة يا ليبنوفيتس؟"

    وبينما كان متمددًا على كرسيه، وكأسًا من النبيذ بجانبه، وينظر إلى كومة من الصحف التي تتناول المحاكمة، سمح المدعي العام لنفسه بابتسامة صغيرة من الرضا.
    ومن كل أصقاع البلاد، ومن كل أطياف الآراء، ومن مختلف التيارات السكانية، صرخ الكتاب ذوو الحدة واندفعوا للدفاع عن معتقداتهم وأفكارهم، ورمي المنجنيق بعضهم على بعض. وكعادته في مثل هذه الحالات، حتى في كلامه عن الآخرين، كل واحد في نفسه معجزة. وسرعان ما امتد النقاش العام وتحول إلى جدل حول صحة واكتمال الرياضيات.

    "دراما المحكمة! أما القانون الثاني فقد تم مقارنته بالشيطان!" - أُعلن العنوان الرئيسي لـ "اكتمال الرياضيات" - "تمكن البروفيسور ليبنوفتز من إثبات أنه، خلافًا لادعاء القانون الثاني، يمكن أن يزداد الترتيب في النظام الرياضي عندما نضيف أعضاء إلى النظام، والإنتروبيا ، كما هو مطلوب، والنقصان."

    "الغرور وسوء النية! الرياضيات بعيدة عن الكمال!" شعور المحرر بـ "عدم اليقين". "في ثلاثينيات القرن العشرين، أظهر كيرت جاديل أن هناك العديد من النظريات في الرياضيات التي لا يمكن إثباتها. حتى قمة الرياضيات النيوتونية، حساب التفاضل والتكامل المتناهي الصغر، تقع على ركبتي الدجاج. ما هو نوع الاكتمال الذي يمكن للمرء أن يتوقعه من التوراة التي حجر الزاوية فيها هو التقسيم المحظور على صفر؟ أطلقوا القانون فوراً مع رسالة اعتذار عن جهل النيابة".

    "وقح! حقير! متهور!" كان محرر "أبانا السماوي" غاضبًا - "ألا يكفي أن القانون الثاني غير الشرعي هو في الأساس متمرد ومرتد، كما أنه يستعين بنظرية التطور الملتوية لمساعدته؟ فعليه أن يسقط على وجهه فورًا، ويتفكر في أفعاله، ويطلب المغفرة من الله عز وجل!

    "القانون الثاني صحيح تمامًا ولايبنوفيتز يربك العقل" - أوضح المعلق العلمي لـ "أبيقور" بشكل معتدل. "لقد أظهر بولتزمان بالفعل في القرن التاسع عشر أن سبب القانون الثاني احتمالي - فالنظام الديناميكي الحراري ببساطة "ينتقل" بين جميع الحالات الممكنة، وماذا يمكننا أن نفعل عندما يكون هناك حالات من الفوضى أكثر بكثير من الحالات المنظمة؟ إن ادعاء البروفيسور السخيف بـ "النظام" يشبه تسليط الضوء على الفائزين الأفراد في اليانصيب، وتجاهل الآلاف العديدة الذين لم يفزوا حتى بفلس واحد. هيا يا ليبنوفيتس، تتحسن."

    "هنا لدينا قائد عظيم في صورة البروفيسور ليبنوفتز" قال محرر "مبادئ الرياضيات" بحماس، "صاحب رؤية بعيدة النظر سيجدد سعي برتراند راسل القديم لإنشاء بنية رياضية مثالية وخالية من العيوب."

    كل هذا لخصه شخص مجهول نقلاً عن كبير كهنة نظرية الأعداد، أندريه ويل: "الله موجود لأن الرياضيات متسقة، والشيطان موجود لأننا لا نستطيع إثبات ذلك".

    سكب المدعي لنفسه كأسًا آخر من النبيذ ورفعه في إشارة تهنئة صامتة.
    "يحيا الحاخام الماكر" بارك نفسه في قلبه. "عاش المتلاعب العظيم بجدران المحكمة. تتصادم الأطراف المختلفة بشكل أعمى مع بعضها البعض، تمامًا كما تصورت خطتي الأصلية. إنهم لا يعرفون - وهذا أمر جيد - أن النتائج محددة مسبقًا، وأن جميع البطاقات الموجودة في المجموعة محددة. ومن يملك صلاحية تحديد تشكيل هيئة المحلفين، مثلي، يمكنه أن يصدر الحكم كما يشاء".
    بنظرة متعجرفة، بدأ ينظر إلى المجلة الشهرية التي كانت تحمل اسم "أخبار الميكانيكا النفسية" وأحس كيف يتجمد الدم في عروقه ببطء.

    "حتى القرن التاسع عشر" - كتب رجل يُدعى "رجل الميكانيكا النفسية" - حاول العديد من العلماء بناء "آلة متنقلة دائمة" - آلة تغذيها الطاقة التي تنتجها بنفسها. وقد تحطم هذا الأمل باكتشاف قانون حفظ الطاقة، المعروف أيضًا بالقانون الأول للديناميكا الحرارية.
    وكانت الخطوة التالية هي محاولة بناء آلة تنتج الطاقة مباشرة من الحرارة، وذلك باستخدام الطاقة الحرارية المخزنة في مياه البحر على سبيل المثال. وقد تحطم هذا الأمل أيضًا مع اكتشاف القانون الثاني للديناميكا الحرارية، الذي يتطلب وجود خزان للحرارة عند درجة حرارة منخفضة لإنتاج شغل من النظام، ويستبعد إمكانية حدوث انخفاض عام في الإنتروبيا.

    "نحن نعتقد أن قوانين مماثلة تنطبق على النظم البشرية، وهذا يعني أن التحسن في نظام واحد يجب أن يسبب تدهورا في نظام آخر. "القانون الثاني للميكانيكا النفسية" - قانون زيادة الإنتروبيا البشرية في الأنظمة الميكانيكية النفسية المغلقة - هو السبب، في رأينا، في زعزعة استقرار العديد من العلماء العظماء، الذين دفعوا صحتهم العقلية وحتى حياتهم من أجل العالم. مساهمة هائلة قدموها للإنسانية. نجوم الفكر مثل بولتزمان، لايبنتز، جيديل، هاردي، تورينج، كانتور، وحتى ديكارت ونيوتن الأسطوريين."

    وواصل وكيل النيابة القراءة بشكل محموم حتى وصل إلى الفقرة التي تسببت في خفق قلبه وسقوط الزجاج من يده.

    "نعتقد أنه في المحاكمة الحالية للقانون الثاني للديناميكا الحرارية، لن ينجح الادعاء في التوصل إلى إدانة، بغض النظر عن نوعية الأدلة أو تكوين هيئة المحلفين. والسبب هو أن المحلفين المعزولين في غرفة يشكلون نظامًا نفسيًا ميكانيكيًا مغلقًا، وبالتالي فإن أي قرار يتخذونه، سواء بالبراءة أو الإدانة، يعني تقليل الإنتروبيا النفسية الميكانيكية الخاصة بهم. تشير الملاحظة المتعمقة جنبًا إلى جنب مع التحليل النفسي الميكانيكي للقانون الثاني، كما يتضح من سلوكه وردود أفعاله في المحاكمة، بوضوح إلى أن الإنتروبيا النفسية الميكانيكية للقانون الثاني للديناميكا الحرارية لا يمكن زيادتها. لذلك، وفقًا للقانون الثاني للميكانيكا النفسية، حُكم على المحلفين بالبقاء في حالة دائمة من عدم القدرة على اتخاذ القرار.

    ربما القانون الثاني ليس بريئًا على الإطلاق، فكر المدعي العام بمرارة وهو يأخذ مكانه أمام المنصة في انتظار استئناف الأدلة والاستجواب.

  337. معجزات.

    عندما كان نيوتن يمارس الكيمياء، كان يعتقد أنه يمارس العلوم الجادة. يزعم العلماء أن أينشتاين كان متورطًا في الهراء في سنواته الأخيرة. اعتقد أرسطو أن هناك أربعة عناصر فقط، وكان مخطئًا بشأنها جميعًا. هل انخرط أرسطو أيضًا في الهراء؟

    وهي هراء في الماضي. في ذلك الوقت كانوا يعتبرون خطيرين للغاية.

    أنا لا أؤمن بالله لأنني لا أرى أي دليل على وجوده، ولكن إذا كان نيوتن قد فعل ذلك فهذا يجعلني أتساءل عما إذا كان مثل هذا العبقري قد رأى ما أفتقده.

    والادعاء بأن هذا كان روح العصر غير مقبول. وفي بداية النهضة كثر الهراطقة، قبل نيوتن وبعده. وفقا لبرونو، وهو من أشد المعجبين بنظرية مركزية الشمس، وجاليليو وأكثر من ذلك.

    ولأغراضنا: هل يمكنك شرح كيف تمكن ماكسويل من استخلاص سرعة الضوء من نموذج الأثير إذا كان النموذج خاطئا؟

    هل يمكنك توضيح لماذا يجب أن يكون للنظام المفتوح اللانهائي نظام راحة مفضل؟

    ويمكنك الاتصال بي ببساطة إسرائيل. لقد فعلت هذا بالفعل عدة مرات.

  338. إسرائيل:
    كل هذا الحديث عن هراء نيوتن لم يكن مبادرتي وعندما سألت عن الهراء الذي ذكره الآخرون شرحت لهم ماذا يقصدون. وأعتقد أيضًا أنه من الخطأ الحكم على شخص دون الإشارة إلى الفترة التي عاش فيها، وأشير بنفس الدرجة من الضحك إلى "البراهين" التي يقدمها الحاخامات الذين يزعمون أنه يؤمن بالله (وبالتالي يجب على الجميع أن آمن مثله).
    وأكثر من ذلك - أعتقد أنه حتى الذين تحدثوا عن هراء نيوتن لم يفعلوا ذلك للحكم عليه بل ليبينوا أن المعرفة تتقدم ويصح في هذا السياق تقديم معتقدات نيوتن على أنها هراء.
    ليس الآخرين فقط - أنا أيضًا مهتم بنظريتك، لكنك لا تعرضها على الإطلاق، فكيف يمكن علاجها. ولهذا السبب كتبت أنك بحاجة إلى تقديمها بطريقة علمية لأنه من الصعب الرجوع إلى استنتاجات نظرية غير معروفة وطالما أنك لا تقدم النظرية - فإن تقديم الاستنتاجات هو التلويح باليد.
    هناك فرق كبير بين "البيع" الذي تقوم به لشيء لا يعرف أحد ماهيته لأنك لا تقدمه وبين النصيحة التي أقدمها في السياقات ذات الصلة بقراءة ما أفكر فيه حول هذا الموضوع أو ذاك بما في ذلك كل ما يتعلق لدي مبرر لأفكاري. على عكس الأشياء التي تقولها عن نظريتك والتي لا أحد يعرف ما هي - كل الأشياء التي يجب أن أقولها حول المواضيع التي أذكرها في المدونة مكتوبة فيها ويمكن لأي شخص أن يقرأها ويقرر ما إذا كان يوافق على تفكيري أم لا .
    يجب أن أقول إنني لا أفهم الشعور بالألم الذي تنقله في ردك علي.

  339. إسرائيل شابيرا
    اعتقد نيوتن أن هناك احتكاكًا في الفضاء وأن الكواكب يجب أن تسقط في الشمس، ولكن هناك إله يدفعها دفعة خفيفة من وقت لآخر.
    كان نيوتن يعمل في الكيمياء لسنوات عديدة وكان لديه مختبر لهذا الهراء.

    أوه – وآمن نيوتن بالله…..الهراء المطلق

  340. إسرائيل
    تحتوي سلعتك على عدد أكبر من المشترين المحتملين مقارنة بالأطراف المهتمة فقط.
    ومن يقوم بحظرك سوف يحصل على بلاغ.
    ماذا عن قصتك التي لا تنتهي؟ هو تنته بعد؟ تجمع في الشمس جلب معه سبورة سوداء وطباشير.

  341. ميخائيل.

    لقد كان نيوتن عالمًا جادًا للغاية كما نعلم جميعًا. ولم ينخرط في "هراء" كما أفهم هذا المصطلح. في ذلك الوقت، كانت الخيمياء وعلم التنجيم تعتبر من العلوم الجادة. لقد درس الموضوع تمامًا كما حاول أينشتاين والعديد من العلماء الآخرين ويحاولون إيجاد نظرية موحدة كبرى. أشياء كثيرة يمكن أن يقال أنها هراء في وقت لاحق، بما في ذلك تجربة MM. في ذلك الوقت بدت شرعية تماما. يؤكد اختبار النتيجة الأفكار التي اعتبرها العديد من الأشخاص الطيبين في ذلك الوقت هراء - قوة جاذبية أثيرية تعمل من مسافة بعيدة (توصل نيوتن إلى استنتاج مفاده أن هذا ممكن من خلال ممارسته في الكيمياء، انظر جليك)، وتجربة هيرتز لإثبات نظرية ماكسويل (المعلم الموصى به لم يصدق الحيلة المتمثلة في إرسال "موجات كهرومغناطيسية" غامضة بين نقطتين، ستنجح)، والهجرة القارية والمزيد.

    نحن جميعًا حكماء عندما ننظر إلى الماضي لنعرف ما هو صحيح وما هو ليس كذلك (انظر السيد نحشون). ولكن هناك بعض الاستثناءات التي يصعب تجاهلها.

    على عكس الموقف المتشكك تجاه نظرية الجذب، عندما صدر "المبادئ" أصيبت أوروبا كلها بالصدمة. وكان قبولها فوريًا بين الأوساط العلمية. والسبب هو أنه من الصعب للغاية الجدال مع البراهين الرياضية الكمية. وهذا هو بالضبط ما جلبه ماكسويل أيضًا في مقالته على موقعه على الإنترنت: دليل رياضي كمي على وجود الدوامات الجزيئية. اشتقاق سرعة الضوء من ثابتي الكهرباء والمغناطيسية.

    فيما يتعلق بـ "نظريتي" - فأنا لا أحاول بيع أي شيء. خطرت لي فكرة لحل مشكلة كما أراها، وذكرت أنني لا أعرف إذا كان الحل صحيحًا أم لا، على الأغلب لا.

    من حقك أن تتجاهل كلامي ورغباتي. ولك أيضًا الحق في عدم قراءة كلماتي. قد يكون هناك من يهتم - أعرب شموليك عن اهتمامه بقصصي عن القانون الثاني والمعجزات و(ب) في قابلية الموقع للانعكاس.

    ومع ذلك، أنا لا أحاول "بيع" أي شيء لأولئك الذين لا يهتمون. المرة الوحيدة التي وصفت فيها "نظريتي" كانت قبل عام ونصف، وذلك بناءً على طلب صريح من R.H. وحتى الآن لم يثبت أحد، بما فيهم أنت، أي خطأ فيما أقول، لا على الأثير ولا على التناقض بين نظرية الانفجار الأعظم والزمن النسبي لأينشتاين.

    أنا لا "أبيع" أو "ألوح" أكثر مما تبيع هي "عربة العلماني" وتلوح باستنتاجاتها. من حيث الكمية - أقل من ذلك بكثير.

    (هل سيتم حظري الآن؟)

  342. المعجزات
    لقد كتبت: "R.H. Rafai.M
    بالمناسبة ماشال على حق"

    لقد التهمت عن طريق الخطأ كلمة Machal في التعليق.

  343. إسرائيل:
    ثلاث ملاحظات قصيرة:
    الملاحظة الأولى:
    لقد كان نيوتن مخطئًا بشأن أشياء كثيرة (مثل تورطه في علم التنجيم والكيمياء) لكنه في الحقيقة لا ينتمي إلى أي مادة، لذلك لم تفهم ما الذي كانوا يتحدثون عنه.
    بشكل عام، يوجد قزم هنا يُطلق عليه عادةً اسم Shemanges "الشبح" وبما أنه يتحدث هراء - غالبًا ما تبدو الإشارة إليه أيضًا هراء.

    الملاحظة الثانية:
    فيما يتعلق بماكسويل - حتى لو استخدم نموذج الأثير، فمن الواضح الآن أن الأثير ليس ضروريًا لحمل الموجات.
    لقد كانت صورة نجحت إلى حد ما وفشلت.
    واليوم، لا أحد يبحث عن وسيلة ما لنقل موجات هايزنبرغ.
    تقدم ماكسويل نفسه أيضًا إلى ما هو أبعد من الإشارة إلى الميزات الدقيقة للموقع.
    علي سبيل المثال على هذا الرابط ستجد النص التالي:
    ومع ذلك، أعرب ماكسويل عن بعض الشكوك المحيطة بالطبيعة الدقيقة لدواماته الجزيئية، ولذلك بدأ في الشروع في اتباع نهج ديناميكي بحت للمشكلة. وكتب بحثاً آخر مشهوراً عام 1864 تحت عنوان "النظرية الديناميكية للمجال الكهرومغناطيسي" كانت فيه تفاصيل الوسط المضيء أقل وضوحاً.[أ3]

    الملاحظة الثالثة:
    يبدو لي أنك "تسعى" لبيع نظريتك على أحد مواقع العلوم الشهيرة في وقت مبكر جدًا، ومن خلال القيام بذلك قد تؤذي نفسك أو الآخرين.
    أقترح عليك حقًا بيع النظرية في الأوساط العلمية أولاً أو على الأقل نشرها بطريقة علمية هنا بدلاً من التلويح باستنتاجاتها.

  344. كان لدى السيد نحشون (تخمينك صحيح) من جهاز التعقب طريقة لتخمين 13 لعبة في توتو: كان يعد المصابيح، ويضربها في طول النتوءات، ويضع الجيران في الداخل. يجب أن يخرج.

    وهكذا خمن النتائج بنفسه 18 مرة على التوالي.

    هل يمكن أن يكون نموذج موقع ماكسويل مجرد حالة ناجحة أيضًا؟ هل اجتازت النموذج؟

    حسنًا، أنا مشغول الآن بإحصاء المصابيح البارزة والمجاورة. يبدو لي أن هذه هي أسرع طريقة لإحضار تاماكا.

    بالمناسبة، أين أخطأ نيوتن؟ ومتى كان يتكلم بالهراء؟

  345. معجزات.

    ما أقوله هو هذا: حتى أينشتاين، كان مفهوم الأثير حجر الزاوية في كل نظرية فيزيائية. تمكن ماكسويل، من ثلاثية نيوتن ماكسويل أينشتاين، من إظهار أنه ليس هناك الأثير فحسب، بل أظهر أيضًا كيف يعمل وكيف يمكن استخلاص سرعة الضوء منه من الاعتبارات الهيدروديناميكية للضغوط والدوامات والجزيئات والتيارات.

    ولو كان نموذج الأثير بعيد المنال أو خاطئا، لكان من المستحيل أن نحصل منه على سرعة الضوء.

    وصل أينشتاين وحكم: لا يوجد موقع.

    فكيف تمكن ماكسويل من أن يكون جميلًا جدًا إذا لم يكن هناك موقع ويب؟

    قليل من الأمور التافهة: حتى بعد سنوات عديدة من نشر أينشتاين للنسبية، استمر العديد من الفيزيائيين في الإيمان بهذا الموقع. ومن بينهم لورنز وميكلسون ومورلي.

    ومع ذلك، فإن سبب رفض أينشتاين لفكرة الأثير هو أنها تتطلب نظام راحة تفضيلي. وهذا لا يتفق مع مبدأ العكسية، ومع أن سرعة الضوء هي نفسها لكل متر.

    ما أحاول إظهاره هو أنه يمكن أن يكون هناك نظام يترك الأثير، ولا تزال سرعة الضوء فيه هي نفسها لكل جهاز قياس. لقد اقترحت فكرة واحدة - نظام مشابه لنموذج ماكسويل للأثير ولكنه مفتوح تمامًا حسب ما يتطلبه موقع متجانس ومتناحي لا نهائي. وهذا لا يعني أن الفكرة صحيحة - فالأغلب أنها خاطئة لأسباب لا أعرفها - ولكن من الممكن أن يكون هناك نماذج أخرى أيضاً.

    تتمثل ميزة النموذج الخاص بي في أنه من السهل نسبيًا محاكاته على الكمبيوتر. كما أنه يعطي نتائج جانبية غير متوقعة: الجاذبية حسب ليساج ولكن دون مشكلة الاحتكاك (المشكلة الرئيسية عند ليساج)، والقصور الذاتي، واحتمال عدم المحلية.

    كما أن لها ميزة أخرى يمكن اختبارها تجريبيا. وهذا ما أحاول فعله الآن، على الرغم من أن الأمر ليس سهلاً على الإطلاق.

    ومن الممكن أيضًا أن يوضح لي شخص يعرف الموضوع ويفهمه أين الخطأ أو طريقة حل المشكلة من قبل الآخرين.

  346. المعجزات
    بالطبع مكال على حق.
    وجميع علماء الرياضيات الآخرين مخطئون أيضًا.
    وأنت تعرف ما الذي تتحدث عنه.
    حياتي…

  347. المعجزات

    لكن هل توافق على أن إسفنجة الكون الحالي ليست متناحية فيما يتعلق بالسرعات؟ هل لها نظام راحة معين؟

    فلماذا هذا؟ لماذا التمييز؟

    لقد أوضحت لك أن هذا الافتراض يؤدي إلى أسئلة لا حصر لها.

    يقول أوكهام: ليس هناك تفضيل.

    هل ألقيت نظرة على نموذج موقع ماكسويل؟ المعادلة 136؟ هل تقبل أنه من غير المعقول أن يتمكن ماكسويل من استخلاص سرعة الضوء من النموذج إذا كان خاطئًا؟

    فكيف يمكنك القول أنه لا يوجد موقع على شبكة الإنترنت؟

  348. إسرائيل
    إن "الكون الإسفنجي" هو بالفعل متجانس ومتناحٍ. إنه هو نفسه في كل مكان وفي كل اتجاه، وليس من الضروري أن يكون هو نفسه في كل سرعة. فكرة ثبات السرعة بأكملها جاءت لاحقًا.

  349. يا علماء الرياضيات، عظام الرقبة، تلك…..كمية الهراء التي يلفظونها أحيانًا…. بحسب أفوجادرو..

    المعجزات

    إذا كنت لا تفهم ما هي اللغة وما هي الرياضيات، فنحن نرحب بك لقراءة هنا:

    http://search.yahoo.com/r/_ylt=A0oG7hYCAy5SYE4A8nJXNyoA;_ylu=X3oDMTEzbDBrMTJiBHNlYwNzcgRwb3MDNARjb2xvA2FjMgR2dGlkA1ZJUDI1MF8x/SIG=13lokfmg0/EXP=1378775938/**http%3a//cms.ed
    ucation.gov.il/NR/rdonlyres/56D4BD7B-A345-411A-9B74-350AA3013481/137123/Unnamed5.doc

    و هنا:
    http://www.limudi.co.il/%D7%90%D7%95%D7%A0%D7%99%D7%91%D7%A8%D7%A1%D7%99%D7%98%D7%90%D7%95%D7%AA/%D7%90%D7%95%D7%A0%D7%99%D7%91%D7%A8%D7%A1%D7%99%D7%98%D7%AA-%D7%AA%D7%9C-%D7%90%D7%91%D7%99%D7%91/%D7%9C%D7%99%D7%9E%D7%95%D7%93%D7%99-%D7%9E%D7%AA%D7%9E%D7%98%D7%99%D7%A7%D7%94/

    (نقلا عن الرابط: "...حسب الكثيرين فإن الرياضيات هي لغة أساسية في العلوم...")

    و هنا:

    Mora-mathemat.com ("الرياضيات هي أيضًا لغة")

    و هنا:

    http://www.youtube.com/watch?v=sKdxqEgeTsk

    و هنا:

    http://www.youtube.com/watch?v=OtupAb3n4wM

    حتى أنني سألت مدرس الرياضيات (لن أذكر اسمه) وكانت إجابته: "يمكن تعريف الرياضيات كلغة. لكن الأصح أن تكون لغة زائفة".

    وأنت مدعو أيضًا لشرح كل هذا للعم "المتخلف" دريسدن.

    قراءة ممتعة.

  350. معجزات.

    "لقد كتبت "لا أعرف أي مجال لا يوجد فيه تفضيل لسرعات معينة." وكتبت أيضًا "ولكن ما السبب وراء تفضيل سرعة معينة في نظام لا نهائي". يقرر"

    انه سهل. لا أعرف أي مجال ليس لديه أفضلية لسرعات معينة، تمامًا كما كتبت. (هل تعلم؟ أعط مثالا).

    ثم كتبت: "ولكن ما السبب وراء تفضيل سرعة معينة في نظام لا نهائي".

    هذا مجال جديد وغير مألوف. هل سمعت من قبل عن مثل هذا المجال؟ انا لا.

    إذن هذا هو قراري. لا أعلم، لكن هنا واحدة جديدة.

    "أول شيء - أنت تفترض أن الكون لانهائي. فإذا أدى بك هذا الافتراض إلى تناقض، وكان أسلوب استنتاجك صحيحا، فاستبدل الافتراض...."

    لا أفترض شيئًا. أزعم أن هذه كانت صورة الكون عام 1887. لا؟ وهذه الصورة تتناقض مع فكرة نظام الراحة التفضيلي.

    "الشيء الثاني - لا يوجد تناقض في فكرة وجود مجال لا نهائي له نظام راحة. مرة أخرى... مثال الإسفنجة اللانهائية لا يسبب أي تناقض."

    ليس هناك تناقض، لكنه ليس نظامًا متجانسًا ومتناحيًا لا نهائيًا. لقد أوضحت لك أن مثل هذا الافتراض (على الرغم من عدم وجود تناقض فيه) يؤدي إلى الكثير من الأسئلة حول سبب عدم وجود الخواص.

    أراد أينشتاين أن يطلق على النسبية اسم "نظرية اختلاف الجزيرة". كان افتراضه أن الكون متجانس بالفعل ومتناحٍ. فلماذا تفضل نظام راحة واحد على الآخرين؟

    ولهذا قلت: أوكهام يصر: "ليس هناك أفضلية لسرعات معينة". فقط لأسباب التناظر.

    الاثنين يا امرأة، الإفطار.

  351. إسرائيل
    لقد كتبت "لا أعرف أي مجال لا يوجد فيه تفضيل لسرعات معينة." وكتبت أيضًا "ولكن ما السبب وراء تفضيل سرعة معينة في نظام لا نهائي". عليك أن تقرر 🙂

    أول شيء - أنت تفترض أن الكون لانهائي. إذا قادك هذا الافتراض إلى تناقض، وكان أسلوب استنتاجك صحيحا، فاستبدل الافتراض...

    الأمر الثاني - لا يوجد تناقض في فكرة وجود مجال لا نهائي له نظام راحة. مرة أخرى... مثال الإسفنجة اللانهائية لا يسبب أي تناقض.

  352. إسرائيل:
    لسوء الحظ، أنا مشغول جدًا جدًا هذه الأيام ولن أتمكن من توسيع الردود.
    يقول شيئًا واحدًا فقط:
    لقد ذكرت أن جزيئات الهيستامين في حالة راحة بالنسبة لجزيئات الشرجين.
    تريد مني أن أعطيهم السرعة فيما يتعلق بهم وأقول ببساطة "لا أريد ذلك".

  353. ב

    سؤالي المعتاد لك:

    و….؟

    ميخائيل.

    ما هو مفقود في Etherin هو Spidin.

    لا شيء في شرجينك يتحرك.

    اختر أي نقطة في الشبكة وسمها بداية نظام المحور ثلاثي الأبعاد، 0,0,0،XNUMX،XNUMX.

    سيشكل العملاق إلى ما لا نهاية للجزيئات قوسًا من السرعات من ناقص ما لا نهاية إلى ما لا نهاية في جميع الاتجاهات (يمكنك القبول بأقل من ما لا نهاية، ولكن من أجل الجدال سوف نستخدم ما لا نهاية)

    لاحظ أنه في أي لحظة، تمر جزيئات الساتامين أيضًا عبر النقطة 0 بسرعات طبيعية تمامًا، بل إن بعضها يكون ثابتًا بالنسبة لها.

    الآن، إذا بدأنا من افتراض أنه فوق سرعة نسبية معينة، لن يكون للجزيئات أي تأثير على كتلة المادة الموجودة عند النقطة 0 (افتراض معقول تمامًا، حتى النيوترونات السريعة جدًا ليس لها أي تأثير على نواة اليورانيوم 235، فقط البطيئة هي التي تفجرها)، لقد قبلنا أنه فيما يتعلق بالمادة فإن نطاقًا محدودًا فقط من السرعات هو المناسب. بالنسبة له، الغاز هو غاز عادي بسرعة 0.

    الآن خذ كتلة أخرى من المادة تتحرك بسرعة معينة بالنسبة إلى النقطة 0 (من أجل التوضيح - في الاتجاه الإيجابي للمحور X).

    وفيما يتعلق بتلك الكتلة من المادة، فهي في نفس حالة الكتلة الأولى تمامًا. ويمكنه تحديد أنه في الواقع عند نقطة جديدة 0,0,0، في نظام متحرك بالنسبة إلى النقطة السابقة 0,0,0. لا يوجد فرق بين الاثنين. ومع أنه في غاز، فلا شيء يعارضه (لا توجد ريح) وهو في راحة بقدر ما هو مهتم.

    متى سيواجه مقاومة لجزيئات الهستامين؟ إذا حاول الإسراع. ولكن بمجرد وصوله إلى سرعة ثابتة، فإنه سيبقى هناك إلى الأبد، لأنه من جميع النواحي يكون في حالة سكون بالنسبة للغاز. (لم يغلق في موضوع التسارع).

    ماذا سيحدث لموجة الأجنة في سيثيمين؟ وسوف يتحرك دائمًا بنفس السرعة بالنسبة إلى نقطة الأصل، لأن كل نقطة متحركة هي تمامًا نفس النقطة الأخرى.

    إذا أزلنا التصادمات بين جزيئات الغاز، فسنحصل على جاذبية وفقًا لـ La Sage ولكن بدون احتكاك (كل جسم ثابت بالنسبة للغاز، فلماذا يحدث الاحتكاك).

    من الممكن إرسال المعلومات بأي سرعة نريدها، بما في ذلك السرعة اللانهائية، لأن الجزيئات تتحرك بكل السرعات.

    ولذلك تلقينا:

    1. نفس سرعة الموجة لكل متر.

    2. الجمود.

    3. الجاذبية.

    جدوى غير محلية.

    إذا كنت لا ترى ذلك، يمكن برمجة النموذج على الكمبيوتر. انها ليست معقدة للغاية. النقطة الأساسية هي "البندول الباليستي" - الافتراض المعقول بأن الجزيئات تتوقف عن إحداث أي تأثير عند سرعة معينة (أولئك الذين لا يصدقون ذلك سيطلقون الرصاص على كتلة من البلاستيسين معلقة على خيط. أما الجزيئات البطيئة فسوف تتحرك ذلك، والأسرع لن يفعل ذلك.

    ماكسويل.

    "لقد طور ماكسويل نظامًا من الصيغ التي توحد جميع النتائج الملموسة التي توصل إليها فاراداي.
    إنه في الواقع الرجل الذي أعطى النتائج بنية رياضية منظمة وبنى في الواقع نظرية المجال الكهرومغناطيسي."

    وفعلت ذلك أثناء استخدام النموذج الهيدروديناميكي للموقع. التيارات والدوامات والجزيئات والضغوط.

    "من بين أمور أخرى، قام بحساب سرعة انتشار الموجات الكهرومغناطيسية وحصل على بعض الصيغة."

    ليس فقط أي "صيغة". لقد أخذ ببساطة الصيغة التي تحسب تقدم الموجة الصوتية في الغاز، والتي تسمى "المعامل الهيدروديناميكي":

    http://he.wikipedia.org/wiki/%D7%9E%D7%94%D7%99%D7%A8%D7%95%D7%AA_%D7%94%D7%A7%D7%95%D7%9C

    تم استبدال معامل صلابة المادة بمعامل العزل الكهربائي وكثافة المادة بالنفاذية المغناطيسية

    . أجرى ماكسويل تشبيهًا بين كثافة هذا الوسط والنفاذية المغناطيسية، بالإضافة إلى تشبيه بين المرونة العرضية وثابت العزل الكهربائي، وباستخدام نتائج تجربة سابقة أجراها فيلهلم إدوارد فيبر ورودولف كولراوش في عام 1856، أسس وجود علاقة بين سرعة الضوء وسرعة انتشار الموجات في هذا الوسط.

    http://en.m.wikipedia.org/wiki/On_Physical_Lines_of_Force

    وهكذا حصل على سرعة الضوء من ثابتي الكهرباء والمغناطيسية

    http://en.wikisource.org/wiki/On_Physical_Lines_of_Force

    الصيغة 136.

    إذن نرى هنا أن معادلات ماكسويل المعروفة وكذلك الحساب الدقيق لسرعة الضوء جاءا من النموذج الهيدروديناميكي للأثير، من افتراض أنه موجة جيبية تتقدم في الدوامات الجزيئية، تمامًا كما تتقدم موجة الصوت في جزيئات الغاز.

    وسؤالي: هل من الممكن أن يصل إلى سرعة الضوء بمثل هذا النموذج الهيدروديناميكي المفصل والمعقد (حوالي 200 معادلة!) والنموذج غير صحيح؟ وإذا كان النموذج صحيحا فكيف لا يكون هناك موقع؟

    هل من المعروف عبر تاريخ الفيزياء حالة أخرى من هذا الحظ السيئ؟

    صحيح أن بور نجح بمسافة A0 في نموذجه الأول وغير الصحيح لذرة الهيدروجين، لكن هذا خطأ بسيط. وفي حالة نموذج ماكسويل، فمن المستحيل ببساطة أن يعطي مثل هذا النموذج نتيجة صحيحة لسرعة الضوء إذا كان النموذج غير صحيح.

    والنموذج هو نموذج الموقع . التيارات، الدوامات، الجزيئات والضغوط، الديناميكا المائية. (المرة السابعة، أعلم، لكن يجب أن نؤكد).

    مايكل، هل بدأت تندم على دخولك الحلقة اللانهائية؟ إذا كان الأمر كذلك، أشر لي، وأنا لا أهان. ر.ح. لقد أتعبني الرجل الفقير لمدة شهرين حتى استسلم واختفى إلى الأبد.

  354. إسرائيل:
    أعتقد أنك مجرد العبث.
    ورغم أن كل ما قلته صحيح، ولكن دعونا ننتقل إلى صورة أخرى.
    أفترض أنك تعرف ما هو الترابط ولكن من أجل جميع القراء سأصف بإيجاز الترابط الذي أريد التحدث عنه.
    يمكنك بناء مثل هذه الشبكة بأي عدد من الأبعاد.
    لنبدأ بخط مستقيم، ونحدد بشكل تعسفي نقطة واحدة عليه، ونضع علامة عليها وعلى جميع النقاط التي تقع على مسافة كاملة منه باللون. هذه النقاط هي الشبكة أحادية البعد.
    سنستمر في الوصول إلى المستوى، ونختار خطًا مستقيمًا ونبني عليه شبكة ثنائية الأبعاد.
    نختار خطاً مستقيماً في مستوى يمر بإحدى نقاط الشبكة ثنائية الأبعاد ويكون متعامداً مع الخط الأول ونختار نقطة التقاطع مع الخط المستقيم الأول وكل النقاط التي تكون على مسافة كاملة من كنقاط في الشبكة ثنائية الأبعاد.
    الآن سنمرر خطوطًا موازية للخط الأول عبر جميع نقاط الشبكة في السطر الثاني وأيضًا خطوطًا موازية للخط الثاني عبر جميع نقاط الشبكة في الخط الأول.
    نختار جميع نقاط التقاطع التي تم إنشاؤها كنقاط شبكية.
    أفترض أنك ترى الصورة - إنها ببساطة نسيج لا نهائي من الرؤوس المربعة (حيث تتم مشاركة كل جانب من جوانب المربع بواسطة مربعين ويتم مشاركة كل قمة بواسطة أربعة مربعات)
    وبطريقة مماثلة، يتم إنشاء شبكة ثلاثية الأبعاد على شكل رؤوس المكعبات.
    حتى الآن قمت فقط بوصف البنية الرياضية.
    لنفترض الآن أن الموقع يحتوي على شبكة لا نهائية ثلاثية الأبعاد حيث تكون جميع نقاط الشبكة مصنوعة من مادة "Shrigin".
    ولنفترض أيضًا أن بقية نقاط الفضاء مملوءة بمادة "ساتامين" الساكنة بالنسبة إلى نقاط "الشرجين".
    هل لي أن أفكر في مثل هذا الموقع؟ نعم!
    هل هناك أي شيء غير متناسق في ذلك؟ لا!
    هل هناك نظام يستحق أن يسمى "نظام الراحة" في هذا الموقع؟ بالتاكيد!
    هل يجب أن نختار نقطة محددة لتكون أصل المحاور؟ بالطبع لا!
    هل لديك المزيد من الأسئلة حول هذا الموضوع؟ لا اتمنى.

    فيما يتعلق بفهمنا لماكسويل:
    طور ماكسويل نظامًا من الصيغ يوحد جميع النتائج الملموسة التي توصل إليها فاراداي.
    إنه في الواقع الرجل الذي أعطى النتائج بنية رياضية منظمة وبنى بالفعل نظرية المجال الكهرومغناطيسي.
    من بين أمور أخرى، قام بحساب سرعة انتشار الموجات الكهرومغناطيسية وحصل على بعض الصيغة.
    هذه الصيغة لم تتضمن سرعة المصدر أو الوجهة ويمكن للمرء أن يطرح السؤال "إلى ماذا تشير السرعة التي استقبلها؟"
    لم يتم طرح هذا السؤال لأن الناس اعتقدوا أن هذه السرعة هي السرعة بالنسبة للموقع.
    من غير المعروف مدى علم أينشتاين بتجربة مايكلسون مورلي عندما طور النسبية الخاصة. ربما ليس بشكل خاص. إحدى الفرضيات حول ما ألهمه لتطوير هذه التوراة هو أنه أدرك أن الأمر في الواقع لا يتعلق بالسرعة بالنسبة للموقع بل بالسرعة بالنسبة لأي مشاهد نريده.
    هذا هو فهمنا الجديد لماكسويل.

  355. إسرائيل:
    يتحرك محوران عدديان متوازيان بالنسبة لبعضهما البعض.
    عند نقطة الصفر لأحد المحاور، يقف الراصد ويقيس سرعتها بالنسبة لبعضها البعض.
    أين المشكلة؟

  356. ميخائيل.

    كلانا متفق على أن تجربة MM صُممت لتحديد حالة سكون الأثير، أليس كذلك؟

    إن نظام الراحة المكون من الهواء أو الماء هو في الواقع ناقل لمتوسط ​​سرعات الجزيئات في حجم معين نقوم باختباره، هل هذا متفق عليه؟

    دعونا نقارنها بأنظمة الراحة المعروفة:

    نظام راحة الهواء الراكد – كداها.

    نظام استراحة المحيط - كما هو مذكور أعلاه.

    نظام راحة الرياح - سرعة الرياح بالنسبة للأرض. نظام استراحة النهر ذو المقاطع العرضية المختلفة - يختلف باختلاف سرعة التدفق.

    لكن في أي منطقة، يكون نظام الراحة هو متوسط ​​سرعة الجزيئات في تلك المنطقة، متفق عليه؟

    ولهذا السبب إذا كنا على طوف في نهر ولا نستطيع رؤيته بسبب الظلام، بغض النظر عن مدى سرعة إبحارنا (يضيق النهر ويتسع وكذلك سرعة الماء) فهو دائمًا عند السرعة 0 بالنسبة لنا.

    هذا لا يعني أنه حتى جزيء ماء واحد لديه سرعة 0 بالنسبة لنا - في الواقع، لديهم سرعات مختلفة - فقط أن متوسط ​​سرعتهم هو 0 بالنسبة لنا.

    إن العائق الذي يتطلب أن يكون الهواء أو الماء بسرعة 0 بالنسبة إلى الأرض هو الجاذبية. إذا كان الهواء في الطائرة فإن سرعته تكون 0 بالنسبة للطائرة.

    الكثير من التوافه. متفق عليه؟

    سوف نأخذ نظامًا لا نهائيًا.

    مساحات لا نهائية، كمية لا حصر لها من الجزيئات، توزيع لا نهائي للسرعة.

    هذه هي صورة الكون في عام 1887 تقريبًا. ربما ليس عددًا لا نهائيًا من الجزيئات، ولكن بالتأكيد ليس هناك حد للسرعات (كان أينشتاين بالكاد قد ولد).

    لماذا إذن يكون لمثل هذا النظام نظام سكون، وهو، كما ذكرنا، متوسط ​​سرعات جميع الجزيئات؟

    السؤال الذي وجهته إليك: ما الجديد في فهمنا لماكسويل؟

    كل الفيزيائيين مغفلون، وكل الكيميائيون متحذلقون، وكل علماء الرياضيات مغفلون.

    (أنا أكره التعميمات).

  357. المعجزات:
    ادعى رفائيم أن الرياضيات لغة.
    على سبيل المثال هنا:
    https://www.hayadan.org.il/ball-state-prez-intelligent-design-not-science-0408135/comment-page-12/#comment-445865
    إن اللامنطقية التي يستخدمها طوال الوقت تظهر أنه شخصيًا ربما لا يمكنه استخدامها إلا كلغة، كما أن سوء فهمه لمثل درازي (حتى هذه اللحظة !!! حتى بعد كل التفسيرات !!!) يثير ابتسامة حزينة.
    وأشرت إلى الرد الذي قال فيه أن الرياضيات لغة وكانت هناك ردود أخرى دافع فيها عن هذا الادعاء ولكن لأنه رأى أنه لا يمكن الدفاع عنه قرر أن يحاول طمس الحقائق وإذا سألت ذلك فهو علامة على أنه استراتيجية إعادة كتابة التاريخ تنجح مع بعض الناس.

    إسرائيل:
    لا أستطيع أن أفهم ما الذي يزعجك.
    ألم تجد إجابة لسؤالك في كلامي؟
    رأيت أنك كتبت أنك وجهت سؤالاً إليّ، لكنني لم أكن في عجلة من أمري لوضع رقبتي في حلقة لا نهاية لها، ولكن إذا لم أفتقد بعض الرد، فيبدو لي أنني أدرجته بالفعل في هذه المناقشة (بالمناسبة - أعتقد أنه جروم حقًا وأسأل نفسي كيف عرفت ذلك 🙂 ).
    والحقيقة أنني أشعر من نظرة واحدة أنك تنسب كلامي في هذا الموضوع إلى المعجزات.
    لماذا لا نجد نظام راحة الموقع؟
    ولم تكن تجربة مايكلسون مورلي مصممة للعثور على نقطة الصفر. لا و ​​لا! وهي مصممة للعثور على نظام الراحة فقط. في نظام السكون هذا، يمكننا وضع الصفر في أي مكان نريده، بشرط أن يكون ساكنًا بالنسبة للموقع.
    لقد أحضرت مثل الهواء - هل لديك مشكلة مع هذا المثل؟
    تخيل أن الأثير يتكون من جسيمات ضخمة، سيخلق المرور من خلالها إحساسًا بالرياح على الأرض (وبالطبع أيضًا الاحتكاك الذي قد يخرجه عن مداره حول الشمس، لكن المقصود من المثل هو توضيح فكرة واحدة فقط ).
    ألا تستطيع تغيير سرعة حركتك في الفضاء حتى لا تشعر بأي رياح؟
    ألن تجد بذلك ما يمكن تسميته "نظام الراحة للموقع"؟
    أنت لا تشعر بالرياح، لكن فكرة أن الموجة تحتاج إلى بعض "المواد" للتحرك فيها قادتهم إلى اختراع الأثير كمادة يمكنها حمل الأمواج ولكن لا تخلق احتكاكًا.
    إذا لم يتم إنشاء أي رياح، فكيف يمكنك العثور على النظام المتبقي في الموقع؟
    لقد ظنوا أن قياس سرعة الضوء في اتجاهات مختلفة يمكن أن يكشف عنها (وسوف يكشفها بالفعل إذا أظهر سرعة مختلفة للضوء في اتجاهات مختلفة - انظر مثال موجات الهواء والصوت).

  358. في مجال درجة الحرارة المتناحية، هناك بالفعل تفضيل لسرعات معينة. لا أعرف أي مجال لا يوجد فيه أفضلية لسرعات معينة.

    ولكن ما السبب وراء تفضيل سرعة معينة في نظام لا نهائي؟ وإذا كان هذا فلماذا لا يكون آخر؟ وإذا كان عشوائيا، فلماذا ينتشر على كامل الفضاء بشكل موحد ولا يتغير عشوائيا؟

    مليون سؤال. قواطع أوكهام: لا يوجد أي تفضيل لأي سرعة.

  359. إسرائيل
    فكر في مجال درجة حرارة متناحٍ، حيث يكون المجال موحدًا في كل مكان وفي كل اتجاه. لكن - إذا كنت في حالة حركة فستجد أن درجة الحرارة تتغير. الخواص لا تتحدث عن السرعة.

  360. معجزات.

    أنت على حق، ولكن ليس عليك أن تفعل ذلك.

    هل توافق على أن الإسفنجة التي تختلف سرعتها لكل حلزون ليست متناحية؟

    ب.

    حتى إسفنجة الحمام العادية لا تفعل ما قلته. إسفنجات الكون - بل وأكثر من ذلك.

  361. إذا كانت الإسفنجة تحتوي على حلزونات سريعة وبطيئة تتحرك في اتجاهات مختلفة، فسوف يقيس كل حلزون سرعة مختلفة عن الإسفنجة، وقد تكون هناك حلزونات تقيس 0 سرعة.

    من حيث نظام الإحداثيات للسرعات، فإن الإسفنج اللانهائي ليس متجانسًا. لديها 0 نقطة التعسفية.

    يمكنني أيضًا أن أتخيل خطًا مستقيمًا لا نهائيًا تكون سرعته 0 فقط بالنسبة إلى مقياس واحد ويتحرك بالنسبة إلى المقاييس الأخرى، ولكنه ليس متناحيًا.

    يمكن للمرء بالطبع أن يقول إن الكون لانهائي ولكنه ليس متناحيًا ومتجانسًا، لكن هذا يثير سلسلة من الأسئلة حول سبب عدم التماثل.

  362. إسرائيل شابيرا
    أنت حتى لا تتناول تعليقاتي، فلماذا يجب أن أقرأ روابطك؟ 🙂
    ….سألتك هل يمكنك أن تتخيل كوناً مصنوعاً بالكامل من الإسفنج، وهو متجانس ولانهائي؟ فهل هناك تناقض منطقي في هذا؟

  363. المعجزات

    سؤالي:

    "كيف يمكن لكون لا نهائي متجانس ومتناحي أن يكون لديه نظام راحة مفضل؟"

    الغرض من التجربة م-م:

    تم إجراء تجربة ميشيلسون-مورلي في عام 1887 من قبل ألبرت ميشيلسون وإدوارد مورلي في ما يعرف الآن بجامعة كيس ويسترن ريسيرف في كليفلاند، أوهايو. لقد حاولت اكتشاف الحركة النسبية للمادة من خلال الأثير المضيء الثابت ("رياح الأثير")

    http://en.wikipedia.org/wiki/Michelson%E2%80%93Morley_experiment

    لا، "لم تكن تجربة ميشيلسون ومورلي مصممة للعثور على "نظام الراحة للأثير". وهو مصمم للعثور على نظام واحد على الأكثر حيث يكون الموقع في وضع الراحة. لكن هذا ليس دقيقًا أيضًا. الوصف الصحيح هو:
    وقد صممت التجربة لقياس تأثير سرعة حركة الأرض بالنسبة للموقع على سرعة الضوء بالنسبة للموقع." كما جادل في، وليس "السرعة القصوى لنقل المعلومات لا تعتمد على النظام المرجعي. وبالصدفة يتحرك الضوء بهذه السرعة، وهذا ما أظهرته تجربة "م-م" كما تدعي.

    الأمر بسيط للغاية: تحقق من سرعة حركة المواد عبر المحيط الأثيري. والمحيط لديه نظام راحة مفضل (الأرض في حالة المحيط الأطلسي والمحيط الهادئ، والمريخ إذا ومتى كان لديه محيطات).

    أنت أيضًا، مثل B، لا تتصفح الروابط؟

  364. إسرائيل شابيرا
    من الممكن أن نبين في تجربة فكرية أن السرعة القصوى لنقل المعلومات لا تعتمد على النظام المرجعي. ومن قبيل الصدفة أن الضوء ينتقل بهذه السرعة، وهو ما أظهرته تجربة م-م.
    أينشتاين لم يقم سوى بالتجارب الفكرية...

  365. إسرائيل:
    لم تكن تجربة ميشيلسون ومورلي مصممة للعثور على "نظام الراحة للأثير". وهو مصمم للعثور على نظام واحد على الأكثر حيث يكون الموقع في وضع الراحة. لكن هذا ليس دقيقًا أيضًا. الوصف الصحيح هو :
    صممت التجربة لقياس تأثير سرعة حركة الأرض بالنسبة للموقع على سرعة الضوء بالنسبة للموقع.
    وهذا الوصف صحيح ومناسب حتى لو كان الموقع في بعض الحركة. إنه فقط يتحقق من الفرق بين حركة الأثير وحركة الأرض وهذا كل شيء. لا يتعلق الأمر على الإطلاق بالنظام المتبقي للموقع.

  366. المعجزات

    أولاً، يشير ادعاءك أيضًا إلى أنه إذا كان الكون لا نهائيًا ومتجانسًا، فإن تجربة MM لا يمكنها اكتشاف النظام الباقي للأثير، أليس كذلك؟

    ثانياً، إذا لم يكن الكون متجانساً، فإن ذلك يطرح سلسلة جديدة من الأسئلة حول طبيعة جزيرة التجانس: هل بيئة درب التبانة هي منطقة يتمتع فيها "المحيط الأثيري" بنظام راحة، وهو نفس نظام الراحة الذي يتمتع به MM حاولت التجربة اكتشاف؟

    وإذا كان الأمر كذلك، فما الذي يتأثر به؟ المذيبات؟ لأنه بعد ذلك سنحصل على أنظمة راحة مختلفة حسب توزيع كتل الحليب.

    وماذا يوجد وراء درب التبانة؟ ألا يوجد نظام راحة بالموقع هناك؟ أم أن نفس نظام الراحة موجود إلى ما لا نهاية؟

    الكثير من الأسئلة. كبار السن أوكهام يتجولون ويؤجلون: لا يوجد نظام استراحة مفضل للموقع.

    ولكن إذا لم يكن هناك، فإن تجربة MM لا يمكنها اكتشاف النظام الباقي للأثير، وبالتالي فهي غير منطقية.

    هو حقاً لم يكتشف...

    استنتاج أينشتاين: لا يوجد موقع.

    لكن هذا استنتاج متسرع إلى حد ما في رأيي، ومن هنا جاء السؤال الذي وجهته إلى مايكل.

    على الرغم من أنه لا يبدو لي أنه في عجلة من أمره لوضع رقبته في الحلقة التي لا نهاية لها ...

  367. من الجيد أن المتخلف ديفيد درازي لم يستبدل كلمة لغة في التلفاز، فمن الواضح بحسب منطقه أنه يتحدث عن مثل رياضي.
    وأعتذر للمتخلفين عن مقارنة ديفيد درازي بك.

  368. مع كامل احترامي (ولا احترامي) أيها الأشباح أنتم من لم تفهموا كلام ديفيد درازي.
    هل سمعت بمصطلح "المثل"؟

    لقد أخذ كلمة "الرياضيات" في إجاباتكما واستبدلها بكلمة "رجل" وأظهر أنه يمكن استخدام نفس مجموعة الحجج (التي استخدمتها لإثبات أن الرياضيات هي لغة) " أثبت" (عيني) أن الإنسان لغة.

    كما أنك لم تفهم الفرق بين الرياضيات ولغة الرياضيات - فالشخصية في المثل لم تفهم الفرق بين الإنسان ولغة الإنسان

  369. ديفيد درازي
    أنت لست متخلفًا فحسب، بل أنت كاذب أيضًا.

    هل يمكنك أن تبين أين كتبت، كما تقول: "الإنسان لغة!" ؟

    حقيقة أنك لا تفهم ما هي اللغة والرياضيات هي مشكلتك، وليست مشكلتي.

  370. شاهد محادثة بيني (ديفيد دارزي) ومتخلف (شبح):

    متخلفا:
    الإنسان لغة!

    ديفيد درازي:
    تعريف اللغة:
    "تستخدم اللغة أيضًا لأغراض التواصل عن طريق تمرير الرسائل اللفظية من المرسل إليه إلى المتلقي، أي شخص يعرف الأصوات والرموز والقواعد النحوية للغة معينة يمكنه إنشاء الجمل"
    "اللغة هي أداة للتفكير والإبداع تساعد على فعل التفكير من خلال السماح باستخدام المفاهيم والأفكار للعمليات المعرفية مثل الكلام الداخلي.
    اللغة يمكن أن تسمح للإنسان بالإبحار في عالم الخيال، كما هو الحال في لعبة رمزية في مرحلة الطفولة أو قراءة رواية في مرحلة البلوغ.
    "اللغة هي وسيلة اتصال تعتمد على نظام معقد من الرموز ذات الشرعية، مما يجعل من الممكن تشفير وتنظيم المعلومات بمعاني كثيرة ومتنوعة. ومن المعتاد التمييز بين الرمز اللفظي الدال وبين المفهوم أو المحتوى الذي يدل عليه، والذي يمكن أن يكون واقعياً أو مجرداً"
    معظم الإعدادات هنا تبعد سنوات ضوئية عن البشر! هل قمت بتصفح المناقشة وقراءة التعليقات "الشبحية"؟ من السهل الانقضاض على شخص آخر دون التعاطف مع الجبن نيابة عنه... والحجج (إذا كان بإمكانك تعريفها بـ "الحجج") موجودة على مستوى المدرسة الابتدائية... آسف على "الشخصية" ولكن إذا الشخص متخلف لن أصوت له؟

    متخلفا:
    ديفيد درازي
    مرحباً بك في التصويت لنفسك كمتخلف، لأنك أعطيت الانطباع بأنك نفسك لا تفهم معنى "اللغة" و"الشخص".
    لقد كتبت: ""اللغة هي وسيلة تواصل تعتمد على نظام معقد من الرموز ذات الشرعية، مما يجعل من الممكن تشفير وتنظيم المعلومات بمعاني كثيرة ومتنوعة. ومن المعتاد التمييز بين الرمز اللفظي الدال والمفهوم أو المحتوى الذي يدل عليه والذي يمكن أن يكون واقعيا أو مجردا" -
    ألا تبدو لك لغة بشرية؟ إذا لم يكن الأمر كذلك فأنت غبي حقا.
    وكيف نعرف أنك ديفيد دارزي حقًا وليس يوسي ألفونسو الذي يتخفى وراء اسم ديفيد دارزي؟

  371. ديفيد درازي
    مرحباً بك في التصويت لنفسك كمتخلف، لأنك أعطيت الانطباع بأنك نفسك لا تفهم معنى "اللغة" و"الرياضيات".

    لقد كتبت: ""اللغة هي وسيلة تواصل تعتمد على نظام معقد من الرموز ذات الشرعية، مما يجعل من الممكن تشفير وتنظيم المعلومات بمعاني كثيرة ومتنوعة. ومن المعتاد التمييز بين الرمز اللفظي الدال والمفهوم أو المحتوى الذي يدل عليه والذي يمكن أن يكون واقعيا أو مجردا" -
    ألا تبدو لك لغة الرياضيات؟ إذا لم يكن الأمر كذلك فأنت غبي حقا.

    وكيف نعرف أنك ديفيد دارزي حقًا وليس يوسي ألفونسو الذي يتخفى وراء اسم ديفيد دارزي؟

  372. "لا يوجد تكافؤ بين وجود فضاء متجانس ومتناحي ولانهائي وبين تحديد الصفر على خط مستقيم لا نهائي.
    تحديد الصفر (هذا ما تفعله - لا تجد الصفر بل تحدده) على خط مستقيم لا نهائي هو حالة خاصة لتحديد نظام الإحداثيات."

    وهذه هي بالضبط المشكلة، في رأيي، مع تجربة أسلوب MM في عالم يعتبر لانهائيًا ومتجانسًا.

    لأنه لم يأت لتحديد نقطة الصفر اعتباطياً كما نفعل في نظام المحور الديكارتي، بل جاء ليكتشف نظام السكون الطبيعي للموقع.

    وهذا ممكن إذا لم يكن الكون لانهائيًا ومتجانسًا، لكنه مستحيل إذا كان كذلك.

    تتحرك الموجات الصوتية في نظامها، نظام راحة اعتباطي - الأرض في حالة الرعد، الطائرة في حالة رسائل القبطان، ناقل الرياح في حالة الريح.

    ولكن ماذا لو كان النظام مفتوحًا تمامًا كما في حالة الكون غير المحدود والمتجانس؟ إذا، كما يقول نيسيم، "يمكنني بسهولة أن أتخيل كونًا متجانسًا لا نهائيًا يوجد فيه موقع يوفر نظامًا مرجعيًا" - فلماذا هذا النظام المرجعي على وجه التحديد وليس نظامًا آخر؟ أين التجانس؟

    مايكل، ما هي الأهمية التي نعرفها الآن عن معادلات ماكسويل؟

  373. تعريف اللغة:
    "تستخدم اللغة أيضًا لأغراض التواصل عن طريق تمرير الرسائل اللفظية من المرسل إليه إلى المتلقي، أي شخص يعرف الأصوات والرموز والقواعد النحوية للغة معينة يمكنه إنشاء الجمل"
    "اللغة هي أداة للتفكير والإبداع تساعد على فعل التفكير من خلال السماح باستخدام المفاهيم والأفكار للعمليات المعرفية مثل الكلام الداخلي.
    اللغة يمكن أن تسمح للإنسان بالإبحار في عالم الخيال، كما هو الحال في لعبة رمزية في مرحلة الطفولة أو قراءة رواية في مرحلة البلوغ.
    "اللغة هي وسيلة اتصال تعتمد على نظام معقد من الرموز ذات الشرعية، مما يجعل من الممكن تشفير وتنظيم المعلومات بمعاني كثيرة ومتنوعة. ومن المعتاد التمييز بين الرمز اللفظي الدال وبين المفهوم أو المحتوى الذي يدل عليه، والذي يمكن أن يكون واقعياً أو مجرداً"
    معظم التعاريف هنا بعيدة عن الرياضيات بسنوات ضوئية! هل قمت بتصفح المناقشة وقراءة التعليقات "الشبحية"؟ من السهل الانقضاض على شخص آخر دون التعاطف مع الجبن نيابة عنه... والحجج (إذا كان بإمكانك تعريفها بـ "الحجج") موجودة على مستوى المدرسة الابتدائية... آسف على "الشخصية" ولكن إذا الشخص متخلف لن أصوت له؟

  374. مما هو موجود في الموقع، أعتقد أن "الفضاء" فارغ ومصنوع من لا شيء، لكن الجزيئات تتحرك ذهابًا وإيابًا عبر الزمن مرات عديدة وبالتالي تشعر بنوع من "الفضاء".

  375. إسرائيل:
    لا.
    ولا يوجد تكافؤ بين وجود فضاء متجانس ومتناحي ولانهائي وبين تحديد الصفر على خط مستقيم لا نهائي.
    تحديد الصفر (هذا ما تفعله - لا تجد الصفر بل تحدده) على خط مستقيم لا نهائي هو حالة خاصة لتحديد نظام الإحداثيات.
    إنها طريقة لتسمية النقاط الموجودة على الخط ولكنها ليست طريقة لتحديدها.
    وفي الرياضيات أيضًا، عندما تتعامل مع الفضاء الإقليدي باستخدام نظام إحداثي معين، فإنك تعامل ذلك الفضاء على أنه فضاء ثابت، وهذا أيضًا أساس التحويل الذي تقوم به في استبدال نظام إحداثي وظيفته بأكملها هي تمكين تمثيل تلك المساحة في نظام إحداثيات آخر.

    تخيل أن الكون كان بالفعل لانهائيًا ومليئًا بهذا النوع من الأثير، وتخيل أن الضوء كان يتحرك بسرعة ثابتة بالنسبة للأثير.
    ليس من الممكن؟
    لماذا؟
    تنتقل الموجات الصوتية عبر الهواء بسرعة ثابتة في أي اتجاه، بغض النظر عن حجم الجسم الهوائي.
    إذا قمت بتعريف نظام الراحة على أنه نظام تنتقل فيه موجات الضوء بسرعة ثابتة في كل اتجاه، فسيكون ذلك منطقيًا تمامًا.
    ثم تقوم بوضع نظام المحاور فيه حسب ما تراه مناسبا، على أن يكون ثابتا أيضا بالنسبة للموقع.

    כמובן שבפועל כל זה לא אפשרי כי ניסוי מייקלסון מורלי הראה שאין מערכת מועדפת כי בכל הנקודות, ללא קשר לתנועתן היחסית, מהירות האור קבועה לכל כיוון אבל אם נתעלם ממייקלסון מורליי ואם נשכח את המשמעות שהיום אנחנו יודעים שהייתה למשוואות מקסוול, לא הייתה בקביעה מסוג זה כל مشكله.
    تجربة صوتية مع الموجات الصوتية لو تم إجراؤها (وأيضا في جسم هوائي ذو حجم لا نهائي) لسمحت لنا بتحديد نظام الراحة المفضل دون أي مشاكل ومن ثم تحديد نظام المحاور فيه بما يرضي قلوبنا .

  376. إسرائيل شابيرا
    لذلك ظنوا أنه لا يوجد بعد درب التبانة. أي أن الفضاء لا نهائي، لكنه فارغ وبالتالي لا معنى له. أي أن مجرة ​​درب التبانة تحتوي على كل النجوم ويجب أن توفر في الواقع نظامًا مرجعيًا. أعتقد أن تجربة MM تهدف فقط إلى تأكيد أن الموقع مرتبط بالفعل بدرب التبانة.

  377. إسرائيل شابيرا
    أنا لا أفهم المشكلة. بادئ ذي بدء - لا أعتقد أن الكون كان يعتبر لانهائيًا في ذلك الوقت. وبعد ذلك بوقت طويل، اعتقدوا أن درب التبانة هي الكون بأكمله، فأين تدعي خلاف ذلك؟
    الأمر الثاني – إذا ظنوا أن النجوم البعيدة لا تتحرك، وعلى حد علمي فقد تحركوا، فلا توجد مشكلة منطقية هنا.

  378. بعد إذنك سؤال آخر.

    وفي عام 1887 اعتبر الموقع حقيقة موجودة مثل الجاذبية وقوانين نيوتن. كانت صورة الكون عبارة عن كون لا نهائي سواء من حيث البعد المكاني (ليس هناك حدود) أو من حيث الزمن (لقد كان موجودًا دائمًا وسيظل موجودًا إلى الأبد).

    وجاءت تجربة مايكلسون مورلي لاكتشاف نظام الراحة للأثير.

    ولكن هنا، في رأيي، ظهرت مشكلة منطقية: كيف يمكن لكون لا نهائي متجانس ومتناحي الخواص أن يكون له نظام راحة مفضل؟ يقال أن مايكلسون كان سيجدها، وأعلن في مؤتمر صحفي أن كوكبة الأسد في حالة سكون بالنسبة إلى الموقع.

    لماذا هذا واحد؟ أين التجانس؟

    أليس العثور على نظام الراحة للأثير في كون لا نهائي ومتجانس يعادل إيجاد مركز خط مستقيم لا نهائي؟

    أو ربما بسبب تحفظ أولبرس لم يكن الكون آنذاك يعتبر لانهائيًا ومتجانسًا؟

    نظرًا لأنه من غير المحتمل أن ماكسويل ولورنتز وميكلسون لم يفكروا في هذه النقطة بالذات، فأنا أود أن أفهم المنطق وراء تجربة م-م.

  379. جيد. والآن بعد محاولات إعادة كتابة التاريخ الذي صنعته الأشباح، أصبح من الصعب حقًا فهمه. ولهذا السبب قمت بالتصويت على التعليقات التي أشعلتها.

  380. المعجزات:
    لم أقل أبدًا أن الرياضيات لغة.
    ومن قال ذلك فهو شبح.
    كنت أتحدث عن قوانين الرياضيات.
    من الواضح أن الأرانب لا تحسب أرقام فيبوناتشي، كما أن حشرات الزيز لا تتحقق من الأعداد الأولية أيضًا.
    ومن الواضح أيضًا أن النجوم لا تحسب مسارها.
    ومن الواضح أيضًا أنه عندما يكون لدي تفاحتان وأعطيهما لشخص ما - لا يوجد حساب هنا يجعل التفاح في يدي صفرًا.
    وكل هذه أمور لا تنتمي إلى الزعم الذي دار عليه النقاش، وإن كانوا يحاولون طمسه تدريجياً.

  381. قلت إن الرياضيات في حد ذاتها هي مهنة إنسانية. وقلت إنها مثل الفيزياء والكيمياء بلاه بلاه….
    أقول أنه من المحتمل أن تكون هناك أرقام في الطبيعة، لكنني لا أعتقد أنها "رياضيات".

    ومرة أخرى - لا أفهم لماذا هذا مثير للاهتمام؟ إذا كانت هناك رياضيات في الطبيعة، فيجب ألا تكون لغة. أنا أميز بين هذين المفهومين. لا أعتقد أن الأرانب تحسب حجم الجيل القادم باستخدام متسلسلة فيبوناتشي…….

  382. حسنًا، بما أن كل شيء يظهر هنا والآن هناك من يحاول إعادة كتابة التاريخ، فأنت مدعو لقراءة التعليقات ذات الصلة.
    جوابي:
    https://www.hayadan.org.il/ball-state-prez-intelligent-design-not-science-0408135/comment-page-11/#comment-445832
    ما الأشباح أجابتها:
    https://www.hayadan.org.il/ball-state-prez-intelligent-design-not-science-0408135/comment-page-12/#comment-445865

    المعجزات:
    لقد أشرت إلى سطر واحد فقط في ردي وكل ما قلته عنه هو "يمكن أن يكون".
    لم يعد "الواضح" نفسه كما كان من قبل، وبالتالي هناك تقدم بالفعل لكنه لا يزال يتجاهل كل ما تبقى من الاستجابة.

  383. ومرة أخرى أكرر:
    إن الظواهر التي كانت (ولا تزال) موجودة في الطبيعة - وتلك التي عرفها الإنسان بالرياضيات - هي ظواهر طبيعية. ولا تعتمد على الشخص.
    ولكن قبل أن يعرّفهم الإنسان بالرياضيات، عرّفهم على أنهم الله. وقبل أن يعرّفهم بأنهم الله، لم يعرّفهم على الإطلاق. تماما مثل السيكادا. الإنسان يعيشها ببساطة. وعندما تطور حاول أيضًا فهمها وتعريفها.

  384. מיכאל
    7 هو أولي حقًا، لكنه ليس بالضرورة موجودًا في الطبيعة. لقد أعطيت مثال السيكادا. إن القول بأنهم اكتشفوا أن الأعداد 13 أو 17 أعداد أولية هو تفسير بشري لما يحدث في الطبيعة. قررنا أن هذا هو سبب كون دورتهم هكذا. ولا نعرف حقًا ما هو مصدر هذه الظاهرة.

    من الممكن أن يكون للأرقام معنى حقيقي، كما ذكرت. على سبيل المثال - بالنسبة لمفهوم التكافؤ أو المضلعات المتقنة في نظرية البلورات. ولكن - ما الذي يهم أي شخص؟

  385. بالمناسبة - من المثير للاهتمام أن رفائيم - أخيرًا - بطريقته المؤذية - قبل رأيي بالفعل.
    قال إن قوانين الرياضيات (وهو الشيء الوحيد الذي تحدثت عنه والذي هاجمه في المقام الأول) كانت موجودة دائمًا وتم تسميتها بأسماء مختلفة.
    لذا فهذا صحيح - فهو يكتب أيضًا هراءً لم أكن لأكتبه أبدًا لأنهم لم يطلقوا على الرياضيات اسم "الله"، ولكن من وجهة نظر أساسية، فجأة يقبل تمامًا ما كنت أدعيه طوال الوقت.
    في الواقع، فإن ادعائه يتضاءل الآن أمام الادعاء بأن قرار تسميته بالرياضيات هو قرار إنساني.
    أنا أقبل هذا تمامًا، ويمكنني بالتأكيد أن أتخيل عالمًا يُسمى فيه الرياضيات "زيربول" وعدد أولي يسمى "كارشوف".
    في هذا العالم، بعض الأشخاص (الأذكى) سيتخصصون في الزيربول وآخرون متخصصون في تشويه سمعتهم سيزعمون أنهم لا يفهمون ما يتحدثون عنه.

  386. المعجزات:
    أنا أعرف العكس.
    كثير من الناس يعرفون العكس.
    وهذا يعني أنه من الواضح أن الشخص يتعامل مع الرياضيات، لكن لم تكن لديه إمكانية أن يقرر أن الرقم 7 ليس عددًا أوليًا.
    قوانين الرياضيات موجودة في الطبيعة قبل وقت طويل من وجود البشر.
    لقد ذكرت عالم الرياضيات وعالم الكونيات ماكس تاغمارك الذي ذهب إلى حد الادعاء بأن الواقع برمته ليس سوى رياضيات.
    لقد ذكرت أن الزيز اكتشف الأعداد الأولية دون مساعدتنا، ويمكنني أن أذكر عددًا لا بأس به من الأنظمة في الطبيعة التي اكتشفت أرقام فيبوناتشي.
    الجميل في الرياضيات هو أن التنبؤات المحسوبة باستخدامها (بما في ذلك تلك التي تحتوي على صفر. بما في ذلك حتى تلك التي تتضمن أرقامًا مركبة) يتبين أنها صحيحة.
    لا يمكن أن يحدث هذا (ويحدث مرارًا وتكرارًا عند كل منعطف) إذا لم تكن الرياضيات شيئًا حقيقيًا موجودًا في الواقع.

  387. إسرائيل
    ما يحاول مكال إظهاره هو أن الرياضيات مخفية في كل شيء في الطبيعة.
    ولكن هذا فقط لأنه عالم رياضيات.
    لم يعتقد أرسطو أن الأمر كله يتعلق بالرياضيات. (ولكن بيننا من هو أرسطو مقابل ماشال: مجرد إرسالية)

  388. إسرائيل
    هل تريد مني أن أشرح الشرح؟ فلماذا لا تسأل عما لم تفهمه؟ لماذا تنسب عدم فهمك لي؟

    نعم. بكلمات اخرى:
    هل تريد التخلص من الناقص الموجود في البنك؟ أبسطها: الموت.

    هناك خيار آخر: لكسب المال؛ كسب قوته؛ ليعمل. ولكن من أجل ذلك عليك أن تكون موجودًا قبل كل شيء - كشخص.

    هل تعتقد أن حشرات الزيز التي "اكتشفت" الأرقام في الطبيعة أطلقت عليها اسم الرياضيات؟

    إن ما اكتشفه الإنسان وأسماه الرياضيات هي ظواهر قبل أن يسميها رياضيات كان يسميها الله.
    إذن الله موجود أيضًا، أليس كذلك؟ وحتى قبل الرياضيات!

  389. من العار أن تجادل مثل البلهاء. ومن الواضح أن الرياضيات هي اختراع بشري، مثلها مثل الفيزياء والكيمياء والأحياء والموسيقى وكرة القدم. هذه كلها مجالات العمل، مجالات الاهتمام. ولا يوجد أي منهم بدون البشر.
    لنأخذ جزءًا صغيرًا من الرياضيات - نظرية الأعداد. تبدو بسيطة ويبدو أنها موجودة في الطبيعة. لكن لا، ليس من الواضح اليوم ما إذا كانت مفاهيم مثل "صفر" و"واحد" و"اثنين" موجودة في الطبيعة.

  390. إسرائيل:
    فيما يتعلق بالتطور - ليس واضحًا تمامًا بالنسبة لي لماذا كتبت ما كتبته، هل هي إشارة إلى شيء قلته؟

    فيما يتعلق بالانتروبيا:
    إن النظام المغلق بقوانين قابلة للانعكاس مع الزمن (وهذا كما ذكرنا ليس هو الحال في الطبيعة) سيعود -إذا عكست حالة مكوناته- إلى الحالة السابقة التي كان عليها.
    ليس هناك شك في الاحتمال هنا. الماضي الذي كان هو الذي ستعود إليه. إذا كان الماضي منظما، فسوف يعود إلى حالته المنظمة. إذا كان الوضع فوضوياً فسوف تعود إلى الوضع الفوضوي.

    الشيء في الكون هو أن هناك شروطًا أولية (انظر إنها عجيبة) فقط في البداية.
    على حد علمنا، فإن العالم لا يعمل "نحو" أي شيء.
    ولهذا السبب يوجد القانون الثاني للديناميكا الحرارية.

  391. أشباح

    أحاول أن أفهم ما قصدته عندما كتبت "لقد اخترع الإنسان الرياضيات المنظمة ليشرح بطريقة منظمة ومنطقية الظواهر في الطبيعة. ليست الطبيعة هي التي ترتب نفسها وفقًا للرياضيات، بل الإنسان هو الذي يقرر الترتيب (وما هو) في الطبيعة كما يراه مناسبًا.

    ويبدو لي أن المعنى هو أن الرياضيات لا توجد بدون الإنسان، كما أن الفن لا يوجد بدون الإنسان.

    وبما أنك لا تشرح، أحاول الخيارات. هل تقصد أن الأعداد السالبة والتخيلية لا وجود لها في الطبيعة ولكننا نستخدم (على الأقل السالب) كل يوم؟ (انظر الناقص في البنك).

    إن أبسط طريقة لشرح ذلك هي هذه.

    ميخائيل.

    تفسير التطور لخلق نظام معقد هو طفرات البقاء والاختيار. نيتي هي أن النظام المعقد للغاية يمكن أن يتطور تلقائيًا من نظام بسيط حتى بما يتجاوز التقلبات العشوائية. المثال الذي قدمته: ترتيب بسيط للأعداد المتتالية (ترتيب صغير) يؤدي تلقائيًا إلى ظاهرة الأعداد المثالية (ترتيب أكبر)

    بخصوص القانون الثاني:

    في النظام المغلق، لا يوجد فقدان للطاقة بسبب الحرارة، ومن حيث المبدأ إذا أخذنا ثلاث حالات من النظام حيث تكون الإنتروبيا في الحالة A منخفضة، وفي الحالة B أعلى وفي الحالة C هي الأعلى، يكون ذلك إحصائيًا من الواضح أن ميل النظام هو زيادة الإنتروبيا، وبالتالي الانتقال تلقائيًا من الحالة B إلى الحالة C.

    ومن نفس الاعتبارات بالضبط، فإن ميلها هو عدم الانتقال من الحالة ب إلى الحالة أ.

    ولم نذكر أي الحالات الثلاثة هو الماضي أو الحاضر أو ​​المستقبل.

    الآن، إذا طبقنا قوانين نيوتن على دورات سهم الزمن فيما يتعلق بالماضي أو المستقبل، فيبدو من الناحية النظرية أن الإنتروبيا يجب أن تزيد أيضًا في اتجاه الماضي.

    ومن الناحية العملية، فإنه لا يرتفع إلا نحو المستقبل.

    لماذا؟

    الحل الذي اقترحته: "القانون الثاني هو قانون لا يمكن تمييزه إلا بسبب الظروف الأولية التي كان فيها الكثير من النظام" هو حل واحد فقط. وفي القرن التاسع عشر، عندما تطورت فكرة القانون الثاني، لم يكن لمصطلح "الشروط الأولية" أي معنى. يعتبر الكون أبديا، وكان دائما وسيظل دائما. أثبت بوانكاريه، عالم الرياضيات الكبير، رياضيًا أن الإنتروبيا يجب أن تنخفض في مرحلة ما.

    وأنا أسأل: إذا أخذنا نظامًا بسيطًا ومغلقًا للغاية، حيث لا علاقة له بالانفجار والقوانين الوحيدة المطبقة هي قوانين نيوتن التي يمكن عكسها بمرور الوقت، فلماذا لا تزداد الإنتروبيا أيضًا في اتجاه الماضي ؟

  392. في الواقع، عندما تتعمق في نفس المقطع مع جاليليو، ترى أيضًا المقصود لأنه هناك يوضح أن فلاسفة ذلك الوقت رأوا أن الرياضيات هي لغة الطبيعة أو حتى لغة الله -أي- على العكس تمامًا للفكرة التي حاول رفائيم التمسك بها وكأنها اختراع بشري

  393. على عكسكم أيها الأشباح، أنا أتأكد من قول الحقيقة.
    أعرف نفسي باسمي وليس بعشرات الأسماء المستعارة، أعترف عندما يتبين أنني كنت في حيرة من أمري، ولا أحاول رمي الرمل والقمامة في عيون البشر.
    قلت إنه عنوان صورة لأنه عنوان صورة. أعتقد أنك تحاول حاليًا إقناع القراء بأن هذا ليس تعليقًا للصورة ولكنك نسيت أنه يمكن لأي شخص الذهاب إلى الرابط ومعرفة الحقيقة.

    ولتوفير بعض الجهد على الناس، سأقتبس هنا بداية الشرح الموجود لسؤال ما هي الرياضيات.
    وهذا التفسير أيضًا له استمرارية ولا يوجد في أي مكان في الشرح وصف للرياضيات كلغة. هذا هراء يظهر فقط كعنوان للصورة.
    "الرياضيات هي مجال المعرفة الذي يتعامل مع مفاهيم مثل الكمية والبنية والفضاء والتغيير. ويقدمه البعض على أنه علم الأنماط (الأنماط الشائعة)، وأن علماء الرياضيات يبحثون عن الأنماط: في الأعداد، وفي الفضاء، وفي العلوم، وفي أجهزة الكمبيوتر، وفي التجريدات الخيالية [1] [2].

    تطورت الرياضيات من المخزون والحساب والقياس ومن الدراسة المنهجية للأشكال وحركة الأشياء الملموسة. لقد كانت معرفة واستخدام الرياضيات الأساسية دائمًا جزءًا طبيعيًا وأساسيًا من حياة الإنسان والجماعة. يمكن العثور على تعميمات للأفكار الأساسية في النصوص الرياضية التي أنتجها المصريون والبابليون والهنود والصينيون واليونانيون والمسلمون. بالفعل في مرحلة مبكرة، برزت ثلاث خصائص تصاحب الرياضيات حتى يومنا هذا:
    التجريد: على الرغم من أن أصل بعض الأشياء الرياضية موجود في العالم الحقيقي، إلا أن المناقشة الرياضية لها تنطوي على قدر كبير من التجريد. قد يمثل الرقم 5 5 أحجار أو 5 تفاحات، لكن الرياضيات تتعامل مع الرقم ككيان مستقل لا يمثل أي شيء. تذكرنا الدائرة بالأشياء الملموسة المستديرة، مثل العجلة، لكن الهندسة تتعامل مع دائرة مجردة، عديمة الوزن وعديمة الحجم ومثالية الشكل.
    التعميم: تدرس الرياضيات ذواتها المجردة برؤية واسعة، مع البحث عن خصائصها العامة. يتضمن مفهوم العدد ضمنه سلسلة من التعميمات: الانتقال من الأعداد الطبيعية إلى الأعداد الصحيحة، ومنها إلى الأعداد النسبية، ومنها إلى الأعداد الحقيقية، ومنها إلى الأعداد المركبة. يحتوي كل نظام من أنظمة الأرقام هذه على النظام الذي سبقه.
    البرهان: يجب إثبات كل ادعاء رياضي، أي تبرير صحة الادعاء من خلال سلسلة من قواعد الاستدلال. يقوم عالم الرياضيات بوضع فرضيات جديدة، يجب عليه إثبات صحتها من خلال البراهين الاستنباطية الشكلية الناشئة عن البديهيات (الافتراضات الأساسية التي تنص على صحتها)، واختيار التعريفات وفقًا لذلك[3]. ظهرت البراهين الرسمية لأول مرة في الرياضيات اليونانية، وخاصة في "الأساسيات" لإقليدس.
    استمر تطور الرياضيات، بطريقة غير منظمة، حتى عصر النهضة في القرن السادس عشر، عندما تفاعلت الابتكارات الرياضية مع الاكتشافات العلمية في ذلك الوقت. وأدى ذلك إلى تسارع البحث الرياضي، وفي الوقت نفسه بدأ التوسع السريع في الرياضيات كعلم مستقل. ويستمر هذان الاتجاهان في التطور حتى يومنا هذا[16].

    تُستخدم الرياضيات كأداة أساسية في العديد من المجالات، بما في ذلك العلوم الطبيعية والهندسة والطب وحتى العلوم الاجتماعية مثل الاقتصاد وعلم النفس. تستمر المشكلات التي تنشأ في فروع العلوم الأخرى في كونها حافزًا ومحركًا للاكتشافات الرياضية الجديدة، وفي بعض الأحيان تتطور مجالات رياضية جديدة تمامًا نتيجة لذلك. وفي الوقت نفسه، تتطور الرياضيات كفرع واسع ومستقل من المعرفة، دون الرجوع إلى تطبيقها في فروع العلوم الأخرى، على الرغم من أن التطبيقات العملية غالبًا ما يتم اكتشافها لاحقًا لاكتشافات بدأت كرياضيات نظرية فقط.

    وفي الحقيقة هناك مقولة أخرى لجاليليو الذي تحدث عن اللغة الرياضية وأسماها "الرياضيات" عندما كتب الجملة التالية:
    "الفلسفة - على أية حال، هي مكتوبة في الكتاب العظيم المنتشر أمام أعيننا منذ الأزل - أعني الكون - لكننا لا نستطيع أن نفهم إذا لم ندرس اللغة ونفهم الرموز التي كتبت بها. هذه اللغة هي الرياضيات."

    ولكن من الواضح لأي شخص لديه خليتين عصبيتين أو أكثر أن هذا مجرد استعارة.

  394. لقد تعبت حقا منك.

    لن تتوقف عن تقديم الأعذار لتبرير نفسك عندما تكون مخطئًا.

    الشيء الجيد أنك لم تقل أنه لم يكن رسمًا توضيحيًا على الإطلاق، بل مجموعة من اللوغاريتمات الحاسوبية.

    المغفل.

  395. بالطبع أحضر أيضًا روابط أخرى حيث لا يظهر حتى هذا العنوان الغبي.
    ومن بيننا - كل إنسان عاقل يعرف أن الرياضيات أكثر بكثير من مجرد لغة وحقيقة أن لها لغة أيضًا هي أن الأفكار يجب أن تكون مكتوبة بطريقة ما وأن اللغة المنطوقة ليست فعالة في التعبير عن الأفكار الرياضية

  396. نموذجي للأشباح!
    يرجى قراءة الرابط ومعرفة ما هو مكتوب هناك.
    هناك صورة بعنوان "الرياضيات هي اللغة" وهناك المدخل الموسوعي الذي يشرح ماهية الرياضيات (وهو بعيد عن أن يكون لغة) لذلك اختار رفائيم أن يبني نفسه على عنوان ماير المؤسف بدلاً من المدخل الموسوعي.
    غبي!

  397. الكذاب الذي يدحض كلامه من رد إلى رد - من كال، يكتب:
    "يمكن لأي شخص التحقق، على سبيل المثال، إذا لم يحاول أن يشرح لنا أن الرياضيات هي لغة.
    ويمكن لأي شخص أيضًا التحقق ومعرفة كيف أحضر روابط كلماته للتأكد من أن الرياضيات ليست لغة."

    مثل هذا الرابط؟ :

    http://he.wikipedia.org/wiki/%D7%9E%D7%AA%D7%9E%D7%98%D7%99%D7%A7%D7%94

    حيث يقول: "الرياضيات لغة"؟

  398. لقد كنت في حيرة من أمري واعتقدت أن شبحًا قد طرح سؤالاً حول أي عالم رياضيات كان يتحدث هراء.
    لقد اعتبرت الإشارة إلى الأشباح في بداية الرد بمثابة اسم المرسل.
    يحدث الارتباك - يحدث - للأشباح ولا يتوقف عن الحدوث ولو للحظة واحدة

  399. الكذاب الذي لا يكل يواصل أكاذيبه.
    هو ببساطة لا يفهم بنفسه ما يقول وعندما يشرح له ذلك يقول أنه لم يقله، ولكن كل شيء موثق - وهذا هو الجميل هنا - يمكن لأي شخص أن يتحقق، على سبيل المثال، إذا لم يقل ذلك حاول أن تشرح لنا أن الرياضيات لغة.
    يمكن لأي شخص أيضًا التحقق ومعرفة كيف أحضر روابط كلماته للتأكد من أن الرياضيات ليست لغة.

  400. "قلت إن رفائيم يعتقد أنه عالم رياضيات لأنه عندما قلت إنه كان يتحدث هراء سأل ""." - هنا هراء آخر قاله عالم الرياضيات.

    وبطبيعة الحال، فإن الشخص الذي سأل "أي عالم رياضيات يتكلم هراء" كان صانعاً للمعجزات.

    لكن كما كتبت في التعليق أعلاه - ليس لدى مكال مشكلة في البدء بالكذب وكتابة الهراء، حتى لو كان فقط ليظهر أنه على حق - حتى عندما يعلم هو نفسه أنه على خطأ.

  401. وطريقته هي أن ينسب للمعلق أشياء لم تقال - كما لو أنها قيلت منه، ثم يحاول أن يضربه بها على فمه. وهذا أسلوب معروف يستخدمه أحمدي نجاد، على سبيل المثال، عندما يقول إنه لم تكن هناك محرقة ثم يحاول إقناع العالم بأن اليهود يريدون بالفعل الاستيلاء على الأراضي الفلسطينية.

  402. قلت إن رفائيم يعتقد أنه عالم رياضيات لأنه عندما قلت إنه كان يتحدث هراء سأل "ما عالم الرياضيات الذي تحدث هراء".
    من الواضح بالنسبة لي أنه لا يعتبر نفسه عالم رياضيات لسبب بسيط وهو أنه لا يعرف حتى ما هو معنى التفكير.

  403. صحيح أنهم لم يتحدثوا هراء في مجال الرياضيات. لكنهم كانوا علماء رياضيات يتحدثون هراء (كما كتبت).
    تمامًا مثل عاشق الرياضيات الذي يتحدث بالهراء عندما يواجه بأخطائه وهو غير مستعد للاعتراف بها. لذلك يبدأ بالتبرز من فمه.

  404. المعجزات
    كيبلر (العبقري) في كتابه "اللغز الكوني" على سبيل المثال. (وصحيح أنه بعد ذلك تحسن علمه، بخلاف مكحل الذي يزيد كبره غباؤه).

    "وحقيقة أن الشبح يعتقد أنه عالم رياضيات هي في الحقيقة مزحة" هي في الحقيقة جملة مزحة. أنا لا أعتبر نفسي عالم رياضيات لأنني لست عالم رياضيات. أنا لست متديناً أيضاً وأنا لست إثيوبيا أيضا. لكن هذا لا يمنع مكال من محاولة ضربي بادعاءات كاذبة وفي النهاية يكتشف أنه هو نفسه يصاب بصداع نتيجة لذلك.

  405. لكن كل شيء واضح في الواقع - بعد كل شيء، لا أحد باستثناء المعالجين يفهم أي شيء، لذلك اختارت مجموعة من الأشخاص الذين لا يفهمون أي شيء واحدًا آخر مثل هذا - صفقة كبيرة!
    هل تتذكرون قصة الرجل الذي يقود سيارته على الطريق السريع ويتحدث مع زوجته في الهاتف ويسمع في الراديو عن شخص يقود السيارة في الاتجاه المعاكس للاتجاه الذي يجب أن يكون عليه ويقول لزوجته "الجميع شخص" أيها الأحمق هناك على الراديو - هل يقود أحدهم في الاتجاه المعاكس؟ الجميع يقودون في الاتجاه المعاكس!

  406. إسرائيل
    "هل تقصد شيئًا ما يتعلق بالأسلوب: على سبيل المثال، لم يكن لدى الغرباء نظام عد عشري؟ أنهم لن يستخدموا أرقامًا خيالية أو لوغاريتمات، لكن كان لديهم طرق حسابية أخرى؟" - ما هذا الهراء. كيف وصلت إلى ذلك؟
    هل يمكنك إظهار أين كتبت ذلك؟

    أنت تنسب لي أشياء لم أكتبها ولم أقصدها. (تمامًا مثلما يفعل مايكل عندما يكتشف أنه كان مخطئًا ولكنه غير مستعد للاعتراف بذلك).

    جملة مثل: كل شيء هو الرياضيات - تتوقع أن تسمع من عالم رياضيات مثل: كل شيء هو الله، وتتوقع أن تسمعه من شخص متدين.
    وفي كلا الحالتين فهو حجة مبنية على الجهل.
    على مر التاريخ كان هناك علماء رياضيات يتكلمون هراء، لا حاجة لإضافة واحد آخر.
    الطبيعة هي أكثر من الرياضيات.
    سأقرأ الآن قصتك.

  407. إسرائيل:
    الشبح لا يعني أي شيء ملموس.
    لقد أراد ببساطة أن يعارضني، وكالعادة أجبره هذا على قول أشياء غير صحيحة.
    لقد تصرف الكون دائمًا وفقًا لقوانين الرياضيات، ولكن لكي يعارضني كان عليه أن يلجأ إلى الديماغوجية التي تقول إن الرياضيات هي لغة ثم يقول شيئًا مثل "كائنات فضائية ستتصل بالأرقام بشكل مختلف" وهذا على الرغم من أنني شرحت له أنها ليست لغة.
    تلعب قوانين الرياضيات دورًا مهمًا في الطبيعة - حتى في علم الأحياء وليس من قبيل الصدفة أن حتى الزيز اكتشف نفس الأعداد الأولية التي اكتشفناها نحن.

    فيما يتعلق بمطالبة الإنتروبيا - يبدو لي أنك مخطئ على عدة مستويات:
    مستوى واحد هو مستوى الفيزياء:
    في رأيي، سهم الزمن موجود على أي حال (وكما ذكرنا - ليست كل قوانين الفيزياء متماثلة في الزمن).
    انكم مدعوون للقراءة هنا
    http://en.wikipedia.org/wiki/Time_symmetry
    وسوف ترى هناك أيضًا، كما قلت، أن القانون الثاني هو قانون لا يمكن تمييزه إلا بسبب الظروف الأولية التي كان فيها قدر كبير من النظام.

    المستوى الثاني: مستوى "قابلية تطبيق القانون":
    ينطبق القانون على الأنظمة الفيزيائية الديناميكية، وليس على أي شيء آخر.
    فهو يربط الحالات المختلفة التي سيجد النظام المادي نفسه فيها في أوقات مختلفة.
    الأعداد الطبيعية ليست نظامًا ديناميكيًا بمعنى أنه لا يوجد فيها تغير يعتمد على الزمن وبالتالي فإن القانون الثاني للديناميكا الحرارية ليس له علاقة بها.
    وحتى لو كان القانون ذا صلة، فإنه لن يربط الأرقام بحسب حجمها، بل بحسب الزمن.

  408. شموليك

    والعكس هو الصحيح: فالقانون الثاني يخضع للرياضيات.

    ولا ينطبق هذا فقط على الديناميكا الحرارية على الرغم من اسمه. لا يوجد مجال في الفيزياء - الميكانيكا، الكهرباء، الكوانتا - لا يخضع للقانون الثاني.

    نظراً للطلب، إليكم بداية فصل "ليبنوفيتز وجيدالهيو". لاحظ أن الأقسام المقدمة حتى الآن بترتيب عكسي.

    ومع اقتراب موعد استئناف الأدلة، تحسنت الحالة المزاجية للمدعي. "كم أنت حكيم وموهوب"، قال لنفسه بمرح وربت على نابلس. يمكن لأي شخص أن يفوز بالمحاكمة حيث يكون لديه ميزة. الحيلة الحقيقية هي هزيمة مالك معقول، أو حتى أفضل.
    صحيح أنه في ظاهر الأمر يبدو كما لو أن القانون الثاني قد تغلب عليه في الجولة الأولى - أولاً بالنقاط وأخيراً بالضربة القاضية - ولكن مع مرور الوقت وانقشاع الغبار، أصبح مختلفاً تماماً وأكثر تفاؤلاً. بدأت الصورة في الظهور.

    "أويش، أويش، كوسكوشون"، نقر المدعي العام على ذقنه بشكل هزلي. هنا يمكننا التعبير عن ثلاثين عامًا من الخبرة، السنوات التي صنع فيها اسمًا لنفسه كمقاتل شجاع للحفاظ على القانون والعدالة والنظام. نعم نعم، هناك عدد قليل من الحيل الأخرى المحفوظة في حقيبة الطيبي المدعي العام القديم الجيد..
    ولا شك أنه -القانون الثاني للديناميكا الحرارية- أثبت تفوقه في عالم قوانين الفيزياء، ولكن ما هذا؟ ما هي الصفقة الكبيرة؟ أتمنى أن يستمر في المناقشة والجدل حول سرعة الضوء والفروق الدقيقة في نظرية النسبية العامة. سيتم تحديد الحكم من قبل محلفين من لحم ودم، يكون هو، المدعي العام، مسؤولاً عن اختيارهم. المحلفون بشر، يخطئون، يتألمون، يعانون، لا يغفرون. هل يهتمون حقًا بما إذا كانت قوة الجاذبية تتناسب عكسيًا مع مربع المسافة أو مربعها؟ ما هي هيئة المحلفين النسائية التي ستقرر - وبالإجماع! - لينتصروا بالقانون مع العلم أن البراءة ستحددهم بحالة حياة قصيرة وسبع سنوات من التهييج تنتهي بالموت؟
    ولا يزال يحتفظ في جيبه بنسخة من جلسات الاستماع الأولية التي عقدت مع القانون المتعلق بهيئة المحلفين. "لك أن تختار من تريد" - قال له القانون البريء - "شريطة أن يجلسوا قبل صدور الحكم على مقاعد البدلاء في غرفة مغلقة ومعزولة لمدة 24 ساعة على الأقل، كما يليق بمحلفي العدالة". فإذا توفرت هذه الشروط سأقبل حكمهم دون استئناف. "ألن تكون جزءًا من اختيار هيئة المحلفين؟" سأل المدعي العام بالكفر. "كلا لماذا؟" أجاب القانون غبي. "إذا كنت على حق، فسوف يحق لي، وإذا لم أكن على حق، فيجب علي ذلك، أليس كذلك؟ اختر المحلفين كما يحلو لك، أنا لست مهتمًا بالعامل البشري. من جهتي، يمكنك الانتظار مع الاختيار حتى تنتهي الشهادة". وماذا عن تنفيذ الحكم؟ ومن سينفذها إذا خسرت؟ "إذا لم أكن على حق وخسرت في المحاكمة، فسيتم تنفيذ الحكم على الفور من قبل سلطة قوية ستتعلم التعرف عليها أثناء المحاكمة"
    "بحق" قلد المدعي العام القانون بأهميته، ورحل "العامل البشري" بازدراء خفي كما فعل القانون. "يالك من أحمق! ألا ترى أن نفس "العامل البشري" الذي تكرهه كثيراً هو الذي سينقش جملتك على القبيلة، وسيكون "العدل" أقل اعتباراته؟".
    ربما يكون هذا هو الحال عندما تقضي الكثير من الوقت بين قوانين الطبيعة الدقيقة والصارمة. تفقد الاتصال بالحياة والناس. لندعه يعتقد أنه ينتصر.. لندعه يثبت بالحجج العلمية المصاغة والدامغة أي حقيقة من حقائق الطبيعة يرغب فيها.. أي حجة منطقية يريدها... سنرسل له أفضل العلماء والمحاضرين. لن نقول لهم إن المعركة خاسرة مقدما، وأن لا فرصة لهم في هزيمة القانون بالعلم أو بالنقاش العلمي.

    وأخيرا، كمحلفين، سوف نحصي الفقراء، والمنكوبين، والأضعف، والأكثر اضطهادا، لأنه ليس هناك شك في أن المحاكمة ستقنعهم بأن القانون الثاني للديناميكا الحرارية وحده هو المسؤول الوحيد عن وضعهم، والقانون الوحيد للديناميكا الحرارية. فرصة التحسن هي قناعة بالإجماع بالقانون.
    "صحيح" ابتسم المدعي العام في اشمئزاز. هل يعتقد القانون الثاني حقاً أنه إذا نجح فقط في أن يثبت للفصل أنه على حق في ادعاءاته وبراهينه، فإن نفس الشيء سيتغير بالنسبة لشخص حياته كلها تعاني من الإهانة والرفض؟ وأي نوع من العدالة هذا على الإطلاق، مع الأخذ في الاعتبار أن القانون هو المسؤول بالفعل عن الحياة البائسة لمحلفي المستقبل؟

    "من هو عبقري أمي؟" قام المدعي العام بقرص خده بمودة. كان عبقريًا وكريمًا وقاسيًا، ابتسم لنفسه وقدم تذكيرًا بإرسال بطاقة تهنئة إلى والديه لتحقيق النجاح. "سأظهر له ما هي العدالة. العدالة مفهوم نسبي، كل إنسان وعدله معه. عدالتي هي فخ، كلما أثبتت مدى ذكائك وتفهمك وعادلك و"عادلك" - كلما عمقت الحفرة التي تحفرها بنفسك. ربما في العالم الطبيعي يجب على الأجساد أن تطيع بشكل أعمى القوانين المطلقة التي كشفت عنها العدالة في النقاش الأكاديمي، كما في حالة غاليليو. لكن في محكمتي، مع المحلفين، نحن من نقرر في القانون ما هو صواب، وصديقنا القانون الثاني للديناميكا الحرارية، الذي لم يتوقف منذ بداية المحاكمة عن الحديث وإجراء المقابلات في كل موضوع وفي كل مجال، من دروس الرقص السوداني لمرحلة ما قبل المدرسة إلى نظرية الجاذبية الكمومية، سيضطر قريبًا إلى إغلاق فمه الكبير - وإغلاقه بإحكام..

    في ذهنه، قارن الخطابات الختامية للادعاء والدفاع: هو، المدعي العام، يتنفس نارًا وقوانصًا، موضحًا للمحلفين أن مأساة حياتهم ومأساة ضحايا القدر الآخرين برمتها، لها واحدة وواحدة. السبب الوحيد: القانون الثاني للديناميكا الحرارية. ثم سيبين للمحلفين التزام القانون بقبول قرارهم دون استئناف، سيصف لهم الحياة الرائعة التي تنتظرهم إذا أدانوا القانون، سيبتسم بسخاء ويستدير ليستمع إلى كلمة دفاع القانون. - في الأساس استسلام - للقانون. فماذا يمكن للقانون أن يطالب في دفاعه؟ ما هو القانون الأساسي بين قوانين الطبيعة والذي ليس له بديل؟ مبتذلة ومملة وغير ذات صلة. لقد ثبت بالفعل أنه من الممكن تنظيم قوانين أكثر نجاحًا. مذنب. ماذا لو كانت سرعة الضوء أصغر؟ صادم حقا. مسؤولة قانونا وهل سيواجهون مصيراً مريراً؟ لقد اخترت الجمهور الأقل ملاءمة للتهديدات. وفي النقطة التي هم فيها الآن، لا يمكن لوضعهم إلا أن يتحسن. هل سيقدم الوعود؟ نفس ما ورد أعلاه. هل سيطلب الرحمة؟ قصتك أيها القانون اللطيف مثيرة للاهتمام وتلامس قلوبنا كثيرا، لكن انتظر حتى تسمع قصتنا إذا كنت تريد أن تعرف ما هي المأساة الحقيقية. مذنب، مذنب، مدان.

    ويجب الحفاظ على السرية، فنظر المدعي العام بعين الريبة إلى الأطراف عند دخوله قاعة المحكمة. لا يُسمح لأحد بتخمين المؤامرة. ليس الشهود، ولا المحلفين، ولا وسائل الإعلام، وعلى الأقل القانون الثاني للديناميكا الحرارية... وبالتأكيد ليس شاهد الادعاء الجديد، لايبنتز الحاد وسريع الغضب، وقود المدفع الجديد في اللعبة الذي لا أحد لكني أعلم أنه مدمن..

    وبخطوات واثقة تقدم وكيل النيابة إلى المنصة وبدأ تحقيقاته في القانون الثاني:

  409. إسرائيل، ما الذي أفتقده هنا؟
    الطبيعة لا "تتكلم" الرياضيات، ولكننا ننجح في صياغتها، في ظل افتراضات كثيرة وإهمالها بطريقة جيدة. من خلال الرياضيات. لست مقتنعا بأنه نفس الشيء. صحيح بالفعل أن الرياضيات مندمجة بعمق في الكون المادي، لكن القانون الثاني يرتبط ارتباطًا وثيقًا بتوليد الحرارة بسبب الاحتكاك الذي لا يمكن عكس اتجاهه وترتيب الأرقام على خط الأعداد ليس كذلك، لذلك ما زلت لا فهم لماذا يجب أن تخضع الرياضيات للقانون الثاني.
    على أية حال، لا تزال القصة مثيرة للاهتمام ورائعة. اخرى!

  410. تقصد شيئًا ما: على سبيل المثال، لم يكن لدى الغرباء نظام عد عشري؟ أنهم لن يستخدموا أرقامًا خيالية أو لوغاريتمات، ولكن لديهم طرق أخرى للحساب؟

    لكن هل تقبل أننا نتفق جميعًا على أن pi هي النسبة بين قطر الدائرة ومحيطها، وأن 3 و7 و11 أعداد أولية، وأنه لو أضاف ياتزك أيضًا عدد الأطراف لكان عليه القسمة بحلول 5؟

  411. أنا فقط أدلي بتعليق بسيط: إذا أحصى ياتسك 4 بقرات، وكان لكل واحدة منها 3 أرجل (لسبب أو لآخر) - فسوف يحصل على موافقة من مدير الحظيرة.

    وفيما يتعلق بقصتك التي لا تنتهي: هل تقتبسها من مكان ما؟

  412. إسرائيل
    ربما لم تفهم (أو أنك "لا تفهم" عمدًا). لم أزعم أن الرياضيات تعتمد على الشخص. فزعمت أن الإنسان هو من اخترع الرياضيات وأن الرياضيات لم تكن موجودة قبل الإنسان (كما زعمت).
    لقد اخترع الإنسان الرياضيات ليصف الطبيعة بطريقة منظمة (لنفترض أننا سنسميها منطقية).
    وكما تصف لغة الرياضيات الطبيعة بأفضل الطرق وأكثرها منطقية - هكذا يكتشف الإنسان (وفقا للرياضيات) أن الطبيعة حقا لا تعتمد على الإنسان.
    كما كتبت: الطبيعة كما هي.

  413. وبخطوات واثقة تقدم وكيل النيابة إلى المنصة وبدأ تحقيقاته في القانون الثاني:

    الذي - التي. أيها المدعى عليه، هل يمكنك أن تخبر المحكمة لماذا تزيد الإنتروبيا دائمًا؟

    أ. بدون سبب يحدث فقط.

    الذي - التي. لكن لماذا؟ هل من المستحيل تركها كما هي؟ الحد منه؟

    أ. ممكن، ولكن من غير المرجح.

    الذي - التي. ماذا يعني ذلك؟

    أ. تعطي قوانين العشوائية الرياضية احتمالية منخفضة للغاية لتقليل الإنتروبيا.

    الذي - التي. لا أفهم هل تزعم أن الرياضيات، وهي البنية الأكثر منطقية وتنظيمًا، هي سبب الفوضى المتزايدة؟ هل ذلك منطقي بالنسبة لك؟ وخاطب المدعي العام الجمهور بسخرية. صفير الجمهور في ازدراء.
    أ. نعم.

    قال المدعي العام باشمئزاز: "لا أصدق ما أسمعه". "لكنني لست عالم رياضيات محترفًا. أود استدعاء البروفيسور ليبنوفيتس، رئيس معهد غاوس للأبحاث الحسابية، للإدلاء بشهادته".

    القانون مملوء بالبهجة. "جمال! من المؤكد أن عالم الرياضيات سيثبت ادعاءاتي وصحتي. أنا أحب البراهين الرياضية. أتمنى أن يكون هذا دليلاً سلبيًا، فهذه هي الأشياء المفضلة لدي." اتكأ على ظهر الكرسي متوقعا.
    أخذ البروفيسور ليبنوفيتس مكانه على المنصة وفتح الباب. "من الواضح أن القانون صحيح. ومن حيث الاحتمالية الرياضية البحتة، هناك حالات فوضى في النظام أكثر بكثير من حالات النظام."
    ابتسم القانون بتواضع قليلاً وهو يقول: "حسناً، ألم أخبرك؟"
    قام الأستاذ بسحب مجموعة من البطاقات المختومة وفتحها أمام الجمهور. "نرى؟ البطاقات في حالة الحد الأقصى للطلب. في البداية سلسلة الوجه، عندما يتم ترتيب البطاقات بالتسلسل من الأصغر إلى الأكبر. وبعد ذلك سلسلة القلب، ثم البرسيم وأخيراً سلسلة الماس عندما يتم ترتيبها جميعها بنفس الطريقة، من الأصغر إلى الأكبر".
    "الآن، إذا قمت بتبديل المجموعة بخفة، فسوف ينخفض ​​الترتيب في المجموعة. ستبقى هناك بالفعل سلسلة صغيرة من البطاقات التي "تلتصق" ببعضها البعض، لكن الترتيب في المجموعة سيتناقص، وسيستمر في الانخفاض مع استمراري في الخلط، حتى تصل إلى حالة من الفوضى القصوى، أو الإنتروبيا.
    وفي كلمته، يخلط الأستاذ الأوراق أمام الحضور، فيما يشمت القانون من كرسيه.
    "نفس الشيء سيحدث لغرفة مرتبة تهب فيها رياح عاصفة. سوف ينخفض ​​مستوى النظام مع مرور الوقت. إذا قمنا بتصوير الغرفة في مواقف مختلفة ثم قمنا بمزج الصور، فيمكننا دائمًا ترتيبها مرة أخرى بترتيب زمني وفقًا لحالة الاضطراب في الصورة: كلما زادت الفوضى أو الإنتروبيا في الصورة، كلما كانت الصورة متأخرة. هل توافق؟"
    هتفت الجماهير بالموافقة، بينما أشرق القانون بالسعادة.
    "وهذا هو السبب أيضًا في تعريف سهم الزمن على أنه اتجاه زيادة الإنتروبيا. في الطبيعة، في نظام مغلق، مع مرور الوقت، تزداد الإنتروبيا، أو تظل ثابتة لسوء الحظ. ولا ينخفض ​​إلا في حالات نادرة وخاصة.
    والسبب في ذلك هو الرياضيات البحتة. هناك ببساطة العديد من الحالات المحتملة ذات الإنتروبيا العالية، أكثر من الحالات المرتبة."
    "خاص." قال القانون بسرور. كنت أعلم أن الرياضيات ستكشف براءتي!"
    "غير صحيح!" قال الأستاذ. الرياضيات تثبت العكس تماما! إنه خطؤك!"
    "ماذا؟" صرخ القانون: "لقد أثبتت لنا للتو أن الإنتروبيا يجب أن تزداد دائمًا لأسباب رياضية بحتة تتعلق بالاحتمال!"
    "صحيح، ولكن هذه مجرد صورة جزئية! ويظل السؤال مفتوحا: لماذا؟".
    قال القانون ببرود: «يبدو لي أنك شرحت الأمر جيدًا بمثال مجموعة أوراق اللعب. "هناك العديد من الحالات ذات الإنتروبيا العالية أكثر من الحالات المرتبة" - نقلاً عن.
    هذا صحيح، ولكن لماذا تتحول الإنتروبيا في الاتجاه الذي تختاره مع مرور الوقت؟ لماذا لا يقوم خلط الأوراق بترتيب الأوراق في المجموعة بدلاً من إفسادها؟ لماذا لا ترتب الريح الغرفة؟ لماذا لا نصبح أصغر سنا مع مرور الوقت بدلا من الشيخوخة؟
    تمتم بالقانون: "كل هذا خطأي، كالعادة".
    "بالضبط" وافق الأستاذ. "أنت في حالة من الفوضى تماما. أنت تعرف فقط كيف تسبب الكثير من المتاعب. هل هناك أي تفسير آخر لماذا تزداد الإنتروبيا دائما مع مرور الوقت؟ قوانين نيوتن للحركة لا تتطلب ذلك! إذا قمنا بتغذية جهاز كمبيوتر ببيانات نظام نجمي معين، فسيكون قادرًا على التنبؤ بدقة كبيرة بحالته خلال ألف عام - وأيضًا بحالته قبل ألف عام! لماذا لا ينطبق مبدأ التكافؤ هذا أيضًا على النظام الديناميكي الحراري؟"
    "كما قلت، الرياضيات .."
    "يا!" وكان الأستاذ متحمسا. "لقد ارتكبت خطأً في هذه المسألة، ويمكنني إثبات ذلك إذا أحضرت لي سبورة وطباشير".
    يمتد الجمهور في مقاعدهم في ترقب متوتر، بينما تهتز الشمس في قاعة السبورة المحمولة.

  414. ميخائيل

    الأبقار - في المرج.

    الجمال - في الصحراء.

    كان لدينا في الحظيرة جاسيك أمين المكتبة، الذي كان يحسب عدد الأبقار في القطيع بسرعة البرق. طريقته بسيطة: احسب عدد الأرجل واقسمها على أربعة.

    أشباح

    أعتقد أنه بغض النظر عن أي كائنات ذكية خارج كوكب الأرض نتواصل معها، فسوف نتفق جميعًا على نفس الأرقام الأولية.

    وهو ما يوضح حجتي لاستقلال الرياضيات عن الإنسان.

    متشكك.

    ربما لم أوضح الفكرة بالشكل الكافي هذه ليست مجرد فلسفة، هناك تشابه هنا بين التطور ونظرية الأعداد. قد يكون عندي خطأ وأريد منكم التنبيه عليه.

    الفكرة الرئيسية للتطور (في فهمي) هي إنشاء نظام معقد من نظام أقل تعقيدا، وإنشاء نظام من نظام أقل. ينص القانون الثاني للديناميكا الحرارية، والذي ربما يكون أبسط قوانين الفيزياء، على أن الأشياء في الطبيعة تميل من الأكثر تعقيدًا إلى الأقل تعقيدًا إلى الفوضى. يحاول البروفيسور ليبنوفيتس في القصة أن يوضح أنه حتى مع الأرقام فمن الممكن الانتقال من ترتيب صغير إلى ترتيب كبير، وهذا يتعارض مع ادعاء القانون بأن الرياضيات تتطلب الفوضى (انظر أدناه).

    وهنا ادعائه:

    "جميع الأعداد، كبيرة كانت أو صغيرة، سواء كانت كاملة أو كسرية، إيجابية أو سلبية، عقلانية أو غير عقلانية - يمكن إدخالها بترتيب تصاعدي بين ناقص اللانهاية واللانهاية. كان من المتوقع أن مثل هذا الترتيب البسيط لن يؤدي إلى أي نمط أنيق بشكل خاص - فماذا فعلنا بعد كل شيء؟ لقد وضعنا 1 في البداية، يليه 2، 3، وهكذا."

    في الواقع، الحد الأدنى من النظام. في المجموع سجلنا : 345,789 ...............1,2,3,4

    وهنا يواصل:

    "من هذا النظام البسيط تمامًا، تم إنشاء نظام أكثر تعقيدًا وإثارة للاهتمام. لقد اتضح أنه لا نهاية للتعقيد الماكر الذي يمكن من خلاله ترتيب هذه الأرقام التي تبدو بسيطة. خذ على سبيل المثال ظاهرة الأعداد المثالية. هذه هي الأرقام التي يكون مجموع الأرقام التي تقسمها مساويًا تمامًا للرقم نفسه."

    الآن، أعتقد أننا جميعًا متفقون على أن ظاهرة مثل الأعداد المثالية هي ترتيب أكثر تعقيدًا من المتسلسلة الكتلية، أليس كذلك؟ ولكن لم يكن من الممكن أن يتم إنشاؤه إذا لم نزود النظام بنفس الترتيب الأدنى، أي الأرقام المتتالية. أولئك الذين لا يصدقون، يجب عليهم تجربة سلسلة عشوائية دون الحد الأدنى من الترتيب، ومعرفة ما إذا كان بإمكانهم الحصول على ترتيب أكثر تعقيدًا.

    أعتقد أن التوازي مع التطور واضح.

    ما هو سبب هذا التطور التلقائي لنظام أكثر تعقيدا؟ بالنسبة للأشخاص المتدينين وأنصار نظرية الخلق الجواب واضح: لقد خلق الله كل شيء، بما في ذلك الرياضيات والأرقام والحمير الوحشية.

    بالنسبة لأولئك الذين يؤمنون بالتصميم الذكي، فإن الإجابة ليست حادة وواضحة. يمكن أن يكون المخطط كائنًا من خارج الأرض بالإضافة إلى جهاز كمبيوتر، وليس بالضرورة كائنًا سلبيًا ومقدسًا. وبالإضافة إلى ذلك، وعلى عكس المتدينين، فإن المخطط الذكي لم يقم بإنشاء الرياضيات. (هل أنا على حق؟ لم أر قط مادة تتناول هذا السؤال، لكنه منطقي بالنسبة لي).

    ادعائي هنا هو أن النظام، وحتى النظام المتقن مثل الأرقام المتقنة، يمكن إنشاؤه تلقائيًا من نظام فضفاض، وليس هناك حاجة لمخطط ذكي لإنشائه.

    يعلم كل لاعب شطرنج أنه إذا كانت لديك بداية جيدة، فإن اللعبة تتدفق بشكل أسهل وأكثر سلاسة مما لو كانت لديك بداية سيئة.

    ما هو الحد الأدنى لعدد التحركات للتحقق؟

    (لا تنظر إلى ويكيبيديا!).

  415. المعجزات:
    وصفك يثبت في الواقع أن هناك منافسة على الموارد.
    وإلا فلن تضطر الأرجل الطويلة إلى التجول.
    وأكثر من ذلك - فمن الواضح أنه في المنطقة التي فروا منها كانت هناك حيوانات قصيرة الأرجل لم تستطع الهروب وعانت منها.
    بل وأكثر من ذلك - الانتقاء الطبيعي - كما قلت أكثر من مرة - إنه أيضًا معدل التكاثر - في الواقع هو معدل التكاثر بشكل أساسي.
    أيضًا وفقًا لطريقتك وعلى الرغم من الأخطاء - كل ما وصفته هو تطور، لذا ليس من الواضح كيف تريد أن تستنتج أنه عند نقطة معينة لم يكن هناك تطور بل شيء آخر.
    ونعم - إنه تطور كلاسيكي تمامًا - لا يوجد شيء مميز.

  416. إريك
    من الصعب بعض الشيء أن نفهم، لكن الانتقاء (سواء كان طبيعيًا أم لا) يبطئ التطور في الواقع. ماذا يفعل الانتخاب؟ يمنع تكاثر بعض الكائنات الحية: فهو يقلل دائمًا من معدل الرؤية.
    لقد تعمدت أن أضرب مثال الضفادع في أستراليا. ورغم عناد صديقي العالم مايكل، إلا أنهم لا يعانون من نقص (للأسف)، لكنهم يتغيرون. هناك نقطة مثيرة للاهتمام هنا - فهي تتطور في عملية ليست تطورية تمامًا :). ما يحدث هو أن الضفادع ذات الأرجل الطويلة تهاجر بشكل أسرع وتتقدم لمسافة أبعد. لذلك اتضح أنه في غرب أستراليا، وهو الحد الأقصى لتقدمهم اليوم، هناك ضفادع ذات أرجل أطول. لكن (!) تعاني هذه الضفادع من التهاب المفاصل - لذلك من الصعب القول إن هذه الضفادع "تتطور".

    باختصار...الشرح معقد بعض الشيء بالنسبة لي وأرى أنه يبدو غير متسق حقًا... لكنه يظهر أن هناك تطورًا حتى بدون نقص. التنمية دون اختيار (اختيار). وفي مرحلة بداية الحياة، من الأرجح بالنسبة لي أن الحياة لا يمكن أن تبدأ في التطور الكلاسيكي. والسبب هو، مرة أخرى، أنه في البداية لم يكن من الممكن استيفاء جميع شروط التطور.

  417. إسرائيل
    لم أقل أنه يمكنك التواصل مع الكائنات الفضائية من خلال الرياضيات. كتبت أنه سيكون من الممكن إنتاج لغة (جديدة) من خلال الرياضيات، ومعها (اللغة الجديدة) سيكون من الممكن التواصل. مثل التكنولوجيا على سبيل المثال. والتي تتكون أساسًا من اللغة الرياضية.
    كل هذا بالطبع بشرط أن يكون هناك بالفعل كائنات فضائية وأنهم سيتواصلون معنا على الإطلاق.

  418. لقد قمت بالاطلاع على المقالة الثانية التي أحضرتها وهي توضح كيف يمكن إنشاء تعقيد لا ينفصم، بشكل عشوائي، من خلال التعقيد المفرط (تم ذكر المفهوم أيضًا في مدخل ويكيبيديا). تم الحصول على النظام الذي لا ينفصل عن نظام أكثر تعقيدا، يتم شرح كل خطوة منه، ثم في الخطوة الأخيرة يختفي جزء من النظام ويصبح النظام لا ينفصل، وبإلقائه على شوتون، المهمة الموكلة إلى الخلقيين، لإثبات أن شوتون هو نظام معقد لا ينفصم، وهو أمر أكثر صعوبة لأنه يتعين عليهم أيضًا رفض جميع المسارات التي تؤدي إلى شوتون من الأنظمة أكثر تعقيدًا من شوتون، ولكن كل خطوة موضحة جيدًا. بالنجاح.

    على المستوى المبدئي، من الواضح أن وجود مكونات ذات تعقيد لا ينفصم يمكن تفسيره بمساعدة مراقب ذكي، وهو ليس الله، والقول إنه هو نفسه تطور بطريقة تطورية (وإلا سيكون هناك تراجع لا نهائي هنا) وبطريقة ما لم يمر عبر التعقيد الذي لا ينفصم وأبعد من ذلك لا يمكن دحض هذا التفسير، وغني عن القول أن السبب الذي جعل الخلقيين يتوصلون إلى فكرة التعقيد الذي لا ينفصم هو الوصول إلى المخطط الذكي الذي هو الله، إذ أنهم ليسوا مهتمين حقًا بالوصول إلى الحقيقة: http://en.wikipedia.org/wiki/Wedge_strategy

    في ويكيبيديا، تحت مدخل IR، يُزعم أنه حتى لو كان هناك تعقيد لا ينفصم، فهذا لا يعني بالضرورة التطور أو المصمم الذكي وأن هذه معضلة زائفة، لكن لسوء الحظ، لم يدرج عارف خيارات إضافية. الخيار الآخر الوحيد الذي أعرفه هو الخيار الذي طرحه ولفرام. وهو يدعي أنه في ظل الانتقاء الطبيعي يوجد مبدأ أعمق (الأتمتة الخلوية) لكنني لم أتمكن من معرفة ما يتحدث عنه بالضبط والأغلبية المطلقة من العلماء لا تقبل ادعائه. أعتقد أنه إذا تبين أن هناك تعقيدًا لا ينفصم، وهو ما يتناقض مع التطور كما نفهمه حاليًا، فإنني أراهن على ظهور نظرية طبيعية تحتوي على هذه الحقيقة قبل أن نقفز إلى استنتاج مفاده أنه لا بد من وجود مصمم ذكي لذلك. سوف أشك في وجود مخطط ذكي إذا أثبت أن هناك أنظمة في الطبيعة يتعارض وجودها مع قوانين الحفظ.

    إسرائيل، إليكم القصة (تذكروا سن 16 عامًا لبضع دقائق):
    http://www.multivax.com/last_question.html

  419. على أية حال - كما قلت - هل يبدأ التطور فورًا أم لا هو سؤال غير مهم.
    لن يتم خلق الحياة قبل أن يبدأ التطور، لذا فيما يتعلق بالحياة، فإن ما خلقها يجب أن يكون التطور.

  420. إريك:
    والحقيقة أن هذه الأشياء التي قالها نسيم لا تتفق مع بقية كلامه، فتجاهلتها.
    ففي نهاية المطاف، كان هدفه كله يتلخص في إظهار أن هناك احتمالاً (بل وأكثر ترجيحاً من البديل في رأيه) بأن التطور لن يبدأ على الفور، لأنه إذا بدأ على الفور، فلن يكون لديه أي حجة أخلاقية.

    فيما يتعلق بمسألة المساحة - فكر في تخطيط التكرارات.
    أولئك الموجودون في المركز ليس لديهم مكان يكررونه.
    إلى جانب ذلك - كما قلت - فإن التنافس على الموارد هو أيضًا تنافس على القدرة على استخدامها بشكل أفضل من أجل إنتاج المزيد من النسل.

  421. מיכאל
    بخصوص كلام نسيم:
    لقد كتب ".. وهذا مكّن من التكاثر السريع جدًا وبالتالي التطور السريع". لم أفهم هذه الجملة.
    بخصوص كلامك:
    يمكن أن يكون هناك موقف تكون فيه الموارد غير محدودة في فترة زمنية معينة. أليس كذلك؟
    أنت كتبت أيضا:
    "من الممكن بالتأكيد أن يكون لدي مساحة كبيرة لأنتشر فيها، ولكن لا يزال - أولئك الذين يجيدون المنافسة هم الذين يفوزون." إذا كان لدي أنا والبقية مساحة كبيرة للانتشار فيها، فلماذا أحتاج إلى التنافس على الموارد؟

  422. المعجزات:
    في الأمثلة التي قدمتها، يوجد صراع على الموارد وكما ذكرنا - الصراع يدور حول المساحة والقدرة على استخدام الموارد بشكل أفضل من أجل إعادة إنتاج المزيد.

  423. إريك
    لقد قدمت بعض الأمثلة حيث لا يوجد نقص في الموارد (اليوم). إذا كان هذا يحدث اليوم، فلا يوجد سبب ليكون حاسما لدرجة أنه قبل 3.5 مليار سنة لم يكن من الممكن أن يكون هناك وضع مماثل.

  424. المعجزات:
    ولهذا قلت إنني أعتقد أنك بالغت عندما قلت إنه لن يأتي شيء (في المستقبل).
    من ناحية أخرى - السؤال هو ما إذا كنا نريد إنشاء الخلق 🙂

  425. وعلى أية حال، كما قلت، فإن مثل هذه المرحلة من الضرب الذي لا مثيل له لن تغير أي شيء جوهري. حتى لو لم تكن هناك منافسة في البداية، فلابد من خلقها لاحقًا.
    إذا كانت الآلية تسمح بالتغييرات (ويجب أن تسمح بذلك إذا كانت "تعتزم" التطور في المستقبل) فمن الممكن أن تكون التغييرات نحو الأسوأ - مثل فقدان القدرة على استخدام الموارد الموجودة بشكل فعال، مما سيؤدي إلى انخفاض وحتى انقراض أولئك الذين لديهم هذا التغيير، وبدلا من ذلك يمكن أن يكون للأفضل - مثل القدرة على استخدام المزيد من الموارد وإنتاج المزيد من النسل (بعد كل شيء، ما يهم في النهاية هو عدد النسل وحتى لو كان) لم يقتلوا بعضهم البعض، بل خلقوا مشكلة ديموغرافية لبعضهم).

  426. تذكرت إمكانية استخدام الحياة الاصطناعية... أعتقد أن الذكاء هو خاصية ناشئة، خاصية يتم إنشاؤها. أنا لا أؤمن بالذكاء الاصطناعي في حد ذاته، لكن من المؤكد أن الحياة الاصطناعية ستعمل على تطوير الذكاء.
    اسمع...إن الوصول إلى مستوى الذكاء الذي خلقهم لا يحتاج إلى أجيال عديدة من التطور...

  427. إريك:
    الآن أرى سؤالك لي.
    ستكون هناك منافسة للأسباب التالية:
    1. لا يوجد شيء غير محدود
    2. إن القيد شديد بشكل مضاعف لأن التكاثر يكون محليا – أي – فحتى لو كانت هناك موارد كثيرة في العالم، فإن هذه الموارد تكون محدودة أكثر بكثير في المكان الذي يتواجد فيه الناسخون.
    3. الموارد وهذا يشمل كل شيء - بما في ذلك المكان الذي سأتواجد فيه والمكان الذي أكون فيه، ولا يمكن العثور على أي شخص آخر.
    4. المكان له تأثير على الموارد الأخرى - إذا كان شخص ما معي، فإنه يحصل على ضوء الشمس أكثر مني وأحصل على المزيد من الموارد من الأرض

  428. מיכאל
    وطالما أن هناك موارد كافية، فلن تكون هناك منافسة. وهذا صحيح بالنسبة لضفادع القصب، وهذا صحيح بالنسبة للبشر. ما الذي يصعب فهمه هنا؟
    في مجرد النسخ المتماثل لا يوجد نقل المعلومات. في الكائن الحي هناك مفاهيم النمط الجيني والنمط الظاهري، وبدون هذه الأشياء لا يوجد (في رأيي) تطور.
    هيا لنتفق على أن نختلف..
    نعم - أوافق تماماً على أن الحياة الاصطناعية ليست حياة حقيقية. وله استخدامات معينة، والدليل على ذلك أننا تعلمنا الكثير عن التطور الحقيقي باتباع برامج مثل Tiara وAbida. لكن أبعد من ذلك لا أعتقد أن له أي فائدة.

    ما يذهلني هو النهج الديني. إذا كانت قصة التكوين صحيحة فإن الحياة نفسها مصطنعة!! لقد بحثت قليلاً في تاريخ الحياة الاصطناعية وكان الموضوع قديمًا جدًا - الغولم من براغ، وحش فرانكنشتاين، بينوكيو، هامش الساحر لغوته، غلاطية (تمثال بجماليون) وأيضًا قصة الخلق.
    التطور أفضل بكثير.....

  429. إريك:
    لم يزعم نسيم أن التكاثر السريع جدًا يؤدي إلى تطور سريع جدًا.
    على العكس من ذلك - فقد قدم مفهوم الضرب السريع دون منافسة (وهو، كما قلت - غير ممكن) ليجادل بأن التطور لا يجب أن يبدأ على الفور.
    لا يعني ذلك أنه سيحدث فرقاً جوهرياً لو كان ممكناً، لكنه كما ذكرنا مستحيل.

  430. المعجزات:
    ليست هناك حاجة لقول الأشياء مرة أخرى لأن قولها مرة أخرى يتركها غير منطقية كما كانت، وعلى الرغم من أنك لم تشرح بعد كيف أنه من الممكن ألا تكون هناك منافسة وأعتقد أنك لن تشرح أيضًا لأنه ببساطة مستحيل.
    علاوة على ذلك، عندما يكون هناك ازدواجية، يكون هناك، بحكم التعريف، نقل للمعلومات - وإلا فلن يكون ذلك ازدواجية.

    عندما تقول أنه لم يخرج شيء من الحياة الاصطناعية، فأنت تبالغ في رأيي، لكن ذلك لا علاقة له بحالتنا.
    والمتعلق بحالتنا هو أنك عندما تقول لم يخرج منه شيء (بصيغة المستقبل) فإنك تقصد أنه حتى الآن لم يخرج منه شيء (بصيغة الماضي) وخاصة أنك تؤكد كلامي أنه على الرغم من وجود كائنات مكررة هنا تنقل المعلومات، لا توجد حياة هنا.

  431. مايكل روتشيلد
    سأتحدث عن رأيي مرة أخرى
    وطالما لم يتم إنشاء آلية الذاكرة (آلية نقل المعلومات بين الأجيال) فلا يوجد ما يمكن الحديث عنه عن التطور.
    ربما لم تكن هناك منافسة في المراحل المبكرة، مما سمح بالتكاثر السريع جدًا وبالتالي التطور السريع

    لقد تناولت في الماضي موضوع الحياة الاصطناعية - وهو مثل الذكاء الاصطناعي تماماً... مثير للاهتمام ولكن لن يأتي منه شيء...

  432. إسرائيل

    الآن أفهم ما يعنيه لايبنتز. وهو يتفلسف حول المفاهيم الأساسية للرياضيات، أي أنه ينسب إليها معنى مختلفا عما هو مقبول. أنا لست مهتما بمثل هذا الاتجاه الفلسفي، ولا أعتقد أنه يحتوي على أي محتوى حقيقي (لأن المفاهيم البديلة تبدو وكأنها مفاهيم غامضة، فهي تحتوي فقط على التلاعب بالألفاظ، وإيماءات اليد بدلا من القسم الفرعي العادي، وما إلى ذلك). بمجرد أن تتخلى عن الأدوات المنطقية، بما في ذلك الطريقة الاستنباطية (التي تشمل الرياضيات)، فإنك تبقى دون أدوات لفهم الواقع المعقد. على أية حال، بعد هذا التخلي، لن يكون لديك أي أدوات لمناقشة مسألة واقعية معقدة مثل "ما هو التعقيد الذي لا ينفصم". إن ممارسة ألعاب الكلمات ذات التفسيرات الغامضة لا يهمني.

    بقدر ما أفهم، لا يمكن تعريف "التعقيد وليس التفريغ" إلا فيما يتعلق بالواقع المادي؛ ليس من الممكن تعريف "التعقيد الذي لا ينفصم" فيما يتعلق بالأدوات العقلية التي ندرك بها الواقع المادي. وهذا خطأ ليبنوفتز (أبعد من تخليه عن المنهج الاستنباطي كما ذكرنا أعلاه). استنتاجاته الغريبة هي نتيجة للأخطاء المذكورة أعلاه.

    لست مهتمًا بمواصلة مناقشة نهج لايبنتز هذا. ما هي مجرد ألعاب الكلمات.

  433. إسرائيل:
    لغز:
    ثلاث بقرات تمشي في الصحراء في عمود خلفي مستقيم.
    تقول البقرة الأولى: هناك بقرتان خلفي.
    البقرة الثانية تقول: بقرة أمامي وبقرة خلفي.
    والثالث يقول: بقرة أمامي وبقرة خلفي.
    كيف يمكن أن يكون؟

    اجابه:
    والثالث يكذب.

    هذا مرتبط بالسؤال الذي طرحته عن الأشباح.

    وبالحديث عن الأبقار الناطقة، يدخل حصان إلى حانة ويطلب كأسًا من البيرة.
    وعندما ينتهي من الشرب يسأله كم ثمنه فيجيبه النادل "10 دولارات، لكن أخبرني ماذا يحدث هنا؟ لم يسبق لي أن رأيت حصانًا ناطقًا."
    أجابه الحصان: "بهذه الأسعار لن ترى المزيد أيضًا".

  434. تصحيح طفيف:
    عندما تحدثت عن البرامج الموجودة على الكمبيوتر كنت أقصد تلك البرامج المصممة لتحقيق هدف معين مثل لعب الشطرنج أو حل مشكلة الوكيل المتجول.
    هناك مجال كامل يسمى "الحياة الاصطناعية" ولكن حتى هناك - على الرغم من أن أولئك الذين يستخدمون كلمة "الحياة" لا يعتقدون أن الأنظمة حية بالفعل.

  435. المعجزات:
    التوقف عن كونها حاسمة جدا!
    لم أقل أن المنافسة ضرورية للنجاح. قلت إن المنافسة تنشأ بشكل طبيعي ولا مفر منها.
    ولم تأت بحجة تناقض هذا الكلام.
    سيكون جميلاً جداً لو كان من الممكن العيش دون استهلاك الموارد على الإطلاق، لكن هذا غير ممكن.
    في بعض الأحيان تكون المنافسة أسهل وأحياناً تكون أصعب ولكنها موجودة دائماً.
    لم تتمكن من تقديم أي نص حيث لا توجد منافسة ومن الواضح أنك لن تنجح.

    إذا أردت تعريف الحياة بأنها كأس من البيرة، فسوف تحدث أشياء مثيرة جدًا للاهتمام، لكن لا أحد يطلق على الكيانات ذاتية التكاثر التي تم إنشاؤها بواسطة الخوارزميات التطورية على الكمبيوتر اسم الحياة، ولا أحد يطلق على كرات زيت لانزيت أو طين كوفمان حياة.

  436. إسرائيل شابيرا
    يجدر التأكيد على ما قلته سابقًا - التعقيد الذي لا ينفصم لا يتطلب "الله". حتى لو كان الأمر يتطلب مخططًا (وهو ليس بالضرورة)، فإن هذا المخطط يمكن أن يكون نتيجة للتطور.

  437. متشكك، أنت في عداد المفقودين هذه النقطة.

    وهي: في الجدل الدائر بين مؤيدي التطور ومنكري التطور، يثير أصحاب التصميم الذكي النقطة التالية: هناك "تعقيد لا ينفصم" في الطبيعة - السياط، وأنظمة المناعة، وما إلى ذلك. وهذا دليل على ادعائهم أن هناك أشياء لا يمكن أن تتطور بنفسها عن طريق التطور، وأن هناك مخططا ذكيا (ما نسميه الله في الحظيرة).

    يحاول البروفيسور ليبنوفيتس أن يثبت أن التخطيط الذكي والتعقيد الذي لا ينفصم موجودان أيضًا في عالم الأرقام، وهذا هو الدليل على وجود الله. (وهذا بالمناسبة رد على ادعاء القانون الثاني بأنه ملتزم بقوانين الإحصاء).

    يعطي البروفيسور الأعداد المثالية كمثال على التعقيد الذي لا ينفصم: فهي جميعها لها خصائص مستقلة على ما يبدو والتي تظهر دائمًا معًا.

    أثبت إقليدس أويلر ومايكل أن هذه الخصائص ليست سوى تعبير عن خاصية واحدة. لكن هذا هو بالضبط مغزى القصة: أن ما يُنظر إليه اليوم على أنه تعقيد لا ينضب، ودليل على التخطيط الذكي (في ظاهره، تبدو الأعداد المثالية وكأنها سحر غامض يتطلب التخطيط)، يمكن اختزاله إلى تعقيد غير معقد وبسيط.

    إذا كنت لا تصدقني، من فضلك اختزل لي الخاصية الإضافية غير القابلة للتحلل المشتركة بين جميع الأعداد المثالية: جميعها زوجية.

    من المؤكد أن هذه الميزة ليست رائعة مثل الميزات الثلاثة الأخرى.

    هل يمكنكم، أيها المعلقون الرائعون، مساعدة المتشككين في تضييق نطاق الميزة 4، أو إثبات عدم وجودها؟

    300,000 دولار.

    سننتهي من ذلك - سننتقل إلى خاصية الأصفار غير التافهة في دالة زيتا. لديهم أيضًا تعقيد لا ينفصم: فهم جميعًا على الخط الحرج 1/2

    (باستثناء تلك الموجودة في القيم السلبية).

    بالإضافة إلى ذلك، أنا متشكك، إذا كان لديك أي شكوى، تعال إلى البروفيسور ليبنوفيتس، وليس إلي. ما أنا والده؟!

    يا..

    جيد.

    وأنت على حق، لم آخذ أي دروس علمية محددة.

    فقط في الرياضيات والكيمياء والفيزياء.

    الأشباح - إذا كنت تدعي أنه يمكنك التواصل مع الكائنات الفضائية من خلال الرياضيات، فكيف يمكنك أن تدعي أن الإنسان هو من خلق الرياضيات؟ ماذا لو كانت الكائنات الفضائية موجودة قبل الإنسان، فكيف تمكنوا من خلق نفس الرياضيات؟

  438. إسرائيل

    أنت تحول النقاش إلى مسألة ما إذا كان من السهل أو الصعب إثبات النظرية (صيغة في 2) من التعريف (صيغة في 1). هذا ليس السؤال الذي كنت أشير إليه.

    من يفهم المعلومات على المستوى الشعبي عن 1 و 2، حتى من دون فهم البراهين، يفهم على الأقل أن: 1 تعريف و 2 جملة وصفية للتعريف في 1. وبمجرد أن يفهم هذا، لن يجزم أن أي خاصيتين مستقلتان. ولا أتفاجأ إذا لم تفهمه، لأنه ليس من الضروري أن تفهمه إذا لم تدرس العلوم الدقيقة.

    ما أتساءل عنه هو لماذا، برأيك، لم يفهم لايبنتز هذا الأمر. إذا كان لديه تعليم جيد في العلوم الدقيقة، فإن كلماته "غريبة" بعبارة ملطفة. هناك احتمال أنك لم تفهم ليبانوفيتز أيضاً، ولهذا السبب أخطأت في صياغة كلامه. هناك أيضًا احتمال أن يكون لايبنيز يتفلسف في صياغاته ويسبب ارتباكًا غير ضروري.

    المطالبة 3 تعادل 2؛ وهذا أمر بديهي لطالب المدرسة الثانوية بمجرد أن يفهم *حقًا* ما تعنيه الصيغة المختصرة لمجموع سلسلة حسابية. كان ينبغي عليك أن تفهم هذا على أية حال، بافتراض أنك حصلت على هذا النوع من التعليم على الأقل وأنك مهتم بالفيزياء العليا.

    فيما يتعلق بسؤالك حول المبالغ الزوجية الأولية (والتي من المفترض أن تحدد جميع المبالغ الزوجية). حسب ذاكرتي، هذا سؤال مشهور لم يتم حله حتى يومنا هذا (لا أتذكر اسم الرجل الذي طرح هذه الفرضية).

  439. מיכאל
    توقف عن كونك حاسمًا جدًا.... من الناحية النظرية، فإن النسخة المتماثلة التي لا تقتصر على الموارد سوف تتكاثر بشكل أسرع بكثير من النسخة المتماثلة التي تتنافس ضد نوعها. انظر إلى ضفادع القصب في أستراليا... لقد تضاعف عددها من حوالي 100 إلى أكثر من 200 مليون خلال 80 عامًا. لا تحتاج إلى المنافسة لتكون ناجحا. وقد تطورت بالفعل - دون أي منافسة على الموارد.

    لقد كتبت أيضًا أنه بمجرد أن يتكاثر البعض، فإن التطور سوف يأخذ الحياة إلى الأمام. انها واضحة.
    وإذا كنت تريد تعريف الحياة كما عرّفها غانتي في ذلك الوقت على أنها كيان مكرر بقاعدة بيانات، فإن الحياة والتطور بدأا معًا بالفعل. وهذا أمر مشروع أيضًا.

  440. ومن المهم أيضًا أن نلاحظ أنه لم يتم تقديم أي تفسير على الإطلاق لتكوين الحياة الذي لا يقع ضمن الفئة التي وصفتها.

  441. من المهم أن نفهم:
    ففي نهاية المطاف، لا أحد يدعي أن التطور هو الذي خلق الأرض.
    والسؤال هو فقط كيف خلقت الحياة على الأرض وكيف يمكن أن تصبح المادة غير الحية حياة.
    بمجرد أن نفهم أن أي نسخ مكررة - في تعريف واسع جدًا للكلمة - يمكن إنشاؤها تلقائيًا - يمكن أن يأخذنا التطور منها إلى الإنسان.
    من المستحيل أن نطرح كل الأسئلة الصعبة حول احتمالية خلقهم على هذه المستنسخات التلقائية، لأنه من الواضح أنهم سيتم خلقهم بأنفسهم.

  442. المعجزات:
    إذن نعم!
    كان من المستحيل عدم وجود منافسة لأنه منذ اللحظة التي قمت فيها بالتكرار، أصبح لديك منافسون على الموارد.
    إن قول "لا" بهذه الطريقة دون دحض الحجة هو بمثابة التلويح باليد.
    لا يحتاج التطور إلى تفسير تكوين الناسخ الأول، لكن هذا الناسخ كان لا يزال بسيطًا وغير حي.
    السؤال هو كيف خلقت الحياة - هل تتذكر؟ وقد تم إنشاء الحياة عن طريق التطور من مكررات بسيطة كانت عبارة عن مادة غير حية.

  443. מיכאל
    هذا كل شيء، لا 🙂 عليك أن تفهم أن الانتقاء الطبيعي يبطئ عملية التكاثر. ولذلك، فمن المرجح أن المكررات الأولى لم تتنافس مع بعضها البعض.

    وعلى أية حال، لا يمكن للتطور أن يفسر تكوين الناسخ الأول. ما هو غير تافه في هذه الجملة؟

    لقد أخبرتك أن الأمر أكثر تعقيدًا من ذلك بقليل: بخلاف الشروط الأربعة التي ذكرتها، هناك شرط آخر هنا - النمط الجيني/النمط الظاهري. وفي رأيي أنه بدون هذا الفصل يكون التطور "أضعف". لديها أساس رياضي مثير للاهتمام - والذي يوضح أيضًا أنه حتى لو كان هناك تخطيط ذكي، فإن الحياة على أي حال كانت ستتطور في التطور وستقضي على الحياة المخططة.

  444. المعجزات:
    غير صحيح!
    يحتاج التطور إلى ناسخات ولكن ليس إلى ناسخات متطورة كما لدينا اليوم.
    لقد ذكرت البلورات، بعض الناس يتحدثون عن الطين، والبعض يتحدث عن البريونات، والبعض يتحدث عن الكرات الدهنية.
    على الأرض، تم إنشاء جميع أنواع الأشياء بشكل عشوائي.
    هذه لم تكن على قيد الحياة بعد، ولكن بعض هذه الأشياء كان لديها القدرة على التكاثر وبعضها استخدم نفس أنواع الموارد لغرض التكاثر وبالتالي "تنافست" وبالتالي في عملية التكاثر والانتقاء الطبيعي تم إنشاء آليات ذلك تم تكرارها بشكل أفضل وأفضل وفي النهاية اكتسبت إحدى هذه الآليات الحق في أن تسمى "الحياة".
    كل شيء هنا طبيعي، والآلية الوحيدة التي تديره هي القوانين الرياضية للتطور، ولا يوجد إله أو أي شيء غامض آخر هنا.
    بالمناسبة - هناك منافسة منذ اللحظة الأولى لأنه منذ اللحظة التي قمت فيها بالنسخ كنت أتنافس مع نسختي المكررة (بعد كل شيء، إذا كانت نسخة مكررة فإنها تحتاج على الأقل تقريبًا إلى نفس الموارد التي أحتاجها لمواصلة النسخ).
    من الممكن أن يكون لدي مساحة كبيرة لأنتشر فيها، ولكن مع ذلك - أولئك الذين يفضلون المنافسة يفوزون.

  445. قرات…

    أنت تفكر مثلي... ولكنك تخطئ مرارًا وتكرارًا - التكاثر شرط للتطور وليس نتيجة للتطور. لقد كتبت بنفسك "التطور يحتاج إلى أشياء تتكاثر".

    أود أن أفهم ما هو التكاثر بالمعنى البيولوجي 🙂 مايكل، نحن نفكر بنفس الطريقة. لكن - أنت تقول أن التطور بدأ الحياة - وها أنت من يقع في الأيدي الدهنية للخلقيين، لأنني أوضحت لك أنه لا يوجد تطور بدون تكاثر (التكاثر أو كما تسميه). وأنا، من ناحية أخرى، أدعي أن هناك عملية كيميائية أدت إلى خلق شيء يتكرر (نظرية إيغان، نظرية كارنز سميث أو أي نظرية أخرى). فهو يتكاثر ويتكاثر بسرعة كبيرة وبدقة شديدة لأنه لا يوجد منافسة !!! وفي مرحلة لاحقة، ظهر أصدقاؤنا ميلثوس في الصورة، وعندها فقط بدأ التطور.

  446. التطور لا يحتاج إلى التكاثر بالمعنى البيولوجي للكلمة.
    إنها تحتاج إلى "أشياء" تتكاثر مع الحفاظ على خصائص معينة و"تتنافس" مع بعضها البعض على الموارد.
    يمكن أن تكون هذه الأشياء جزيئات أو مخازن جزيئات أو بلورات وما إلى ذلك.

  447. إن حقيقة ادعائك بأنه يمكنك الشرح ليست تفسيرًا.
    طالما لا يوجد تفسير، فهذه هي حركات اليد.
    إن تعريفي للتطور ليس واسعًا جدًا. ما هو "واسع جدا" في هذا السياق؟
    هل قرأت الروابط التي أعطيتك إياها؟

  448. מיכאל
    حقا حقا لا. لقد شرحت لك لماذا لا. أنا لا أرى كلماتي بمثابة التلويح باليد، أستطيع أن أشرح كل ادعاء من ادعاءاتي.
    من المحتمل أن يكون تعريفك للتطور واسعًا للغاية، وبالتالي فإنك تلقيه على تكوين الحياة.

    للتوضيح - سأطرح عليك سؤالًا بسيطًا: إذا كان التطور يحتاج إلى التكاثر، فكيف يخلق التكاثر في المقام الأول؟
    من المؤسف أنك لم تحاول فهم ما كتبته من قبل ... هذا ملخص لنصف أطروحتي في بضع جمل 🙂

  449. كم مرة كررت أن التعقيد الذي لا ينضب يمكن بل ويجب أن ينشأ في كون لا نهائي، لن يغير حقيقة أن الكون محدود أو حقيقة أنه غير محتمل جدًا جدًا جدًا جدًا جدًا.
    بعد كل شيء، لقد قلت بنفسك أنه إذا وجدت دباسة على المريخ، فسوف تعتقد أن مصممًا ذكيًا شارك في إنشائها، فماذا حدث لهذا الكون اللامتناهي بأكمله؟

  450. المعجزات:
    أنت مخطئ.
    من الممكن أن تكون الحياة قد نشأت من مادة غير حية من خلال التطور، وهناك عدد لا بأس به من السيناريوهات التي توضح مدى حدوث ذلك تقريبًا.

    على سبيل المثال، اقرأ هذا المقال والروابط الموجودة فيه:
    http://sciam.co.il/archives/1833

    أو مشاهدة المحاضرة التالية:
    http://www.youtube.com/watch?v=f7iByyEqgtY&feature=youtu.be

    إن التلويح بيديك والقول إن هناك حاجة إلى آلية أخرى هنا ليس حكمة.
    لا يوجد ما يستبعد إمكانية التطور في خلق الحياة، ولا توجد آلية أخرى يقترحها أي شخص كتفسير لأصل الحياة.

    إن الادعاء بأن التطور لا يمكنه معالجة أصل الحياة هو أحد الأخطاء الشائعة لمؤيدي العلم لأنهم بفعلهم هذا فإنهم في الواقع يحولون الساحة في هذا المجال إلى الله.

  451. مايكل روتشيلد
    يمكن أيضًا إنشاء تعقيد لا ينفصم بشكل عشوائي. قلت في وقت سابق - حقيقة أن هذا "غير مرجح" لا يستبعد هذا الاحتمال. في الكون اللانهائي، يجب أن يحدث ذلك.

  452. מיכאל
    دعونا نذهب قليلا إلى آلية التطور. لكي يحدث التطور يجب توافر عدة شروط، وهناك 4 شروط من هذا القبيل. وشروطها هي: التكاثر، والمنافسة، والتنوع، والوراثة. هذه شروط كافية وضرورية. في مقال مؤطر سأذكر فقط أنني شخصياً أعتقد أن الأمر أكثر تعقيدًا من ذلك بقليل... ولكن في وقت آخر.

    وبعبارة أخرى، "لبدء" الحياة، تحتاج إلى آلية مختلفة تماما. نحن نعرف عددًا من هذه الآليات، لكننا لا نعرف أي منها هو الصحيح، أو أنها آلية مختلفة تمامًا.

    إنها خدعة غامضة من أنصار نظرية الخلق لربط التطور بخلق الحياة - لا تقع في هذا الفخ 🙂

  453. وفيما يتعلق بردك اللاحق، يبدو لي أنك انحرفت عن الموضوع.
    لم أقل أنه لا يوجد أي إفرازات من شأنها أن تظهر وجود الله. قلت فقط إنك سترى وجود مخطط ذكي.

  454. المعجزات:
    أنا لا أفهم إلى أين أنت ذاهب.
    لقد قلت منذ وقت طويل أنني أعتقد أن هناك تطور.
    أنا ثابت كالجحيم لكنك لا تتبع الاتساق.
    معظم أشكال الحياة على الأرض (تلك التي لم نخلقها من خلال التصميم الذكي) نشأت عن طريق التطور.
    وكل الشواهد تشير إلى هذا.
    ولكن إذا تم العثور على دليل على وجود آلية لا تنفصم (وشرحت لماذا من غير المرجح أن تجد مثل هذا الدليل لأن ادعاء عدم القابلية للانفصال عادة ما يكون حجة من الجهل) في كائن حي اعتقدنا أنه تم خلقه من خلال عملية تطورية العملية - سيكون هذا دليلًا موثوقًا للغاية على تورط مصمم ذكي في إنشاء هذا الحيوان.
    وهذا من شأنه أن يتناقض مع ادعائنا الشامل بأن كل أشكال الحياة على الأرض جاءت عن طريق التطور.
    إذا تم العثور على آلية لا تنفصم في جميع الكائنات الحية، فسوف تتعارض مع تكوين الحياة بشكل عام في عملية التطور.

    ونقطة مهمة:
    التطور هو الآلية الوحيدة التي يمكننا تقديمها حاليًا كتفسير لتكوين الحياة.
    رغم أننا لا نعرف ما هي العملية ومن الممكن جداً أننا لن نعرف أبداً، لكن ليس لدينا أي تفسير غير تطوري (ولا أرى التخطيط الذكي تفسيراً لأن التخطيط الذكي يتطلب وجود شيء ما) المعيشة التي سيتم التخطيط لها في المقام الأول)

  455. מיכאל
    سأشرح وجهة نظري مرة أخرى. لنفترض أن الشوتون البكتيري ذو تعقيد لا ينفصم، وبالفعل سيتم إثبات أنه لا يمكن أن يكون قد تطور في التطور. هراء...ولكن لنفترض.
    لذلك فمن الأرجح أن يكون Shotton قد تم تصميمه بواسطة مخلوق من عالم آخر، دعنا نقول كروبوت نانو للأغراض الطبية... من هنا المسافة إلى تلوث الأرض قصيرة.
    والله لم يخترع بعد....

  456. מיכאל
    لكن لكن لكن... اتفقنا على أنه لا يوجد أي خطر... لأنه حتى لو كانت هناك آلية معقدة للغاية فمن الممكن أن يكون مخلوق آخر هو الذي خلق هذه الآلية. أنت لست من المتابعين....

    والنقطة المهمة هي أن التطور لا يرتبط بخلق الحياة. هناك العديد من الآليات المحتملة لتكوين الحياة، وقد لا نعرف أبدًا أيًا من هذه الآليات أنتج الحياة بالفعل على كوكبنا.

    الحياة شرط أساسي للتطور – لذلك ليس من المنطقي أن يكون التطور هو أصل الحياة.

  457. وبالطبع نحن متفقون على ذلك.
    إن الخطر على التطور (كتفسير لتكوين كل أشكال الحياة على الأرض) لا يكون إلا إذا تبين أن الحياة على الأرض لها آليات لا تنفصم (وأعني الحياة التي لم يخلقها الإنسان: حتى اليوم، لا يفسر التطور كل شيء). أشكال الحياة الموجودة لأن البكتيريا التي أنشأها كريج فينتر تم إنشاؤها بمشاركة التخطيط الذكي).

  458. ففي نهاية المطاف، لن يتبادر إلى الذهن دحض نظرية التطور كآلية.
    سينجح الأمر حتى لو تم إنشاء كل أشكال الحياة على الأرض من خلال تصميم ذكي.
    الأمر نفسه ينطبق على التخطيط الذكي وأفعالنا الذكية تثبت ذلك - فالتخطيط الذكي ممكن وموجود.
    عند مناقشة مسألة "التطور مقابل التصميم الذكي" فإننا نعني فقط مسألة ما إذا كانت الحياة على الأرض هي نتيجة التطور أم التصميم الذكي.

  459. صحيح أنها معجزات، لكن لا تتشوش.
    بعد كل شيء، قلت إننا نحاول خلق حياة متساوية من خلال التصميم الذكي.
    عندما أتحدث عن حقيقة أن شيئًا ما قد تم إنشاؤه بواسطة تصميم ذكي (وهناك الكثير من هذه الأشياء) - فهذا لا يتعارض مع الادعاء بأن شيئًا آخر تم إنشاؤه عن طريق التطور. إنه يناقض نفسه فقط لأنه خلق عن طريق التطور.

  460. أشباح:
    أنا لا أطلب منك أن تقرر ما إذا كانت الرياضيات مجرد لغة كما قلت من قبل أو ما نقلته للتو.
    أنت لا تفهم معنى أي من هذه الادعاءات المتناقضة، فكيف يمكنك أن تقرر؟

    أنا تعبت منكم.

  461. من * آل
    حقيقة أنك تعرف كيفية خربشة الأرقام والحروف لا تجعلك شخصًا يفهم شيئًا عن طبيعة الحياة.

    http://en.wikipedia.org/wiki/Mathematics :

    "يقوم علماء الرياضيات بتحليل صحة أو كذب التخمينات عن طريق البرهان الرياضي. عندما تكون الهياكل الرياضية نماذج جيدة لظواهر حقيقية، فإن التفكير الرياضي يمكن أن يوفر رؤية أو تنبؤات حول الطبيعة. - هو قال. الرياضيات تشرح الطبيعة بطريقة منطقية ومن خلال البراهين. والطبيعة لا تشرح الرياضيات (كما اعتاد سام كال أن يعتقد).

    اصدقاء:
    http://en.wikipedia.org/wiki/Definition_of_mathematics
    أنتم مدعوون للقراءة والشرح لمايكل أين هو مخطئ.
    "
    الموضوع الذي لا نعرف أبدًا ما نتحدث عنه، ولا ما إذا كان ما نقوله صحيحًا. (من الرابط)

  462. ماشيل
    حقيقة أنك تعرف كيفية خربشة الأرقام والحروف لا تجعلك شخصًا يفهم شيئًا عن طبيعة الحياة.

    http://en.wikipedia.org/wiki/Mathematics :

    "يقوم علماء الرياضيات بتحليل صحة أو كذب التخمينات عن طريق البرهان الرياضي. عندما تكون الهياكل الرياضية نماذج جيدة لظواهر حقيقية، فإن التفكير الرياضي يمكن أن يوفر رؤية أو تنبؤات حول الطبيعة. - هو قال. الرياضيات تشرح الطبيعة بطريقة منطقية ومن خلال البراهين. والطبيعة لا تفسر الرياضيات (كما اعتاد مايكل روتشيلد أن يعتقد).

    اصدقاء:
    http://en.wikipedia.org/wiki/Definition_of_mathematics
    أنتم مدعوون لقراءة وشرح لمايكل روتشيلد أين هو مخطئ.
    "
    الموضوع الذي لا نعرف أبدًا ما نتحدث عنه، ولا ما إذا كان ما نقوله صحيحًا. (من الرابط)

  463. نعم:
    أنا أتفق مع حقيقة أن المخلوق الذي تم إنشاؤه في تطور داروين سوف يصمم آليات لا تنفصم، ومن الواضح أن هذه الآليات ستكون آليات تم إنشاؤها بمشاركة التخطيط الذكي

  464. مايكل روتشيلد
    سأعطيك واحدًا منهم فقط …… للمرة الثانية.
    ليس هناك ما يمنع مخلوق خلقه تطور داروين من تصميم آليات غير قابلة للتدمير، أليس كذلك؟ هل توافق على هذا

  465. מיכאל
    إذا رأيت مشبكًا على المريخ سأقتنع أن مخلوقًا ذكيًا هو من فعل ذلك.
    لكن - لقد أعطيتني نصين يتعارضان مع ادعاءك. أنت لم تستبعدهم. انا مخطئ؟ و بالتأكيد هناك المزيد…

  466. والمعجزات:
    والآن باليد على القلب:
    إذا أتيت إلى المريخ ورأيت هذا الترتيب من الأكواب والسكاكين كما صورته: هل ستستنتج أنه تطور في التطور؟ هل حدث ذلك بشكل عشوائي؟ أم أن مخلوقاً عاقلاً فعل ذلك؟

  467. المعجزات:
    أنا لا أقبل الحظر الخاص بك.
    مسموح لي أن أطالب بما أريد، كما يُسمح لي أن أدعي "غير معقول".
    إن مصطلح "الآلية الضرورية للحياة" ليس هو ما يلعب دوراً في التطور. ما يلعب فيه هو الآليات التي تسمح لك بالبقاء على قيد الحياة بما يتجاوز تقديراتك. من الممكن أن نعيش دون رؤية حتى في بيئة مخيفة، فهل البصر ضروري للحياة؟ إنه ضروري كما ذكرنا فقط للمنافسة مع المخلوقات الأخرى وإذا رأوا فلن تتخلى عن بصرك.
    ولكن ماذا عن الحياة في الظلام؟
    هناك بالفعل حيوانات تفقد قدرتها على الرؤية تدريجيًا.
    إن الآلية غير القابلة للفصل والتي ليس لها أي وظيفة في الحياة هي شيء غير محدد.
    ماذا يعني "اخرج جزء وتتوقف الآلية عن العمل" إذا لم تعمل في المقام الأول؟

    أشباح:
    إن معرفتك بكيفية كتابة الكلمات لا تعني أنك تفهم اللغة العبرية. الحقيقة هي أنك لم تفهم سؤالي المكتوب باللغة التشاتشا العبرية على الإطلاق.

  468. مايكل روتشيلد
    هناك خياران فقط؟ وماذا عن العشوائية مثلا؟ إذا كانت جميع الآليات غير القابلة للتحلل بسيطة بما فيه الكفاية، فلا يوجد ما يمنعها من التولد بشكل عشوائي. وفي الكون اللانهائي لا يُسمح لك بادعاء "غير محتمل".
    الاحتمال الآخر هو أنه إذا لم تكن هذه الآليات التي لا تنفصم ضرورية للحياة (وليس لدي أي سبب للتفكير بطريقة أخرى)، فإن شكلاً آخر من أشكال الحياة، التي لا تحتوي على آليات لا تنفصم، هو الذي خلق هذه الآليات. ومرة أخرى - ليست هناك حاجة لمخطط ذكي مسبق.
    وهذا قبل أن أنتهي حتى من تناول البيرة......

  469. أنا لا أفهم العبرية أيضاً؟ إذن ما هي اللغة التي كتب بها هذا التعليق؟ فرنسيه؟
    كما قلت…. حسنًا، في الواقع لا أريد أن أكرر الأشياء لأنك سوف تشعر بالإهانة.
    (وليس لدي أي نية لإهانتك، ولكن فقط لإظهار خطأك)

  470. أشباح:
    لقد تمت صياغة سؤالي باللغة العبرية، لكن اتضح أنك لا تفهم هذه اللغة أيضًا (وهي في الواقع مجرد لغة).

  471. المعجزات:
    لأنه في الوقت الحالي لا يوجد سوى احتمالين: إما أن الاستخبارات كانت متورطة في التصميم أو لا.
    فإذا ثبت أنه بدون ذكاء (من خلال التطور) لا يمكن خلق هذا (وهذا ما ثبت عند اكتشاف التعقيد الذي لا ينفصم لأن كل ما يخلقه التطور هو تعقيد غير معقد)، فإنه يتم إنشاؤه بالتخطيط الذكي.

  472. سؤالك (المقصود منه، في نظرك، التحقق مما إذا كان المعلق الذي أمامك متديناً أم لا) ليس في محله.

    وحقيقة أنني أضعك في مكانك هو سبب آخر لكتابة هراء مثل: "أنت غير قادر على التفكير بشكل صحيح".

  473. ومرة أخرى أوضح (لأولئك الذين لم يفهموا): الاعتقاد بأن الطبيعة لا تخضع لقوانين الرياضيات هو هراء.
    لكن الاعتقاد بأن الرياضيات كانت موجودة قبل البشر هو هراء أكبر.

  474. أشباح:
    حسنًا - لنفترض أن الرياضيات لغة.
    أخبرنا بهذه اللغة لماذا في أي مجموعة من الأعداد الطبيعية أقل من ألف ومضاعفها المشترك الأصغر لأي اثنين منها أكبر من ألف، يكون مجموع المعكوسات (مقلوب العدد هو واحد مقسوم على نفس العدد) من أعضاء المجموعة أقل من واحد ونصف.
    السبب وراء عدم معرفتك كيفية الإجابة ليس لأنك لا تعرف اللغة (أنت تعرف كل الكلمات اللازمة للإثبات) ولكن لأنك غير قادر على التفكير بشكل صحيح.

  475. ماشيل
    لقد كتبت: "بمجرد إنشاء شيء ما من خلال عملية تطورية - فهذا دليل على أن تعقيده هو تحلل (لأن العملية التي خلقته هي تحلل للتعقيد)." حتى الان جيدة جدا.

    ثم كتبت "وبالتالي فإن إثبات وجود تعقيد لا ينفصم في الطبيعة سيكون دليلاً على التصميم الذكي". لا أفهم من أين أتيت "لذلك"... لا أرى كيف يترتب على الادعاء السابق.

    أبعد من ذلك، أنا أيضًا لا أقبل أن الدليل "أ" يؤدي إلى "ب".... دون أن يكون هناك ارتباط مثبت بين الأمرين... فلنفترض أن هناك آلية أثبتنا أنها ليست تفريغاً. من أين يأتي المخطط الذكي؟
    أنا لا افهم ما تقول"

  476. أشباح:
    فقط طبق نصيحتك على نفسك وسيكون كل شيء على ما يرام.
    الرياضيات ليست مجرد لغة. لها لغة ولكن معرفة اللغة لا تعادل معرفة البرهان أو معرفة الحقائق.
    لكن قلت لا أريد أن أدخل في جدال سخيف والجدل معك مبني على مثل هذا التعريف لذا أتوقف هنا

  477. أعتقد أنه سيكون من الممكن إنشاء لغة مشتركة مع الكائنات الفضائية من خلال اللغة الرياضية. لكن لا تتحدث عن الرياضيات مع كائنات فضائية لأنها بحتة.

  478. عندما يقول شخص ما إنه لا يريد الدخول في جدال سخيف، فلا ينبغي له أن يتحدث بالهراء.
    الرياضيات هي لغة.
    http://he.wikipedia.org/wiki/%D7%9E%D7%AA%D7%9E%D7%98%D7%99%D7%A7%D7%94
    ("إن معرفة واستخدام الرياضيات الأساسية كانت دائمًا جزءًا طبيعيًا وأساسيًا من حياة الإنسان والجماعة. ويمكنك العثور على تضمينات للأفكار الأساسية في النصوص الرياضية التي أنشأها المصريون والبابليون والهنود والصينيون واليونانيون والمسلمون. ")
    لذلك، عندما يدعي شخص ما أن الرياضيات كانت موجودة قبل البشر وأن الطبيعة أطاعت الرياضيات قبل البشر، فهو ببساطة يتحدث عن هراء.
    لكن هذا الهراء ينبع من الارتباك - لأن الطبيعة تخضع لقوانين الرياضيات ليس لأن الرياضيات تملي الطبيعة - ولكن لأن الإنسان يملي الرياضيات وفقا لقوانين الطبيعة.

    إسرائيل
    ببغائي لا يتحدث لغة الرياضيات. (تستطيع سؤاله)
    كما أنني لا أعلم أن الحيوانات المجنحة كانت تتحدث الرياضيات قبل وجود الإنسان.
    إن الاعتقاد بأننا سنتحدث اللغة الرياضية مع كائنات فضائية هو أمر ساذج.

  479. بالمناسبة - من الجميل أن أرى شرحي لموضوع الأرقام المفصلة يتردد في التعليقات الأخرى.
    أي شيء رأيت الدليل عليه يمكن تقديمه على أنه تافه.

  480. إسرائيل:
    لقد امتنعت عن الإشارة إلى ذلك حتى الآن، لكنني اكتشفت أيضًا صيغة مجموع عمود حسابي حتى قبل الصف السادس.
    بالطبع أنا أتفق مع الادعاء بأن إله الفجوات هو إله رديء. أردت فقط أن أضع الأمور في نصابها الصحيح رياضيا.
    فيما يتعلق بالإنتروبيا - فقد قدمت إجابة على مسألة انعكاس الزمن (والتي بالمناسبة - لا توجد في جميع العمليات الفيزيائية ولكنها موجودة في معظمها) وقلت إن كل ما يحتاج إلى تفسير هو حقيقة أن الإنتروبيا كانت منخفضة لنبدء ب. بمجرد أن نشرح ذلك، سيتم حل كل شيء ولن يكون عكس الزمن مشكلة.
    كما ذكرت أن بنروز اقترح حلاً للمسألة ومن بين كل الحلول التي رأيتها يبدو لي أن هذا الحل هو الأكثر إقناعاً (في الكتاب الذي أشرت إليه يشرح أيضاً لماذا في رأيه الحلول المقبولة مبنية على على تسطيح الفضاء عن طريق التضخم ليست مقنعة).
    بالمناسبة - قد يتبين أن فرضية جولدباخ خاطئة أو سيثبتها شخص ما (العم بيتروس؟)، لكن جودل يؤكد لنا أن إله الفجوات سيكون لديه دائمًا مكان للاختباء.

    المعجزات:
    تعليقي على شموليك واقعي تمامًا وليس له أي علاقة بالتكاثر الجنسي.
    بمجرد أن وجدت مسار التطور التطوري الذي نجح في الانتقاء الطبيعي - وجدت التفكك - وأكرر وأذكرك بتعريف بيهي الثاني للتفكك.

  481. متشكك

    الأمر ليس بهذا الوضوح للوهلة الأولى. للعين غير المدربة تبدو الميزات مستقلة. هل كان بإمكانك إثبات الخاصية 2 (القوى) بنفسك دون إقليدس وأويلر؟

    وإذا كان الأمر كذلك، هل يمكنك أيضًا أن تشرح لماذا كل عدد زوجي هو مجموع عددين أوليين؟ أيضا ما هي الميزات التي تعتمد؟

    سأعطيك شخصيا نصف مليون دولار.

    النصف الآخر (الجائزة) - للديون.

  482. إسرائيل

    من هو ليبنوفيتس؟ أتمنى من أجله أنه *لم* يدعي أن الخاصية 3 مستقلة عن 2، أو أنه قرر أن يتفلسف حول هذا الموضوع.

    2 و 3 هو نفس الادعاء بالضبط في صيغ مختلفة. (إن استخدام صيغة مجموع المتسلسلة الحسابية - عند تطبيقها على الأعداد الطبيعية في الحالة السابقة - أمر بديهي ولا يحتاج إلى ذكره صراحة).

    بحسب ما هو مكتوب في ويكيبيديا العبرية، الميزة 2 تتبع مباشرة من التعريف. (لذلك: إذا كانت الخاصية 1 عبارة عن صيغة للتعريف، فإن 2 تتبع من 1. وحقيقة أن 2 تتبع مباشرة من التعريف أثبتها أويلر كما افترضت.)

    باختصار: كل ادعاءات عدم الاعتماد كاذبة.

  483. مايكل روتشيلد
    تعليقك على شموليك خارج الموضوع قليلاً: "إذا كان A يؤدي إلى B، فإن B لا يؤدي إلى C".

    أرجو أن تفهم أنه ليس كل شيء في الكائنات الحية يجب أن يتطور في خطوات صغيرة...... على سبيل المثال في التكاثر اللاجنسي قد تكون هناك تغييرات كبيرة (نسبيًا) لن تضر بالقدرة على التكاثر.

    ومن أجل القضاء على مشكلة القوباء المنطقية البكتيرية مرة واحدة وإلى الأبد - نبيع 3 أنواع مختلفة تمامًا من القوباء المنطقية. عندما أرى نفس الوظيفة تنفذ 3 مرات في برنامج كمبيوتر، أسميها "فوضى" وليس "تخطيط".
    وهو المطلوب.

    دعونا نمضي قدما؟

  484. مايكل، أنت مذنب.

    ماذا، ألا تدرك بنفسك أنني عندما أكتب "كل الأعداد الصحيحة هي سلسلة من الأعداد المتتالية" فأنا أقصد المجموع؟ منذ متى وأنت تعرفني

    وإينيل درابكوم لهذا المتجر الذي باعني كتابا عن عجائب الأرقام. ألا يمكنهم تحديد أن جميع الميزات مرتبطة؟ أهكذا توبيخني أمام الجميع؟

    مقاضاتهم.

    على كل حال، أحسنت وشكرا على الحل. إذا كنت أتذكر بشكل صحيح، استخدم غاوس البالغ من العمر 7 سنوات أيضًا صيغتك ليحسب في بضع ثوانٍ مجموع الأرقام المتتالية من 1 إلى 100.

    ولسوء الحظ، فإن هذا لا يجيب على المعضلة التي طرحها البروفيسور ليبنوفيتس في القصة.

    يُظهر الميزات الثلاثة على أنها غير مرتبطة. أثبت إقليدس وأويلر أن الأولين مرتبطان، وأنت وجاوس أثبتا الثالث أيضًا.

    لكن ماذا لو لم نعرف الحل؟ إذا لم يكن إقليدس وأويلر ومايكل وجاوس موجودين في العالم؟

    (مايكل، أنت لم تشعر بالإهانة من هذه المقارنة، أليس كذلك؟).

    كنا نتجول مع شعور بأن هناك نوعًا من القوة المتفوقة هنا، وأن جميع الميزات الثلاثة تركز على أرقام عشوائية لا يمكن التنبؤ بها، وأن Xingua القديم الجيد سيتحدث عن "التصميم الذكي" في عالم الأرقام.

    تمامًا كما أننا لا نستطيع أن نفسر لماذا كل زوج هو مجموع عددين أوليين (مايكل، لن تزعجني بحل غولدباخ أيضًا، أليس كذلك؟)

    عجائب العجائب، سحر الساحرات، معجزات المعجزات.

    وهذه هي وجهة نظري والمغزى من القصة: حتى لو لم نتمكن بعد من تفسير أشياء كثيرة في التطور، حتى لو بدت أشياء كثيرة وكأنها تخطيط ذكي، فهذا لا يعني أننا لن نكون قادرين على تفسيرها في المستقبل في المستقبل. طريقة مفهومة وبسيطة، كما فعل إقليدس أويلر ومايكل فيما يتعلق بالخصائص الإعجازية للأعداد المثالية.

    فيما يتعلق بزيادة الإنتروبيا مع مرور الوقت، كتبت: "يجب أن تزيد مع مرور الوقت لأسباب رياضية".

    المشكلة هي أن الزمن في جميع معادلات نيوتن قابل للعكس في كلا الاتجاهين - الماضي والمستقبل. فلماذا إذا أخذنا نظامًا ديناميكيًا حراريًا في حالة معينة وطبقنا عليه معادلات نيوتن، ألا تزداد الإنتروبيا أيضًا في اتجاه الماضي؟

    هذه، بالمناسبة، هي المشكلة التي عذب بها كوينتر بوانكاريه بولتزمان المسكين حتى قضى عليه. أثبت بوانكاريه رياضيًا أن الإنتروبيا الكونية يجب أن تنخفض في النهاية، وأن سهم الزمن سينعكس.

    ولم يتم اكتشاف الحل المحتمل للمشكلة إلا في وقت لاحق.

    الأشباح - لا أذكر أنني قلت إن الكون نظام مغلق. قلت إن إسرائيل عبارة عن نظام مغلق من الناحية النفسية والميكانيكية، وبالتالي فإن الأمل في السلام العالمي ساذج.

    وأعتقد أن الرياضيات موجودة بشكل مستقل عن الإنسانية، وأنه إذا تواصلنا مع كائنات فضائية فيمكننا التواصل معهم من خلال الرياضيات بغض النظر عن اللغة. وهو نفسه في كل مكان في الكون.

    في اعتقادي، حتى أولئك الذين يؤمنون بالتصميم الذكي لا يدّعون أن المصمم الذكي هو الذي خلق الرياضيات (وهذا على عكس المتدينين والخلقيين الذين يعتقدون أن الله خلق كل شيء). لذلك من المهم إظهار أنه وفقًا لمنطق التصميم الذكي، من الممكن إظهار وجود تصميم ذكي في الرياضيات أيضًا (غولدباخ على سبيل المثال)، مما يسقط أساس الحجة بأكملها.

    شموليك - سعيد لأن القصة أعجبتك. قرأت "السؤال الأخير" عندما كنت في السادسة عشرة من عمري، وأحببته مثل معظم قصص عظيموف.

    تجدون الفصل الأول من "قانون القانون الثاني للديناميكا الحرارية" على الرابط التالي:

    http://www.amalnet.k12.il/machine/show_item.asp?item_id=14600&id=1425&level=1&num=3

    ليلة سعيدة وسنة جديدة سعيدة من لوس أنجلوس.

  485. شموليك:
    بمجرد أن يتم إنشاء شيء ما في عملية تطورية - فهذا دليل على تفكك تعقيده (لأن العملية التي خلقته هي تفكك التعقيد).
    ولذلك فإن إثبات وجود تعقيد لا ينفصم في الطبيعة سيكون دليلاً على التصميم الذكي.

  486. وفيما يتعلق بالإنتروبيا: فإنها يجب أن تزيد مع مرور الوقت لأسباب رياضية.
    القانون الثاني للديناميكا الحرارية هو قانون رياضي احتمالي.
    مصطلح "النظام" هو مصطلح أكثر تعقيدًا مما يميل الناس إلى التفكير فيه، ومن حيث "النظام" المستخدم في تعريف الإنتروبيا، فإن درجة اضطراب البنية الكروية أعلى من درجة اضطراب توزيع آخر لنفسها كتلة.
    السؤال المثير للاهتمام ليس هو سبب زيادة الإنتروبيا، بل هو ضرورة لأسباب رياضية.
    السؤال الوحيد المثير للاهتمام هو لماذا كانت الإنتروبيا منخفضة في المقام الأول.
    وهذا أيضًا سؤال مهم لأنه لولا هذا الترتيب الأولي لما حدث شيء مهم في الكون، ولما كانت الحياة قد خلقت فيه بالتأكيد.
    هناك العديد من المحاولات للإجابة على هذا السؤال، ومن أكثرها إثارة للاهتمام وجديدة ما ورد في كتاب روجر بنروز: دورات الزمن

  487. لا أريد أن أتورط في حجة سخيفة أخرى، ولكن بالطبع الرياضيات صحيحة بغض النظر عن الإنسان والطبيعة أطاعت قوانينها حتى قبل وجود البشر وستستمر في طاعتها حتى بعد انقراضها.
    ليس من قبيل الصدفة أن علماء الرياضيات في جميع أنحاء العالم يتوصلون في نهاية المطاف إلى نفس الاستنتاجات حتى عندما يعملون بشكل مستقل. إنهم لا يخترعون قوانين الرياضيات، بل يكتشفونها.

  488. إسرائيل
    كيف نعرف أن الكون نظام مغلق؟ ربما الكون كله محاط بثقب أسود ضخم وهو عبارة عن طاقة مظلمة وباردة جدًا؟ 🙂
    ربما لم أفهم قصتك، لكن كما أراها: الطبيعة هي ما هي عليه. اخترع الإنسان الرياضيات المنظمة ليفسر بطريقة منظمة ومنطقية الظواهر في الطبيعة. ليست الطبيعة هي التي ترتب نفسها وفقًا للرياضيات، بل الإنسان هو الذي يقرر الترتيب (وما هو) في الطبيعة كما يراه مناسبًا.

  489. إسرائيل،
    قصة رائعة، من أين حصلت عليها وإذا كانت لك، رائعة جدًا رغم أنني لم أفهم لماذا، في المقام الأول، يجب أن تخضع النظرية الرياضية للقانون الثاني للديناميكا الحرارية؟ إذا كنا نتحدث عن قصة عن القانون الثاني، فمن المستحيل عدم ذكر قصة أسيموف الرائعة غدا الساعة التاسعة (الاسم باللغة الإنجليزية أفضل بكثير): "السؤال الأخير":
    http://en.wikipedia.org/wiki/The_Last_Question

    علم الأحياء الرياضي,
    ما هو غير مفهوم؟ أنت تزعم أن السوط غير قابل للكسر، فأثبت ذلك.
    أنت فقط تعتقد أنك دحضت ميلر. وهذا رده: http://www.millerandlevine.com/km/evol/design2/article.html
    حتى لو فشل، فهذا لا يزال يعني أنه كان لطيفًا أن يحاول ويفشل. فشله لا ينقل عبء الإثبات عليه. مرة أخرى (ربما سيتم التقاطه هذه المرة). تأكيد الادعاء بأن شوتون لديه تعقيد لا ينفصم، سوف تثبت.
    والأكثر سخافة هو محاولتك معالجة هذه القضية (بعد الفشل المتكرر) في محاولتك لإثبات وجود الله، في حين أن هناك قضية أفضل: إن مسألة كيف أصبحت الكيمياء علم الأحياء أكثر إثارة للاهتمام وأين لا يزال العلم يفعل ذلك؟ لا تملك إجابة مرضية. قم بذلك (ما هي حجة إله الفجوات الأخرى بالنسبة لك؟)

    فيما يتعلق بالتعقيد الذي لا ينفصم،
    إنني أدرك الطبيعة الإشكالية للتعريف وأن ادعاءات الوجود هي تعقيد لا ينفصم، وهو ادعاء مبني على الجهل، وبسبب الصعوبة الكبيرة في التعريف، يحاول الخلقيون نقل عبء الإثبات إلى خبراء التطور، كما يوضح علم الأحياء الرياضي على مر السنين. ومرة أخرى، بطريقة محيرة، ولكن دعونا نفترض للحظة أن هناك تعقيدًا لا ينفصم (وفقًا لأي تعريف تريده). هل وجود بنية بيولوجية ذات تعقيد لا ينفصم يؤدي بالضرورة إلى مصمم ذكي؟
    يوضح الرابطان التاليان (أحدهما قد أحضرته بالفعل) أن الأمر ليس كذلك، ولكن هذه محاكاة حاسوبية.
    http://skeptico.blogs.com/skeptico/2005/02/irreducible_com.html
    http://www.complexity.org.au/vlab/evolution/irreducible-complex/icTechnicalReport.pdf

    ما رأيك؟

  490. إسرائيل:
    بالطبع عليك أن تحكي ما سبق لشخصيات القصة (التي لم أقرأ تفاصيلها) لأنه في هذه اللحظة - عندما تكتمل صياغة سؤالك - يكون حله قد اكتمل أيضاً.
    لا توجد معجزة هنا والظاهرة مطلوبة حسب قوانين الحساب

  491. أعتقد أن الأمور انتهت على خير.
    إذا لم يكن الأمر واضحًا لشخص ما - دعه يقول ذلك وإذا لزم الأمر، سأقوم مع ذلك بإعداد ملف خاص في بيئة تسمح بتسجيل الصيغ

  492. إسرائيل:
    من الواضح لك أنني أعرف ما هي الأرقام المتتالية، لكنك كتبت "جميع الأرقام المثالية هي سلسلة من الأرقام المتتالية".
    هذه هي صياغتك السيئة حيث أن عبارة "كمية" مفقودة وبالتالي يتم تفسيرها على أنها سلسلة من الأرقام المفصلة وليس كما قصدت.
    فيما يتعلق بالحقيقة التي تتحدث عنها، فهو تشخيص مثير للاهتمام ولكن من السهل جدًا إثباته.
    المشكلة هي أنه من الصعب كتابة التشتيتات هنا، لكنني سأحاول على أي حال.
    إذا لم ينجح الأمر سأقوم بإنشاء ملف خاص.
    مجموع الارقام من 1 الى k هو
    ك*(ك+1)/2
    الآن - مكان ك أنت
    (2 أس ن) ناقص 1
    وسوف تحصل على ما تحتاجه بالضبط.

  493. قال الأستاذ: "دعونا نتفحص ادعاء القانون الثاني بأن الرياضيات تتطلب اضطرابًا متزايدًا".
    لقد رسم خطًا مستقيمًا على السبورة من النهاية إلى النهاية، وحدد نقطة في وسط الخط وكتب فوقها "0".
    "هنا، هذا هو محور الأعداد. يبدأ من سالب اللانهاية عند الطرف الأيسر من الخط الذي رسمته، ويمر عبر الصفر وينتهي عند زائد اللانهاية، عند الطرف الأيمن من الخط. ويمكن مقارنتها إلى حد ما بالخط الزمني في الطبيعة، حيث تمثل اللانهاية المستقبل البعيد للغاية ناقص اللانهاية مثل الماضي. "
    وقد وضع الأستاذ في كلمته علامة ∞- على يسار السطر و∞ في أيامنا هذه.
    "جميع الأعداد، كبيرة كانت أو صغيرة، سواء كانت كاملة أو كسرية، إيجابية أو سلبية، عقلانية أو غير عقلانية - يمكن إدخالها بترتيب تصاعدي بين ناقص اللانهاية واللانهاية. كان من المتوقع أن مثل هذا الترتيب البسيط لن يؤدي إلى أي نمط أنيق بشكل خاص - فماذا فعلنا بعد كل شيء؟ لقد وضعنا 1 في البداية، يليه 2، 3، وهكذا."
    "إذا كان ادعاء القانون الثاني صحيحًا، فسيكون هناك اضطراب متزايد مع تقدمنا ​​من الصفر نحو اللانهاية، وستنخفض جودة العلاقات الرياضية بين الأرقام، كما هو الحال في مجموعة أوراق اللعب. ولكن هنا لدينا مفاجأة سارة. من هذا الطلب البسيط تمامًا، يتم إنشاء نظام أكثر تعقيدًا وإثارة للاهتمام. لقد اتضح أنه لا نهاية للتعقيد الماكر الذي يمكن من خلاله ترتيب هذه الأرقام التي تبدو بسيطة. خذ على سبيل المثال ظاهرة الأعداد المثالية. هذه هي الأرقام التي مجموع الأرقام التي تقسمها يساوي الرقم نفسه تمامًا. فالرقم 6، على سبيل المثال، يقبل القسمة على 1 و2 و3، و6=1+2+3. الأرقام 28 و496 و8128 هي أيضًا أرقام مثالية. وهنا يتبين أن جميع الأعداد الكاملة تخضع للقاعدة التي اكتشفها إقليدس: كل عدد كامل هو مضاعف لعددين، أحدهما قوة 2 والآخر هو قوة 2 ناقص 1. على سبيل المثال:
    6=2¹ س(2²-1)
    28=2² س(2³-1)
    496=24 × (25 -1)
    س(27 -1) 8128=26
    .
    . .
    . . .
    -2216090) =2216090 ×(2216091 -1) 2432181 )

    يتكون هذا الرقم من أكثر من 130,000 رقم! وهو يطيع اقليدس!"
    "وهذا ليس كل شيء. وتبين أيضًا أن جميع الأعداد المثالية هي سلسلة من الأعداد المتتابعة:
    6=1+2+3.
    28=1+2+3+4+5+6+7.
    496=1+2+3+4+5…+30+31.
    8,128=1+2+3+4+5…+126+127.

    وواصل الأستاذ كتابة الأرقام على السبورة بينما صفير الحضور إعجاباً.
    "لذلك نرى أنه على الرغم من أننا بدأنا بأقل قدر من الترتيب، إلا أن سلسلة الأرقام عديدة، والترتيب يتزايد كلما أضفنا المزيد من الأعضاء إلى سلسلة الأعداد المثالية. هذا مجرد مثال واحد من العديد من الأمثلة على النظام والكمال في مجال الرياضيات والتي تسمى نظرية الأعداد. هناك العديد من الأرقام الجميلة - الأرقام الصديقة على سبيل المثال - وهي عبارة عن أزواج من الأرقام يساوي كل رقم بينها مجموع قواسم شريكه. على سبيل المثال، الرقمان 220 و 284، المعروفان باسم روميو وجولييت، من الأرقام الصديقة، ويعتبران رمزاً للصداقة والحب، لأن أجزاء العدد 220 هي: 20,22,44,55،1,2,4,5,10,11،110،284 284،1,2,4,71،142،220 وXNUMX وXNUMX وXNUMX ومجموعها XNUMX أجزاء XNUMX هي XNUMX وXNUMX وXNUMX وXNUMX وXNUMX ومجموعها XNUMX. هل تشعر بعدم الانسجام أو أي اضطراب في عالم الأرقام؟ "خاطب الأستاذ الجمهور.
    وضجت القاعة بالتصفيق والهتافات الإيقاعية: "لايبنوفيتش! لايبنوفيتش!
    سحب الأستاذ صوته ببطء وابتسم بلطف للقانون المتجمد في كرسيه. "هل تريد منا أن نستمر؟ لا يزال لدينا أعداد أولية، وأعداد أولية توأمية، وأعداد محاصرة..."
    "أعطه الهندسة!" القديس المدعي الانتقامي.
    "أوه، نعم، الهندسة." قام الأستاذ برسم مثلث مسدود بدائرة على السبورة. "ملاحظة: عندما تكون زوايا المثلث عشوائية تمامًا، فإن العلاقة بين أضلاع المثلث تكون عشوائية أيضًا. ولكن عندما تصل إحدى الزوايا إلى 90 درجة -" قام الأستاذ بتحريك الطباشير على طول محيط الدائرة، مما أدى إلى زيادة الزاوية المركزية في المثلث بشكل مطرد،" تحدث الظاهرة المعروفة لنا من نظرية فيثاغورس: مجموع مربعي الضلعين المتقابلين يساوي مربع الوتر. 90 درجة هي، من قبيل الصدفة تمامًا بالطبع، نسبة رقمية دقيقة تبلغ 2/1 مجموع الزوايا في المثلث و4/1 مجموع الزوايا في الدائرة. أين الخلل هنا بالضبط؟ وهنا ظاهرة أخرى من مجال الهندسة "لقد رسم كرة على السبورة باستخدام المنظور. "أم ينبغي أن نقول: القياس المجسم؟ هل خطر لك يومًا عزيزي القانون، لماذا تستخدم الطبيعة، التي تدعي أن طبيعتها هي زيادة الإنتروبيا، التناظر الكروي كثيرًا على الرغم من أنه هو التناظر الأكثر كمالًا وانتظامًا؟

    "هل تمانع أن تشرح للمحكمة ما هو التناظر الكروي؟" كان المدعي مصممًا على استخدام الميزة حتى النهاية.

    "التناظر الكروي هو التناظر الوحيد الذي يسمح لنا جميعًا بأن نكون "في القمة". ضحك الأستاذ في نفسه مستمتعًا بكل لحظة. "إن الشخص الذي يقف على قمة جبل على الأرض سوف يرى نفسه أطول من شخص آخر يقف على قمة جبل آخر بنفس الارتفاع، وهذا بسبب الانحناء المنتظم، على الأقل من الناحية النظرية، لسطح الأرض. سوف يرى الشخص الآخر نفسه على أنه أعلى، وهكذا، مع أي عدد مطلوب من الأشخاص.
    "شكل الكرة، الذي يعتبر الشكل الهندسي الأكثر مثالية، هو الشكل الأكثر شيوعا بين الأجرام السماوية الكبيرة، وهو مثال آخر على النظام في الكون المضطرب على ما يبدو."

    "إنه بسبب الجاذبية، التي تجعل الأجسام لها مساحة سطحية أصغر، كما في حالة فقاعات الصابون .." بدأ القانون.

    "يا! جاذبية!" صاح الأستاذ بحماس: "وما الذي يسبب التناظر الشعاعي المثالي للجاذبية؟ كيف يحدث أن قوة الجاذبية تتناسب طرديا مع معكوس مربع المسافة دون باق ودون حد إضافي في معادلة الجاذبية؟

    كان القانون صامتا.

    "ما الذي يجعل الأرقام تخلق مثل هذه الهياكل المثالية من النظام الأكثر مرونة؟
    ما الذي يجعل مربعي الضلعين المتقابلين في المثلث القائم الزاوية يساويان تمامًا مربع الوتر؟ لماذا لا توجد مثلا نسبة 1.147 بينهما؟
    ما الذي يجعل ثوابت الفيزياء تتوافق معًا بطريقة سلسة بحيث أن أي انحراف طفيف عنها كان سيمنع تكوين الكون؟

    ما الذي يسبب تكوين الحياة، وتكوين الحياة الذكية على وجه الخصوص؟"

    "حسنا، التطور .." افتتح القانون.

    "إلى DM، هذا هراء مطلق!" كان الأستاذ يصرخ الآن بأعلى رئتيه. "هل تعرف احتمال التكوين التلقائي لخلية حية واحدة قادرة على تكرار نفسها؟ صفر، صفر مطلق!"

    وطالب الجمهور بتفسير.
    "هنا" نزع الأستاذ ساعته من معصمه وألقاها بغضب على الحامل. "إذا وجدت هذه الساعة ملقاة على الأرض على كوكب بعيد. هل تفترض أن الساعة خلقت بنفسها من خلال التطور؟ قوى الطبيعة من خلال التجمع العشوائي للجزيئات؟ لا بالطبع لأ. كان على شخص ما أن يصنعها."

    أومأ الحشد بالاتفاق.
    "وهل أنت مستعد لتصديق أن الإنسان الحي، المفكر، الشعور، الخالق، المكون من مليارات الخلايا، كل خلية منها تتكون من الساعة مليون مرة، قد خلق تلقائيا دون خالق خارجي؟"

    ساد الصمت التام في القاعة.

    "يمكنني أن أشهد، كعالم رياضيات، أن احتمال وجود مثل هذا الكون المثالي، وكذلك احتمال التكوين التلقائي للحياة، يساوي تمامًا الصفر."

    "هل تلمح إلى ذلك..." كان صوت المدعي العام أجشًا من الإثارة.

    "أنا لا أعني، أنا أقول ذلك صراحة. إن الله تعالى هو المسؤول عن الكمال المذهل في الرياضيات والهندسة والفيزياء والكيمياء، ونتيجة لذلك علم الأحياء. ولأننا كائنات بيولوجية، فإننا ندين بحياتنا لكل شيء”.

    هتف الجمهور لفترة طويلة.

    لقد عاد القانون إلى رشده. "حسنًا، فليكن، لا أنوي الإضرار بحرية معتقدك. حسنًا، هناك إله! هل يمكننا العودة إلى المنزل بالفعل في سبيل الله؟"

    قال المدعي العام بحزم: "لا. السؤال الذي تُحاكم عليه لا يزال مفتوحًا: لماذا تزداد الإنتروبيا بمرور الوقت؟"

  494. إسرائيل شابيرا
    أفهم ما تقصده ولكن ربما الله (أو الله أو براهمان أو زيوس أو مردوخ أو …..) صمم أيضًا الأعداد الطبيعية (كما قال كرونكر….).

    تتميز الساعة بالعديد من خصائص الجسم المصمم. على سبيل المثال - لديه مهمة. كل ما نعرفه كما هو مخطط له له هدف، أليس كذلك؟
    لكن - الحياة ليس لها هدف ندركه. وهذا يضعف الادعاء بأن الحياة مخططة.

    ودعونا ننظر إلى مستوى آخر: كل مكون من مكونات الساعة له غرض فريد (حتى أن ويليام بالي ذكره في مقالته). الأعضاء في الجسم الحي ليس لها غرض واحد. على سبيل المثال - لذيل الفقاريات استخدامات عديدة، لكن ليس له وظيفة محددة.
    مرة أخرى - ضعف حجة المخطط الذكي.

    هذا التفسير يمكن أن يستمر. ولتضعف حجة المخطط الذكي أكثر فأكثر... :*

  495. المعجزات

    لا أعلم أنه لا يوجد مستقلون.

    لكن احتمال أن تحتوي جميع الأرقام المعقدة البالغ عددها 48، بما في ذلك الأرقام التي تتضمن مئات الآلاف من الأرقام، على جميع السمات بشكل عشوائي، هو احتمال منخفض للغاية، هل توافق على ذلك؟

    وفي الوقت الحالي (على حد علمي) ليس لدينا تفسير مناسب لكيفية حدوث ذلك، تمامًا كما ليس لدينا أي تفسير لماذا جميع الأعداد الزوجية التي نعرفها هي مجموع رقمين أوليين، أو لماذا تتم إزالة الأصفار في دالة زيتا من المحور السيني قبل أن يصلوا إليه.

    فلماذا نذهب إلى أبعد من ذلك وندعي أن الساعة يجب أن يصممها مصمم ذكي؟ يكفي أن ننظر إلى عالم الأرقام لاكتشاف انتظامات مذهلة وغير قابلة للتفسير (حتى الآن).

    لم تجرب المعجزات؟

  496. ميخائيل

    الأعداد المتتالية هي الأعداد التي فرقها واحد.

    جميع الأعداد الأولية هي مجموع أعداد متتالية:

    6=1+2+3.
    28=1+2+3+4+5+6+7.
    496=1+2+3+4+5…+30+31.
    8,128=1+2+3+4+5…+126+127.

    ولم أر في براهين أويلر وإقليدس إشارة إلى هذه النقطة التي تبدو مستقلة. أدلتهم تتعلق فقط بالاعتماد بين الخاصيتين الأخريين.

    إذًا كيف يمكن أن يحتوي كل عدد مثالي أيضًا على هذه الخاصية المستقلة؟

    لا نوبات سحرية؟

  497. كيف فزعت؟؟ تقوم بإخراج حجر واحد (أي حجر) فينهار... وبالتالي فهو في الواقع معقد بشكل لا ينفصم.

    وأما التعريف الثاني - وهو أنه لا سبيل للوصول لنفس المنتج على مراحل - فكما أن هناك طريقة لبناء القوس على مراحل، كذلك هناك طريقة لبناء العصا البكتيرية (وبقية اختراعات بيهي) في مراحل.

    فأين يكمن خطر الخسارة؟

  498. المعجزات:
    أفضّل طريقة المرجعية التي وصفتها - ليس لأنني أحب الخلقيين ولكن بسبب الحاجة إلى توخي الحذر وعدم إعطائهم أساسًا لمجادلتك حول قضايا غير ذات صلة.
    في هذه الحالة - يمكن للمبدع أن يوضح لك أن القوس شيء غريب وبما أنه غريب حقًا فلن يكون لديك إجابة وسيخلق الانطباع كما لو أنه فاز بالحجة.

  499. مايكل روتشيلد
    لم أسمع بعد حجة واحدة ضد التطور تأتي من النزاهة. لذا، أنا آسف إذا كنت أبدو غير متسامح قليلاً مع كل أنواع الهراء.
    إن فكرة "المخطط الذكي" برمتها تزعجني، لأنها مجرد خدعة قذرة من الدعاة المسيحيين. لم أسمع بعد حجة جدية واحدة حول هذا الموضوع وبالتأكيد ليست حجة بيهي الملفقة....

  500. ومن الواضح، بقدر ما يهمني، أن التطور صحيح.
    أعتقد فقط أنه عند الدفاع عنها يجب على المرء أن يفعل ذلك بأمانة وأن ينسب الحد الأدنى من الغباء إلى الطرف الآخر.
    إن الادعاء بأن مصطلح "التعقيد الذي لا ينفصم" لا معنى له هو أن ننسب إليه أقصى درجات الغباء. إن الادعاء بعدم وجود مثل هذه الظواهر في الطبيعة، أو الادعاء بوجود مغالطة منطقية من نوع "حجة الجهل" في استخدام هذا المصطلح، هو التعامل معه بحد أدنى من الغباء.

  501. لا أعتقد أن بيهي يتمتع بأي وضع خاص فيما يتعلق بمعنى المصطلح كما يفهمه الجمهور، ولكن، كما ذكرنا، فإن الاقتباس الذي قدمته يوضح أنه كان يعني ما يعنيه الجميع.

  502. المعجزات:
    إنك تتجاهل جزءًا مهمًا جدًا من ردي، وهو اقتباس من تعريف آخر لنفس الكتاب المقدس.
    الاقتباس الإضافي يساعد على توضيح ما كان يقصده أصلا ولا داعي لمواجهة الصياغة غير الواضحة بدقة عندما تكون هناك صياغة واضحة

  503. مايكل روتشيلد
    لا أصر على تعريف بيهي - لكن بيهي هو من حدد هذا المفهوم ولا أفهم لماذا نبحث عن تعريف آخر 🙂
    هنا هو التعريف من هذا الويكي….
    يعرّف أستاذ الكيمياء الحيوية مايكل بيهي، منشئ مصطلح التعقيد غير القابل للاختزال، النظام المعقد غير القابل للاختزال بأنه "نظام يتكون من عدة أجزاء متفاعلة ومتطابقة جيدًا تساهم في الوظيفة الأساسية، حيث تؤدي إزالة أي جزء من الأجزاء إلى تعطل النظام" التوقف عن العمل بشكل فعال"

    أعتقد أنه من الضروري أن نكون دقيقين في المصطلحات، هذا كل شيء.
    ما تتحدث عنه هو "حجة العين" لداروين.

    ولكن - أعتقد أننا جميعا متفقون على أنه بغض النظر عن التعريف الذي نتخذه، فإن صحة التطور ليست محل شك.
    هل أنا على حق؟

  504. المعجزات:

    لذلك دعونا فرز الإعدادات.
    http://en.wikipedia.org/wiki/Irreducible_complexity

    وستجد هناك تعريفا آخر لنفس البيهي:
    المسار التطوري المعقد وغير القابل للاختزال هو المسار الذي يحتوي على خطوة واحدة أو أكثر غير منتقاة (أي، واحدة أو أكثر من الطفرات الضرورية ولكن غير المنتقاة). درجة التعقيد غير القابل للاختزال هي عدد الخطوات غير المحددة في المسار.

    لكن حتى بدون هذا التعريف (وأنا أكرر - بنفس بيهي) فمن الواضح أن بيهي كان يتحدث عن التطور وأنه لم يعرف كيف يعبر عن نفسه بشكل واضح لا يغير شيئا في الأمر.

    هناك مشكلة أخرى في الصياغة التي يستند إليها شطا والتي يبدو أن الناس لا يلاحظونها، وهي استخدامه ذاته لكلمة وظيفة

    يختبئ هنا الافتراض الخاطئ بأن الكل لديه وظيفة أساسية وُجد من أجلها، وهذه ليست الطريقة التي تعمل بها الأشياء في التطور.
    أعتقد أن بعضكم يعرف أن أعضائنا السمعية، والتي تعتبر مهمة جدًا لقدرتنا على السمع، تطورت في المقام الأول من نظام عظم الفك.

    إذا رجعت إلى مثال القوس وأصررت على نفس صيغة بيهي، فاسمح لي أن أسألك ما هي الوظيفة الأساسية للقوس، ولماذا خلق ولماذا تضر طريقة تشكيله التي وصفتها بذلك؟ الوظيفة الأساسية.

  505. وبالتالي فإن حجة بيهي غبية جدًا.

    إن نظرية التطور (الحديثة) لا تدعي أن كل جزء من أجزاء الجسم الحي تطور على مراحل. سوف تقرأ عنها في كتاب الذهب...

  506. استمع للحظة
    أقول مرارًا وتكرارًا - لقد حدد بيهي التعقيد الذي لا ينفصم في كتابه "صندوق داروين الأسود". كان يقصد شيئًا محددًا للغاية، وهو أنه بمجرد تفكيك شيء (أي شيء...) من عضو لا ينفصل، فإنه سيتوقف عن العمل. لذلك - قوس قزح ليس تفريغًا.
    أنت تتحدث عن شيء مختلف تمامًا، عن تطور هذا العضو.

  507. إسرائيل،
    لقد تحولت للتو إلى سؤالك حول الأرقام المعقدة.
    العلاقة بين السمتين الأولين موجودة وقد تم إثباتها، لذلك لا أفهم الادعاء القائل بأنه لا يوجد أي اتصال "على ما يبدو".
    الخاصية الثالثة ليست واضحة بالنسبة لي (الأرقام ليست متتالية وقوى 2 التي تحددها ليست أرقامًا متتالية أو حتى أعدادًا أولية متتالية. الشيء الوحيد الذي يمكن التحدث عنه باسم "أرقام متتالية" هو أرقام مارتن التي هي عامل فيها ولكن بما أنه من المعروف أن أي رقم يحتوي على هذا النوع من النمط فهو مثالي بعد كل شيء، فهذه الميزة بديهية وليس هناك ما يثير الدهشة في ذلك.)
    كل ما سبق صحيح فقط بالنسبة للأعداد الزوجية المثالية (لا يُعرف ما إذا كان هناك أي أرقام فردية على الإطلاق).
    باختصار - في الواقع ليس هناك شك هنا.

    http://he.wikipedia.org/wiki/%D7%9E%D7%A1%D7%A4%D7%A8_%D7%9E%D7%A9%D7%95%D7%9B%D7%9C%D7%9C

  508. للمعجزات
    "إن مثال القوس الحجري هو بالضبط مثال على التعقيد الذي لا ينفصم - أخرج كل حجر وسوف ينهار القوس."
    قطعاً لا، فأنت تتهم الآخرين بعدم الفهم وأنت نفسك لا تفهم:
    مثال القوس الحجري هو مثال للتعقيد -المتكشف- الذي *يبدو* وكأنه تعقيد غير قابل للكسر، لماذا هو تفريغ؟ لأنه يمكن بناؤه تدريجيا. ومن ناحية أخرى، في مثال سكاكين مايكل، الادعاء هو أنه لا توجد وسيلة في العالم لتصنيعها تدريجيا. وحتى لو كانت هناك طريقة وفي الواقع يتعلق الأمر بتفكيك التعقيد، فهذا لا يعني أنه لا يوجد مثل هذا الكائن وأنه لا توجد طريقة في العالم لبنائه تدريجيًا، لاحظ: حقيقة أنك وجدت الأشياء التي كان يُعتقد أنها لا تنفصل وتبين أنها لا تنفصل، لا يعني أنه لا يوجد تعقيد لا ينفصم حقًا في العالم، بحيث حتى لو لم يكن هناك قدرتنا على التحقق من ذلك، فلا يزال من غير الممكن الوصول إليه تدريجيًا.
    هناك فرق بين الادعاء بأننا لا نستطيع/لدينا القدرة على اختبار ما إذا كان التعقيد تحللًا أم لا، والادعاء بأنه بغض النظر عن اختبارنا، يوجد/لا يوجد كائن في العالم الحقيقي له تعقيد غير قابل للاختزال.

  509. المعجزات:
    لقد قلت أنك ترغب في معرفة الفرق، لذا استعد لتكون سعيدًا، ولكن قبل أن أشرح الفرق، أريد أن أكرر وأؤكد على الأشياء التي سبق أن قلتها من قبل حول هذا الموضوع:
    الأول هو أن تعريف شيء ما على أنه ذو تعقيد لا يمكن فصله عادة ما يكون حجة من الجهل، بمعنى أننا ندعي عدم قابليته للفصل فقط لأننا لا نعرف كيفية تحليله.
    ولهذا السبب أوضحت أن الأمثلة التي قدمتها يمكن أن تندرج أيضًا ضمن هذه الفئة، ولكي نعرف حقًا أن هذا تعقيد لا ينفصم، يجب على المرء أن يخطط لعالم خاص يمكن فيه المطالبة بهذا لأنه في عالمنا يمكن إنشاء مثل هذا الهيكل نظريًا حتى بشكل عشوائي (وشرحت الطريقة بشكل أو بآخر) - قلت كل هذا في ردي:
    https://www.hayadan.org.il/ball-state-prez-intelligent-design-not-science-0408135/comment-page-9/#comment-445517
    الآن عن القوس:
    عادة ما يشير الناس فقط إلى المكونات الرملية والحجرية التي طلبتها في الطبيعة. إنهم لا يتناولون عنصرًا مهمًا جدًا يجعله قوسًا.
    هذا المكون هو "الثقب" الذي يسمح لك بالمرور من تحته.
    هذا هو بالضبط المكون الذي يمكن "إزالته" للحصول على التصريفات.
    يبدأ القوس كمجرد مجموعة من طبقات الصخور والتربة التي تلتصق ببعضها البعض بواسطة القوى الكيميائية والميكانيكية.
    في هذه المرحلة لا يبدو كالقوس، لكن في مرحلة ما لاحقاً - عملية طبيعية تتسبب في انحراف أو سقوط جزء من الطبقات السفلية، ومن ثم ينفتح الثقب ويسمى ما تبقى "قوساً" رغم أنه كان موجوداً قبل.
    هناك أشكال أخرى من التشكيل تتضمن مراحل أكثر مثل تكوين الكهف عن طريق انجراف أو سقوط المواد وعندما يسقط جزء من سقف الكهف فيما بعد، يتكون قوس - ولكن المبدأ كله واحد.
    لم يتم إنشاء الهيكل الذي وصفته بهذه الطريقة، ومن الصعب أن نتخيل أنه سيتم إنشاؤه بهذه الطريقة.
    انتبه إلى عبارة "يصعب وصفها".
    هذه هي بالضبط نفس حجة الجهل وفي الرد الذي أشرت إليه أعلاه، أوضحت أيضًا كيف يمكن أن يحدث نفس الشيء "الذي يصعب وصفه".
    ولهذا السبب -أكرر- شرحت أنه لكي ندعي حقًا أن هذا هيكل غير قابل للتدمير، نحتاج إلى التخطيط لعالم خاص حيث يمكن إثبات ذلك عمليًا وليس مجرد الادعاء عن الجهل.
    ومع ذلك، حتى في عالمنا هناك أسباب وجيهة للاعتقاد بأن الهيكل غير قابل للتدمير وهذا بسبب المواد المصنوعة منها، وطريقة معالجتها، ولاحقًا مزيجها المتناقض إلى حد ما حيث يكون السكين A تحت السكين B، والسكين B تحت السكين C لكن السكين C تحت السكين A.
    لذلك، ليس من المستغرب أن الأقواس موجودة في الطبيعة، ولكن مثل هذا الهيكل من السكاكين ليس كذلك.
    على ما يبدو، ليس نحن فقط، ولكن الطبيعة أيضًا، لم تجد طريقة لإنتاج مثل هذا الهيكل والسبب في عدم عثور الطبيعة عليه هو أنه لم يتم تفريغه.

  510. علم الأحياء الرياضي
    مثالك جيد ولكن تم دحضه لأن نفس التسلسل (على سبيل المثال) 35 بروتينًا ربما تم استخدامه لغرض آخر.
    هذا هو:
    1) هذا النظام ليس تفريغًا بالفعل
    2) هذا النظام لا يستبعد التطور لأنه من الممكن بالفعل أن يكون قد تم إنشاؤه عن طريق التطور. وتبين أننا نعرف عدة احتمالات لتطوير هذا التسلسل.

    سأطرح عليك سؤالاً (عادة لن تتمكن من الإجابة عليه): أنت تقول إن الرجل (وهو مصمم ذكي) فشل في إنشاء سيارة ذاتية التكرار. لقد تمكنت الطبيعة من خلق شيء يكرر نفسه. Argo - كائن غير عقلاني تمكن من خلق مخلوق مكرر...
    السؤال هو - ما الخطأ في ما قلته؟

  511. مايكل روتشيلد
    مثال القوس الحجري هو بالضبط مثال على التعقيد الذي لا ينفصم - أخرج كل حجر وسوف ينهار القوس. وهذا هو معنى العبارة كما عرفها بيهي، وكما استخدمها علم الأحياء الرياضي.
    لقد صنعت قوسًا من السكاكين - أود أن أفهم الفرق الأساسي في الأمثلة التي قدمناها - باستثناء أن قوسي موجود في الطبيعة وبالتالي يدحض حجة بيهي تمامًا.

  512. مايكل والمنتدى.

    سؤال خطير حول الأعداد المركبة.

    كل واحد منهم لديه 3 ميزات تبدو غير ذات صلة:

    1. هذه هي الأرقام التي مجموع الأرقام التي تقسمها يساوي الرقم نفسه تمامًا.

    2. كل عدد مثالي هو مضاعف لعددين، أحدهما يمثل قوة للعدد 2 والآخر هو للقوة التالية للعدد 2 ناقص 1.

    3. جميع الأعداد المثالية هي سلسلة من الأعداد المتتابعة.

    1. هل يمكنك توضيح لماذا إذا كان الرقم يحتوي على خاصيتين من الخصائص الثلاثة التي تمت مناقشتها، تكون الخاصية الثالثة مطلوبة أيضًا؟

    2. هل يمكنك العثور على أي رقم يحتوي على خاصيتين فقط من أصل 2 خصائص؟

    3. هل يمكنك العثور على صيغة من شأنها أن تسفر عن أرقام معقدة؟

    هذا ليس مشابهًا للمثلثات، حيث أنه دائمًا عندما يكون هناك ساقان متساويتان، يكون هناك أيضًا زاويتان متساويتان للقاعدة والعكس صحيح أيضًا، ولا يمكن أن يكون للمثلث خاصية واحدة فقط من هذه الخصائص. وهذا أمر بديهي ويمكن إثباته أيضًا إذا كنت أتذكر بشكل صحيح. في حالة الأعداد المعقدة، أخذنا أبسط سلسلة على الإطلاق، وهي سلسلة الأعداد، ومن خلال مجموعة متتالية معينة من الأعداد، حصلنا بشكل غير متوقع على خاصيتين إضافيتين من أرقامها لا ترتبطان ببعضهما البعض.

    كيف يحدث هذا؟

    الأشباح - في الكيبوتس الخاص بنا، كان المثلث هو إله المجتمع بأكمله، كبيره وصغيره.

  513. وبالمناسبة، بالنسبة لأولئك الذين يعتقدون أن التعقيد الذي لا ينفصم لم يتم إثباته في تجربة معملية:

    http://www.evolutionnews.org/2006/04/do_car_engines_run_on_lugnuts_1002157.html

    "طفرة واحدة، وجزء واحد خرج، ولا يستطيع السباحة. أعد هذا الجين مرة أخرى لنستعيد القدرة على الحركة. نفس الشيء هنا. نضع، نتخلص من جزء واحد، ونعيد نسخة جيدة من الجين مرة أخرى، ويمكنهم السباحة. بحكم التعريف فإن النظام معقد بشكل غير قابل للاختزال. لقد فعلنا ذلك مع جميع مكونات السوط الـ 35، وحصلنا على نفس التأثير."

  514. علم الأحياء المحفور:

    "لقد ضربت مثلا لم يدحضه أحد، وفي رأيي لن يدحضه أبدا. مركبه."

    سيارة - اخترعها الإنسان.

    لقد اخترعها الإنسان خطوة بخطوة.

    مما يدحض ادعائك.

  515. مجرد ملاحظة أخيرة حول الهياكل غير القابلة للتحلل. حتى داروين نفسه فهم أنهم إذا أثبتوا وجود أنظمة غير قابلة للفصل في الطبيعة، فسيتم دحض نظريته (وبالطبع يضيف أنه لم يجد أي منها، لكن ذلك كان في ذلك الوقت، عندما لم يكونوا يعرفون حتى ما هو الجين). كان). والسبب في ذلك بالطبع هو الاحتمال المنخفض جدًا لظهور مثل هذا الهيكل مرة واحدة. لقد رأيت تقييمات من خبراء التطور الذين يزعمون أنه حتى 100 هكتار تعتبر قفزة كبيرة جدًا (وفي رأيي أقل من ذلك، وليس لدي مشكلة في التوسع في هذا لعدة صفحات). ويتكون البروتين الواحد من 300 هكتار في المتوسط. ناهيك عن النظام الذي يتطلب عددا من البروتينات. المشكلة هنا كلها تتعلق بالتطور. لأنه من دون نظرية تشرح بشكل لا لبس فيه كيف يمكن للروبوت العضوي أن يتطور، فليس هناك أي معنى للادعاء بأن الطبيعة تطورت من خلال عملية طبيعية. وليس من قبيل الصدفة أن يضيف دوكينز أنه لا يستطيع أن يتخيل كيف كان من الممكن أن تكون ملحدًا قبل العام الذي نشر فيه داروين كتاب أصل الأنواع. وليس عبثًا أن يضيف أنه إذا لم يكن بين قرائه من يعتقد أن مثل هذه الدرجة من التعقيد تعني الوعظ، فإنه يرفع يديه.

    حتى عالم RNA تم اختراعه لحل مشكلة الاحتمالية (الدجاجة والبيضة). لذا نعم، هذه مشكلة خطيرة. وإذا لم يكن لدى أنصار التطور مشكلة مع الاحتمالية، فليس لديهم مشكلة في قبول التكوين المتزامن للكائن الحي.

    أنت قلت :

    "في المحاكمة والنقاش العام بين ميلر والباهي، قدم ميلر بدائل بما في ذلك بديل للادعاء بأن مصيدة الفئران غير قابلة للكسر (استخدم مصيدة فئران محايدة كحامل ربطة عنق). يدعي علم الأحياء الرياضي أنه دحض أمثلة ميلر ولكن هذا لا يهم. ويقع عبء إثبات أن السوط غير قابل للكسر على عاتق المدعي "-

    فلماذا لا يثبت ميلر أن السوط غريب الأطوار؟ لماذا دحضت كلامه بهذه السهولة؟ هل تفهم الآن أنه وفقًا لمعاييرك، يجب على ميلر تقديم الأدلة، أم تعتقد أن مؤيدي التخطيط هم فقط من يجب عليهم القيام بذلك؟

    لقد ضربت مثلا لم يدحضه أحد، وفي رأيي لن يدحضه أبدا. مركبه. وكما أن السيارة لها أنظمة لا يمكن فصلها، فإننا نجد مثل هذه الأنظمة في الطبيعة. كل بروتين هو في الواقع نظام لا ينفصل. لديها حد أدنى معين لا يمكن أن تعمل دونه. آلية تخثر الدم هي مثل هذه، المكوك، توربين atp، نظام ttss، الهيموجلوبين أو الميوجلوبين، تخليق جزيء "الهيم"، السيتوكروم c و b، هيستون h4 و h3 وآلاف من البروتينات وبراءات الاختراع المختلفة الأخرى.

    والشيء المضحك هو أن التعقيد الذي لا ينفصم (من الواضح أنهم لا يستخدمون المصطلح أعلاه) بدأ يظهر في الأوراق العلمية لعلماء الأحياء التطورية. ربما ليس لديهم خيار.

  516. المعجزات:
    لقد ذكرت اسمك لسبب وجيه جدًا وليس عبثًا.
    لقد قدمت أيضًا أمثلة على التعقيد غير القابل للتحلل، وحقيقة أنك وجدت أن شخصًا ما أعطى أمثلة خاطئة للتعقيد غير القابل للتحلل لا تظهر شيئًا ونصف فيما يتعلق بالأمثلة التي قدمتها.
    أنت تخلط هنا بين الادعاء بأن كل شيء له تعقيد لا ينفصم ويكفي له مثال مضاد واحد، والادعاء بأن هناك أشياء لا يمكن فصل تعقيدها ولا يمكن لأي مثال مضاد أن يناقضها.
    تم اكتشاف مدى تعقيد الأقواس بالكامل ونعرف بالضبط كيف يتم تشكيلها. إن مجرد إرباك شخص ما واعتقاده أن تعقيده لا ينفصم، لا يجعله يجرم مسألة التعقيد الذي لا ينفصم برمتها.

    ومن المثير للاهتمام أن نرى كيف يكرر الجميع ما قلته بالفعل بشأن (عدم) صحة حجة التعقيد الذي لا ينفصم، بل ويثبتون لأنفسهم حقيقة أنهم مرتبطون (عن جهل - ولذلك يطلق عليها حجة من الجهل ) تعقيد لا ينفصم للهياكل المعقدة التي يتم تفصيل تعقيدها.
    إن الادعاء بوجود أنظمة بيولوجية لم تتطور على مراحل هو أيضًا حجة من الجهل.
    على الأكثر - من يدعي أنهم لم يتطوروا على مراحل، لا يعرفون ما هي المراحل التي مروا بها في تطورهم.

    وكذلك الأمثلة في المقال الذي أحضره شموليك ليست أمثلة على التعقيد الذي لا ينفصم.
    قد لا يتحلل التعقيد كما هو متوقع، ولكن إذا اتبعت خطوة بخطوة العملية التطورية للقدرة على الاتصال - فلا شك أنه سيتم العثور على التحلل المناسب للتعقيد.

  517. المعجزات
    انت كتبت
    "لقد كنت ذات مرة في مكان توجد به "قاعة" مصنوعة من الحجارة الكبيرة تحيط بمجلد يمكنك المشي فيه. تشكل الحجارة قبة فوق هذه المساحة مثل القوس - حرك حجرًا آخر (يزن طنًا ...) وسيقع عليك كل شيء.
    كيف حدث ذلك؟ كان هناك انهيار أرضي كبير من الهاوية القريبة. سقطت حجارة كبيرة على تلة صغيرة من الأرض الناعمة. وبعد ذلك، لسنوات، جرفت مياه الأمطار التربة الناعمة وهكذا تم إنشاء القاعة".

    المثال الذي قدمته هو مثال لبنية بشرية هي بنية غريبة: القاعة التي يتم إنشاؤها أحيانًا لغرض معين من قبل الشخص، يمكن إنشاؤها من خلال العمليات الطبيعية.
    أبدأ من افتراض يبدو غير معقول، وهو أن هناك هياكل خلقها الإنسان ولا يمكن عملياً خلقها تدريجياً. ومن المنطقي السبب: فالإنسان لا يخضع لهذا القيد، مقارنة بالطبيعة التي "تخضع" له. في الواقع، يبدو أنه من غير الممكن إثبات بشكل إيجابي أن بنية معينة في الطبيعة هي ذات تعقيد لا ينفصم، لأنه كما تكتب، وكما أثبت شوتون هاين وآخرون، فإن الهياكل التي تبدو أحيانًا غير غريبة، تتحول إلى كن غريب الأطوار.

  518. فيما يتعلق بخطأ "غياب الدليل هو بالضرورة دليل على الغياب"، فمن الواضح أنه نظرا لأن الشبيحي لم يثبت بشكل إيجابي أن هناك تعقيدا لا ينفصم في الطبيعة (كيف يمكن القيام بذلك؟ ليس لدي أي فكرة) فإن هذا خطأ حجة من الجهل، لا يزال، نظرا للتعقيد الذي لا ينفصم، حتى لو أثبت ذلك، في ظاهر الأمر هذا هو التخطيط.
    لقد كتبت أن كلامي يذكرك بالإجابة: إنه أصم لأنه لا يسمع. وسأكون ممتنا لو أمكنك شرح القياس.
    علاوة على ذلك، إذا وافقت على أن كائنًا معينًا معقد بشكل لا ينفصم، فلماذا تعتقد أنك لن *تحتاج* إلى استنتاج أن شخصًا ما صممه؟ هل يمكنك أن تشرح بكلماتك الخاصة؟

  519. "التعقيد الذي لا ينفصم" هو ديماغوجية لا ثاني لها.
    وما دام لا يوجد تعقيد لا يمكن تحليله - في الواقع المادي المعروف (وأنا لا أتحدث عن الأرقام والرياضيات الافتراضية) - فلا يوجد تعقيد لا يمكن تحليله.

  520. شموليك
    لقد كتبت "كل ما لا يتعارض مع الفيزياء الأساسية في هذا الكون، كل شيء ممكن" - حتى أن ماري جيلمان توسعت وقالت (في سياق الكون اللانهائي) "كل ما هو ممكن ضروري"

  521. إريك
    لا لا لا! البناء غير القابل للتدمير ليس دليلاً على التخطيط!! مثال واحد يكفي لمناقضة هذا الهراء - وقد قدمت مثل هذا المثال.
    سأعطيك مثالا وأرجو قضاء 5 دقائق في التفكير وفهم الهراء الذي قلته....
    لقد كنت ذات مرة في مكان توجد به "قاعة" مصنوعة من الحجارة الكبيرة تحتوي على مجلد يمكنك المشي من خلاله. تشكل الحجارة قبة فوق هذه المساحة مثل القوس - حرك حجرًا آخر (يزن طنًا ...) وسيقع عليك كل شيء.
    كيف حدث ذلك؟ كان هناك انهيار أرضي كبير من الهاوية القريبة. سقطت حجارة كبيرة على تلة صغيرة من الأرض الناعمة. ثم، على مر السنين، جرفت مياه الأمطار التربة الناعمة، وبالتالي تم إنشاء القاعة.
    فقط لا؟

  522. إذا سألت قطة ما هو الله، فسوف تجيب: قطة كبيرة.
    إذا سألت مثلثًا ما هو الله، فسيجيب: مثلث كبير.
    لقد أصبح الدين - في نظر الحداثيين وما بعد الحداثيين - منتجا استهلاكيا لوقت الفراغ. وتجذرت كثقافة بين الطبقة الوسطى.

  523. إريك،
    يجب أن أعترف أن بداية كلامك تذكرني بالجواب: إنه أصم لأنه لا يسمع. والسؤال هو: هل هناك حقا مثل هذا اليقين؟ سوف أنتقل للحظة إلى الأكوان المتعددة (نظرية التضخم التي تحدث ببطء، تشير الفيزياء إلى مثل هذا الاحتمال) وأجادل أنه في ظل افتراض وجود عدد لا نهائي من الأكوان، فإن كل شيء لا يتعارض الفيزياء الأساسية في هذا الكون كلها ممكنة. في عالمنا مثلا كسر قوانين الحفظ سيكون دليلا جيدا على مخطط ذكي ولهذا كتبت لو أريتني نظام وجوده يتناقض مع قوانين الحفظ أقول لك والله هناك أشياء فيه بس خلاص بطلت الانجراف ورجعت للواقع.

    عندما أقول أننا في حجة "التعقيد غير القابل للاختزال" نعتمد على اللغة، أعني أننا في بعض الأحيان ننجرف في النظر تحت المصباح. يرى الخلقيون الشوتون ويفهمون أنه يشبه المحرك لأنه من السهل علينا تحديد الوظيفة في كلمة مألوفة لنا من الحياة اليومية، وبالتالي، هذا هو الادعاء، التطور الكامل للشوتون هو يتم ذلك مع تحديد أن المكون يريد دائمًا أن يكون محركًا. لكن من الذي قرر أن نفس الشوتون، ناهيك عن الجزيء، ليس له وظيفة، وهي وظيفة لا نعرف كيف نحددها أو نسميها؟ في هذه المرحلة يرتكب الخلقيون خطأً مفاده أن "غياب الدليل هو بالضرورة دليل على الغياب". في محاكمة شفهية ومناقشة عامة بين ميلر والباهي، قدم ميلر بدائل بما في ذلك بديل للادعاء بأن مصيدة الفئران غير قابلة للكسر (استخدم مصيدة فئران محايدة كحامل ربطة عنق). يدعي علم الأحياء الرياضي أنه دحض أمثلة ميلر ولكن هذا لا يهم. يقع عبء إثبات أن السوط غير قابل للكسر على عاتق المدعي وليس على ميلر الذي استجاب للتحدي. هذا لحسن التدبير.

    الآن، أذهب إلى الشمال ومن أجل الجدال، أوافق على أن هناك تعقيدًا لا ينفصم وأتساءل، إذا كان هناك تعقيد لا ينفصم، فهل الحل الوحيد هو بالضرورة مخطط ذكي؟ من الواضح أن هذا يمكن أن يشير إلى مخطط ذكي، ولهذا السبب اخترعت البشرية آلهة البحر منذ آلاف السنين. إنهم لا يعرفون شيئًا سوى أنهم هم أنفسهم يبنون الأشياء، وبالتالي فإن كل شيء من حولهم قد بناه الله. أستطيع دائمًا أن أقول إن الله خلق كل شيء وكل شيء يمكن أن يشير إليه، إنه الأسهل ولكن منذ ذلك الحين تعلمنا بعض الأشياء والرابط الثالث الذي قدمته يوضح أن الهياكل غير القابلة للكسر يمكن إنشاؤها عن طريق الانتقاء الطبيعي على الرغم من أنني أعترف بذلك. لا أعرف إلى أي مدى يمكنك الاعتماد على المحاكاة الموصوفة هناك لاتخاذ القرار، لكن على أي حال، الموضوع مثير للاهتمام ولا أوافق على أن التعقيد الذي لا ينفصم يشير بالضرورة إلى مصمم ذكي.

  524. فقط لتوضيح شموليك: بما أن هناك أشياء في عالمنا خلقها الإنسان لغرض محدد والتي بالتأكيد لا يمكن أن تتطور بشكل طبيعي، فيمكن تعريفها بأنها "غير قابلة للكسر" فهذه مسألة تعريف. لذلك من الناحية النظرية، إذا كان هناك هيكل في الطبيعة أنشأه مصمم ذكي مثلنا على الأقل، فمن الممكن أن يكون له هيكل "غير قابل للتحلل" وذلك ببساطة لأن قابلية التحلل هي قيد لا ينطبق على المصمم. إذا كان هناك مثل هذا المخطط، ففي الواقع هناك سبب لوجود مثل هذه الهياكل غير القابلة للتحلل، ولنفترض أن هناك علامة تشير إلى الهيكل غير القابل للتحلل، فهي تشير أيضًا إلى المخطط.

  525. شموليك
    ما الذي "يعتمد على اللغة البشرية"؟ من الناحية النظرية، إذا كان من الممكن إثبات أن بنية معينة في الطبيعة غير قابلة للتدمير، فهذا دليل على التصميم. من الصعب أن نفهم كيف يمكن إثبات مثل هذا الشيء، لكنه صحيح من الناحية النظرية.

  526. مقدمًا، يعد موضوع "التعقيد الذي لا ينفصم" إشكاليًا للغاية لأنه يعتمد إلى حد كبير على اللغة البشرية ويفترض المغالطة المنطقية القائلة بأن نقص الأدلة هو دليل على النقص. إنه يعتمد على اللغة لأنه يبحث عن هدف للفعل، ولكن فقط للأفعال التي نعرف كيفية تعريفها بالكلمات (التخثر، المحرك، إلخ) ولكن من قال أنه لا توجد طريقة للنشاط، وأننا لا نعرف كيفية القيام بذلك؟ حدد موقع النشاط أو حدده كميًا وأعطيه اسمًا، وفي غطرستنا الكبيرة (على الأقل غرور الخلقيين) قررنا ما هو ليس نشاطًا. أعتقد أنه إذا رأيت نظامًا يتعارض تطوره مع قوانين الحفظ (على سبيل المثال)، فسوف أبدأ في الشك في احتمال وجود مخطط ذكي كعامل خارج الطبيعة.

    من أجل المضي قدمًا في المناقشة قليلاً، بحثت عن مقالات تطرح السؤال التالي: إذا كان هناك تعقيد غير قابل للتحليل، فهل الإجابة الوحيدة بالضرورة هي مخطط ذكي (بطبيعة الحال، من الصعب جدًا التحدث عن التعامل معه لأننا لسنا متأكدين من ذلك) هناك أنظمة غير قابلة للتحلل) وإليك بعض الروابط:
    http://en.wikipedia.org/wiki/Irreducible_complexity#False_dilemma
    http://schneider.ncifcrf.gov/paper/ev/behe/
    ورابط للمقال الأكثر إثارة للاهتمام الذي رأيته، على الرغم من أنه يتعلق بالمحاكاة الحاسوبية وأنا دائمًا متشكك في المحاكاة التي تحاول نمذجة الطبيعة، بمعنى أنه ليس من الواضح إلى أي مدى يمكن استقراء مثل هذه المحاكاة في الحياة الواقعية . مشبوهة ولكنها ليست مستبعدة بالطبع (المحاكاة أداة ضرورية وفعالة للغاية في تطوير التقنيات):
    http://skeptico.blogs.com/skeptico/2005/02/irreducible_com.html

  527. مايكل روتشيلد
    لماذا تذكر اسمي الكاذب؟
    "التعقيد الذي لا ينفصم" هو مفهوم حدده مايكل بيهي، في أعقاب فكرة طرحها داروين في كتابه أصل الأنواع. نية بيهي هي نظام يتوقف عن العمل بمجرد فقدان جزء واحد، لذلك لا يمكن بناؤه على مراحل. لاحظ أن تعريف بيهي لا يتحدث عن طريقة البناء، بل عن الوضع الحالي. إذا كنت لا تصدق إقرأ الكتاب..

    وبالتأكيد لا علاقة لها بنظرية الأعداد. لا يوجد رقم زوجي رقمه الأخير ليس 0 أو 2 أو 4 أو 6 أو 8... هل هذا أيضًا مركب غير قابل للتحليل؟ 🙂

    يجدر توضيح 3 نقاط بسيطة:
    1) لا نعرف حاليًا أي نظام بيولوجي يلقي ظلالاً من الشك على التطور. وهذا يشمل العصا البكتيرية وآلية تخثر الدم ونظام الهيموجلوبين وغيرها الكثير من الاختراعات والهراء لبيهي وديمبسكي وغيرهم من المشعوذين.
    2) هناك بالفعل هياكل في الجيولوجيا لها تعقيد لا ينفصم - على سبيل المثال الأقواس. أو كومة من الأغصان في حديقتي تقف مثل خيمة تيبي... تحرك واحدة فتسقط جميعها 🙂
    3) يجب ألا ننسى "متلازمة الفيلسوف" لدينيت - لا تخلط بين الافتقار إلى الخيال والاعتراف بالضرورة. سأشرح، بالنسبة لعلم الأحياء الرياضي....لقد وجدت آلية بيولوجية لم تنجح فيها، كيف تم إنشاؤها عن طريق الانتقاء الطبيعي، وبالتالي هناك بالضرورة مخطط ذكي.

    أقترح إعادة القراءة 1) .....

    مايكل - مثال واحد يكفي لدحض نظرية، أليس كذلك؟ أعطى كيرنز سميث المثال الشهير للأقواس في الطبيعة، قبل 10 سنوات من كتاب بيهي.

    ونقطة أخيرة - من المعروف أن الأنظمة البيولوجية لم تتطور على مراحل. وليس هناك مشكلة هنا بالنسبة لنظرية التطور.
    وهو المطلوب

  528. ميخائيل

    هل تشك في موثوقية اللاعبين رفيعي المستوى مثل بلوندي وتوكو؟

    بالتأكيد هناك تعقيد لا ينفصم في عالم الأرقام! ومن المؤكد والأكيد أن المخطط الذكي هو سبب التعقيد! ألم تقرأ رمز الكتاب المقدس؟ ألم ترى كيف كانوا دقيقين في كل شيء؟

    (حسنًا، هذا صحيح، كانت هناك بعض الأخطاء غير المهمة في جزء من التوقعات للمستقبل. لم يتم اغتيال بيبي في عام 2000 ولم تندلع حرب ذرية في العقد الأول من القرن الحادي والعشرين. لكن يجب أن نعترف بذلك في كل شيء آخر ضربوا عين الثور.

    كما يقول توكو:

    عندما تضطر للإطلاق، اطلق. لا تتحدث

  529. المعجزات
    بدلاً من تقدير جهود بيهي في صياغة تعريف واضح لهيكل يعكس التخطيط، أنتم تقللون من شأنه؟ بالمقارنة مع "علم الأحياء الرياضي" الذي يستخدم حدسًا لا أساس له من الصحة من أجل بيع منتجاته، فهو على الأقل يحاول استخدام الحجج العقلانية، وبما أنه يستخدم الحجج العقلانية فإنه يخدم التطور بشكل غير مباشر لأنه من الممكن فهم كيفية عمله بشكل أفضل.

  530. إسرائيل:
    إن حقيقة وجود نظريات في الرياضيات، وأنه كلما توافرت شروط النظرية، تحققت نتيجتها أيضًا، ليس ما يعرف بـ "التعقيد غير القابل للاختزال" في الطبيعة.
    يمكنك أيضًا القول بأن هناك تعقيدًا لا ينفصم في الهندسة الإقليدية، على سبيل المثال - دائمًا عندما يكون هناك منصفان متساويان، يكون هناك أيضًا زاويتان متساويتان للقاعدة والعكس صحيح أيضًا، ومن غير الممكن أن يكون للمثلث زاوية واحدة فقط هذه الخصائص.
    أيضا التورية المعجزات ليست إشارة جدية إلى هذا الموضوع.
    ومن الأمثلة على التعقيد الذي لا ينفصم والذي يتطلب التخطيط، على سبيل المثال، هيكل 4 زجاجات تشكل رؤوس مربع وأربعة سكاكين طول كل منها أقل من جانب المربع، موضوعة بنهاية كل منها منها على رأس إحدى الزجاجات والطرف الآخر مدمج مع الآخرين في هيكل من القماش يمنع الأربعة من السقوط ويجبرهم على تشكيل جسر بين رؤوس الزجاجات.
    أتمنى أن يكون الوصف مفهوما لأنني لا أملك القوة لرسمه.
    من الممكن وصف موقف سقط فيه عدد كبير من السكاكين على شكل كومة بين الزجاجات وفي مرحلة ما تشكل مثل هذا الهيكل في الأعلى ثم تم غسل السكاكين الموجودة بالأسفل بواسطة تيار من الماء يمر عبر المكان ولكن من الممكن التخطيط لعالم خيالي (وحتى عالم تتساقط فيه السكاكين باستمرار من السماء) حيث لا يكون مثل هذا الشيء ممكنًا، وسيكون المخطط الذكي لا غنى عنه.

  531. التعقيد الذي لا ينفصم في الرياضيات.

    خذ على سبيل المثال ظاهرة الأعداد المثالية (المعقدة). هذه هي الأرقام التي مجموع الأرقام التي تقسمها يساوي الرقم نفسه تمامًا. فالرقم 6، على سبيل المثال، يقبل القسمة على 1 و2 و3، و6=1+2+3. الأرقام 28 و496 و8128 هي أيضًا أرقام مثالية. وهنا يتبين أن جميع الأعداد الكاملة تخضع للقاعدة التي اكتشفها إقليدس: كل عدد كامل هو مضاعف لعددين، أحدهما قوة 2 والآخر هو قوة 2 ناقص 1. على سبيل المثال:
    6=1^2( 1- 2^2)
    .
    . .
    . . .
    ( 1- 216091^2 ) 216090^2 = (216090^2 – 2432181^2)

    يتكون هذا الرقم من أكثر من 130,000 رقم! وهو يطيع إقليدس!
    "وهذا ليس كل شيء. وتبين أيضًا أن جميع الأعداد المثالية هي سلسلة من الأعداد المتتابعة:
    6=1+2+3.
    28=1+2+3+4+5+6+7.
    496=1+2+3+4+5…+30+31.
    8,128=1+2+3+4+5…+126+127.

    ومن المستحيل تفكيك خصائص هذه الأعداد - فمثلا العدد المعقد سيحتوي فقط على بعض الخصائص التي تمت مناقشتها وليس كلها، مما يدل على نحو ألف شاهد على تعقيد لا ينفصم وبالتالي على مخطط ذكي في العالم من الأرقام.

    وهذا هو بالضبط سبب اختيار المخطط الذكي للرقم المثالي الأول - 6 - باعتباره عدد الأيام التي تم فيها خلق العالم المثالي.

    وإذا كان هذا الدليل الواضح والحاسم غير كاف للمشككين، فإن كلينت إيستوود في الخير والشر والقبيح يقول أيضًا "ستة - رقم مثالي" قبل أن يذهب مع توكو للقضاء على عيون أنجل وأصدقائه.

    ولا عجب العجائب؟

  532. مايكل روتشيلد
    هناك تعريف جميل لـ "التعقيد الذي لا ينفصم" - وهو تعريف بسيط للغاية بحيث لا يمكن تبسيطه أكثر من ذلك - ومع ذلك فإن الخليقة غير قادرة على فهمه.

  533. إن عبارة "التعقيد الذي لا ينفصم" هي عبارة لها معنى، وفي رأيي، إذا كان هناك شيء في الطبيعة لا يمكن فصل تعقيده، فإنه يدل على مخطط ذكي.
    النقطة المهمة هي أنه حتى يومنا هذا لم يتم العثور على أي شيء في الطبيعة يبرر هذا الوصف، وكل شيء ادعى أنه معقد بشكل لا ينفصم، فقد وجد في الواقع أنه مقسم إلى مكونات صغيرة، كل منها في حد ذاته يعطي ميزة تطورية.
    لم يتم العثور على أي سمة حيوانية تتطلب من ناحية العديد من التغييرات الجينية المنسقة ومن ناحية أخرى لا يوجد ترتيب يمكن من خلاله إجراء هذه التغييرات واحدة تلو الأخرى مع اكتساب ميزة في كل مرحلة.
    ولكن أكثر من ذلك - حتى لو لم يتم دحض جميع ادعاءات الخلقيين بعدم إمكانية فك التشفير فعليًا - فإن هذا لن يشير إلى وجود تعقيد غير قابل للفك.
    لقد كانت مجرد إشارة إلى أننا لم نجد طريقة للتحلل ولكن حقيقة أننا لم نعثر على شيء ما لا تثبت عدم وجوده.
    بمعنى آخر - حجة التعقيد الذي لا ينفصم هي ببساطة مغالطة منطقية من النوع "حجة من الجهل"

  534. شموليك
    "التعقيد الذي لا ينفصم" هو محاولة سيئة من قبل واعظ مسيحي فقير يدعى مايكل بيهي. لقد جعل هذا التعبير أحد القضاة في الولايات المتحدة يضحك ذات مرة قبل 10 سنوات، ولكن هذه هي نهاية الأمر. هذا المفهوم ليس له معنى علمي.

    هذه هي القشة الأخيرة بالنسبة للمؤمنين بالفودو، مثل علم الأحياء الرياضي. الآن - لنفترض أنه تم العثور على مثل هذا المكون بالفعل (يصر عمال الفودو على البكتيريا، حسنًا، هذا كل شيء...). فهل هذا يعني أن هناك مخططا ذكيا؟ لا - لا يعني ذلك. نحن نعرف المكونات "غير القابلة للكسر" في الطبيعة، ونعرف بالضبط كيف تشكلت.

    لو أن رجلاً مؤمنًا فقط سيأتي ببعض الحجج المعقولة... ربما لا

  535. علم الأحياء الرياضي
    ما هي بالضبط العلاقة بين السيارة والتعقيد الذي لا ينفصم؟
    لقد طرحت عليك عدة أسئلة ولم تجب عليها - هل لي أن أفهم أنه ليس لديك إجابات؟

  536. نفخ الماء
    معنى الحياة، في رأيي، هو أن دور كل شخص هو ترك عالم أفضل للجيل القادم. تخيل ماذا سيحدث لو فكر الجميع هكذا؟ ما هو العالم الرائع الذي سنعيش فيه اليوم؟ ليس من الضروري أن تصاب بالجنون، ولكن يمكن لأي شخص أن يفعل شيئًا صغيرًا..

  537. كانت هناك أيضًا حلقة منشفة من Stola (تذكرنا قليلاً بعالم الأحياء الرياضي)

    نفخ الماء:
    قتلتني بتعليقك

  538. علم الأحياء الرياضي,
    ما الفائدة من الجدال؟
    إذا كنت تعتقد أنه يجب تأكيد أو إثبات نظرية محدودة، وأن معظم المعلمات التي تم اختبارها معروفة، عن طريق الملاحظة المباشرة، كاستدلال على موضوع لا تكون غالبيته معروفة، فلا بأس. استمر. ومن الواضح لي أن الاثنين ليسا متساويين، وكذلك العلم وفلسفة العلم والمنطق. كم مرة يجب أن أشرح ذلك وكم مرة يمكنني أن أرسلك إلى مدخل ويكيبيديا للحصول على كلمة "تأكيد" حتى تفهم ذلك؟ لذا، فنحن نرحب بك لتستمر في إخبار نفسك أن الاستدلال يستحق الإثبات، وتعتقد أنك من خلال القيام بذلك قد أثبتت وجود المصمم الذكي الذي لا يمكن دحضه، وبالتالي لا يمكن إثباته. انا لا.

    إذا كنت تعتقد أن وظيفتي هي إثبات عدم وجود التعقيد الذي لا ينفصم، فسأتمنى مرة أخرى ألا يتم اتهامك، ولكن إذا كان الأمر كذلك، أتمنى أن تفهم سبب وقوع عبء الإثبات على عاتق الدولة، أي على الشخص تقديم المطالبة. أنت تدعي أن هناك عقدة لا تنفصم. اثبت ذلك. وعدم إظهار الإفرازات ليس دليلاً على ذلك. وهذا مجرد دليل على فشله (مهما كان الفاشل لا يتفق معك أنه فشل وهو خبير عالمي في المجال، لكن ما المهم).

    إذا كنت تعتقد أنه حتى لو كان هناك تعقيد لا ينفصم، فيجب بالضرورة أن يكون هناك مخطط ذكي، حسنًا. أثبت أن هذا هو الحال ولا، فالسؤال: "وماذا يمكن أن يفسر هذا" لم يثبت. في ساوث بارك كانت هناك حلقة حيث يصادف الأطفال أقزامًا يسرقون الملابس الداخلية. كان لدى الأقزام خطة من ثلاث خطوات، وكانت نصها: الخطوة الأولى: سرقة الملابس الداخلية. الخطوة الثالثة للثراء. الخطوة الثانية كانت علامة استفهام (؟). أنت تسرق الملابس الداخلية وتحاول أن تصبح ثريًا بينما تتخطى الخطوة 2. لن تنجح. http://austrianeconomists.typepad.com/.a/6a00d83451eb0069e20120a962e645970b-pi

    وهذا ما قصدته عندما قلت أنه لا توجد حجج جديدة. دورة من الادعاءات العلمية الزائفة والإقناع الذاتي. الاستمرار في الجدال ليس بالأمر الذكي.

  539. شموليك، لقد أحضرت كمثال نظرية الأرض الإهليلجية. وهي نظرية ثبت صحتها فيما بعد مع التقدم التكنولوجي.

    أي أن مبدأ البرهان حقيقة ممتازة، ومن ثم فإن التصميم الذكي يكون علميًا.

    فيما يتعلق بالحجج الجديدة. لا أفهم ما الحجج التي تقصدها. لقد أعطيت مثالاً على المشكلة النفسية الجسدية (سيارة تم إنشاؤها في غرفة مغلقة). وبالطبع الدحض الإحصائي في شكل تعقيد لا ينفصم، والذي أذكرك أنه قائم ولم تدحضه بعد من خلال إظهار الخطوات التدريجية للسيارة.

  540. المعجزات:
    كان من المهم بالنسبة لي أن أكشف عورة الكاذب، لأن الكذب في رأيي هو من أفظع الأشياء التي يمكن القيام بها في هذا النوع من المواقع.
    عندما أدركت أنني حققت الهدف (جزئيًا بفضل تعليقك) قلت إنه كان مضيعة لوقتي وتوقفت عن الإشارة إليه.
    أنت تقترح أن أستقيل بعد أن استقيل ولم تقترح أن أستقيل من قبل.
    هذا يعني أنني، على الأقل بالنسبة لك، توقفت في اللحظة المناسبة.

  541. آسف نسيم، مجرد سؤال فاتني منك وهو مرتبط إلى حد ما، إذا كان من الممكن أن تكرر لي معنى حياتك لأنني لم ألاحظ، شكرا

  542. المراوغ كاذب!
    أنتم مدعوون للحضور مع القائمة التالية من الأشخاص:
    العاقل
    تطور لوجهك
    الى عمانوئيل
    اللدغة
    الشوك
    كاليبروس

  543. وكما لاحظت منك: أنت مراوغ.

    انكم مدعوون لاستخدام جميع المعلومات المتاحة لك. هل مازلت تريد الاستمرار في التهرب؟ إذا كان الأمر كذلك، فأنت تثبت لي أنني على حق.

  544. يعرف ماذا؟
    تعال مع المزيد من الأشخاص من بين أولئك الذين تدعي أنهم يستجيبون من نفس الكمبيوتر والذين سيشهدون على هويتك.
    سيؤدي هذا بالفعل إلى تقليل حالة عدم اليقين كثيرًا!

  545. ويستمر التهرب.
    لقد قلنا بالفعل أنه لا يوجد أي دليل في العلم.
    تعال مع سارة إلى جهاز كشف الكذب وسنحصل على تأكيد لنظريتي أو النظرية التي تخترعها لنا.

  546. باختصار - تتجنب.
    يعرف ماذا؟ تعال مع سارة أيضا. سيؤدي هذا إلى زيادة اليقين في الاختبار. أرك لاحقًا!

  547. خانتريشكار:
    هل أنت مستعد للخضوع للاختبار بواسطة جهاز كشف الكذب أو هل تعلم أنه سيتم القبض عليك؟

  548. أعلم أن لديك بعض المعرفة بأجهزة الكمبيوتر (وأنا أيضًا) لذا طلبت منه أن يكتب البريد الإلكتروني بهذه الطريقة.
    لأنني عرفت أن هذا هو بالضبط ما ستعتمد عليه!

    أنت مجرد كاذب ومهرج نتن.

  549. وهنا لدينا دليل آخر على أن ما يراه الشبح يأتي مباشرة من عقله المحموم.
    بالمناسبة - لا يتعلق الأمر فقط بعنوان IP (وبالمناسبة - يستخدم Rafaim أكثر من عنوان IP واحد) ولكن أيضًا بأسلوب عنوان البريد الإلكتروني (أسمح لنفسي بالإشارة إلى ذلك لأنه في اختراقه الأخير أثبت بالفعل أنه اكتشفه) وإلا كنت سأستمر في إخفاء الأمر عنه).
    وبطبيعة الحال، فإن الهراء الذي يكتبه يثبت ذلك أيضًا.
    والأطرف أنه يلقب الآخرين بلقب "الكذابين".
    يعرف ماذا؟ أشباح! نرحب بك للكشف عن اسمك الكامل والمجيء معي لإجراء اختبار كشف الكذب. التكلفة على عاتقي، ولكن إذا تبين في الاختبار أنك كريه الرائحة، فسوف تدفع لي 10 أضعاف نفس التكلفة.
    يوافق؟

  550. أرى أن مخال غير قادر على إثبات أي شيء.

    من أجل الإفصاح الكامل:
    هذا هو عنوان IP الذي أكتب منه عادةً.
    الكمبيوتر الذي ردت منه "سارة" هو نفس الكمبيوتر الذي قمت بالرد منه عدة مرات. (والتي يعلق عليها المزيد من الأشخاص - على الرغم من أنني لا أعرف إذا كنت في هذا المنتدى أم لا).

    طلبت من المعلق "سارح" أن يقرأ رسائل مايكل روتشيلد النتنة. وسوف يستجيب كما يشاء.

    بالطبع عائلة روتشيلد تعرف كل الحقائق. ولابد أنه كان يعرف هذه الحقيقة أيضًا. (أليك)

    ولكن هذا ما يحدث يا روتشيلد، عندما تصاب بجنون العظمة، وتكون متغطرسًا ومفتخرًا، كما أن الأمر كريه الرائحة عندما تتعرض للظلم.

    منذ اليوم، تم تصنيفك أيضًا على أنك كاذب.
    لأن أي شخص يمكن أن يرى أنه عندما تكون مخطئًا، فلا مشكلة لديك في البدء بالكذب. (في النهاية، أنت تعرف كل شيء، ومن المؤكد أن الآخرين سوف يصدقونك حتى عندما تكذب)

    لقد تجاوزت مهزلتك كل الحدود ويمكنك أن تكتفي بالمعلقين مثل Nisim وMouthole (الذين ثبتت سمعتهم السيئة هنا بالفعل) لمواصلة تصديقك.

  551. المعجزات:
    هذه مرة أخرى من الأوقات التي أشعر فيها بالأسف لعدم وجود زر "أعجبني" هنا.
    لقد أثبت ذلك مرارًا وتكرارًا ويواصل الآن محاولته النتنة لانتحال الشخصية.
    هناك موقف مثير للاهتمام هنا: لقد تمكن من أن يثبت لك أنه أحمق، ولكن يمكنني أن أثبت لك أنه شبح بمجرد حصولك على امتيازات المشرف على الموقع. مع هذه الأذونات من الممكن أن نرى أنه هو نفس الشخص (إذا افترضنا أنه شخص).

    بالمناسبة - ليس كل المتدينين أغبياء وهذه حقيقة.
    ومنهم من يبلغ من الذكاء درجة أنه يتمكن من التفوق رغم الدين، رغم أن ذلك يحد من قدراته في بعض المجالات.

  552. مايكل روتشيلد
    إنه شيء واحد أن تعتقد أن المتدينين حمقى (أعتقد أنهم ليسوا جميعًا حقًا….).
    إنه شيء آخر عندما يثبتون ذلك مرارًا وتكرارًا 🙂

  553. بالمناسبة - لدي الكثير من الصبر، ولكن فقط للأشخاص الذين يريدون أن يفهموا.
    الأشخاص الذين يفتقرون إلى المعرفة والفهم والذين يصرون على البقاء هكذا بأي ثمن ويجرون الناس وراءهم يضايقونني حقًا.
    تكمن المعضلة دائمًا في ما إذا كان يجب إدخالهم في منتجات الإسهال الفموي أو تجاهلها.
    الجواب ليس بسيطا دائما.

  554. فقط لأكون واضحا:
    سارة هي شبح في إحدى محاولات الاحتيال المعتادة.
    يجب أن أعترف أن هذا الاسم يناسبه أكثر.
    فتحة الفم صحيحة.
    لقد تحول الشبح من الرسوم المتحركة إلى القزم إلى القزم.
    إنه لا يفهم شيئًا مما يقال له أو مما يحدث في الطبيعة ولكنه يتحدث كما لو كان رئيس الله.
    من الذاكرة المباركة.

  555. ويمكن أيضًا استعارة المعجزات
    يمكن أن تكون الذاكرة الخالية من المتاعب والسرعة والجيدة هي "مصيرك" فيما يتعلق بالمخطط الذكي

  556. نفخ الماء
    لماذا توافق على هذا ؟؟؟؟ ولماذا نعتقد أن البشر يعرفون ما هو "الخير"؟ للعيش لأطول فترة ممكنة؟ الأقل؟ مضاعفة قدر الإمكان؟ الأقل؟ قتل الأنواع الأخرى أم لا؟ قتل الآخرين أم لا؟

    لذلك 1) لا أرى أي سبب للموافقة على شيء وهمي ولا أساس له من الصحة
    و 2) كيف أعرف أنا أو أي شخص آخر (ومن يحدد من هو هذا الشخص؟) ما هو "الخير"؟

    اسمع، نظريتي أبسط بكثير، وتشرح العالم، وليس بها تناقضات، وتعطي معنى للحياة.
    لقد بدأت رحلتك سيئة للغاية، ومن هناك لم تتقدم إلى أي مكان، على حد علمي.

  557. يأتي الأمر على مراحل، في البداية نظرية، ثم تشعر بها فعليًا، وكذلك الأكوان الموازية
    ابدأ بالذهاب خطوة أخرى إلى الأمام من خلال الاتفاق على أن الأشخاص يريدون الأفضل لكي يعمل الكمبيوتر "بشكل جيد"

  558. نفخ الماء
    أنا أتفق معك. النقطة المهمة هي كما يلي: في علم الآثار على سبيل المثال، إحدى الطرق لمعرفة ما إذا كان هناك شيء مخطط له هي محاولة فهم غرض المخطط. إذا قلت أن الأمر لا معنى له هنا، وربما كنت على حق، فكيف يمكنك القول بأن الحياة تبدو مخططة؟
    أستطيع أن أفهم ما هو الغرض من كائن حي معين وبالتالي أفهم ما يفعله. لكنني لا أرى أي سبب للاعتقاد بأن هناك من خطط لحياته... دون البحث عن سبب لهذا التخطيط.
    اسمحوا لي أن أقول ذلك بطريقة أخرى: إذا لم تكن هناك طريقة لمعرفة أي شيء عن هذا المخطط الذكي (مصطلح "ذكي" هو أيضًا إشكالي، ولكن المزيد عن ذلك في وقت آخر) فهذا يعني أنه ليس له أي تأثير على حياتي. إنه مثل الحديث عن عالم موازٍ لن نتواصل معه أبدًا. جميل فكريًا ولكنه مضيعة للوقت على المستوى اليومي.
    افهم - لا يوجد سبب، وفقًا لك (!) للاعتقاد بأن المخطط يريد مصلحتي، أو مصلحة الإنسان بشكل عام. على العكس من ذلك، نحن أدوات لغرض معين لا نستطيع أن نفهمه أبدًا.

  559. السيد نسيم المحترم
    إنه يشبه إلى حد ما الكمبيوتر والإنسان، الكمبيوتر نفسه لا يدرك غرضه، المخطط، الإنسان فوقه. وكذلك موقف الإنسان "والمخطط الذكي"

  560. لا أعرف ما إذا كان الشبح عبارة عن رسم كاريكاتوري مصمم للتأكيد على نفاد صبر ميشال من خلال نفاد صبره المجنون أو إذا كان يعاني حقًا من متلازمة توريت (وليس ذكيًا جدًا أيضًا)

  561. "الخوارزميات التطورية" هي كائن حي؟ - في الواقع هراء.
    وأتساءل أين رأيت مثل هذا المخلوق "الخوارزمية التطورية".
    بالتأكيد أقول أنها موجودة لأن الإنسان هو من صنعها. لكن بالطبع لن تفهم أن الإنسان مخلوق وليس تطور. لكن لا يهمني إذا كنت تفهم أم لا، فإن المصاب بجنون العظمة مثلك لن يقبل أبدًا حقيقة أنه كان مخطئًا.

  562. علم الأحياء الرياضي,
    أنا حقًا لم أصل إلى عمق عقلك، لكن إذا فهمت المغزى من كلامك، فأنت تستمر في محاولة دحض التطور، لكنني لا أدافع عنه. وفي هذا الصدد، التطور لا يهمني. كل ما أقوله هو أن الرئيس كان على حق.
    لا يوجد شيء اسمه مبدأ التأكيد، ولا يوجد شيء اسمه مبدأ الإثبات. ببساطة لا يوجد. هناك نظريات أقوى وأقل قوة، لكن أبدا، لم يتم إثباتها (وأنا لا أتحدث عن نظرية العلق في الحائط، ولكن عن النظريات التي لا نعرف فيها كل البيانات). أنت على الأكثر تستنتج ولا تثبت وجود مخطط ذكي من ملاحظتك (السطحية)، لكن يجب أن تعلم أنك تفعل ذلك من أجل شيء لا سبيل إلى نقضه. هذه مغالطة منطقية.
    كنت أتمنى الحصول على حجج جديدة فيما يتعلق بالمصمم الذكي. ذلك لم يحدث

  563. سأتابع شيئًا واحدًا.

    أنت قلت:

    "وينطبق الشيء نفسه على أي سيارة طبق الأصل. أنت تخترع آلات غير موجودة."-

    إذا لم تكن موجودة، فأنت لم تشاهد إنتاجها، أو قام شخص ما نيابة عنك. ومن هنا مرة أخرى - في رأيك، مثل هذه السيارة ليست دليلاً على وجود مصمم. وإذا لم يكن من الممكن، وفقًا للتطور، معرفة ما إذا كانت السيارة المكررة تتطلب مصممًا، فإن هناك خطأً كبيرًا في النظرية.

    الشيء نفسه ينطبق على مبدأ المصادقة. تريد أن تسميه مبدأ الإثبات. ليس هذا ما يهم.

  564. في الواقع، هراء شديد.
    العالم كله يسميها خوارزميات تطورية فقط لأن واحد زائد واحد يساوي اثنين ويسميها واحد زائد واحد يساوي اثنين باسم التطور لأن الشبح لم يستقر عليها.
    يعرف ماذا؟
    في الواقع من الواضح بالنسبة لي أنك لا تعرف ماذا!

  565. هذا ما كتبته:
    "ومن المرغوب أيضًا توضيح أن كل هذه "الروبوتات المكررة" هي نتاج فكر البشر وخلقهم، وليس تطور كائن حي آخر في الطبيعة". - استعارة لما يشبه ذلك: الإنسان لم يخلق الديناصورات - بل التطور.
    الروبوتات القادرة على التكاثر هي من صنع البشر. وليس تطور كائن حي يسمى الروبوت. لأنه ببساطة لا يوجد مثل هذا المخلوق.

    لكن عندما تحدد مسبقًا أن واحد زائد واحد هو تطور، سيكون من الصعب أن تثبت لك أن واحد زائد واحد يساوي اثنين.

  566. لذلك دعونا فرزها:
    عندما قدمت مثالاً للروبوتات التي تتطور من خلال التطور، وجدت أنه من المناسب "توضيح" أن الروبوتات المتماثلة التي ذكرتها ليست روبوتات مكررة (وإلا لماذا علامات الاقتباس؟) وأنها نتاج الفكر البشري و خلق.
    وبما أنني قلت من قبل أنه نظام تطوري من صنع الإنسان ولم ترغب فقط في تكرار ما قيل، فقد استنتجت - كما كان سيستنتج أي شخص عاقل - أنه عندما تقول إنهم نتاج فكر بشري و الخلق - تقصد أن التطور لم يخلقهم.

    ومن ناحية أخرى - أنت تقول أنني لم أفهم وأنني أكذب عندما أقول أن هذه هي الأشياء التي حاولت توضيحها.
    لذلك أقترح أن توضح لنا (وهذه المرة باللغة العبرية!):
    1. هل، كما زعمت، استنساخ الروبوتات التي هي ثمرة التطور؟ من فضلك أجب بنعم أو لا.
    2. إذا كانت إجابتك "لا" فأخبرني ما هو الخطأ في تقديم مطالبتك. إذا كانت إجابتك "نعم" فأخبرني ما الذي تهدف إجاباتك إلى تحقيقه.
    علاوة على ذلك، أنتم مدعوون للتصويت لنا على أي مساهمة منكم في أي مناقشة.

  567. الهراء الوحيد الذي أفعله في كل هذه المناقشات هو الإشارة إليك.
    لم أر بعد أنك قد ساهمت بأي معلومات أو فكرة أو فكرة أصلية في أي مناقشة.
    كل ما يهمك هو نفسك والترويج لخرافاتك.

  568. لقد كتبت للتو هراء.
    فقط لأنك لم تفهم كلماتي قررت ببساطة تحريفها.
    (هل فكرت يومًا في الذهاب إلى كوريا الشمالية واستبدال كيم جونغ أون؟ - كلاكما يعاني من جنون العظمة)

  569. من المستحسن أن نفهم أن الذي لم يفهم هو شبح، وبما أنه ربما لن يفهم أيضاً، فلا داعي للجدال معه.
    قوانين الرياضيات هي جزء من قوانين الطبيعة وهي الشيء الرئيسي الذي يعمل أيضًا في تطور الحيوانات.
    وحقيقة أن المادة المعطاة لهم ليست عضوية (وبالمناسبة - حتى هذا ليس صحيحا بالنسبة لجميع الأمثلة التي ذكرتها، وأحدها مواد عضوية) لا تنتمي إلى مسألة التطور على الإطلاق.
    إن كون الإنسان خلق البيئة هو ببساطة لكي تكون هناك بيئة تشجع على تحقيق الأهداف التي لا يعرف هو نفسه كيف يحققها.
    باختصار - إنه تطور لكل شيء وكل ما يريدك الشبح أن "تفهمه" هو أنه ليس تطورًا وأن الروبوتات لا يمكنها التطور من خلال التطور - أي أنه يريدك أن "تفهم" أشياء خاطئة.

  570. ينبغي توضيح أن مايكل روتشيلد ببساطة لم يفهم. يحدث…

    "وبعبارة أخرى: البشر خلقوا البيئة التي تتكاثر فيها الروبوتات وتفاصيل كيفية وراثة السمات" - كما كتبت - وكما صحح ماشيل نفسه بعد ذلك.

    "لكن كل شيء آخر هو نتيجة للقوانين الرياضية للتطور." - بالضبط، وليس نتيجة لتطور كائن حي آخر في الطبيعة. كما شرحت.

  571. أي: البشر هم الذين خلقوا البيئة التي تتكاثر فيها الروبوتات وتفاصيل كيفية وراثة السمات، أما كل شيء آخر فهو نتيجة للقوانين الرياضية للتطور.
    ولم يتمكن البشر من التنبؤ بما سيتم إنشاؤه هناك.

  572. اصدقاء:

    أنا مندهش أنك لا تزال تأمل في إعادة "علم الأحياء الرياضي" إلى عقلانيتك.

    أنا شخصياً تخليت عن النقاش معه لأنني وقفت سريعاً إلى جانب طبيعته: الحقائق لا تهمه. انه مجرد القزم.

    وبالمناسبة - هناك روبوتات مكررة تقوم بأشياء مفيدة وليست نتيجة تخطيط لأنها نتيجة تطور.

    وبما أنه قانون رياضي لا يعتمد على كون الناسخين على قيد الحياة، فإن نظرية التطور تستخدم أيضًا لتطوير البرمجيات والروبوتات.

    ويتم ذلك، من بين أمور أخرى، عن طريق الخوارزميات التطورية:

    http://en.wikipedia.org/wiki/Evolutionary_algorithm

    إن الإنجازات في هذا المجال ليست قليلة، وهي في الواقع قوانين رياضية لا تفسر الواقع المادي فحسب، بل تخلقه في الواقع.

    من بينها يمكننا أن نجد:

    البرامج التي تلعب الشطرنج: http://www.ynet.co.il/articles/0,7340,L-3566594,00.html
    تطوير الروبوتات تطوريًا:

    http://www.plosbiology.org/article/info:doi/10.1371/journal.pbio.1000292
    العالم الآلي:

    http://ccsl.mae.cornell.edu/sites/default/files/Science09_Schmidt.pdf

    والمزيد.

    كل هذه الأمور - بالطبع، ليست أدلة على النظريات الرياضية، لأنها أثبتت رياضيا بشكل مستقل.

    إنه تطبيق تكنولوجي للنظريات عن طريق خلق الظروف التي تظهر فيها النظريات أن التطور سيحدث - بشكل مصطنع - من أجل تلقي التطور.

    مجال آخر موجود في مجال التطبيق وفي مجال عالم الحيوان هو تطوير الأدوية من خلال عملية تطورية لاختيار أنواع مختلفة من المواد الكيميائية التي تتفوق أكثر فأكثر في التعامل مع المرض أو مع السم، كما على سبيل المثال في هذا المثال:

    https://www.hayadan.org.il/freedom-movement-weizmann-2604114/

  573. علم الأحياء الرياضي,
    فهمت. لن تجيب على سؤالي حول ما إذا كنت تعتقد أن الاستدلال دليل، لذلك سأجيب. الجواب بالطبع ليس.
    1. من كلامك: الإنسان فاعل عاقل. الإنسان يخطط للأشياء والأشياء التي يخطط لها معقدة. تتكون الطبيعة من هنا (تلميح، هذا هو المكان الذي تبدأ فيه عملية الاستدلال) وقد صممها عامل ذكي. ادعائك هو استنتاج وليس دليلا. التدفئة.

    2. لا يوجد شيء اسمه مبدأ التأكيد. لقد أوضحت لك ما يعنيه مفهوم التأكيد وما لا يعنيه. انتبه إلى لغتك العبرية، لقد كتبت: "بالنسبة لي، التأكيد هو دليل على نوع ما من الادعاء". ويسعدني أنه من وجهة نظرك، ولكن من وجهة نظر المتحدثين الآخرين بالعبرية، ليس هذا ما يعنيه مفهوم التثبيت. وبما أنك اخترعت مبدأ غير موجود، فيمكنك أن تدعي بموجبه أنه على الرغم من استحالة دحض المصمم الذكي، إلا أنه يمكنك تأكيده. هذه هي الطريقة التي تخترع بها مخططًا غير موجود. التأكيد بمبدأ غير موجود.

    3. عبء إثبات المطالبة: القانون معقد للغاية ويقع على عاتقك وحدك. وحقيقة أن العلماء حاولوا تفكيك الشوتون، ولم ينجحوا في نظرك، على الأكثر يدل على عدم كفاءة هؤلاء العلماء ولا شيء غير ذلك. آمل ألا تقاضيك الولاية أبدًا بسبب أي شيء، ولكن إذا فعلت ذلك، فسوف تفهم على الأقل سبب أهمية الحفاظ على عبء الإثبات. بالإضافة إلى ذلك، أنت تحاول أن تنقل لي، بطريقة غريبة، عبء الإثبات، بأن تكرار سفينة الفضاء ليس دليلاً على التصميم. في أي عالم بالضبط من المفترض أن أثبت وجهة نظرك، فيما يتعلق بجهاز لا أعرف عنه شيئًا؟

    4. إن واجب إثبات أنه إذا كان هناك تعقيد لا ينفصم يؤدي بالضرورة إلى عامل ذكي فهو يقع على عاتقك وحدك. أنا أزعم أنه إذا كان هناك نظام من التعقيد الذي لا ينفصم، فسيتعين على نظرية طبيعية أخرى أن تتناول هذه الحقيقة وتضمينها. ولكي أبين أن هناك أشخاصا لا يهربون إلى الله عندما يصعب عليهم الأمر، حدثتكم عن ستيفان ولفرام (مطور برمجيات رياضية ومحرك بحث يحمل اسمه) والذي يدعي أنه وجد مبدأ طبيعيا أعمق من ذلك. الانتقاء الطبيعي، والذي، حسب رأيه، يحل مشكلة الاحتمال. الغالبية العظمى من الخبراء في هذا المجال يختلفون عليه، ولكن ماذا في ذلك؟ الغالبية المطلقة من العاملين في هذا المجال تختلف معك أيضًا، لكن على الأقل أوضحت أن هناك نظريات أخرى متداولة في هذا المجال.

    5. התגובה שלך בנוגע לצלחת המעופפת ומכוניות משוכפלות היתה מתחמקת, משונה ומעידה על כך שאתה שוב ושוב מחמיץ את הנקודה, וחבל כי לאחר שהויכוח הזה יתסיים, בעוד כמה שבועות תצוץ לה כתבה אחרת ושוב נתחיל את הויכוח הזה מחדש (אולי תבחר, שוב, שם مختلف). على الأقل الاستعداد. النقطة المهمة هي، مرة أخرى، أن الاستدلال ليس دليلاً وبما أنك لسبب ما لا تفهم هذا، فأنت تحاول أن تجعلني أستنتج (لأن الاستدلال بالنسبة لك هو دليل) أن هذا تخطيط باستخدام مثال سيء، دعنا لا أقول الغوغائية الرخيصة. أنت تحاول رشوتي للتفكير في التصميم من خلال طرح الآلات التي بنيناها والتي لا وجود لها في الطبيعة وسؤالي عما إذا كان من المرجح أن يكون هناك عميل ذكي قام ببنائها. ومن المؤكد أن شخصًا ذكيًا هو الذي صنع طبقًا طائرًا، ولكن ذلك لأني قد رأيت أشخاصًا صنعوا مثل هذه الآلات ورأيت أيضًا: الطائرة هي طبق طائر. سفينة الفضاء هي طبق طائر. هذا المثال لا يقول شيئًا عما لم أر الناس يخططون ويبنونه، بل إنني أتفق معك في أنه يجب على المرء أن يكون حذرًا مع المعيار الذي أحضرته، لأنه حتى مظهر أعيننا يمكن أن يكون مضللاً. حسنًا، هذا يعزز حجتي، والتي بدون معلومات كافية، لا يمكن اتخاذ قرار بشأنها. بالإضافة إلى ذلك، أوضحت لك أن مثالك سيء لأنه يتناقض مع التطور، ففي نهاية المطاف، ما هي الميزة التطورية للانفصال عن الكوكب والطيران لملايين السنين في أصعب الظروف التي يمكن تخيلها في الأمل الأعمى بمواجهة كوكب؟ ؟ شرحت لك أن المواد التي يصنع منها الصحن الطائر الذي أتخيله (لم تذكر أي مواصفات) هي أقل المواد شيوعا في الكون. لقد أخبرتك أنه إذا أعطيت تفاصيل مختلفة، فقد أفكر بشكل مختلف. لهذه الأسباب (وهناك المزيد) سأفترض أنني إذا رأيت طبقًا طائرًا سأعتقد أن شخصًا ذكيًا هو من صنعه، لكن ليس بسبب تعقيد الآلة. وهذا في حد ذاته لا يعني شيئا. وينطبق الشيء نفسه على أي سيارة طبق الأصل. أنت تخترع آلات غير موجودة، وليس لدي أي فكرة عن كيفية تناسبها مع الطبيعة (كيف تعيش، ماذا تأكل، وما إلى ذلك، وما إلى ذلك) وكأنك تتوقع مني أن أجد لك حلاً لكيفية التطور قمت بإنشائها وعندما تفشل، بسبب نقص البيانات، فإنك تحتفل. هذا أمر مثير للسخرية تماما. على أية حال، حتى لو كنت أعتقد أن الأرجح أن الطبيعة قد صممت ولم تخلق عشوائيا، فهذا لا يعني أنني أثبت ذلك. وبما أن المخطط الذكي لا يمكن دحضه وكما شرحت لك لا يوجد ما يمكن الحديث عنه من حيث إثباته أيضا، فقرار الرئيس بعدم تدريس التخطيط الذكي في دروس العلوم هو صحيح تماما (من الضروري تدريس التخطيط الذكي في دروس العلوم وكيف لا ينبغي بناء النظريات العلمية)

  574. يمكن الجدال على مستوى التخطيط الذكي
    أرى أن اختلاف الآراء في الموقع يكون على مستوى التجربة والخطأ، وهو التخطيط الذكي، والذي يمكن القول أيضًا بأنه تطور التفكير!

  575. الأحياء غير الواقعية:

    "من لا يرى التخطيط في روبوت عضوي مكرر، ربما لا يرى التخطيط في أي شيء." - الشخص الوحيد الذي يرى روبوتًا عضويًا يتكاثر هو أنت. وتراه في خيالك وليس في الواقع. لأنه في الواقع الحالي لا يوجد شيء من هذا القبيل.

  576. وبما أن معظم الأسئلة هنا تمت الإجابة عليها، وبما أنني لا أحب الخوض في الماء، فلا أرى ضرورة لتكرار الادعاءات التي تم دحضها. أي شخص لا يرى التخطيط في الروبوت العضوي المتماثل ربما لا يرى التخطيط في أي شيء. ولا أستطيع تغيير ذلك.

    علاوة على ذلك، حتى السيارة الثابتة يمكن أن تتطور في غرفة مغلقة. لأنه إذا كان بإمكان الإنسان الذكي أن يتطور من خلال عملية طبيعية، فيمكنه أن يتطور في غرفة مغلقة نظرية ضخمة (تحتوي على ضوء الشمس والماء وما إلى ذلك) ويطور سيارة. طوبى للمؤمن.

    عوفر، عندما تثبت أن الكتاكيت يمكن أن تتطور بعملية طبيعية من مادة جامدة أو كائنات سبقتها، فسيكون لادعائك وزن. لأنه عندها يمكننا أن ندعي أننا نعرف العمليات الطبيعية التي تنتج الكتاكيت بينما السيارات لا تفعل ذلك. وحتى ذلك الحين، فإن التوازي مثالي.

  577. السيرة الذاتية. مات.

    كما شرحت - السيارة ليست ذات صلة - في رأيك أيضًا. أجب عن الأسئلة مباشرة - ولا تتهرب.

    1. قلت: "هناك عمليات طبيعية تكون الحجارة" - فهل تؤيد هذا الادعاء؟
    2. إذا رأيت حجرًا يتم تصنيعه في أحد المصانع، فهل ستغير مطالبتك من القسم 1؟

    إذا أجبت مباشرة، فراجع السؤال
    1. إن إنشاء حجر في مصنع لا علاقة له بالأصل الطبيعي للأحجار.
    2. ولهذا السبب فإن إنشاء الكتاكيت في المصنع لا علاقة له بأصل الكتاكيت في البرية.
    3. وحقيقة أن السيارة المقلدة ليست ذات صلة، فهي غير ذات صلة.

  578. علم الأحياء الرياضي
    وهذا هو بالضبط وجهة نظري. قنديل البحر معقد للغاية، ولهذا السبب نحن كبشر لا نراه كما هو مخطط له. أقول مرارا وتكرارا وأنت ترفض الاستماع - البساطة هي علامة التخطيط، وليس التعقيد.

    ما تفعله هو أخذ رأيك واحتوائه على العالم. أنت تقول "هناك خالق" وأنت تعدل العالم الذي تراه حسب ذوقك الخاص. لقد جربته أيضًا - وتوصلت إلى تناقض. أتعلم؟ جئت إلى بحر من التناقضات. كنت أتوقع أن يبدو العالم مختلفًا إذا كان لديه مخطط ذكي.

    وما هو المضحك؟ أنت بغطرستك تعزز شعوري مرارًا وتكرارًا :). لقد وصفت لي مدى تعقيد العالم الحي ومدى هشاشته. والأمر المضحك أنك تقدم لموقفك حججًا قابلة للدحض، وهي حجج الدعاة المسيحيين البعيدين جدًا عن روح اليهودية. هل تحاول أن تثبت لي أن المسيحيين على حق؟؟

    علامة أخرى على التخطيط هي إظهار أن المخطط لديه هدف. سأطرح عليك سؤالاً: لأي غرض رسم المخطط الذكي الحياة؟

  579. عوفر،

    ومرة أخرى من زعم ​​أن الكتاكيت لا تحتاج إلى تخطيط لأنها تتطور بنفسها. وهكذا تتطور السيارة أيضًا من تلقاء نفسها في مصنع يصنع السيارات. التشبيه صحيح وبالتالي وحسب ادعائك - السيارات لا تحتاج إلى تخطيط.

    هل تريد الادعاء بأن هناك عمليات طبيعية يمكنها خلق ما تراه في العالم الطبيعي؟ من فضلك، أحضر هنا دليلا قاطعا على أن أي مخلوق يمكن أن يتطور من خلال عملية طبيعية من مادة غير حية. أو حتى من مخلوق سبقه يحتوي على أنظمة مختلفة (ليس عن طريق نقل الجينات كما في مثال شموليك، الذي لم يتطور فيه شيء فعليا بل تم نقله فقط من مكان إلى آخر). طالما لم تفعل ذلك، فالتوازي صحيح تمامًا ولا يمكنك تقديم القسمة.

    شموليك،

    "بشكل عام، لقد زعمت أنه من المرجح أن يتم تصميم سفينة فضائية بواسطة عميل ذكي، لكنك لم تثبت ذلك."

    شكرا. الآن نحن نمضي قدما. كما قلت - من وجهة نظرك (ومن وجهة نظر جميع أنصار التطور)، فإن الاستنساخ ليس دليلاً على التصميم. وهذا الادعاء الخطير يحتاج إلى دليل جدي. إن نظرية التطور ببساطة لا تملك الدليل المطلوب لإثبات هذا الادعاء، وبالتالي لا تشكل تفسيرا علميا.

    "تعتقد أنه من المرجح أن يكون الصحن الطائر قد تم بناؤه بواسطة المخابرات، لأنك، من بين أمور أخرى، رأيت، أو رأى شخص ما نيابة عنك، أشخاصًا يصنعون الصحون الطائرة"-

    أنا آسف لإحباطك. لم أرى أحداً يصنع أطباقاً طائرة. ليس من أجلي وليس من أجل أختي. مع العلم أنه ليس لدي أخت.

    "وهذا بالضبط ما ادعيته (الملاحظة المباشرة تساوي إثباتًا في اللغة اليومية) وبالتالي سأوافق على أنه إذا رأيت طبقًا طائرًا فضائيًا، فسأفترض أن الفضائيين هم من بنوه، لكن هذا ليس دليلاً، بل افتراضًا." -

    مرة أخرى، حتى وفقًا لطريقتك، فمن المرجح أن يكون شخص ما قد صمم الطبيعة.

    إن ادعاءك بشأن المراقبة المباشرة لشخص ما وهو يصمم سيارة هو أيضًا مشكلة كبيرة. يرى بعض الناس أشياء كثيرة لا يراها الآخرون (أعتقد أنك سمعت عن الدكتور ابن إسكندر). هل تعتقد أن كل ما يرونه هو دليل على وجودهم؟ هل تعتقد أن ابن الإسكندر زار الجنة فعلاً؟ إذا كانت إجابتك "لا"، فلا فائدة من ادعاءك بالملاحظة المباشرة. وبالتالي لا يمكنك في الواقع إثبات أي شيء.

    ونعم، أنا أتحدث بالفعل عن مركبة فضائية بيولوجية. ليس هذا ما يهم. ولو أن البلاستيك كان المادة الأكثر شيوعاً في الكون وموجودة في الطبيعة. فهل تعتقدون أننا نتوقع رؤية مركبة فضائية مصنوعة من البلاستيك في مثل هذه الحالة؟

    "لم يكن الهدف اختبار قنديل البحر على مشبك ورق. نظرًا لأن مشبك الورق يتطلب مخططًا لمشبك الورق، وهو أكثر تعقيدًا بكثير من مشبك الورق وربما أيضًا من ميدوسا، فإن المثال يهدف إلى إنشاء أعلى بكثير من مجرد مقارنة."-

    وحتى أنه بعيد المنال. لأنه إذا لم يكن للمخطط الأول بداية، فهو لا يحتاج إلى التخطيط على أية حال.

    معجزات، نعم سأفاجأ بميدوسا أكثر من سفينة الفضاء. هل سبق لك أن شاهدت العمليات التي تحدث في قنديل البحر على المستوى الجزيئي؟ أود أن أقترح عليك أن تبدأ هنا:

    http://www.youtube.com/watch?v=yqESR7E4b_8

    وهذا في الواقع مجرد غيض من فيض.

  580. لقد قلت فكرة دينيت... أنا أستخدمها بشكل مختلف قليلاً. ما أقوله هو أن الجسم الأبسط سوف يفاجئنا ويجعلنا نعتقد أن هناك مخططًا على نفس الكوكب. مرة أخرى - أنا أتحدث عن بساطة العظم. وهذا ما يوضح لنا أنه مخطط له. إنه حلم كل مهندس...
    لم يتعامل علم الأحياء الرياضي بشكل جيد مع هذه الفكرة... ولا مع فكرة الاحتمال التي طرحتها. يحدث هذا دائمًا في مثل هذه الحجج، لأنه ليس لديهم أي حجج جدية حقًا. هكذا هو الحال عندما تؤمن بشيء دون أي أساس..

  581. علم الأحياء الرياضي,
    هل الرأي دليل؟

    أنت لم تدحض أي شيء، لقد زعمت للتو أنه من المرجح أن يتم تصميم سفينة فضائية بواسطة عميل ذكي، لكنك لم تثبت ذلك. الرأي ليس دليلاً، ولو كنت طفلي لسألت نفسك: لماذا يجب أن أشرح لك نفس الشيء مراراً وتكراراً. تعتقد أنه من المرجح أن يكون الصحن الطائر قد تم بناؤه بواسطة الذكاء، لأنك، من بين أمور أخرى، رأيت، أو رأى شخص ما نيابة عنك، أشخاصًا يصنعون الصحون الطائرة، وهذا بالضبط ما ادعيته (الملاحظة المباشرة تساوي الدليل في اللغة اليومية) ، لذلك سأوافق على أنه إذا رأيت طبقًا طائرًا فضائيًا، فسوف أفترض أن الفضائيين هم من صنعوه، لكن هذا ليس دليلاً، بل رأي. حجتي لا تعني أن ما لم تراه لم يكن بالضرورة من صنع عامل ذكي. هل تدعي أنك رأيت، أو هل رأى أحد نيابة عنك، المصمم الذكي الذي يخلق الطبيعة؟

    لاحظ بالمناسبة أنك لم تحاول فحص ما إذا كانت سفينة الفضاء متوافقة حتى مع التطور (لماذا ستكون هناك ميزة تطورية لمغادرة الكوكب؟) وأنا أميل إلى فكرة وصف مثالك بالديماغوجية الرخيصة منذ الحقيقة هو أن الطبيعة كلها مبنية من المواد المشتركة في الكون، في حين أن سفينة الفضاء (التي أتخيلها على الأقل) مصنوعة من المواد الأقل شيوعا في الكون، مما يدل على أن عامل ذكي هو الذي خلقها. هل تريد تصحيحي وإخباري أن هذه مركبة فضائية بيولوجية؟ إذا كان الأمر كذلك، قم بإعطاء التفاصيل. من المحتمل أنه مع وجود مواصفات فنية مختلفة، فمن الأرجح أنه لم يقم أي عميل ذكي ببناء سفينة الفضاء.

    مشبك ورق أم قنديل البحر؟
    إليكم المقال الذي يناقش موضوع (معجزات، هل لديكم صلة بالآخرين؟). ابحث عن الكلمات أشعل النار البطلينوس
    http://www.investigatingatheism.info/danieldennettchapter.html
    لم يكن الهدف اختبار قنديل البحر على مشبك ورق. نظرًا لأن مشبك الورق يتطلب مخططًا لمشبك الورق، وهو أكثر تعقيدًا بكثير من مشبك الورق وربما أيضًا من ميدوسا، فإن المثال يهدف إلى أعلى بكثير من مجرد المقارنة. يستخدم المقال هذا المثال ليقول إن تفسير "المخطط الذكي" يؤدي إلى حلقة مفرغة أو انحدار لا نهائي، وبالتالي لا يفسر شيئًا

  582. علم الأحياء الرياضي
    إذا كانت جميع الإنزيمات والآليات والباتينيتات المعجزة موجودة في قنديل البحر - فإنها تبقى على قيد الحياة بصعوبة كبيرة. قم بتغيير الملوحة أو تركيز الأكسجين أو أشياء أخرى لا حصر لها بنسبة واحد بالمائة وسوف يموت قنديل البحر. ففي نهاية المطاف، هذا هو بالضبط ادعاء المؤمنين بالتصميم الذكي. ويسمى بمبدأ الضبط الدقيق، ومثل كل ادعاءات المؤمنين، فهو ببساطة خطأ...

    ولكن في صلب الموضوع - دعونا نستبدل مشبك الورق بعلامة إعلانية لشركة كوكا كولا باللغة اليونانية. ما زلت تزعم أنك ستتفاجأ أكثر من قنديل البحر. دعونا نجري تغييرًا آخر في التجربة - سفينة الفضاء التي تهبط على الكوكب البعيد هي سفينة غير مأهولة، وبالطبع لم تطأ قدم أي إنسان هذا الكوكب.
    هل ستظل متفاجئًا أكثر من قنديل البحر؟

  583. "والسؤال هو كيف تم إنشاء كل الآليات المذكورة، ولا توجد إجابات حقيقية".
    ولديك الجواب "شخص صممها" والدليل غير العلمي هو "لأنها أكثر تعقيدا من الساعة"
    ولدي الجواب "تطور طبيعيا" والدليل غير العلمي هو "لأنه موجود في الطبيعة"
    وأكرر وأسأل لماذا إجابتك أنجح من إجابتي.

    لا تحاول صرف انتباهك عن الإجابة على ما يحدث في هذا المصنع أو ذاك، لأنه ليس له صلة بالموضوع. تمامًا كما لو أن الآلة تنتج حجرًا في المصنع، فلن تغير ادعاءك بأن "هناك عمليات طبيعية تنتج الحجارة". فحتى حسب رأيك، إذا اخترع شخص مصنعًا لإنتاج الكتاكيت، فهذا لا علاقة له بأصل الكتاكيت في البرية. إن مصنع إنتاج السيارات (والسيارة تختلف تمامًا عن الكتكوت، حتى لو كانت تتكاثر) لا علاقة لها بأصل الكتاكيت في البرية.

    أنا آسف، كلما كررت حجة تكرار السيارة أو عدم تكرارها، كلما بدا الأمر أكثر غباءً. تريد أن تقول أن الكتاكيت تشبه السيارات المقلدة لأنها معقدة، وأنا أقول أن الكتاكيت تشبه الحجارة لأنها موجودة في الطبيعة. خيالك هو لشيء غير موجود على الإطلاق. مخيلتي هي شيء يعرفه الجميع ويصادفه. عذرًا، كما قلت بطريقة غير علمية تمامًا، "من الواضح للجميع أن" ثم "من الواضح للجميع أن" الكتاكيت والحجارة والجداول والأشجار هي أشياء طبيعية، في حين أن "السيارة الافتراضية ذاتية التكاثر" هي بالضبط الشيء الذي ليس واضحا.

  584. عوفر، أنت الذي زعمت أنك تعلم أن الساعات تحتاج إلى مصمم لأنك تعلم أن الإنسان يصنع الساعات. وبهذا المعيار فإن السيارة المكررة لا تعتبر دليلا على التصميم.

    والآن أتيت بحجة جديدة – في حالة المسيحيين في الطبيعة. إن ادعائك يذكرني بادعاء كاميرون دياز، الذي بموجبه يعتبر نمو الطفل دليلاً على التطور. ومن الواضح أن هذا الادعاء خاطئ، لأن مئات الآليات فائقة التطور هي المسؤولة عن هذه العملية المعقدة. والسؤال هو كيف تم خلق كل الآليات المذكورة، ولا توجد إجابات حقيقية.

    أرى أيضًا سيارات يتم تصنيعها بنفسها في مصنع لتصنيع السيارات. برأيك هذا دليل على أن السيارة ليست نتاج تخطيط؟

    معجزات، بالطبع سأكون منبهرًا بقناديل البحر أكثر من المشبك. بالتأكيد عندما أستكشف كلاهما على المستوى الجزيئي وأرى كل تلك المئات من الإنزيمات وبراءات الاختراع تعمل بكفاءة وسرعة ستجعل رأسك يدور.

    شموليك، نكرر نفس الإدعاء مرة أخرى. كان معيارك هو أننا يجب أن نرى شخصًا يصنع سيارات لنعرف أن السيارة تحتاج إلى تصميم.

    تم دحض هذا الادعاء في رأيي بسهولة تامة: وفقًا للمعيار المذكور أعلاه، حيث أنني لم أر قط أي شخص يصنع طبقًا طائرًا باستخدام تكنولوجيا فضائية، إذا رأينا طبقًا طائرًا، فسيتم ادعاء أنه ليس دليلاً على التصميم. هل تعتقد أن الصحن الطائر ليس دليلاً على التصميم؟

    اسمحوا لي أن أقدم مطالبة أخرى. هل تعتقد أنه يمكن إنشاء سيارة عادية في غرفة مغلقة؟

  585. شموليك
    تتمثل فكرة دينيت في إظهار الحجة في فكرة المصمم الذكي. سنتفاجأ أكثر بمشبك الورق، على الرغم من أن تصميمه أبسط. لماذا؟ نظرًا لأن المشبك يبدو مصممًا بالفعل... فهو يتمتع ببنية هندسية بسيطة، ومصنوع من مادة متجانسة، وله وظيفة محددة - أي غرض محدد.
    الكائن الحي لا يبدو هكذا، ولا يبدو مصممًا.
    لقد قمت بتوسيع الصورة لإظهار أن تشبيه بالي بالساعة هو في الواقع خاطئ تمامًا. تبين أنه لا يوجد تخطيط في الطبيعة !!!

  586. علم الأحياء الرياضي,
    أنت حر في ابتكار أي لغة تريدها، لكنني أوضحت لك ما تعنيه كلمة التثبيت باللغة العبرية وما هي حدود مفهوم التثبيت باللغة العبرية. لقد شرحت لك الفرق بين التأكيد المبني على الملاحظة المباشرة والاستدلال (وهو ما تفعله) في المواضيع التي لا تعرف فيها جميع المعلمات. لقد أوضحت لك أن هناك قيودًا على الحث على شكل أكوام من التراب: وفقًا لك، إذا قام شخص ما بتخطيط وإنشاء كومة من التراب، فإن جميع أكوام التراب في العالم قد أقامها بالضرورة شخص وفي بينكما تمكنت من تفويت معنى معياري. يقدم المعيار الخاص بي ادعاءً إيجابيًا حول متى يمكنك التأكد من أنه عميل ذكي وليس متى يمكنك التأكد من أنه ليس عميلاً ذكيًا. والسبب في وجود اتجاه واحد فقط هو أن الاتجاه الإيجابي يتلقى رياحًا خلفية من الملاحظات بينما الاتجاه السلبي لا يفعل ذلك. حتى هنا نذهب مرة أخرى. أدعي أنني أستطيع أن أقول عن شيء ما أنه تم تصميمه وبنائه بواسطة وكيل ذكي إذا رأيت أنا، أو أي شخص نيابة عني، نفس العميل الذكي يصمم ويبني المكون المعني (الملاحظة - تمامًا كما فعلت مع Kdvaa). هل تدعي أنك رأيت، أو رأى أحد نيابة عنك، المصمم الذكي وهو يبني شيئًا ما؟
    إذا كنت توافق على أنه ليس لديك ملاحظة مباشرة، فإنك لم يسبق لك أن رأيت المصمم الذكي يبني الشوتون أو الطبيعة بأكملها، فكل ما لديك هو استنتاج والاستدلال ليس دليلاً.

    نسيم (فيدنت)، سؤال عظيم!
    يبدو مشبك الورق أبسط من شكل قنديل البحر، لكن أود أن أقول إنه قبل إنشاء مشبك الورق، كان لا بد من إنشاء أداة إنشاء مشبك الورق. هل الهدف من السؤال الإشارة إلى الانحدار اللانهائي في حل المصمم الذكي؟ أي أن المخطط الذكي لا يحل أي شيء لأنه يدعو إلى السؤال من الذي خلقه.

  587. علم الأحياء الرياضي
    لقد بنينا بالفعل آلات استنساخ بدرجة أو بأخرى. الإنسان خلق من الطبيعة، أرجو أن الآلات المذكورة خلقت من الطبيعة...
    أنت فخور جدًا بقدرة التكاثر البشري، لكنه نظام ضعيف جدًا، وهناك الكثير والكثير من المشاكل. وبدون مساعدة العلم، هناك وفيات هائلة أثناء الولادة، سواء الأطفال الرضع أو الأمهات.
    كتبت لك سابقاً، وكعادتك تتجاهل ما يصعب عليك، أنك لا ترى "التخطيط" في العالم - عندما أقصد التخطيط بالمعنى المقبول للكلمة.
    سألتك عن اختراع أشياء عن كوكب بعيد، فهل هذا أيضًا صعب عليك الإجابة عليه؟ اساعدك؟

  588. ". وفقًا لمعاييرك ومعايير شموليك" - لم أقدم أي معايير مع شموليك، أنا آسف.

    "إنها لحقيقة أن هناك عمليات طبيعية تصنع الحجارة، أما الساعة فلا"
    وهناك حقيقة أخرى وهي أن هناك عمليات طبيعية تخلق الحيوانات، والساعة لا تفعل ذلك - ففي كل يوم يتم إنشاء فراخ جديدة في الحظيرة، وأؤكد لك أن الدجاجة لم تخطط لأي دور فيها... لم يقم أحد بذلك رأيت أي شخص يخطط لفرخ وفي المقابل، يخطط الناس للساعات.

    لذا اخترت معيارًا غير محدد لـ "التعقيد"، وقلت إنه نظرًا لأن الساعة معقدة، والحياة "أكثر تعقيدًا بحوالي ألف مرة" فإن "أي شخص يمكنه أن يرى" أنها مصممة. ثم عندي معيار آخر (ليس معيار شموليك) - ما يخلق في الطبيعة يوميا ليس مخططا له وما ينتجه الإنسان مخطط له. يستطيع كل طفل أن يرى أن الكتاكيت والأحجار تنتمي إلى الطبيعة، ويتم خلقها هناك بشكل يومي، بينما الساعات تنتمي إلى مكان آخر. أثبت الآن لماذا يعتبر معيارك أكثر نجاحًا من معياري.

    معياري أيضًا ليس علميًا، لكنه أصح بكثير من معيارك.

  589. بيولوجيا هارتابارت:
    "برأيك، ما المدة التي تعتقد أن الطبيعة ستستغرقها لإنشاء سيارة مكررة؟" - إذا كنت تقصد استنساخ السيارة بواسطة الإنسان: فالطبيعة استغرقت حوالي 2 مليار و1900 سنة أخرى (إذا حسبت حسب التقويم الميلادي)

  590. عوفر،

    مرة أخرى، لم أرى أحداً يصمم سيارة طبق الأصل. وفقًا لمعيارك ومعيار شموليك، يجب أن نستنتج أن مثل هذه السيارة النظرية ليست دليلاً على التصميم؟

    الحجر ليس أكثر تعقيدا من الساعة. إنها لحقيقة أن هناك عمليات طبيعية تنتج الحجارة، لكن الساعة لا تفعل ذلك. ليس بالضرورة أن يكون المركب مشتقًا من العديد من المكونات، ولكنه أيضًا من تصميمها ووظيفتها وتعاونها مع المكونات الأخرى.

    شموليك،

    رأي بيهي حول الأصل المشترك معروف لي. ماذا تفعل عندما تشير الأدلة العلمية ضد ذلك.

    التأكيد هو، بالنسبة لي، دليل على بعض الادعاءات. وكما أستطيع أن أؤكد اليوم أن الأرض ليست مسطحة. لقد أثبت ادعائي من خلال تقديم دليل إيجابي للمصمم - محرك مكرر يدور حوالي 1000 مرة في الثانية. تحتوي على توقف وحساسات بتكنولوجيا لا يمكن إلا أن يحلم بها الإنسان. ولا تحتاج إلى معرفة المعلمات لتحديد أن المحرك يتطلب التخطيط.

    معجزات,

    "أعطني مليارات السنين وتريليونات المحاولات وسأقوم بتطوير أي شيء تريده"-

    قد تكون كذلك (على الأرجح لا، نظرًا لوجود العديد من القيود التقنية). عملية طبيعية ليست كذلك. هل حقاً تقارن ذكائك بنقص ذكاء التطور؟

    في رأيك - ما المدة التي تعتقد أن الطبيعة ستستغرقها لإنشاء سيارة مكررة؟

  591. علم الأحياء الرياضي
    أعطني مليارات السنين وتريليونات المحاولات وسأفتح لك أي شيء تريده 🙂

    لم تفهم شيئا مما قلته….. لماذا نفس الآلية التي وصفتها معقدة للغاية؟ كل شيء في الجسم الحي معقد للغاية.
    ما أقوله هو أن الشيء الذي نراه كبشر تخطيطًا لا وجود له في الطبيعة. أنا لا أقول أنه لا يوجد مخطط، بل أقول إننا لا نرى علامات على ما نسميه التخطيط.

    سأخبرك بمثل - ربما بعد ذلك سوف تفهم. رحلة علمية تذهب إلى كوكب بعيد. وبعد سنوات تصل المركبة الفضائية وتهبط على سطح الكوكب، وهي المرة الأولى التي يصل فيها الإنسان إلى هذا الكوكب البعيد. يكتشف هناك شيئين: مخلوق يشبه قنديل البحر يعيش في أعماق البحر... ومشبك ورق. ما الذي سيفاجئك أكثر؟ و لماذا؟

  592. علم الأحياء الرياضي,
    كان لدي العديد من النقاط التي أوضحت فيها لماذا لا ينبغي تدريس التصميم الذكي الذي لم تتناوله. على أية حال، تمكنت من فهم مستوى الحجج المستخدمة لمعارضة المخطط الذكي. ولكم أنتم مدعوون للاستماع إلى بيهي ورأيه في التطور، لتسوية الخط:
    http://www.youtube.com/watch?feature=player_detailpage&v=V_XN8s-zXx4#t=3602

    نظرًا لأن مبدأ التأكيد مهم جدًا بالنسبة لك، فإليك تفسيرًا آخر لعدم ارتباط التأكيد بالموضوع. التأكيد، كما يطلق عليه، يقوي ثقتك في النظرية لكنه لا يثبتها. في حالتك، لقد قررت أن كل شيء مخطط له وبالتالي تزداد ثقتك في المخطط الذكي، لكن هذا لا يثبت بأي حال من الأحوال وجود المخطط الذكي. اقرأ قليلاً عن الكلمة التي تستمر في استخدامها: http://he.wikipedia.org/wiki/%D7%90%D7%99%D7%A9%D7%95%D7%A9
    بالإضافة إلى ذلك، فإن الحجة القائلة بأن الإنسان يخطط لأشياء معقدة وبالتالي فإن الطبيعة، كونها معقدة، تتطلب مخططًا ذكيًا، هي في معظمها حجة وليست دليلاً على صحتها (والأهم من ذلك، أن هذه الحجة غير محددة بشكل جيد). لماذا تعتقد أن تقديم الحجة هو نفس إثباتها؟ وعلى أية حال فهو غير ضروري على الإطلاق، لأن الادعاء بوجود المصمم الذكي هو ادعاء لا سبيل إلى دحضه. لماذا تحتاج أكثر؟
    لقد أوضحت لك بالفعل أن التأكيد بالملاحظات "يثبت" و"يؤكد" بالفعل النظريات التي نتحكم فيها بجميع المعلمات، تمامًا كما هو الحال في مثال الحائط الخاص بي، لكن هذا المثال لا ينطبق عندما يتعلق الأمر بالنظريات التي لا تكون فيها جميع المعلمات المعلمات معروفة. اقرأ بعناية مرة أخرى رابط ويكيبيديا الذي أرسلته. بالمناسبة، إذا كنت قد تحدثت بالفعل عن الملاحظات، أي الأشياء التي نراها بشكل مختلف عن الاستدلال (وهي في الواقع حجتك، نظرًا لأنك تستنتج أن هناك مصممًا ذكيًا، فأنت فقط تخشى استخدام هذه الكلمة) إذن سأحاول مرة أخرى: أدعي أنني أستطيع أن أقول عن شيء تم تصميمه وبنائه بواسطة وكيل ذكي إذا رأيت أنا، أو أي شخص نيابة عني، نفس العميل الذكي يصمم ويبني المكون المعني (ولقد رأينا بالفعل العديد من المباني التي صممناها). هل تدعي أنك رأيت، أو رأى أحد نيابة عنك، المصمم الذكي وهو يبني شيئًا ما؟

  593. علم الأحياء الرياضي,
    "أنت تعلم أنه مصمم لأنه معقد للغاية" - ليس صحيحًا، أنت تعلم أنه مصمم لأن الناس صنعوه.

    الحجر البسيط أكثر تعقيدًا بكثير من الساعة، لأنه يتكون من مليارات الجزيئات مرتبة بترتيب محدد للغاية مما يجعله يتمتع بوزن معين، ولون معين، وصعوبة معينة، وقدرته الخاصة على الامتصاص الماء، والمغناطيسية الخاصة به، ودرجة حرارة الطلاء الخاصة به، وأكثر من ذلك بكثير. هل تدعي أن الحجر البسيط مصمم أيضًا؟

  594. شموليك، لقد تناولت بالفعل كل النقاط التي أثرتها وأضفت المزيد من نقاطي، حتى أنني دحضتها. لديك الحق في التفكير بشكل مختلف. كما أثبتت أنه لا توجد مشكلة في مبدأ التأكيد - إذ من الممكن تأكيد الادعاء بأن الأرض بيضاوية الشكل. وينطبق الشيء نفسه على الدليل الإيجابي للمخطط.

    لقد أضفت أيضًا أن التطور "حقيقة". وسألتك ماذا تقصد بقولك هذا. ولم أحصل على إجابة.

    نسيم، إذا كنت راقصًا، فأنت غير قادر على إنشاء مليون من براءات الاختراع الموجودة في العالم الحي (على سبيل المثال، حاول إنشاء آلية تخثر الدم الخاصة بك، دون الاعتماد على ما تراه في الطبيعة، سوف نراك)، كيف يمكنك تحديد أن التخطيط في الطبيعة ليس شيئا؟

    عوفر، الاستنتاج غير صحيح. إذا كانت أمامك ساعة، فأنت تعلم أنها مصممة لأنها معقدة للغاية، حتى بدون تحديد الاحتمالية الدقيقة. وإذا كان أمامك سفينة فضاء بداخلها ساعة. يشير استنتاج التصميم إلى أن المركبة الفضائية أكثر تعقيدًا. وينطبق الشيء نفسه على آليات الحياة، التي هي أكثر تعقيدا وتطورا من أي شيء حلم به الإنسان من قبل.
    .

  595. لقد كتبت: "ربما لا يكون من الممكن الإشارة إلى احتمال دقيق بأن هذه الكائنات تتطلب التخطيط. لكن من الواضح للجميع أن الروبوت أو الصحن الطائر دليل واضح على ذلك".

    أعتقد أن هذه الجملة تقول كل شيء. بشكل عام، دليلك هو "أن هذا واضح للجميع"، ثم تضيف وتقول ذلك نظرًا لأن الآليات الحية "أكثر تعقيدًا بحوالي 1000 مرة"، فهي أيضًا واضحة.

    ومن المفيد الاستفادة من "مبدأ التحقق" في مجالات أخرى. على سبيل المثال، "من الواضح للجميع أن" 91 هو عدد أولي لأنه يمكن لأي شخص أن يرى الأسبقية فيه. الرقم 88119 أكبر منه بحوالي 1000 مرة، لذلك لا بد أن يكون أوليًا. بالفعل علمية.

  596. علم الأحياء الرياضي
    لذلك دعونا نصل إلى وجهة نظرك.
    اسمع، أنا مهندس. أنا أقدر التخطيط عندما يكون التخطيط بسيطًا. هناك أقلية (نسبيًا) من الأجزاء المنفصلة. كل جزء متجانس - يؤدي وظيفة واحدة فقط بشكل جيد. أحب رؤية الخطوط المستقيمة والدوائر الجميلة. أحب أن أرى قطعًا متجانسة ذات هندسة بسيطة. إذا تم استخدام نفس الجزء أكثر من مرة في التصميم، فأنا أريد أن تكون جميع مثيلاته هي نفسها.

    انظر إلى جسد حي. إنها في الواقع لا تبدو كنتيجة للتخطيط... بل تبدو أشبه بمجموعة من التصحيحات، نتيجة لعدد لا يحصى من التجارب التي تم إجراؤها على مدى فترة طويلة من الزمن. يشبه إلى حد ما نظام التشغيل Windows - :)…….

  597. علم الأحياء الرياضي
    أنا أفهم أنه ليس لديك إجابة بالنسبة لي
    لذلك سوف نكتب واحد صفر لصالحي.

  598. علم الأحياء الرياضي,
    أنا آسف، لكنك تجاهلت كل ما هو مكتوب هنا. لقد أوضحت أنه لا يوجد شيء اسمه مبدأ التأكيد. شرحت أن كل ما لديك هو المطالبة. لقد شرحت لك أنك تستوعب مفاهيم غير محددة وتبني عليها نظرية غير علمية كاملة. لقد أوضحت لك لماذا لا يمكن دحض هذه النظرية ولماذا لا تقول شيئًا في الواقع وتتظاهر في الواقع، وأعطتك مثالاً عن سبب استمرار الناس في البحث عن طرق طبيعية إضافية لشرح الانتقاء الطبيعي.

  599. هذه المرة سأكون موجزا حقا.

    تدور معظم المناقشات حول التعقيد الذي لا ينفصم، وعلم التصميم الذكي، وما إذا كان هناك دليل على أن العالم مصمم.

    وبينت أن النظرية يمكن اعتبارها علميا أيضا وفقا لمبدأ التأكيد، لأنها تقدم اختبارا للتحقق من صحتها (مثال آخر - إذا رجعنا إلى الأيام التي كانوا يعتقدون فيها أن العالم مسطح، فإننا نكتشف أدلة تراكمية) أنه ليس كذلك، حتى يتم تأكيد هذه النظرية أخيرًا). وهذا على النقيض من نظرية التطور التي لا تقدم أي اختبار لأي شيء.

    قدمت كدليل على تصميم سيارة مكررة. أو حتى طبق طائر بتكنولوجيا غير معروفة. إن الأشياء الرئيسية التي تثبت التخطيط من هذه الأشياء هي مكوناتها المتعددة، وعمل مكوناتها معًا، والتصميم المرئي في تركيبها. نفس التصميم موجود حوالي ألف مرة في الآليات الحية (كما ذكرنا - شوتون وهو محرك فائق التطور يمكن أن يصل إلى 1000 دورة في الثانية، أو إذا اتبعت النهج المادي، حتى القرد كروبوت عضوي مكرر). قد لا يكون من الممكن الإشارة إلى احتمال دقيق بأن هذه الكائنات تتطلب التخطيط. ولكن من الواضح للجميع أن الروبوت أو الصحن الطائر هو دليل واضح.

    أولئك الذين يريدون الدفاع عن نهج طبيعي، يستمتعون. ليس من قبيل الصدفة أن غالبية العلماء يختلفون معك. بسبب الاحتمالات الواضحة التي كنت أتحدث عنها (إذا أردت أن تبدأ نقاشاً حول احتمال حدوث هذه الطفرة أو تلك، استمتع بوقتك).

  600. علم الأحياء الرياضي,

    1. أين أرسلتك لدحض الانتقاء الطبيعي؟ كتبت أنني لا أريد العبث بها وكتبت أنه إذا كنت تريد، تحقق من ويكيبيديا. أكرر وأكتب مرارا وتكرارا أنني لم آت للدفاع عن التطور، من بين أمور أخرى، لأنني لست في وضع يسمح لي بذلك. لقد جئت للدفاع عن قرار عدم تدريس التصميم الذكي في دروس العلوم وشرحت السبب. يبدأ الشرح وينتهي بالمخطط الذكي، وذلك لأنه لا يوجد ما يسمى "التخطيط الذكي" كنظرية علمية، لأن عبارة "التخطيط الذكي" تصف الخدع التي قام بها المخطط الذكي. فالمصمم الذكي عامل خارج عن الطبيعة ولا يمكن دحضه. يمكنك دائمًا القول بأنه في الممارسة العملية تم تصميم كل شيء بواسطته، حتى لو كان التطور قادرًا على إنتاج مخلوقات جديدة. وكما أوضحت، فإن حقيقة إمكانية الوصول إلى حيفا عبر الطريق رقم 2 لا تعني عمليًا أنني لم أصل إلى حيفا عبر الطريق رقم 4. يمكنك دائمًا الادعاء بأن المصمم الذكي خلق جميع المخلوقات في وقت واحد، بحيث تظهر نحن الناس من مختلف الأعمار. يمكنك دائمًا الادعاء بأننا خلقنا بواسطة المصمم الذكي، منذ 4 ثوانٍ، مع ذكرى تاريخنا بأكمله. כפועל יוצא מכך, לא ניתן לדבר על התכנון התבוני כעל תיאוריה מדעית כי היא לא עומדת בפני עצמה ואין לה קיום במנותק מהמתכנן התבוני (בדיוק כפי שלא ניתן לדבר על תהליך בניית הלגו, בידי הבן שלי, כתכנון תבוני. התכנון לא עשה דבר אלא הבן שלי القيام به).

    2. انتبه إلى صياغة حجتك: الأشياء من تصميم الإنسان. الإنسان مخلوق ذكي. ولهذا السبب، فإن أي شيء يبدو لي أنه مخطط له، بالضرورة، يجب أن يتم التخطيط له بواسطة عميل ذكي. أولا، هذه حجة وليست دليلا على صحة الحجة. يجب عليك إثبات هذه الحجة وفي اللغة اليومية يسمى هذا استدلالًا وليس دليلاً.

    3. حجتك استقرائية ويجب على المرء دائمًا أن يكون حذرًا في مثل هذه الحجج. انظر: رجل يبني أكوامًا من الرمل. ولهذا السبب، كان لا بد من بناء كل تل رملي بواسطة الإنسان. ومن الواضح أن هذا ليس هو الحال.

    4. يتم حل مسألة الكائنات الفضائية والغوميم وكل شيء بسهولة: الإجابة هي أننا نحتاج حقًا إلى فحص كل حالة على حدة، وفقًا للأدلة ومحاولة فهم ما تم إنشاؤه عشوائيًا بطبيعته (كعامل أعمى) و ما صممه عامل مقصود. وجهة نظري هي أنني لست سريعًا في الحكم وأنت كذلك. أنا لا أتحدث عن المطلقات، نوع من المبدأ الذي سيصف ما يتم إعداده وما لا يتم إعداده، وأنت كذلك. ويكمن جزء من المشكلة في أن مصطلحي "مصمم" و"معقد" غير محددين بشكل جيد. نحن نفهم مفهوم "المصمم" فقط على أنه شيء يفعله الإنسان (وربما بعض الحيوانات أيضًا)، لذا تقف حجتي وتنتبه إلى ما تقوله وما لا تقوله. إن ادعائي لا يعني أن الأشياء التي ليس لدينا معلومات عنها لم يتم تصميمها بالضرورة بواسطة كائن ذكي (وهذا، بالمناسبة، هو على وجه التحديد الحجة ضد التصميم الذكي. فحتى لو كان التطور قادرًا على إثبات بناء الهياكل المعقدة، فهذا لا يعني أنها لم تكن مصممة من قبل وكيل ذكي). وهنا حجتي مرة أخرى: لكي أتمكن من القول بأنني أثبت وجود تخطيط ذكي، أدعي أنني أستطيع أن أقول عن شيء تم التخطيط له وبنائه بواسطة عميل ذكي إذا كنت أنا أو أي شخص ينوب عني، رأيت نفس العميل الذكي يصمم ويبني المكون المعني (ولقد رأينا بالفعل العديد من المباني المخطط لها من قبلنا). هل تدعي أنك رأيت، أو رأى أحد نيابة عنك، المصمم الذكي وهو يبني شيئًا ما؟

    5. يقع عبء الإثبات فيما يتعلق بوجود التعقيد الذي لا ينفصم على عاتقك وحدك. حاول ميلر أن يوضح أن شوتون يتكون من هياكل أبسط وتعتقد أنك نجحت في دحض ادعائه. جيدة بالنسبة لك. على الأكثر، هذا يعني أنه فشل في تفسير تفريغ القضيب، لكنه لا يقول شيئًا عما إذا كان القضيب غير قابل للتدمير حقًا. إن عبء إثبات أنه لا توجد طريقة في العالم لإنتاج Shoton يقع على عاتقك وحدك. إذا أظهرت أن خلق شوتون يتعارض مع قوانين الحفظ، ويتعارض مع القانون الثاني للديناميكا الحرارية، بالإضافة إلى حصوله على جائزة نوبل، فإنه سيخلق صداعًا حقيقيًا لدعاة التطور. أبعد من ذلك، فإن موضوع التعقيد الذي لا ينفصم، إذا كان هناك شيء من هذا القبيل، سوف ينتمي إلى نظرية طبيعية أخرى يجب أن تقدم تفسيرا والقفزة التي إذا كان هناك تعقيد لا ينفصم لمصمم ذكي غير صحيحة. على سبيل المثال، يزعم ستيفان ولفرام أنه في ظل الانتقاء الطبيعي يوجد مبدأ أكثر أساسية ويناقش مسألة التعقيد. لم أتمكن حقًا من معرفة ما يدور حوله وأعتقد أن معظم الخبراء يعارضون ادعائه ولكن هنا مثال على كيفية وجود أولئك الذين يحاولون البحث عن تفسيرات طبيعية ولا يقفزون مباشرة إلى الله. تفضل بالدخول http://www.kurzweilai.net/reflections-on-stephen-wolfram-s-a-new-kind-of-science
    ابحث عن الكلمات الانتقاء الطبيعي هناك.

    6. لا توجد مشكلة في الطريقة التي شرحت بها عندما يمكن تأكيد النظريات. وفي النهاية، يعود الأمر كله إلى القيود اللغوية. إن كلمة تأكيد وإثبات وما إلى ذلك هي كلمات يومية ومطلقة ويمكن استخدامها بحرية فقط في الأمور اليومية (وبالتالي فإن كلمة تأكيد تناسب الرابط الذي قدمته أو مثال الاصطدام بالحائط). كمبدأ، ليس من الممكن إثبات نظرية في الطبيعة (على عكس النظرية الرياضية)، ولكن فقط لتعزيز ثقتنا بها وتطوير التقنيات القائمة عليها.

  601. علم الأحياء الرياضي
    ادعاءك بشأن المعقد يسقط على الفور لأنك تفترض شيئًا أكثر تعقيدًا. أنتم تخلقون تراجعاً، وهو لا يتقدمنا ​​في شيء.

    فيما يتعلق بادعائي - أنت تسحب إجابة دون أي دليل. لإعطاء حجة احتمالية تحتاج إلى إعطاء الاحتمالات ..........
    إجابتك على مستوى "لماذا؟" هكذا!".
    أنت تتحدث عن "علماء التطور" - هل هناك علماء يرفضون التطور؟

    لدي عدة نقاط أخرى حول هذا الموضوع، ولكن دعونا نركز على نقطة واحدة فقط. النقطة المهمة هي "الحجة الاحتمالية". إذا لم تتمكن من إعطاء أرقام فإن هذه الحجة ستفشل. إنها لا تثبت أو تدحض أي شيء، ولكنها حجة أقل يمكن استخدامها.

  602. السيد المحترم علم الأحياء الرياضي
    و. من الجيد أنك تمتص
    ب. في طريق العودة، كان لدي الوقت للقاء سيارة مماثلة لسيارتي (نسخة طبق الأصل)، ما علاقة ذلك بما كتبته، لم يكن لدي القوة للقراءة

  603. حسنًا، سأحاول التلخيص لأن المناقشة أصبحت رتيبة بعض الشيء.

    إن برهاني على وجود مخطط عالمي يعتمد على تعقيد السيارة النظرية ذاتية التكرار.

    أما ادعائك فيعتمد على أننا نعرف مسبقاً أن الإنسان يخطط للسيارات، ومن هذا يجب أن نستنتج أن السيارة مطلوبة للتخطيط، وهذا الادعاء مرفوض في نظري من عدة نقاط. الشعور حر في دحضه:

    أ) وفقًا لهذا المنطق، إذا رأينا مثلاً طبقًا طائرًا أو سفينة فضائية مزودة بتكنولوجيا كائنات فضائية لم نر صانعيها من قبل، فيجب أن نستنتج أنها خلقت من تلقاء نفسها.
    ب) رأينا أيضًا بشرًا يصنعون غمازات. لذلك، وفقًا للمعيار المذكور أعلاه، سيتعين علينا أن ندعي أن الجينومات في الطبيعة قد تم تصميمها
    ج) لم يسبق لي أن رأيت سيارة طبق الأصل. كما أنك لم تقدم دليلاً على وجود مثل هذه السيارة. لذا مرة أخرى، وفقًا للمعيار المذكور أعلاه، فإن السيارة المكررة ليست دليلاً على التصميم. إلا إذا كنت تقصد أن مجرد وجود السيارة هو أساس صالح لتكرار السيارات أيضًا. هل هذا ما تعنيه؟

    لقد أرسلتني للبحث عن محاولة لدحض الانتقاء الطبيعي، حتى عندما كنت أتحدث عن ادعاء الأصل المشترك. ليس لها علاقة بالموضوع.

    الآن أنت تدعي أن التطور "حقيقة". هل تقصد ادعاء الأصل المشترك الذي ليس له دليل واحد؟ أم تقصد تعريف التطور بأنه تغير في تواتر الأليلات (وهي حقيقة ملحوظة وصحيحة حتى لو تم خلق جميع الأنواع دفعة واحدة بواسطة مصمم).

    "أو كيف يتعامل خبراء التطور مع شوتون"-

    تقصد خبراء مثل ميلر؟ بعد كل شيء، لقد دحضت ادعاءاته، فلماذا الاستمرار في الوثوق به وأعين مغلقة؟

    "في القضايا الصغيرة نقوم بتطوير النظريات وبالطبع نؤكد هذه النظريات لأنفسنا، ولكن ليس في القضايا الكبيرة. "-

    والآن تقترح التقسيم التعسفي إلى "قضايا كبيرة" و"قضايا صغيرة"؟ النظرية هي نظرية، فترة. لا يمكنك اللعب بها. ولا يهم أيضًا. أي نظرية قابلة للاختبار، بحكم تعريفها، تعتبر علمية لأنها توفر طريقة لاختبارها. يمكنني على سبيل المثال أن أدعي أن وحش بحيرة لوخ نيس يمكن أن يكون نظريًا نوعًا من البليزوصورات. ويمكنني أيضًا أن أقترح طريقة لاختبار هذا الادعاء - على سبيل المثال: تجفيف البحيرة أو نشر شبكة ضخمة على طول البحيرة، وما إلى ذلك.

    فيما يتعلق بالتعقيد الذي لا ينفصم. يدعي خبراء التطور أنه يمكن إثبات أن هذا التفسير لا أساس له من الصحة. ومن ناحية أخرى، دحضت تفسيرهم بسهولة تامة. لذا فإن عبء الإثبات يقع الآن عليهم وليس على عاتقي. وعلى العموم فإن عبء الإثبات يقع على من يدعي أي ادعاء. ومن الغريب أنك لا تقترح على خبراء التطور أن يدعموا ادعائهم بشأن الأصل المشترك لجميع الكائنات الحية. بالنسبة لك، هل التطور خالي من أي عبء إثبات؟

    2″. إذا كان هناك تعقيد لا ينفصم، فسوف ينتمي إلى النظرية الطبيعية التي سيتعين عليها التعامل مع هذه الحقيقة.

    وبما أن التطور في جوهره ليس نظرية طبيعية (مرة أخرى، وإلا فلن تكون هناك حاجة لطريقة السقالات أو سيكون من الممكن الادعاء بأن الأنواع تطورت في مكة))، فإن وجود بعض التعقيد الذي لا ينفصم يشكل مشكلة إحصائية/تقنية. دحض نظرية التطور .

    لم تظهر تجربة لينسكي أي تغيير ملحوظ. ما يوضحه هو ميزة جديدة. ولهذا ليست هناك حاجة حتى للذهاب إلى مختبر لانسكي. تظهر ميزات جديدة حتى في الكائنات غير الحية. فالسيارة على سبيل المثال سوف يتغير لونها إذا تركتها في الشمس لفترة طويلة.

    "لذا، إذا كان ادعاءك صحيحًا، وفشل العلماء مرارًا وتكرارًا في إرضائك، فإن ذلك يفسر تمامًا الصعوبة الهائلة في إثبات التطور في المختبر،"-

    ليست مشكلتي. إما أن يكون التطور قابلاً للاختبار أم لا. وإذا لم يكن الأمر كذلك - فإنهم لا يدرسونها كعلم في دروس العلوم. تماما كما تدعي عن التصميم الذكي.

    معجزات,

    إن احتمال وجود خالق ذكي يعادل على الأقل احتمال أن السيارة ذاتية التكرار تحتاج إلى مصمم. وبالمناسبة، فإن علماء التطور أنفسهم لا يقبلون أي احتمالية على الإطلاق. وإلا لن تكون هناك حاجة للتطور.

  604. علم الأحياء الرياضي
    أنت تكتب لفترة طويلة جدًا ويصعب متابعتها. يمكنك أيضًا مزج الكثير من الأشياء معًا، أشياء من مجالات مختلفة.
    سأقدم مطالبة واحدة بسيطة ويرجى محاولة فهمها.

    والحجة هي أنه بما أنه لا يمكن عزو أي احتمال لوجود خالق، فلا يمكن رفض التطور بسبب "احتماله المنخفض".

    لدحض ادعائي عليك أن تعطيني رقما يشير إلى احتمال وجود خالق (في لحظة الخلق بالطبع، وليس في الوقت الحاضر) نظرا للكون كما نراه.

    أتمنى أن تكون حجتي واضحة.... أنا فقط أقول - طالما أنك لا تملك الاحتمالية لتعطيني، فلا يوجد ما تقوله "غير محتمل".
    نعم؟

  605. علم الأحياء الرياضي,
    نحن نتحرك هنا على طائرتين متوازيتين، في إحداهما من المؤكد أنك قد قدمت حجة رابحة في شكل سيارة مكررة (إذا كنت تريد، أعد كتابة الحجة لأنني لا أستطيع أن أفهم لماذا لا تفهم وجهة نظري دحضًا) بينما في الآخر دحضته مرارًا وتكرارًا. على مستواي، لقد سبق أن شرحت لك خطأك وكتبت بالفعل أنني لم آت للدفاع عن التطور وإذا كنت ترغب في ذلك، فإن البحث في ويكيبيديا والإنترنت سوف يجيب على سؤال كيفية دحض الانتقاء الطبيعي ( التطور في حد ذاته حقيقة، بقدر ما يمكن الحديث عن الحقائق في الطبيعة. نقل الجينات الأفقي هو مجرد مثال واحد على ذلك) أو كيف يعامل خبراء التطور شوتون، لكن هذا لا يهمني حقًا لأنه خلال هذه المناقشة أنا فقط دفاعاً عن قرار الرئيس بعدم السماح بدراسة التصميم الذكي في دروس العلوم ومن أجل محاولة التقارب، لن أرد إلا على ما يروج لهذه القضية مع الإشارة إلى بعض ما كتبته.
    أول ما يهمني أن أتطرق إليه هو مبدأ التأكيد الخادع، الذي تبني عليه نظرتك للعالم بالكامل، ولذلك سأعود مرة أخرى: لا يوجد شيء اسمه تأكيد نظرية والادعاء بإثباتها. في القضايا الصغيرة نقوم بتطوير النظريات وبالطبع نؤكد هذه النظريات لأنفسنا، ولكن ليس في القضايا الكبيرة. على سبيل المثال، إذا صادفت شيئًا ما في الليل، فإنني أفترض أنه جدار، وبعد ذلك، لإثبات نظريتي، أقوم بتشغيل الضوء. بعد أن قمت بتشغيل الضوء، تم إثبات نظريتي (أو دحضها) ويمكنني تأكيدها من خلال محاولة إعادة إنتاج الضرر، من خلال التحقق مرة أخرى مما إذا كان هناك أي شيء آخر يمكن أن أواجهه وينطبق الشيء نفسه على الرابط الذي قدمته لي . ابتكر العلماء نظرية صغيرة لأنفسهم وأكدوها. في القضايا الكبرى، تلك التي تصف حدود معرفتنا، لا يمكننا إلا أن نعزز ثقتنا في النظرية من خلال ملاحظات إضافية ولكن لا يمكننا أبدًا أن نقول إننا أثبتنا نظرياتنا، وفي الواقع، فإننا نبدي ملاحظات إضافية على أمل أن نتمكن من ذلك. يمكن أن نكتشف معلومات جديدة تتعارض مع معرفتنا الحالية وتكشف عن قطعة أخرى من اللغز وتدفعنا إلى الأمام. ولهذا السبب نقوم ببناء مسرعات أقوى، لاختبار متى تنهار نظرياتنا الحالية. من الواضح أنه من أجل تطوير التقنيات، ليست هناك حاجة لانتظار الدليل (الذي لن يأتي). بالنسبة لبناء المباني، نيوتن جيد بما فيه الكفاية. الفرق بين المواضيع السهلة والموضوعات الكبيرة هو أنه في أحد المواضيع تكون جميع المعلمات معروفة (أو الأغلبية المطلقة) ومحددة جيدًا وفي الآخر لا، في الواقع لا. في القضايا الكبرى، لن نعرف أبدًا ماذا سينتج يومًا وما هي المعلومات الجديدة التي سيتم اكتشافها، وهذا هو السبب في أن النظريات العلمية هي تلك التي بطبيعتها لا يمكننا إثباتها أبدًا، بل ننفيها فقط حتى نتمكن من ذلك. ولتسميتها نظريات علمية، يجب أن توفر القدرة على دحضها. المخطط الذكي لا يسمح بذلك. وبما أنه لا يوجد وجود مستقل للتخطيط الذكي - وهو الوصف الوظيفي للمخطط الذكي - فإن التخطيط الذكي غير صالح أيضًا للاستخدام في دروس العلوم.
    وبالمناسبة، فإن اسم التخطيط الذكي مضلل أيضًا. لا يوجد تصميم ذكي هنا لأنه إذا كان المصمم الذكي قد خلق كل شيء، فهو ببساطة قرر أن الأمر سيكون كذلك. وليس كأنه جلس في غرفة بها جهاز كمبيوتر وذكاء مفعل، لأن الذكاء مفهوم للوجود الإنساني ولا يمكن فرض هذا المفهوم على المخطط.

    1. يقع عبء إثبات وجود أنظمة ذات تعقيد لا ينفصم على عاتق من يدعي أن هناك تعقيدًا لا ينفصم. وكما كتبت بالفعل، ليس من الممكن إثبات كيف تم إنشاؤه في أي بنية. لا يمكن للمرء إلا أن يتكهن بكيفية تشكله، ولكن عدم القدرة على شرح كيفية تشكل أي هيكل بالتفصيل لا يمثل مشكلة. إذا رأيت بركة من الماء على الأرض، هل يمكنك معرفة كيف تبدو كتلة الجليد التي ذابت لتشكل البركة؟ هل حقيقة عدم قدرتك على ادعاء وجود كتلة من الجليد هناك أمر غير قابل للتصديق؟ بالطبع لا. وهذا ما تفتقده مرارًا وتكرارًا. التطور لا يناقض وجود الله، لكنه يقول أنه ليس هناك حاجة إليه. في هذا السياق، يجب أن أشير إلى أنني لم أفهم حجتك حول سبب وجود عبء الإثبات على أن هناك هياكل معقدة بشكل لا ينفصم على أنصار التطور.

    2. إذا كان هناك تعقيد لا ينفصم، فسوف ينتمي إلى النظرية الطبيعية التي سيتعين عليها التعامل مع هذه الحقيقة.

    3. حقيقة أننا صممنا الأشياء لا تعني أن ما تراه في الطبيعة قد صممه بالضرورة كيان خارج الطبيعة، لأننا صممنا الأشياء. وهذه الحجة في أغلب الأحوال حجة وليست دليلاً على صحتها، ومشكلتها أنني لا أستطيع تفنيدها. وحتى لو تبين في المختبر، بطبيعة الحال، أن الشوتون يمكن أن يتشكل من تلقاء نفسه، فإن هذا لا يعني أنه من الناحية العملية لم يخلقه عامل خارجي. حقيقة أنه يمكنك الوصول إلى حيفا من الطريق رقم 4 لا تستبعد إمكانية وصولي بالفعل عبر الطريق رقم 2.
    وأنت غبي أيضاً عندما تصر على عدم فهم ما أقوله عن السيارة. أنت تعرف بالضبط من يصنع ويصنع السيارات، ومن العار أن تكون مدعيًا. وبما أنك تعرف بالضبط من يصمم ويصنع السيارات، فيمكنك القول أنه عندما ترى سيارة في الشارع، فهذا يعني أن شخصًا ذكيًا هو من صممها وصنعها. لا يجوز لك أن تستخدم هذه الحجة في الطبيعة، لأننا لم نر كيف خلقت، فسؤالي هو: أستطيع أن أقول عن شيء أنه صممه وبناه عاقل عاقل إذا رأيت أنا أو من ينوب عني يقوم ذلك العميل الذكي بتصميم وبناء المكون المعني (في الواقع رأينا الكثير من المباني التي صممناها). هل تدعي أنك رأيت المصمم الذكي يبني شيئًا ما؟
    4. الشيء الذي أود أن أتناوله هو تجربة Lenski التي ذكرتها وأنا على دراية بها بشكل فضفاض. التجربة تثبت أنه بالتأكيد حدثت تغيرات لا تصدق بين الأجيال المختلفة، ربما هذا لا يرضيك (لأنك وحدك من تحدد ما هي العملة الأساسية التي تعتبر تغييرا ولا يسمح لي، ما يعتبر معقدا ولا أنا) مسموح به) ولكني أريد تسليط الضوء على جوانب أخرى. النقطة الأولى هي أنك سترى مدى صعوبة إجراء مثل هذه التجربة التي خلقت 45 ألف جيل. بحاجة إلى تمويل لمدة 20 عاما من العمل. انظر إلى مدى صعوبة العمل لفهم التسلسل الجيني الذي تم إنشاؤه هناك، ولأنه من الصعب جدًا التقاط مثل هذه التجارب، لم يكن هناك سوى تجربة واحدة مثل هذه (ربما هناك تجارب أخرى، ولكن بالتأكيد يمكنك عدها على يد واحدة) فإذا كان ادعائك صحيحًا، وفشل العلماء مرارًا وتكرارًا في رغبتهم فيك، فإن هذا يفسر تمامًا الصعوبة الهائلة في إثبات التطور في المختبر، بطريقة (لا) تأمل في رؤيتها. النقطة الثانية هي أنك ستلاحظ مدى إشكالية التجربة المبهرة في حد ذاتها، لأنها لا تحاكي الطبيعة على الإطلاق، لأنه على سبيل المثال، جميع العينات موجودة في جرار، مما يعزلها عن البيئة، وعن الضغوط المختلفة، وعن البيئة. الإشعاع الكوني الناتج عن التفاعلات الهائلة مع البكتيريا الأخرى. كل هذا يمكن أن يفسر سبب عدم حدوث ما كنت تريد رؤيته (ليس كثيرًا)، ولكن مما لدي انطباع، يتبنى خبراء التطور هذا البحث ويؤكد 🙂 الكثير من الأفكار.

    حاولت أن أقصره ولم أنجح وأنا آسف قليلاً لذلك لأنه مرة أخرى ستفوت نقاط كلامي، ومرة ​​أخرى سنستمتع باختصار:
    لا يوجد شيء اسمه تخطيط ذكي لأن هذه الكلمات في مجملها تعبر عن عمل المخطط الذكي (حسب نظرتك للعالم) وهو لم يفعل شيئاً ذكياً (وهي كلمة تنسب أساساً إلى الجنس البشري) بل قرر ببساطة أن هكذا ستكون الأمور. دع النور يعبر إلى هناك. لا يوجد شيء هنا للتدريس في المدارس، في دروس العلوم.
    إن عبء إثبات التعقيد غير القابل للاختزال يقع على عاتقك وحدك. إذا كان هناك تعقيد لا ينفصم، فهذا شيء يجب على النظريات الطبيعية التعامل معه
    الادعاء ليس دليلاً على صحة الادعاء، وبالتالي فإن الادعاء بأن شيئًا ما يبدو أنه قد تم التخطيط له بواسطة وكيل ذكي (لأن البشر أيضًا يخططون للأشياء) هو في الغالب ادعاء ولا يمكن الجدال فيه.

  606. أهلا بك مرة أخرى شموليك،

    قلت: "حجتك هي كما يلي: هناك أشياء لا يستطيع التطور تفسيرها بسبب احتماليتها المنخفضة (وهي في حد ذاتها حجة غير مرضية لأن من قال إن الأحداث غير المحتملة لا يمكن أن توجد) ويعزو التعقيد الذي لا ينفصم إلى تلك الهياكل التي تعتقد أن لها احتمال منخفض جدا. عند هذه النقطة ترفعون أيديكم وتقولون: إذًا، من الواضح بالنسبة لي أن هناك عاملًا خارج الطبيعة، هو الذي صمم هذه الهياكل. أنا أعارض هذه القفزة."-

    وأنصار التطور أنفسهم يعارضون اعتراضك على حد علمي. لماذا تعتقد أن داروين نفسه كتب أنه إذا أثبت شخص ما أن بعض الأعضاء لا يمكن أن تتطور تدريجياً فسيتم دحض النظرية؟ ووفقاً للمعيار أعلاه – فحتى القطة التي خلقت في مكة أمام أعيننا لن تكون دليلاً على وجود مخطط في نظرك. كل شيء ممكن.

    "أنت تقول أن هناك فجوة في التطور وتملأها على الفور بالله. كيف لا ترى أن هذه حجة إله الفجوة، من الصعب بالنسبة لي أن أفهمها وهذه هي الإجابة العامة على سؤالك المكرر عن السيارة"-

    عدة اشياء:

    أ) كل التطور مليء بالثغرات. في كل مجال ممكن تقريبا.
    ب) تعتمد حجة الفجوات على "لا نعرف بعد". حيث قدمت دليلاً إيجابيًا على تكرار تصميم السيارة. ولم تدحضه بعد، كما سنرى لاحقا.

    "حتى الآن كنا دائمًا قادرين على تفسير كل ظاهرة نراها باستخدام التفسيرات الطبيعية وليس هناك سبب لعدم القيام بذلك الآن، على الرغم من أن الإجماع العام يدعي أننا فعلنا ذلك بالضبط" -

    لقد قدمت عددا من الأسباب. حيث يبدو أنك الآن تتجه إلى الإجماع، وبما أن هذا الإجماع يمثله نفس العالم من الفيديو السابق. وبما أنني رددت دعواه وبينت مغالطاته فليس في دعوى الإجماع شيء على الإطلاق.

    "ليس هناك شيء اسمه مبدأ التأكيد، ولا توجد نظرية في العلوم الطبيعية تم تأكيدها على الإطلاق"-

    ما هي العلاقة الآن بالعلوم الطبيعية؟ نحن نتحدث عن نظرية علمية. والنظرية القادرة على توفير طريقة لإثبات ذلك (على سبيل المثال عن طريق التجربة، كما يدعي أنصار التطور فيما يتعلق بالتطور نفسه)، هي نظرية علمية بالتأكيد. لا تظن ذلك؟

    "لا يمكنك إثبات التعقيد الذي لا ينفصم، ولن تنجح محاولة نقل عبء الإثبات."-

    وقد أثبت ذلك بالفعل ولم أتلق منك إجابة على هذا الإثبات. أسأل مرة أخرى - هل تعتقد أنه سيكون من الممكن في المستقبل إنشاء سيارة تدريجياً عندما تكون كل خطوة فعالة في حد ذاتها في الطريق إلى السيارة؟ وإذا لم يكن الأمر كذلك، فلماذا تعتقد أن مثل هذا المسار موجود في التطور؟

    "إن واجب الإثبات، بأن هناك هياكل لا يمكن إنشاؤها من تلقاء نفسها تحت أي ظرف من الظروف، يقع على عاتقك وحدك، لأنك زعمت أن هناك هيكلًا غير قابل للتدمير ويجب على المدعي إثبات ذلك".

    يزعم أنصار التطور أيضًا أن الأنظمة غير القابلة للتحلل *نعم* يمكن أن تتطور تدريجيًا. ورأينا طبيعة الأدلة على ذلك في الفيديو السابق. إذا كان الأمر كذلك، في رأيك، فحتى مؤيدو التطور لديهم عبء الإثبات الخاص بهم.

    "تمامًا مثل طلب بيهي الذي نقوم بإنشائه في مختبر شوتون، والذي ندعي أن تكوينه استغرق ملايين السنين. كيف بالضبط؟ أخبرني من أين أبدأ؟"

    لا بد أنك سمعت عن تجربة لانسكي. إذا كانت كل خطوة في قضيب أو أي نظام معقد آخر لديها فرصة واحدة في المليار لهذه المسألة، فيجب أن نرى بسهولة البكتيريا أو حتى مجرد تسلسلات عشوائية في أنبوب اختبار تتطور تدريجيًا إلى نظام معقد، حيث تكون كل خطوة فعالة شيء آخر. في الواقع، العلماء يفعلون ذلك بالضبط، ويفشلون مرارًا وتكرارًا.

    "يدعي الخبراء أن التطور يقدم تفسيرًا عامًا يشرح كيف يمكن أن يكون كل هيكل نراه قد تم إنشاؤه"-

    هل تقصد هؤلاء الخبراء مثل ميلر؟

    "لكن من المؤكد أن عدم القدرة على وصف كيفية خلقه بالضبط لا ينفع معه لأنه من المستحيل إعادة بناء كل الشروط والقيود"-

    إذن التطور ليس علمًا لأنه لا يمكن فحصه. ولذلك لا ينبغي تدريسها في المدارس. هل تتفق مع هذا البيان؟ لا يوجد سبب لعدم القيام بذلك.

    "بما أننا نتحدث عن مليارات الخطوات على الأقل، ولا توجد طريقة لمعرفة، في المختبر، أن الخطوة الأولى على طريق الصلاح قد تم اتخاذها بالفعل" -

    أولاً، لا يتعلق الأمر بمليارات الخطوات. تتضمن الجرعة الدنيا ما يقرب من 20 بروتينًا. وهذا يعني ما مجموعه بضع عشرات من الخطوات. في هذا الصدد، ليست هناك حاجة أيضًا إلى أن يكون النظام قيد التطوير على وجه التحديد. أي نظام آخر يوضح التعقيد الذي لا ينفصم وطريقة السقالات يجب أن تكون كافية. وينظر أيضا في نظام من خطوتين.

    "فيما يتعلق بالسيارة المقلدة (أنا حقًا لا أفهم الحجة ولكنني سأبذل قصارى جهدي للإجابة)، فنحن نصنع سيارات مكررة" -

    حقًا؟ سترى إشارة واحدة إلى أن شخصًا ما قام بإنشاء سيارة مكررة تحتوي على الحمض النووي وآليات النسخ وتكون أيضًا قادرة على التحرك بمحرك.

    ودون رؤيته. وفقًا للمعيار المذكور أعلاه - رأيت أشخاصًا يصنعون كومة من الرمال. من هذا يجب أن أستنتج أن كومة الرمل تحتاج إلى مخطط؟ بالطبع لا. الآن سوف تدعي بحق أننا نعرف العملية الطبيعية التي تخلق كومة من الرمل. وبالتالي لم يتم التخطيط بالضرورة لكل كومة رمل. وأقول مرة أخرى أننا لم نر قطًا يتطور في عملية طبيعية أيضًا. إذن لقد عدت إلى المربع الأول.

    وبشكل عام، لكي أعرف أن السيارة المقلدة تحتاج إلى مصمم، هل يجب أن أرى المصمم؟. لم يسبق لي زيارة مصنع لتصنيع السيارات. وحتى بدون ذلك، أعلم أنني إذا رأيت غدًا سيارة من نوع لم أره من قبل (بتقنية تبدو غريبة وغير مألوفة بالنسبة لي، فكر على سبيل المثال في "اليوم الثالث")، فسيكون ذلك دليلاً على ذلك. التخطيط بالنسبة لي. لأنني أعلم أن الأشياء المعقدة مثل السيارات تتطلب التخطيط.

    "لا توجد نظرية تم إثباتها على الإطلاق بسبب الملاحظات الإيجابية."-

    ومن الغريب أن خبراء التطور أنفسهم يقدمون مثل هذه الأدلة على ادعائهم:

    http://www.sciencedaily.com/releases/2013/06/130606191001.htm

  607. لا يوجد شيء اسمه التخطيط الذكي لأنه في مجمله الوصف الوظيفي للمخطط الذكي ولا توجد عملية بدون المخطط الذكي. العودة إلى ابني وهو يصنع سيارة ليغو. ابني هو المخطط الذكي في سياق الليغو. وبعد أن انتهى من بناء السيارة، فهل هناك فائدة من الحديث عن عملية بنائها بمعزل عنه وكأنها غير موجودة؟ هل العملية التي جرت لها وجود مستقل؟ بالطبع لا وهذا في الواقع وجهة نظرك! ميلة كانت من أتباع التخطيط الذكي الذي يعتقد أن هناك تطور والمخطط يساعده في النقاط الصعبة (وبيهي هو في الواقع واحد من هؤلاء) ولكنك أكثر تطرفا، من النوع الذي يدعي أن المخطط الذكي ليس ذكيا بما فيه الكفاية ليخطط لعملية تتطور بدونه وهو الوحيد القادر على إدخال كائنات جديدة على سطح الأرض (هذا على الرغم من أنني أوضحت لكم كيف أن هذا غير صحيح في الواقع، من خلال النقل الجانبي الجينات). ولهذا السبب لا يوجد شيء اسمه تخطيط ذكي، بل يوجد فقط مخطط ذكي يفعل ما يريد.

    حجتك هي كما يلي: هناك أشياء لا يستطيع التطور تفسيرها بسبب احتماليتها المنخفضة (والتي في حد ذاتها حجة غير مرضية لأنه من قال أن الأحداث غير المحتملة لا يمكن أن توجد) ويعزو التعقيد الذي لا ينفصم إلى تلك الهياكل التي تعتقد أن احتماليتها منخفضة للغاية. عند هذه النقطة ترفعون أيديكم وتقولون: إذًا، من الواضح بالنسبة لي أن هناك عاملًا خارج الطبيعة، هو الذي صمم هذه الهياكل. وأنا أعارض هذه القفزة. إن حجة المصمم الذكي، التي نسميها الله في اللغة اليومية (وأشعر بالحرج لأننا نستمر في كتابة المصمم الذكي وليس الله) كانت موجودة حتى بدون معرفتنا المتقدمة في علم الأحياء، وكانت دائمًا ما يتم تفنيدها من قبل أعظم العلماء في العالم. ، عندما لم يتم شرح عناصر معينة في مقدمة المعرفة في إطار النظرية التي طوروها. معظم العلماء الكبار حصروا معرفتهم بالله، وفي كل مرة كنا نكتشف اكتشافًا جديدًا يخلق تفسيرًا جديدًا يتناقض مع التفسير القديم ويدفع الله بعيدًا. أنت تقول أن هناك فجوة في التطور وعلى الفور املأها بالله. كيف لا ترى أن هذه حجة إله الفجوة، من الصعب علي أن أفهمها وهذه هي الإجابة العامة على سؤالك حول السيارة المكررة: حتى الآن كنا دائمًا قادرين على تفسير كل ظاهرة نراها باستخدام التفسيرات الطبيعية والتفسيرات الطبيعية. لا يوجد سبب لعدم القيام بذلك الآن (على الرغم من أن الإجماع العام يدعي أننا فعلنا ذلك بالضبط)

    الآن بالنسبة لبعض الديون:

    لا يوجد شيء اسمه مبدأ التأكيد، ولا توجد نظرية في العلوم الطبيعية تم إثباتها قطعاً وليس عن الله. إن فكرة أنك تستطيع إثبات الله، الذي لا يمكن دحضه، هي مغالطة منطقية من الدرجة الأولى. كيف لا يمكنك رؤيته؟

    الطريقة التي تفعل بها ذلك مليئة بالثغرات أيضًا. لا يمكنك إثبات التعقيد الذي لا ينفصم ولن تنجح محاولة نقل عبء الإثبات. إن واجب الإثبات، بأن هناك هياكل لا يمكن إنشاؤها من تلقاء نفسها بأي حال من الأحوال، يقع على عاتقك وحدك، لأنك ادعيت أن هناك هيكلًا لا ينفصل ويجب على المدعي إثبات ذلك. لا يمكنك تقديم ادعاءات ضعيفة مثل "يبدو أنه مصمم" أو القول بأنه بما أننا نستطيع أيضًا بناء محرك، فيجب أن تكون هذه هي الطريقة التي تم بها صنع البندقية ونسميها علمية تقريبًا (كما لو كان هناك شيء يمكن تسميته علميًا تقريبًا ) ونتوقع منا أن نصل إلى طريق مسدود. وهذا أمر مثير للسخرية، تمامًا مثل طلب بيهي الذي أنشأناه في مختبر شوتون، والذي ندعي أن تكوينه استغرق ملايين السنين. كيف بالضبط؟ قل لي من أين أبدأ؟

    يزعم الخبراء أن التطور يقدم تفسيرًا عامًا يشرح كيف يمكن أن يكون كل هيكل نراه قد تم إنشاؤه (وتستخدم هذه الحجة أحيانًا ضده لأنه من الصعب جدًا دحض مثل هذا الادعاء، ولكن لحسن الحظ، هناك آخرون، لكنه لا يهمني حقًا الآن) ولكن من المؤكد أن عدم القدرة على وصف كيف تم إنشاؤه للتو ولا يعمل ضده لسبب أنه من المستحيل إعادة بناء جميع الظروف والقيود (على الأرض وخارجها) وكل ما يزعم خبراء التطور أنه لا يوجد منع مبدئي (أي أنه لا يوجد انتهاك لقوانين الحفظ هنا) بل ويقدمون تفسيرًا طبيعيًا يضع رأيهم في معظم الخبراء. وحقيقة وجود بعض التفسيرات لا تعني أنه من السهل إثبات كيفية تشكل أي بنية خطوة بخطوة، لأنها تتطلب مليارات الخطوات على الأقل، ولا توجد طريقة لمعرفة ذلك في المختبر. إن الخطوة الأولى في الطريق إلى الحجر (على سبيل المثال) تم اتخاذها بالفعل لأنك لا تعرف كل الخطوات المحتملة التي ستؤدي في النهاية إلى شوتون، وبالإضافة إلى ذلك، لماذا تعتقد أن هناك الكثير من المختبرات في جميع أنحاء العالم تحاول لإنتاج شوتون؟

    فيما يتعلق بالسيارة ذاتية النسخ (أنا حقًا لا أفهم الحجة ولكنني سأبذل قصارى جهدي للإجابة)، فنحن نبني سيارات ذاتية النسخ (على سبيل المثال، مثل خوارزميات الكمبيوتر) وهذا ممكن بالتأكيد، من أجل المتعة. ، لتصميم روبوت لبناء روبوت، ولكن على أي حال، قمنا ببرمجة أو بناء الجيل الأول لذلك يمكن القول أن عميلاً ذكياً صمم وبنى ولا أفهم لماذا تعتقد أنه لا يمكننا قول ذلك حول التكنولوجيا التي ننتجها بأنفسنا. ومن ثم فإن سؤالي لا يزال قائمًا: أستطيع أن أقول عن شيء ما أنه تم تصميمه وبنائه بواسطة وكيل ذكي إذا رأيت أنا أو أي شخص نيابة عني نفس العميل الذكي يصمم ويبني المكون المعني (ولقد رأينا بالفعل العديد من المباني المصممة بواسطتنا). هل تدعي أنك رأيت المصمم الذكي يبني شيئًا ما؟

    وفي الختام، لا يوجد شيء اسمه تخطيط ذكي، هناك فقط مخطط ذكي. إن القفزة من "التطور لا يمكن تفسيره" إلى "المصمم هو من فعل ذلك" غير منطقية ويخفيها التاريخ البشري، وعلى الأكثر يمكنك القول أنك تعتقد أن المصمم هو من فعل ذلك ولا يوجد شيء اسمه مبدأ التأكيد الذي يثبت شيئًا ما. لا توجد نظرية تم إثباتها على الإطلاق بسبب الملاحظات الإيجابية.

  608. شموليك،

    ليس من أجل لا شيء قمت بربطك بالفيديو. لم يوضح ميلر أن كل مكون من مكونات القذيفة له وظيفة مستقلة. وهذا ببساطة غير صحيح في الواقع.

    أ) تحتوي قاعدة القضيب على بروتينين متماثلين على الأقل. يعني لن تجدهم في أي مكان
    ب) يدعي ميلر أن المكونات المفقودة من Shotton ستمنحنا نظام حقن ttss. هل هذا صحيح؟ مرة أخرى، هذا ببساطة غير صحيح في الواقع. انتقل مباشرة إلى الدقيقة 5.02. انظر بنفسك كيف يدعي في الجيب فجأة مطالبة مختلفة. وفجأة أصبح "المتماثلون". وما هي تلك المتماثلات؟ هذه بروتينات *مختلفة* عن تلك الموجودة في شوتون.

    كما ذكرنا أعلاه، فإن ميلر أخطأ مرتين في نفس الفيديو. هذه الحقيقة يجب أن تشعل الضوء الأحمر.

    لذا، إذا كان ذلك يجعلك سعيدًا (وحتى لو كان ذلك يعطيك تلميحًا)، فسوف أقبل ادعاءاته. لماذا لا يهم بالنسبة لي؟ لأن واجب إثبات وجود عضو له تعقيد لا ينفصم، يقع عليك وحدك."-

    أ) يقع عبء الإثبات أيضًا على الجانب الآخر، لإثبات أن هناك مسارًا تدريجيًا لتطور المحكمة. لذا أفترض أنك تتفق مع نيل من الجانب التطوري أيضًا.

    ب) لقد قدمت تفسيراً علمياً شبه واقعي، وهو أنه لا توجد خطوات تدريجية لتطوير نظام معقد. من جانبك، زعمت أنه ربما في المستقبل سنكتشف كيف يمكن للسيارة، على سبيل المثال، أن تتطور تدريجياً. وردًا على ذلك سألتك ما الذي يجعلك تعتقد أنه في المستقبل سنكون قادرين على إظهار شيء كهذا. وسأكون ممتنا ردكم.

    "حتى لو أثبت أن شوتون لديه تعقيد لا ينفصم، فإن ذلك سيثبت فقط الطبيعة الإشكالية للانتقاء الطبيعي ولا شيء غير ذلك."-

    بالطبع. يشرح ميلر بنفسه في نفس الفيديو لماذا يتطلب التطور وجود خطوات وظيفية في تطور القضيب. وكل ذلك بسبب الاحتمالات المنخفضة. وإلا فلن يتمكن التطور من تفسير كيفية تطور هذه الأنظمة. وسيكون التطور في حالة يرثى لها.

    "إن المطالبة بحق التخطيط الذكي هي في الواقع ادعاء بحق المصمم الذكي، وهو ما اتفقنا عليه بالفعل، ولا يمكن دحضه، ولهذا السبب لا ينبغي تدريس التخطيط الذكي في المدارس والجامعات، في العلوم الطبقات إلا كمثال لكيفية عدم بناء نظرية."-

    ومرة أخرى، تمسكت بمبدأ التأكيد. إذا كانت هناك طريقة لإثبات نظرية فهي في نظركم ليست علمية ولا يجوز تدريسها في المؤسسات التعليمية؟

    "(وهنا حجة بيهي الدحضية مثيرة للشفقة وسخيفة، لأن الادعاء بأن الشوتون تطور بطريقة تطورية، عبر ملايين السنين ويريدنا أن نختبره في المختبر؟؟؟)"-

    نعم. والسبب في ذلك هو أنه حتى وفقًا للنظرية، يجب أن تتطور العصا تدريجيًا. عندما تكون كل خطوة فعالة في حد ذاتها على طريق الصلاح. ومن ثم، إذا كان التفسير التطوري صحيحًا، فلا ينبغي أن تكون هناك مشكلة في توضيح كيفية تطور هذا النظام تدريجيًا خطوة بخطوة. إذا افترضنا أن فرص الخطوة الأولى هي واحد في المليار. وهذا شيء من المفترض أن نراه يحدث في كل مختبر اليوم أمام أعيننا.

    "فيما يتعلق بادعائك بوجود روبوت مكرر، فأنا لا أفهم حقًا ما تريد قوله هنا. لقد رأيت، أو رأى شخص ما بالنيابة عني العوامل التي صممت الروبوت المكرر"-

    لكن هذا ما لم نره. هل سبق لك أن رأيت أحداً يصمم سيارة طبق الأصل؟ انا لا. ووفقا للمعايير التي قدمتها - مثل هذه السيارة ليست دليلا على التخطيط.

  609. بيولوجيا،
    ترتيب عكسي:
    ليس لدي أي فكرة عن سبب عرضك لي موقع يوتيوب يوضح إلى أي مدى تنهار حجتك فيما يتعلق بالشوتون، لأن ما يقدمه ميلر هو أن المكونات المختلفة، التي تشكل الشوتون، لها وظيفة مستقلة.
    أنا لست في وضع يسمح لي بالموافقة أو الاختلاف معه، لذا، إذا كان ذلك يجعلك سعيدًا (وحتى لو كان يجعلك تضحك)، فسوف أقبل حججه. لماذا لا يهم بالنسبة لي؟ لأن واجب إثبات وجود عضو له تعقيد لا ينفصم، يقع عليك وحدك. إن فكرة التعقيد الذي لا ينفصم، مثيرة للاهتمام في حد ذاتها، لكننا سنرى أنك تثبت وجودها. يجب عليك تتبع تاريخ الفضاء والفضاء الخارجي بأكمله وإثبات أنه لا يمكن أن تكون هناك ظروف بيئية ولا قيود ولا عنصر عرضي من شأنه أن يسمح بإنشاء الشوتون. ميلر، قم بالعمل الذي كان ينبغي عليك القيام به، قبل أن تدعي أن الشوتون يُظهر تعقيدًا لا ينفصم، وقد فعل ذلك لأنه، كما ذكرنا، الفكرة نفسها مثيرة للاهتمام بالتأكيد وتحدي فكري رائع. قبل أن تتوصل إلى أي نظرية بينك وبين نفسك، حاول أن تناقضها من كل اتجاه ممكن. والأمانة الفكرية تقتضي منكم أن تفعلوا ذلك ومن الواضح أننا لن نفعله.
    حتى لو أثبتنا أن شوتون لديه تعقيد لا ينفصم، فإن ذلك سيثبت فقط الطبيعة الإشكالية للانتقاء الطبيعي ولا شيء غير ذلك. إذا كان هذا هو الحال بالفعل، فإن التعقيد الذي لا ينفصم سيصبح جزءًا من نظرية طبيعية أخرى، وسيكون إسناد التصميم الذكي إلى التصميم الذكي أمرًا ديماغوجيًا. والسبب هو أنه لا يمكن فصل التخطيط الذكي عن المخطط الذكي، وفي الواقع لا يوجد شيء اسمه تخطيط ذكي ولكن هناك فقط مخطط ذكي هو الذي خلق كل شيء. إذا قام طفلي ببناء سيارة ليغو، فلا فائدة من الحديث عن العملية التي تم من خلالها بناء الليغو، ولكن فقط عن طفلي الذي قام ببنائها. لم يتم إنشاء السيارة من لا شيء، بل صنعها طفلي. إن المطالبة بحق التخطيط الذكي هي في الواقع ادعاء بحق المصمم الذكي، وهو كما اتفقنا لا يمكن دحضه، وهذا هو السبب وراء عدم تدريس التصميم الذكي في المدارس والجامعات، في العلوم الطبقات إلا كمثال على كيفية عدم بناء النظرية.

    ومن الاتجاه الآخر، حتى لو كان الشوتون يمكن أن يتطور من خلال التطور (وهنا حجة تفنيد بيهي مثيرة للشفقة وسخيفة، لأن الادعاء بأن الشوتون تطور بطريقة تطورية، عبر ملايين السنين ويريدنا أن نختبره في المختبر) ؟؟؟) هذا لا يعني أن هذه هي الطريقة التي تطورت بها بالفعل. من المحتمل بالتأكيد أن المخطط الذكي هو من خلقه وكما سبق أن كتبت فإن حقيقة إمكانية الوصول إلى حيفا عبر الطريق السريع رقم 4 لا تتعارض مع وجود الطريق السريع رقم 2 وهذا هو السبب وراء عدم إمكانية دحض التخطيط الذكي وهذا هو سبب آخر لعدم تدريس التخطيط الذكي في المدارس والجامعات، وفي فصول العلوم إلا كمثال على كيفية عدم بناء نظرية.

    فيما يتعلق بادعائك بوجود روبوت مكرر، فأنا لا أفهم حقًا ما تحاول قوله هنا. لقد رأيت، أو رأى شخص ما بالنيابة عني العوامل التي صممت الروبوت المتماثل، لذلك أنا لست منزعجًا من أن الجيل العشرين من الروبوت المتماثل هو الذي أنشأ الجيل التاسع عشر لأنني أعرف من قام ببناء الجيل الأول، لذا فإن ادعائي هو:
    أستطيع أن أقول عن شيء ما أنه تم تصميمه وبنائه بواسطة وكيل ذكي إذا رأيت أنا أو أي شخص نيابة عني نفس تصميم الوكيل الذكي وقام ببناء المكون المعني. هل تدعي أنك رأيت المصمم الذكي يبني شيئًا ما؟
    بناءً على كلامك، لا أفهم لماذا تعتقد أن الاستطلاع يستحق أي شيء؟ هل ستقنعني دراسة استقصائية للعلماء بأن ميكانيكا الكم لا تتنبأ بالواقع بشكل صحيح؟ بالطبع لا. لماذا تعتقد أن دراسة استقصائية للأشخاص ذات صلة بشيء ما؟

    فيما يتعلق بالعصا والمحرك، مرة أخرى، تريد الادعاء بأنه نفس المحرك البشري، ادعي ذلك. كنت أشير للتو إلى الفرق الموجود بين المحركات التي صنعها الإنسان مقابل الهياكل التي تم إنشاؤها من خلال عملية تطورية. هنا بالمناسبة، فاتك نقطة أخرى. بينما لا نعرف حقًا كيف نفسر كيف تحولت الكيمياء إلى بيولوجيا تنظم العناصر الضوئية (وهنا لن أطعن في ادعائك بعدم وجود بروتينات على الأرض إلا في الكائنات الحية، لكنني سأحيلك إلى http:// www.space.com/10498-life-building-blocks-surprising-meteorite.html)، العناصر التي تشكل البروتينات هي الأكثر شيوعًا في الكون، وهو بالضبط ما تتوقعه من عملية عمياء: الاستفادة مما هو متاح. فهل هذا دليل على شيء، بالتأكيد لا، ولكن سيكون من المدهش لو كان غير ذلك.

  610. شموليك،

    أنت قلت:

    "سأعود مرة أخرى: أستطيع أن أقول عن شيء ما أنه تم تصميمه وبنائه بواسطة وكيل ذكي إذا رأيت أنا أو أي شخص نيابة عني نفس تصميم العميل الذكي وقام ببناء المكون المعني. "-

    وهذا يعني أن تكرار السيارة/الروبوت/الساعة ليس دليلاً على التصميم. لأننا لم نر أحداً يصنع واحدة من قبل. نعم. وأعتقد أن السيارة المكررة هي بالتأكيد دليل على الكثير من التخطيط، حتى بدون أن أرى شخصًا يصنعها. في الواقع، إذا طرحت هذا السؤال على أي شخص في الشارع أو حتى قمت بإجراء استطلاع بين العلماء. أعتقد أن الجواب سيكون لا لبس فيه.

    "من المؤكد أن Shotton يقوم بعمل مشابه للمحرك، لكن هذا لا يقول شيئًا عن كونه محركًا صممه مصمم ذكي. "أحد الاختلافات الأساسية بين الذي صممه الإنسان والشوتون الذي يُزعم أنه تم إنشاؤه من خلال عملية تطور طبيعية هو أن الشوتون مبني بالكامل من العناصر الأكثر شيوعًا الموجودة في الكون"-

    يتكون القضيب من البروتينات. لم يتم ملاحظة البروتينات على الأرض من قبل إلا في الكائنات الحية. مما يعني أن البروتينات أندر من الذهب والماس. مما يعني أنه وفقًا للمعيار المذكور أعلاه، يعتبر القضيب أكثر خصوصية من المحرك العادي. وهنا أيضا أنا لا أتفق مع هذا الادعاء. إذا قمت بتصميم سيارة غداً مصنوعة من عناصر مشتركة. ألا يمكنني أن أسميها سيارة؟ منذ متى كان نوع المادة ضروريًا لهوية الشيء؟

    "أنت لا تتمكن حتى من إظهار أن هناك شيئًا مثل التعقيد الذي لا ينفصم لأنه ليس لديك ما يمكنك من خلاله استعراض جميع الخيارات التي كان من الممكن أن يتم إنشاء السيف فيها ومعارضتها واحدًا تلو الآخر."-

    هذا من فم ميلر. الوقود المباشر للدقيقة الثالثة:

    http://www.youtube.com/watch?v=m2alpk8PUd4

    هل تتفق معه ومع تصريحاته؟

    وأما سؤالك الأخير، بما أنني لست من هواة مناقشة الفلسفة، فسأتجنبه بأدب :)

    يا أبي، ليس هناك شيء اسمه "نصف عين". لأنه لا يوجد شيء اسمه نصف كاميرا. حتى أبسط عين في الطبيعة تتكون من مئات المكونات المختلفة (بقعة العين البسيطة يمكن أن تصل إلى 200 جين). وهناك حاجة إلى العديد من المكونات للحصول على الحد الأدنى من العين. فضلا عن الحد الأدنى من الكاميرا

  611. التبشيري,
    هذه هي النقطة بالضبط، كما شرحت مراراً وتكراراً. وحقيقة أنه يمكن القول دائمًا أن المصمم الذكي هو الذي خلق الخلية الأولى، هو الذي خلق المادة الأولى، هو الذي خلق كل شيء، يجعل الأمر غير علمي. يمكنك دائمًا ادعاء ذلك باعتباره ورقة رابحة وليس هناك ما يمكنني فعله لدحض ذلك. الكيان الذي يُزعم أنه صمم (وبنى وفقًا لبعض المؤمنين) كل ما نراه حولنا يسمى في اللغة اليومية، الله. ولهذا السبب، لا يجوز تدريس هذه النظرية في المدارس أو الجامعات، في دروس العلوم إلا كمثال لكيفية عدم بناء نظرية علمية. كل الروابط التي جلبها علم الأحياء الرياضي أظهرت ما حوله من غش وإكراه. بدءاً بالطريقة التي تقترح بها عناصر معينة كيفية مناقضة هذه العناصر، وانتهاءً بالقفزة المنطقية "السهلة" من محاولة (جديرة بذاتها) لدحض التطور إلى عامل خارج الطبيعة.
    التطور لا يتناول كيفية تشكل الخلية الأولى، وعلى أية حال، فإن نقاط الضعف/ثغرات التطور هي نقاط ضعف/ثغرات في التطور وليست دليلاً على أي نظرية أخرى. أنت منزعج من نقاط الضعف/الثغرات هذه، فلا تملأها بالهراء ولكن اجلس وفكر في كيفية ملء هذه الثغرات بمحتوى يمكنك القيام بشيء ما به (أو السماح للعملية العلمية بالقيام بعملها). ينتشر وباء شلل الأطفال حاليًا في إسرائيل. هل تقترح أن نرفع أيدينا ونجلس مكتوفي الأيدي وندعو الله أن يعتني المخطط الذكي بالأمر، أو أن نحاول التفكير وخلق حل علمي فعال وغير خارق للطبيعة للطاعون (هناك بالفعل) ؟
    كل التقدم الذي صنعناه لأنفسنا هو نصب تذكاري للعكس، وفوق كل شيء، فإن الاقتباس من كلمات مارك توين مناسب هنا: "اقرأ توقعات الطقس قبل أن تصلي من أجل المطر"

  612. المبشر:
    أنا أكتب من هاتفي المحمول في المطار وليس لدي وقت للهراء لأنني سأغادر بعد لحظة.
    لقد بدأت بالطماطم وعندما شرحت لك أن هذا هراء "أرجعتني" إلى مكان لم نكن فيه - الخلية الحية الأولى.
    وحتى الخلية الحية الأولى يمكن الوصول إليها عن طريق التطور من مادة غير حية.
    صحيح أنه لا توجد حفريات من هذه الفترة، لكن لا توجد أيضًا حفريات للخالق الذكي ولا يوجد دليل على وجود هذا الخالق.
    وكما ذكرت فإن فكرة الخالق الذكي أيضا لا تعطي أي إجابة على السؤال حول أصل الحياة. هل تقرأ حتى ما هو مكتوب لك؟
    المضحك هو أن نكتة التصميم الذكي برمتها هي طريقة مراوغة لإنقاذ الله، لكن الحمقى الذين يسلكون هذا الطريق ينسون أنهم لا يتحدثون عن أي شيء مختلف على الإطلاق عن إله اليهودية.

  613. إلى مايكل
    سأعيدك إلى النموذج الأولي للخلية الأولى
    كان من الممكن أن يكون التصميم الذكي هو الذي خلق الخلية الأولى على مستوى التصميم التطوري
    (لاحظ أن كل ما أدعيه وغيره هو نموذج)

  614. المبشر:
    انا اعتقد انك تفتقد الى المقصد.
    لا أحد يدعي أن التصميم الذكي غير ممكن. والعكس صحيح! نحن أنفسنا - كبشر - منخرطون في التخطيط الذكي للحيوانات وقد حققنا بالفعل نجاحات معينة في هذا الصدد.
    انظر على سبيل المثال هنا:
    http://www.jcvi.org/cms/research/projects/first-self-replicating-synthetic-bacterial-cell/overview/

    إن الجدل حول الخلق الذكي لا يدور حول ما إذا كان ممكنًا، بل حول ما إذا كانت الأدلة التي نجدها في الطبيعة تشير إلى التصميم الذكي أو التطور.
    الجواب واضح تمامًا لأي شخص يعرف الحقائق: كان هناك تطور ولم يكن هناك تصميم ذكي.
    وبالطبع لو وجدنا دليلاً على التخطيط الذكي، فلن نصمت كما يفعل المؤمنون، بل سنبحث عن كيفية خلق المخطِّط الذكي.
    يبدو أن هذا الموضوع لا يهم على الإطلاق الهوزيين من التصميم الذكي ويدافعون عنه بالرغم من عدم وجود دليل على وجوده ورغم أنه حتى لو كانت حياتنا نتيجة للتصميم الذكي - فإنه لن يجيب على سؤال كيف تم خلق الحياة لأنه وفقًا لادعاء التصميم الذكي - في الوقت الذي خلقنا فيه كان هناك شخص ما على قيد الحياة بالفعل وهو الذي خلقنا (وبالتالي فإن طريقة تكويننا ليست إجابة لسؤال كيف تم خلق الحياة).

  615. سؤال لشموليك وروتشيلد
    في ذلك سوف نأخذ بذور الطماطم
    سننشئها بالفكر والتكنولوجيا (الازدواجية) نسخة طبق الأصل
    هل تعتقد أنه سيكون التخطيط الذكي - أو أي شيء آخر؟؟؟

  616. علم الأحياء الرياضي,

    سأعود مرة أخرى: أستطيع أن أقول عن شيء ما أنه تم تصميمه وبنائه بواسطة وكيل ذكي إذا رأيت أنا أو أي شخص نيابة عني نفس تصميم العميل الذكي وقام ببناء المكون المعني. فيما يتعلق بكل تلك الأشياء التي لم أرها، أو لم يراها أحد لسببي، والتي خطط لها وبناها عميل ذكي، لا أستطيع أن أقول ذلك. هل تدعي أنك، أو أي شخص نيابة عنك، رأيت المصمم الذكي وهو يخطط ويبني الشوتون (أو أي شيء آخر)؟

    من المؤكد أن الشوتون يؤدي عملاً مشابهًا للمحرك، لكن هذا لا يقول شيئًا عن كونه محركًا صممه مصمم ذكي. أحد الاختلافات الأساسية بين ذلك الذي صممه الإنسان والشوتون الذي يُزعم أنه تم إنشاؤه من خلال عملية تطورية طبيعية هو أن الشوتون مبني بالكامل من العناصر الأكثر شيوعًا الموجودة في الكون (أنا أخمن هنا ولكنه يتكون بالتأكيد من الكربون والهيدروجين والنيتروجين والأكسجين وما إلى ذلك) وهو بالضبط ما تتوقع أن يفعله التطور. هذا (العمل مع المواد الشائعة في الكون) يُقارن بالمحرك البشري المبني من مواد غير شائعة. وهذا هو أحد الأسباب التي تجعل هذا التوازي الفضفاض لا يثبت شيئًا. على أية حال، كما سبق أن كتبنا مرارًا وتكرارًا، حتى لو كان من الصعب على التطور تفسير ذلك، فهذا يعني فقط أن التطور يواجه صعوبة في تفسيره. هذا كل شيء. هذا يحول التطور إلى نظرية بها ثقوب، ولكن كما كتبت بالفعل، لا يهمني هذا حقًا (يمكن أيضًا قراءة الدفاع عن التطور ضد الشوتون، وتفسير الاحتمالات، وما إلى ذلك على ويكيبيديا) لأنني مهتم بفهم سبب تأكدك من وجود التصميم الذكي أو كيف يمكن دحضه ولا يمكنك فعل ذلك وذاك. أنت لا تتمكن حتى من إظهار أن هناك شيئًا مثل التعقيد الذي لا ينفصم، لأنه ليس لديك ما يمكنك من خلاله استعراض جميع الخيارات التي يمكن من خلالها إنشاء العصا ومعارضتها واحدًا تلو الآخر. ليس لديك طريقة لمعرفة ما هي الحاجات التطورية التي ربما تكون قد خلقت بعض الهياكل في الماضي حتى تم إنشاء الشوتون لكي تدعي، بشكل مطلق، أن خلق الشوتون يتناقض مع الفيزياء في شكل تناقض الحفظ القوانين. حتى لو أظهرت أن هناك ما يسمى بالتعقيد الذي لا ينفصم، فإن ذلك لن يجعلك أقرب إلى المصمم الذكي.

    إذا كنت تريد مني أن أشير إلى ميلر، سأكون ممتنًا لو تمكنت من توجيهي إلى الرابط والدقيقة المعنية.

    وبالمناسبة، كيف ترى حياتك كشخص يعتقد أنه مخلوق صممه مصمم ذكي؟ هل أنت سعيد بذلك؟

  617. باعتباري شخصًا أصبحت عيناه أضعف فأضعف حتى اضطررت للخضوع لعملية جراحية لإزالة المياه البيضاء، يمكنني أن أشهد أنه حتى ربع العين مفيد. هذا صحيح، لا يمكنك قراءة الخطوط الصغيرة، ولكن من أجل البقاء.
    اقرأ على سبيل المثال هنا أيضا
    https://www.hayadan.org.il/seeing-sea-stars-the-missing-link-in-eye-evolution-080713/

  618. شموليك،

    وفق معيار أن وضع نسخة مكررة من الساعة/الروبوت/السيارة لا يحتاج إلى تخطيط. ذلك لأنني لم أر قط أي شخص يصنع سيارة أو ساعة طبق الأصل.

    "القبول شيء جميل، لكنه في حد ذاته لا يثبت شيئًا. حقيقة أنك قررت أن شوتون هو محرك،" -

    أخبرني بنفسك - لماذا تعتقد أن العصا ليست محركًا؟

    "1. وأن التطور، في كل مرحلة، يجب أن ينتج ميزة تطورية."-

    إذا كنت تقصد كل خطوة على مستوى النوكليوتيدات فأنت على حق. لكن بشكل عام، يتطلب التطور تدرجًا انتقائيًا. وإلا فإنه يعادل أن العين مثلاً خلقت في مكة. لماذا ؟ لأنه إذا لم تكن هناك ميزة البقاء في أي مرحلة في الطريق إلى العين (على الرغم من أنه ليس من الضروري أن يكون كل نيوكليوتيد ضغطًا انتقائيًا، ولكن بالتأكيد بشكل عام)، فإن فرصة أن تتطور العين من تسلسل عشوائي *بدون الانتقاء الطبيعي* في الطريق إلى العين المعقدة، يشبه العين التي تتطور فجأة من تسلسل عشوائي. ولذلك، لا يتفق أي خبير في التطور مع هذا الادعاء. ولهذا السبب تم اختراع طريقة السقالات أيضًا.

    "الادعاء هو أن معظم المراحل لها مبرر تطوري بخلاف استخدامها كمحرك (إذا ذهبت إلى مناظرة بن ميلر-ليبيهي، فقد أثبت ميلر ذلك)"-

    ولم يثبت ميلر هذا. لقد تصور أن هناك نظامًا آخر به أجزاء مشتركة بين شوتون. هل هذا يثبت أن الأنظمة كان من الممكن أن تتطور من بعضها البعض؟ بالتأكيد لا. كما شرحت مع سيارة وطائرة.

    "إن واجب الإثبات، أن كل مرحلة في حد ذاتها، (كل مرحلة) غير ممكنة من وجهة نظر تطورية، هو فقط من جانبك. "-

    أعطيتك مثالاً من محرك أو سيارة حقيقية. إذا لم تكن هناك درجة انتقائية في الطريق إلى سيارة مصنوعة من مادة مكررة، فلماذا يكون هناك درجة انتقائية في محرك شوتون؟

    شيء آخر، يدعي ميلر أن الادعاء أعلاه يستحق النظر فيه. هل تختلف معه؟

  619. بيولوجيا هاتاباراتي:
    نعم بالتأكيد.
    (هذه المناقشات كانت موجودة بالفعل على هذا الموقع من قبل. ربما فاتتك هذه المناقشات. إذا كان الأمر كذلك، فنحن نرحب بك للبحث في أرشيفات الموقع وقراءتها.
    وخاصة لتتعلم منهم لماذا أنت مخطئ. فقط لا تخلط بيننا.)

  620. علم الأحياء الرياضي,
    لا، لا يوجد شيء اسمه "كما تعلمون". السبب الوحيد الذي يجعلني أعرف أن محرك السيارة قد صممه إنسان هو أنني رأيت بأم عيني (أو من ينوب عني) التصميم المصمم ومن ثم بناء المحرك. هل تدعي أنك رأيت المخطط (أو أي شخص نيابة عنك)؟
    أبعد من ذلك، القبول شيء جميل، لكنه في حد ذاته لا يثبت شيئا. إن حقيقة أنك قررت أن Shuton هو محرك هو ادعائك، وبعد ذلك، إذا كنت تدعي أن لديه تعقيدًا لا ينفصم، فهو أنه يتعين عليك إثبات ادعاءين:
    1. أن التطور، في كل مرحلة، يجب أن ينتج ميزة تطورية. لا يدعي أي من دعاة التطور أن كل خطوة في حد ذاتها يجب أن تكون فعالة. الادعاء هو أن معظم المراحل لها مبرر تطوري بخلاف استخدامها كما نحن (إذا ذهبت إلى مناظرة بن ميلر إلى بيهي، فقد أثبت ميلر ذلك) وهذا الادعاء قدمه أنصار التطور ولكنه حدث لأن أنصار التطور استجابوا إلى التحدي. إن عبء الإثبات، أن كل مرحلة في حد ذاتها، (كل مرحلة) غير ممكنة من وجهة نظر تطورية، يقع على عاتقك فقط. نظرًا لأنه لا أحد يدعي أنه من هذه اللحظة، سيكون اسمها في نهاية شوتون، بدأت في التطور وكانت هناك نية (موجهة مسبقًا) لإنتاج شوتون، يجب عليك إثبات أنه لا توجد طريقة لإنشاء شوتون. وعبء الإثبات يقع عليك فقط وأنت لم تفعل ذلك. لقد قدمت للتو مطالبة.
    2. أن ما سبق لا يمكن أن يكون قد تشكل بالصدفة وبطريقة سهلة، فإذا كانت بعض المراحل مستقرة تطورياً، فهل من الممكن أن تكون المرحلة النهائية قد حدثت بالصدفة؟

  621. شموليك، سأحاول الرد على معظم ادعاءاتك.

    أولاً، يقع عبء الإثبات على عاتق من يدعي. وليس لدي مشكلة في إثبات أن الطبيعة تتطلب التخطيط. لا يوجد نقص في الأدلة الإيجابية للتخطيط. واحد منهم هو السوط الحركي العضوي. يمكن لهذا المحرك المتطور أن يدور حوالي 1000 مرة في الثانية. والمحرك، كما نعلم، هو دليل على التصميم. حتى لو كانت مكونة من مادة عضوية أو قادرة على عمل نسخ من نفسها.

    إن الادعاء بأن المحرك يتطلب التخطيط هو ادعاء علمي. خلافا للادعاء المعاكس بأنه ليس تطورا.

    يشير التصميم الذكي إلى وجود أنظمة لا يمكن تفسيرها تدريجيًا. وهي أيضًا التي تميز وتفصل بين الكائنات المختلفة (باستثناء ما هو مذكور في البكتيريا، حيث إنها تقوم بالنقل الأفقي، حيث يكون العمل أكثر تعقيدًا لأنه من الضروري تتبع مصدر الجينات المنقولة). وكما ذكرنا، واحد منهم هو السوط. والسبب في عدم إمكانية إنشاء هذا المحرك تدريجياً هو أن فقدان أجزائه (كما هو موضح تجريبياً) يؤدي إلى خلل بيولوجي. ومن ثم، إذا قمنا بإعادة بناء تاريخها التطوري فسوف نواجه نهاية الطريق.

    فكر في الأمر بهذه الطريقة - ما الذي يميز موديلات السيارات المختلفة؟ من حيث المبدأ، هذه أنظمة مختلفة، وتتكون من مكونات مختلفة.

  622. علم الأحياء الرياضي,
    الجزء الأول
    والحجة بسيطة: ليس من الممكن إثبات، بأي شكل من الأشكال، أنه لا يمكن دحضها مقدمًا، ومن المؤكد أنه ليس من الممكن الحد من سلطة المخطط. إن محاولة عكس الترتيب الصحيح للأشياء لن تغير أي شيء. ما كتبته، لا يمكن دحضه ولكن يمكن إثباته، هو كلام كاذب. ما لا يمكن دحضه لا يحتاج إلى دليل ولن يغير شيئا على أي حال. كل ما أقوله لن يتغير: لن أستطيع دحض وجوده. ولكن إذا كان مسموحًا لك بالمطالبة بالأشياء نيابةً عنه، فلماذا لا أفعل ذلك؟ ما هي المعلومات التي تتعرض لها والتي لا أعرفها؟

    اتفقنا على أن المخطط هو الذي خلق التصميم الذكي. في جميع الروابط التي قدمتها لي فيما يتعلق بالتصميم الذكي، لم أر طريقة واحدة لدحضها، حيث أن نظريتك ستتوافق مع كل ما نراه من حولنا وأي نتائج جديدة ستكون مقبولة بالضرورة (من الواضح بالنسبة لي أنك ستحاول لطرح نفس السؤال عن التطور، ولكن في هذا الجزء من المناقشة، أنا لا أشير إليها. فمرحبًا بك لطرح الأسئلة على الآخرين). وما يمكن دحضه هو مبدأ التعقيد الذي لا ينفصم، ولدي عند هذه النقطة ثلاث تعليقات:
    1. حتى لو لم يكن هذا التعقيد غير موجود، فسيظل من الممكن الادعاء بأن التصميم الذكي موجود وأنه أكثر ذكاءً مما افترضنا لأنه خلق آلية متطورة مثل التطور.
    2. إن واجب إثبات أن المكون الذي تم تصميمه بواسطة مصمم هو ملكك وحدك وليس وظيفتي هي إثبات ذلك. في الواقع، إن دور أنصار التطور هو تقديم تفسير لكيفية قيام التطور بذلك، ولكن هذا ليس دوري أيضًا، وقدرة التطور أو عدم قدرته على تفسير نقاط معينة لا تساهم بأي صحة لنظرية التصميم الذكي.
    3. القيود التي حاولت روابطك ادعاء وجودها على التصميم الذكي هي قيود قسرية وتعسفية وكأنها تم إنشاؤها للسؤال: "كيف يمكن دحض التصميم الذكي؟" تقول هنا - أرني أنه كان هناك تنمية بدون تخطيط ذكي، وقد دحضت التخطيط الذكي، ولكن هذا غير صحيح. لقد حددت، بشكل تعسفي، ما هي العملة الأساسية التي تسمى التقدم (معك حديقة جديدة، إذا فهمت بشكل صحيح)، حددت، دون أن تخبر، ما هو المخلوق ومتى يتحول إلى مخلوق آخر وحددت، دون أن تخبر لنا، ما هو التخطيط ثم تأتي وتطلب مني أن أظهر أنه كان هناك تقدم، دون تخطيط، دون أن أعرف ما هو التخطيط أو عندما يتوقف الكائن عن أن يكون هو نفسه. يبدو الأمر كما لو أنني سأقوم بإنشاء نظرية شموليك النسبية، وهي نفس نظرية أينشتاين، لكن نظرية أينشتاين لا تعمل بشكل صحيح على مادة تسمى جازبين وأنا أفعل ذلك. ابحث عن Jazbin ويمكنك دحض نظريتي أو القول بأن نظرية أينشتاين خاطئة. هذه ليست الطريقة التي تبني بها النظرية. إذا قمت بطرح سؤال الروبوت الخاص بك مرة أخرى، فلن تحصل على شيء، فالسؤال ليس تعريفًا.
    وبما أنه كذلك، إذ لا يمكن دحضه، لأنه انتحال، إذ لا يخدم شيئا، ولا يقدم تنبؤا واحدا، ويأتي مع الأب التخطيطي الذكي، فيمنع تدريس التخطيط الذكي في المدارس، في دروس العلوم، باستثناء كيفية عدم بناء النظرية.

    جزء ثان
    وأسأل مرة أخرى، ما علاقة هذا بما يمكن أن يفعله الإنسان الذكي؟ هناك أشياء تمكنا من التخصص فيها وهناك أشياء لم نتخصص فيها بعد. حقيقة أن الطبيعة تفعل أشياء كثيرة أفضل منا، لا تعني أننا لسنا أفضل في بعض الأشياء. ما المشكلة في ذلك؟
    ماذا تقصد أن الوقت لا يلعب دورا؟ يساعد زمن الماضي في تفسير سبب عدم كون ما نراه هو نتيجة الصدفة. ماذا تقصد بالمخطط الذكي الذي أظهر أنه لا يستطيع... لا أفهم على أي مبدأ تقلل من قدرة المخطط الذكي ولماذا تعتقد أن ما كان سيكون بالضرورة.
    وقد سبق أن أشرت إلى أنك أنت من يحدد كل شيء، بما في ذلك ما هو التخطيط، بما في ذلك إضافة مبادئ لا يقبلها التطور، على سبيل المثال الادعاء بأن كل خطوة في التطور يجب، في حد ذاتها، أن تكون خطوة إيجابية ( من أين لك ذلك؟) ومع ذلك فإنني أسأل، كيف يمكن للمرء أن يثبت عدم كفاءته؟ وإذا وجد ذلك ممكنا في المستقبل، فسوف تتخلى عن التخطيط الذكي (لكن التخطيط الذكي سيبقى حيا، وهو ما لا يمكن دحضه، تذكر)؟ بالإضافة إلى ذلك، هل تريد حقًا أن تخبرني أنه من المستحيل تصميم مخلوق ليتحول إلى مخلوق آخر وكل خطوة في التغيير ستكون مبررة (رغم أن هذا ليس مبدأ موجودًا في التطور)؟ ولا حتى كتمرين نظري؟ .
    حتى لو كنت أجد صعوبة في فهم كيفية تعريف المخلوق، ما زلت لا أفهم لماذا لا تسمي المخلوق الذي تم إنشاؤه نتيجة لنقل الجين الجانبي مخلوقًا جديدًا؟ علاوة على ذلك، لقد تجاهلت تماما حجة النسخ التناظرية الخاصة بي. هل تريد إعادة تعريف ما هي حدود المخطط الذكي؟

  623. إن كون موليك، كما ذكرنا، مخططًا ذكيًا عامًا لا يمكن دحضه حقًا. هذا لا يعني أنه لا يمكن إثباته.

    "لقد حدثت العملية التطورية على مدى مليارات السنين، ببطء وبشكل مرعب، وما أهمية ما يستطيع أو لا يستطيع الشخص الذكي فعله؟"

    بالطبع يهم. التطور كمبدأ هو تفسير طبيعي. وإذا كان المصمم الذكي غير قادر على إثبات التطور التدريجي لنظام جديد، فكيف يمكن للمرء أن يصدق أن العامل الطبيعي يستطيع ذلك؟ منذ متى يلعب الوقت دورًا هنا؟ وفقًا للمعيار أعلاه - لا يستطيع الشخص الذكي إنشاء سيارة طائرة. وبالتالي، خلال مليارات السنين، يمكن إنشاء مثل هذه السيارة بنفسها.

    أول شيء، يمكننا القيام بذلك قريبًا إن لم يكن بالفعل، وثانيًا، المخطط الذكي يستطيع ذلك، لأنه لا شيء يحده. هل تزعم أن المصمم الذكي محدود في قدراته؟

    هل تريد دعم المصمم الذكي الذي ابتكر كل شيء بطريقة تظهر لنا أنه لا يتدخل؟ من فضلك. فقط اعلم أن معظم علماء الأحياء التطورية يدعمون التطور بدون مصمم. على الأقل بقدر ما أعرف. تأتي نظرية التطور بأكملها لتشرح كيف يمكن إنشاء أنظمة معقدة بدون مصمم.

  624. علم الأحياء الرياضي,
    ولم أحصل على إجابة: هل نتفق، في رأيك، أن التصميم الذكي (مهما كان) هو من صنع المصمم الذكي، وهو أمر لا يمكن دحضه؟

    فيما يتعلق بالمادة:
    أما كون العملية معروفة فأنا سعيد بذلك، فهذا يعني أن العلماء يقومون بعملهم. وما تم إثباته هو نقل المادة الوراثية، التي لا يمكن أن تنتقل بشكل كامل، كمبدأ (كما بينت)، بين كائنات مختلفة أثناء خلق كائنات جديدة، وليس من خلال المخطط الذكي (وهذا هو بيت القصيد). وحقيقة أن هناك نظامًا على الجانب الآخر يمكنه تلقي المادة الوراثية لا يغير شيئًا، فهو دائمًا كذلك. وكما هو معروف (في ضوء المقال الذي أحضرته)، فإن المادة الجينية التي تمر تنتقل دون تخطيط مسبق، مما يسمح للكائن الجديد بالبقاء على قيد الحياة بشكل أفضل، وحتماً فإن المادة الجينية التي تمر لن تمر بالكمال الرقمي (هل تعلم؟ أعتقد خلاف ذلك؟) والتي ستؤدي حتما إلى تغييرات إضافية على طول الطريق.

    أما ما "تحدثت عنه بوضوح" فلا شيء واضح. هل تريد تأهيل حجتك القائلة بأن المصمم الذكي هو وحده القادر على إنشاء مخلوقات جديدة؟
    ש
    بخصوص الرابط الذي أرسلته، ماذا تريد مني أن أقرأ هناك؟ ومن ناحية أخرى، جاء في المقال الذي نشره العالم: "في بحثي، قمت باختبار قدرة نوع واحد من العتائق على مقابلة العتائق التي تختلف قليلاً عنها. لقد تحققت مما إذا كان هناك عائق فيما يتعلق بالقدرة على نقل الحمض النووي. رأيت أن الحواجز منخفضة جدًا، وهذا في الواقع يثير تساؤلات حول تطور هذه المخلوقات. أثناء التطور، يحتفظ المخلوق بحمضه النووي ويسمح له بالتطور. إذا كان لدى مخلوق معين القدرة على تلقي الحمض النووي وإعطاء الحمض النووي للآخرين، فإن ذلك يؤثر على نوع التطور الذي سيخضع له. يغير عملية التطور تماما. إذا لم تكن مقيدًا بحمضك النووي، ويمكنك الحصول على سمات كاملة من الآخرين، فإن هذا يسرع التطور بشكل كبير.
    https://www.hayadan.org.il/edit-naor-interview-310713/

    فيما يتعلق بما يمكن للتطور أن يفعله أو لا يستطيع فعله، ليس من المهم حقًا بالنسبة لي أن أخوض في التفاصيل، لأن التصميم الذكي يجب أن يعتمد على مزاياه الخاصة والنظرية لا تحصل على حالة الصحيحة افتراضيًا.
    بالنسبة لسؤالك: "لماذا يمكن للتطور أن يفعل ذلك بسهولة... ولا يستطيع الشخص الذكي..." فهو سؤال غريب. لقد حدثت العملية التطورية على مدى مليارات السنين، ببطء وبشكل مرعب، وما أهمية ما يستطيع أو لا يستطيع الشخص الذكي فعله؟ أول شيء، يمكننا القيام بذلك قريبًا إن لم يكن بالفعل، وثانيًا، المخطط الذكي يستطيع ذلك، لأنه لا شيء يحده. هل تزعم أن المصمم الذكي محدود في قدراته؟

    أريد الحصول على إجابات للأسئلة التي طرحتها عليك في أعلى رسالتي وبالتالي على السؤال الذي طرحته منذ زمن طويل: إذا كنت أؤمن بما تؤمن به (التصميم الذكي) مقترنًا بالتطور، فهل أؤمن بالتصميم الذكي؟ وهذا يختلف عن لك؟

  625. بالمناسبة، قلت

    "يناقض ادعاءاتك بأن المصمم الذكي هو وحده القادر على إدخال كائنات تحتوي على مادة وراثية جديدة"-

    ليس تماما. ففي نهاية المطاف، لن تمر نفس المادة الجينية دون أن يقبلها نظام معقد. من الواضح أنني كنت أتحدث عن مخلوقات ليس لها انتقال فيما بينها. وبالإضافة إلى ذلك، فإن تبادل البلازميد هو عملية معروفة ومعروفة. إنها لحقيقة أنه حتى أنصار التطور لا يقدمونه كدليل على التطور وهم محقون في ذلك:

    http://en.wikipedia.org/wiki/Bacterial_conjugation

  626. شموليك، النقطة المهمة هي أن تطوير نظام جديد بمكونات جديدة لم يتم إثباته مطلقًا، كما تدعي T.Evolution. إذا كنت تعتبر أن نقل السمات الموجودة بين الكائنات الحية هو شيء يثبت ذرة من الادعاء المذكور أعلاه، فيرجى القيام بذلك. فقط اعلم أنه لا يقترب حتى. بالأحرى، إذا كان التطور يستطيع بسهولة إنشاء نظام معقد تدريجيًا. فكيف لا يستطيع ذلك حتى الشخص الذكي؟

  627. علم الأحياء الرياضي,
    هل تختلف مع حقيقة أن التصميم الذكي هو بالضرورة نتاج مصمم ذكي؟
    لم أتوصل بعد إلى تعريف ما هو التخطيط الذكي، وبالمناسبة، لا أنت ولا أي شخص يملك أي نظرية. فقط بعد أن أتلقى إجابة منك، يمكننا المضي قدمًا والبدء في مناقشة التصميم الذكي.

    بخصوص المقال: حسنًا، تنتقل المادة الوراثية من بكتيريا إلى أخرى عندما تكون أهمية المادة المنقولة حاسمة لبقاء البكتيريا.
    بكتيريا/بكتيريا هي كلمات باللغة العبرية تثير النقاش. وأمامنا ثلاثة مخلوقات: واحد له خاصية معينة، واثنان متطابقان، دون المادة التي يوشك المخلوق أ أن ينقلها إلى أحدهما. وبعد فترة زمنية معينة، تنتقل المادة الوراثية من الكائن الحي (أ) إلى الكائن الحي (ب). يحتوي المخلوق B الآن على مادة وراثية مختلفة عن المخلوق A والمخلوق C (المخلوق B ليس المخلوق A). في الواقع، سيحتوي المخلوق (ب) ونسله على مادة وراثية جديدة، والتي ليس المصمم الذكي مسؤولاً عنها، وهذا يتناقض مع ادعاءاتك بأن المصمم الذكي وحده هو الذي يمكنه إدخال كائنات بمواد وراثية جديدة. وبالمناسبة، إذا حاولت نقل الطعام من طبق إلى طبق، فلا بد أنك رأيت أنه من غير الممكن نقل جميع محتويات الطبق الأول إلى الطبق الثاني بشكل كامل، لذلك إذا كنت تعتقد أن المادة الوراثية المنقولة قد تم نقلها في السلامة الرقمية (مع تصحيح الأخطاء، والمجموع الاختباري، وما إلى ذلك) فأنت مخطئ، وبالتالي فإن المادة المنقولة ليست بالضرورة نفس المادة الأصلية، وبالتالي، ستعرف ما هي التأثيرات الإضافية التي يمكن أن تنشأ عن مثل هذه النسخة الكيميائية المشوهة. من الواضح أن عملية النسخ تنجح عادةً بحيث يتم امتصاص الميزة بواسطة البكتيريا الجديدة، لكن النقطة المهمة هي أنه لا توجد فرصة لأن تتم عملية النسخ بشكل مثالي ومن هذه الأخطاء، يمكن في بعض الأحيان إنشاء تقدم متبقٍ وأحيانًا لا يحدث ذلك.

  628. أحيائي
    الخطأ هو فهمك للتطور. وليس التطور . لا يزال أمامك الكثير لتتعلمه (على الرغم من أنك تحاول بالفعل تعليم الآخرين أن التطور خطأ)

  629. إن خطط موليك، كما ذكرنا بشكل عام كما ترسم، أمر لا يمكن دحضه بالفعل. لكن الأوصاف الحالية للمخطط هي في الواقع غير قابلة للدحض. ومن هنا وجب تعليمهم.

    ومن الواضح أنه يمكن نقل البلازميدات بين البكتيريا، وما إلى ذلك. ولكن وفقا للتطور، تطورت جميع الجينات المذكورة أعلاه تدريجيا من جينات أخرى. في حين أن المثال الذي قدمته يوضح عملية نقل بسيطة للجينات من بكتيريا إلى بكتيريا.

  630. الغوغائية:
    هذا ليس ما قلت. حاول ثانية.
    يعرف ماذا؟
    في الواقع، لا تحاول - أنا آسف على الوقت الذي أمضيته في الدردشة معك

  631. علم الأحياء الرياضي,
    أنا لا أقبل مبدأ التأكيد ولكن هذا لا يهم والأمر لا يتعلق بالتخطيط الذكي مقابل التطور بل يتعلق فقط بالتخطيط الذكي. وأكرر مرة أخرى، لم أؤكد صحة أي ادعاء بسبب التطور.
    ونواصل: إذا كان من المستحيل دحض هذا المخطط، فبأي مبدأ تقللون من قدراته؟ فهو لا يخضع لأي قوانين.

    بخصوص المقال: لا أفهم إصرارك وسبب إضافة الكلمات واستخدام الكلمات للتهرب (هذه المرة أضفت: المقال الذي ربطته في الواقع لا يوضح جدوى نظام جديد دون تخطيط).
    كل ما أقوله هو أن هناك آلية في الطبيعة تقوم باستيعاب مادة وراثية في البكتيريا لم تكن موجودة فيها من قبل. إن البكتيريا التي تحمل المادة الوراثية الجديدة هي، حسب مبدأك، إنتاج جديد. على سبيل المثال، تتمتع البكتيريا الآن بقدرة جديدة على الدفاع عن نفسها ضد دواء جديد، وذلك عن طريق إدخال مادة وراثية فيها.
    أعني أننا خلقنا مخلوقات جديدة (وفقًا للغتك، لأنه تم إدخال مادة وراثية جديدة إليها) وليس بواسطة المصمم الذكي. لقد كتبت في الماضي أن المخطط الذكي وحده هو الذي يمكنه القيام بذلك.
    هل كتبت شيئا غير صحيح في الواقع؟ هل توافق على أن هناك آليات تسمح بإدخال مادة وراثية جديدة إلا عن طريق المصمم الذكي؟

  632. شموليك، في مثل هذه الحالة، من المستحيل بالفعل دحض مثل هذا المخطط. لكن تذكر أن الأمر يتعلق بالتصميم الذكي أو الخلق مقابل التطور. وإذا كنت تصر على مخطط عام فأنا أحيلك مرة أخرى إلى مبدأ التأكيد.

    المقالة التي قمت بربطها لا توضح حقًا جدوى النظام الجديد دون تخطيط. تقوم البكتيريا ببساطة بنقل الجين الذي يرمز إلى إنزيم يكسر المضادات الحيوية (بيتا لاكتاماز على سبيل المثال). مثلما يمكنك نقل العجلات من سيارة جيب إلى سيارة. لا يوجد إنشاء نظام جديد هنا.

    يا مايكل، قلت: "من الواضح أنه إذا تبين أنه لا توجد علاقة بين الشجرة الناشئة من الجين أ والشجرة الناشئة من الجين ب (وهو ما هو متوقع إذا لم يكن هناك تطور) فسيتم دحض التطور" "-

    بمعنى، إذا وجدت قطعة جينية في الشمبانزي، أقرب وراثيًا إلى كلب مثلًا من الإنسان، فهل هذا يدحض التطور بالنسبة لك؟

  633. الكشكشولوجية:
    الجملة "فيما يتعلق بالعلم الحقيقي، لقد تم دحض التطور منذ فترة طويلة بالنسبة لي" مثيرة للاهتمام حقًا.
    يجب عليك أن تقرر ما إذا كنت تتحدث عن "من حيث العلم الحقيقي" أو "من حيث أنت" لأنه لا يوجد أي اتصال بين الاثنين.
    هناك علماء حقيقيون في العالم وبالنسبة لهم - ليس هذا علمًا حقيقيًا فحسب، بل إن النظرية صحيحة بما لا يدع مجالًا للشك.
    انكم مدعوون لقراءة هذا:
    http://www.interacademies.net/File.aspx?id=6150
    ومن الواضح أنه إذا تبين أنه لا يوجد أي ارتباط بين الشجرة الناشئة من الجين أ والشجرة الناشئة من الجين ب (وهو ما هو متوقع إذا لم يكن هناك تطور) فسيتم دحض التطور.
    ومع ذلك، فمن الواضح أن النتائج تشير إلى عدم وجود علاقة ارتباط فحسب، بل إنها مرتفعة بالفعل.

  634. علم الأحياء الرياضي,
    انتقل إلى الأعلى فيما يتعلق بإجابتي فيما يتعلق بمعيار التأكيد وفي ما يتعلق بالتطور، فأنا لست في مستوى رفضه وقبول رأي الإجماع ولكن في النقاش معك، لم أقم بأي ادعاء على أنه صحيح على أساس التطور.

    دعونا نفعل ذلك ببطء، دون إضافة الكثير من المطالبات:
    أنت لا تفهم، أنا لا أتحدث عن نظرية الخلق أو التخطيط الذكي ولكن عن المخطط الذكي وهذا لا يمكن دحضه، ولا حتى حسب الرابط الذي قدمته بنفسك. هل توافق على هذا الادعاء (من فضلك لا تجيبني بخصوص التصميم الذكي أو الخلق، فهما لا يهمانني في هذه المرحلة)؟

    بخصوص المقال: أنت مخطئ. حسب رأيك، تم خلق "مخلوقات جديدة" لأن البكتيريا تختلف قليلا عن بعضها البعض وتقوم بنقل المادة الوراثية من واحدة إلى أخرى، وهي مادة تمتلكها إحداها ولا تمتلكها الأخرى. ليس لدي أي فكرة عما إذا كانت المادة الوراثية قد تم نقلها كليًا أم جزئيًا فقط، ولكن من الواضح أن هذا هو الادعاء. ومن بين أمور أخرى، هناك ادعاء في الرابط التالي بأن هذه هي الطريقة التي تطور بها البكتيريا مقاومة للأدوية.
    http://en.wikipedia.org/wiki/Horizontal_gene_transfer#Viruses

    هل تغير هذه المقالة رأيك بشأن الادعاء القائل بأن المصمم الذكي وحده هو الذي يمكنه إنشاء مخلوقات جديدة؟

  635. مايكل، من حيث العلم الحقيقي، التطور منذ فترة طويلة تم دحضه بالنسبة لي. من حيث العلوم الزائفة - في الواقع، لا يمكن دحض النظرية التي لا تحتوي على اختبارات تفنيد. وللدليل: ما هي النتيجة التطورية التي تدحض الادعاء الذي ربطته في المقال؟

    شموليك، مرة أخرى، فإن نظرية الخلق هي في الواقع علمية. ففي نهاية المطاف، إذا أثبتت أن المخلوقات تطورت تدريجياً، فسوف تدحض الادعاء بأن نظرية الخلق صحيحة. لذا، فيما يتعلق بالمعيار الذي يعتبر أن وضع نظرية الخلق كتصميم علمي وذكي ليس كذلك. إن التقييد المتعلق بمركب لا ينفصل ليس تعسفيًا. لأنه لا يوجد عامل ذكي قادر على إنشاء نظام وظيفي تدريجيًا. لقد زعمت أن ذلك سيكون ممكنا في المستقبل، ولكن هذا الادعاء لا يدعمه أي دليل.

    علاوة على ذلك، هل توافق على أن التطور ليس علميًا أيضًا؟ ففي النهاية، إذا لم يكن كلاهما علميًا، فلماذا يتم تدريس التطور في فصول العلوم؟

    وبالمناسبة، مازلت أنتظر إجابتك فيما يتعلق بمعيار التأكيد.

    فيما يتعلق بالمقال الذي أحضرته، ليس من الواضح بالنسبة لي ما الذي يثبت التطور هناك بالضبط. بعد كل شيء، إذا كان الأمر يتعلق بنقل الجينات الموجودة، فلا يوجد تطور لنظام جديد هنا.

  636. علم الأحياء الرياضي,
    أنت تستمر في الادعاء بأن التخطيط الذكي يؤدي حتماً إلى المخطط الذكي، لكن هذا ترتيب خاطئ لعرض الأشياء. الترتيب الصحيح للأشياء: مخطط ذكي (فهو مصدر كل شيء) -> تخطيط ذكي -> العالم الذي نراه حولنا.
    نظرًا لأنه ليس من الممكن، مقدمًا، على الإطلاق، دحض المخطط الذكي (هكذا يقول أيضًا الرابط الذي أحضرته بنفسك)، فإن أي شيء تقول إنه قادر على فعله، سيذهب. هذه هي المشكلة بالضبط وهذا هو بالضبط السبب وراء عدم تدريس التصميم الذكي في دروس العلوم.
    لقد طرحت الجدال بين ميلر وبيهي باعتباره مسرحية لإثارة إعجاب بيهي، عندما ادعى أن ميلر كان يوضح أن المصمم الذكي يمكن ويمكن دحضه. لقد أوضح ميلر، بحق، أنه دحض أمثلة بيهي للتعقيد الذي لا ينفصم، لكنه أوضح أن المصمم الذكي نفسه لا يمكن دحضه، لأن ما يمكن دحضه هو ما يوجد ضمن النظام الطبيعي للأشياء، وليس ما هو خارج عنه، مثل مصمم ذكي. وأكرر مرة أخرى، حتى لو لم يكن هناك شيء اسمه تعقيد لا ينفصم، فلا يزال من الممكن القول بأن هناك مخططًا ذكيًا، وهو الذي اقترحته، والذي يعرف باسم الله الذي ليس له حدود.

    وما زال يحيرني هو حقيقة إصرارك على أن هناك أشياء لا يستطيع المصمم الذكي الله أن يفعلها (مثل اختراع التطور)، وذلك بقيود تعسفية لا يوجد سبب لإنكارها.

    بالمناسبة، هنا مقال يشير إلى خلق كائنات جديدة (حسب لغتك: المخلوق الجديد هو مخلوق ذو جينات جديدة)؟ https://www.hayadan.org.il/edit-naor-interview-310713/
    ما رأيك بالمقال؟

  637. لقد قلت هذا ولكني لم أذكر بالتفصيل كيفية استخدام هذه الاختبارات وقد قمت بتفصيل ذلك في ردي الأخير.
    لماذا تعتقد أنني كنت أدعي أنني لست من قال ذلك؟
    ومن ناحية أخرى - أنت الذي قلت أنه لا توجد اختبارات دحضية ثم زعمت أنها ليست موجودة فحسب، بل إنها تدحض.

    بالمناسبة، المقالة التي استشهدت بها لا تشير على الإطلاق إلى أن المقالة التي استشهدت بها لا توفر اختبار دحض.
    في الحقيقة أنت تتظاهر بالإشارة إلى اختبار دحض آخر (! أذكرك أنك تفعل ذلك بعد ادعائك بعدم وجود مثل هذا الاختبار!).
    لا يوجد تفنيد هنا لأنها طريقة مختلفة لتحليل شجرة التطور التي لا يزال من غير الواضح ما إذا كانت نتائجها موثوقة (كما يشير الباحث نفسه وكما تتأكد من الجبس).

  638. مايكل، أنت من قال: "إذا كان عدد الجينات هو N، فإنه فقط على أساس الفكرة المقدمة هنا يكون لديه 2 قوة اختبارات دحض N"

    قبل أن تتمسك بادعاءات أخرى مثل تلك التي قدمتها، هل تتراجع عن هذا الادعاء؟

    شموليك. دعنا نذهب إلى طريقتك للحظة. إن اختبار التعقيد الذي لا ينفصم هو في الواقع أكثر توافقًا مع نظرية الخلق. وذلك لأنه وفقًا لنظرية الخلق، تطورت جميع الحيوانات والنباتات كما هي. لذا، إذا وجدت طريقة لإثبات أن الحياة تطورت تدريجياً، فقد تم دحض النظرية. أو بدلاً من ذلك، كما قلت، من السهل جدًا إثبات وجود مخطط ذكي للعالم الطبيعي. في الواقع، يمكن أن يسمى هذا حقيقة وليس نظرية. فيما يتعلق بمناقشة بيهي مع ميلر: يقدم ميلر نظام TTSS للدحض، والذي يتضمن عدة أجزاء مشتركة مع السوط. ما لم يظهره (وما كان يجب أن يظهر في الواقع)، هو أن هناك بالفعل خطوات تدريجية بين ttss والسوط. دحض ادعائه - يحتوي كل من الهاتف الخليوي والساعة على بطاريات وشاشة وأسلاك وما إلى ذلك. ولكن لا يوجد انتقال تدريجي بين الساعة والهاتف الخليوي.

    إريك، لقد دحضت الإدعاء حول الجينات الشمية في الدلافين. لم أرى كيف يمكنك التوفيق بين ذلك وادعاء كوين. تبين لك أكثر الأخطاء التي تظهر في كتابه؟

  639. كتب جيدة:
    "السمكة بداخلنا"
    "لماذا التطور صحيح"
    يوجد في نهاية الكتب مصادر والكثير من المواد لمزيد من القراءة، يوصى بها بشدة.

  640. علم الأحياء الرياضي,
    شكرا على الرابط. فأجاب على سؤالي هل يمكن دحض المصمم الذكي؟ من المقال:
    صحيح أنه لا توجد طريقة لتزييف التأكيد الصريح على وجود مصمم كوني.
    ما هو هناك للقول هنا؟
    وبما أنني لا أستطيع دحض وجود المصمم الذكي، الذي هو الأساس الأساسي لنظرية التصميم الذكي، إذ كان على شخص ما أن ينفذ التصميم الذكي (هل تدعي خلاف ذلك؟)، فإن هذا يفسر لماذا يُمنع منعًا باتًا تدريس هذه النظرية في دروس العلوم. هذا يجيب على سؤالي الطريقي والجواب هو، بشكل لا لبس فيه: حتى لو كان التطور قادرا على إنتاج شوتون، فإنه لن يدحض وجود المصمم الذكي. ومن ثم، فلا سبيل إلى وضع قيود اعتباطية لا تقبل الجدل على المصمم الذكي، كما تفعل أنت (عندما تزعم أن المصمم الذكي غير قادر على إنتاج نظام يتطور). إن مفهوم المصمم الذكي الذي لا يمكن دحضه يسمى في اللغة اليومية: الله.

    أبعد من ذلك، لم تكن المقالة مقنعة فيما يتعلق بأي من ادعاءاتها (ليس لدي القوة للجدال حول مستواها المنطقي، لكن من السهل القيام بذلك. إنها مليئة بالأخطاء المنطقية. وكان القسم الخاص بعلم الفلك غريبًا):
    1. لم يذكر في أي مكان في المقالة أن التصميم الذكي سوف ينهار إذا تمكن التطور من إنتاج شوتون، ولكن ما سينهار هو ادعاء بيهي بالتعقيد الذي لا ينفصم.
    2. الافتراض بأن كل خطوة في التطور في حد ذاتها يجب أن تكون مبررة، فهي لك وحدك وهذا غير صحيح. ليس كل خطوة في التطور يجب تبريرها. لا يوجد إله يقتل على الفور مخلوقًا يبدو أقل ملاءمة لبيئته وفقًا لبعض الخوارزميات التي يديرها. إن العملية التطورية تدريجية وبطيئة للغاية، وعلى طول الطريق، يمكن للجينات التي كانت محايدة في مرحلة معينة أن تصبح الأساس لخلق قدرات مفيدة وجيدة.

    ماذا عن المقالة التالية؟ https://www.hayadan.org.il/edit-naor-interview-310713/

  641. في فكرة أخرى - يبدو لي أنه لم يحاول أحد بعد وضع القزم في شجرة التطور.
    موضوع مثير للاهتمام للبحث!

  642. وبالمناسبة، بالطبع التخطيط الذكي (لو كان هناك شيء من هذا القبيل) لا يدل على أي ذكاء لـ "المخطط" الذي أعطى النعام أجنحة، والذي جعل الأفاعي والحيتان تنمو أرجلها تحت الجلد، والذي أعطى الطيور جينات معاقة للأسنان و - كدت أنسى - من الذي جعل مخلوقات مثل هذا المبرمج ممكنة "علم الأحياء الرياضي"

  643. علم الكشكوش في البلطيق:
    لقد قلت أنه لا توجد اختبارات دحض، لذا تحاول الدحض الآن؟
    هذا يعني أنك كذبت عندما قلت أنه لا يوجد اختبار دحض.
    وبما أن هذه ظاهرة إحصائية، فيجب فحصها إحصائيًا أيضًا، حتى لو كان هناك انحراف هنا وهناك - فعندما تتطابق الأغلبية، تحصل على تأكيد بأن فرصة وجود أي تطابق بالصدفة هي ببساطة صفر.
    فيما يتعلق بالشوتون - أنا مندهش: أنت لا تنظر حتى إلى ويكيبيديا؟
    http://en.wikipedia.org/wiki/Evolution_of_flagella

  644. شموليك، عندما تتمكن من إظهار مدى فائدة كل خطوة، فسوف تدحض ادعاء التخطيط الذكي. حتى الآن، لم يفعل أحد. وعلى حد علمنا فإن هذا مستحيل.

    بخصوص اختبار النقض، انظر هنا:

    http://www.evolutionnews.org/2006/01/intelligent_design_is_empirica001819.html

    والصحيح، إذا ثبتت نظريتك فهي حقيقة موجودة. بالنسبة لي، ومن حيث الأدلة العلمية، فإن التصميم الذكي هو بالفعل حقيقة موجودة.

  645. علم الأحياء الرياضي,
    نعم، يمكننا أن نصنع سيارة تدريجياً:
    1. عندما تكون كل خطوة مفيدة
    2. أن تكون بعض التدابير مفيدة والباقي محايدة
    3. عندما تكون بعض التدابير مفيدة، فإن بعضها محايد، وبعضها ضار.
    سنكون قادرين على فعل أي شيء وسيكون مستوى سيطرتنا على المستوى الذري إن لم يكن أقل من ذلك. إذا كنا نستطيع، فلماذا لا يكون المخطط الذي خطط لنا؟

    لا يوجد شيء اسمه اختبار تأكيد: إذا تم إثبات نظريتك، فإنها لم تعد نظرية بل حقيقة موجودة، لكن في العلم لا يوجد شيء اسمه نظرية مثبتة بل فرضيات بمستوى أو آخر من مستويات القوة. ومن الواضح أن الادعاء الذي لا يمكن دحضه ليس ولا يمكن أن يكون نظرية علمية، ومثل هذا الادعاء لا يستحق أن يدرس في دروس العلوم.

    بعض الأسئلة التي لم تتم الإجابة عليها:
    1. يرجى تقديم رابط للمقال الذي ينص كبديهية على أن المصمم الذكي لا يمكنه إنتاج نظام سيخضع للتطور. وحقيقة أن بيهي قال هذا (كما كتب: "حسب فهمه" لا يجعله صحيحا. فهو لم يفهم بشكل صحيح. بالنسبة لي يبدو وكأنه قرار تعسفي غير مدعوم في أي مكان، إلا في الرابط القديم لقد قدمت، ما لم تريني، في جملة رياضية، أنه على المستوى الأكثر مبادئًا، ليس من الممكن تصميم نظام سيخضع للتطور، وعدم الاحتمالية، مهما كانت متطرفة، ليست عائقًا أو قيدًا أمام المصمم الذكي.
    2. لا أفهم اختبار التفنيد الخاص بك: لماذا، إذا كان التطور قادرًا على إنتاج مركب شراعي، فهل يدحض ذلك وجود المصمم الذكي؟ إذا كان بإمكاني الوصول من تل أبيب إلى حيفا عبر الطريق رقم 4، فهل يعني ذلك أن الطريق رقم 2 غير موجود؟
    3. إذا كنت أؤمن بمصمم قادر على فعل ما يستطيع مصممك وكذلك تطور التصميم كما يصفه العلماء، فهل أؤمن بمصمم ذكي ليس مصممك؟

    شكر

  646. لقد قلت أنا وأبي لتومي أن مسائل العلم تتم مناقشتها بين العلماء. هل المحاكم هي السلطة في المسائل العلمية؟ جيد ان تعلم. ما هي الخطوة التالية؟ وبعد ذلك، عندما يكون هناك نقاش علمي، هل نلجأ إلى وكلاء التأمين؟

    ميكا، المقالة التي قمت بالربط بها مبنية على دراسة من عام 82. ومنذ ذلك الحين تدفقت مياه كثيرة في مصب هاردينج وكانت معجزة، حيث تم العثور على نتائج تتناقض مع ما جاء في المقال:

    http://www.nature.com/news/phylogeny-rewriting-evolution-1.10885

    وبما أنك قررت بنفسك أن كل حديقة تشكل في حد ذاتها اختبارا دحضيا. وبما أن هذا المقال فيه عدة مئات من التناقضات، أو على حد تعبيره:

    "لقد بحثت في الآلاف من جينات microRNA، ولم أتمكن من العثور على مثال واحد من شأنه أن يدعم الشجرة التقليدية."

    ففي نهاية المطاف، يتم دحض التطور بشكل مباشر بشكل مباشر....

    شموليك، أسأل مرة أخرى - هل يستطيع المصمم أن يصنع سيارة تدريجياً عندما تكون كل خطوة فعالة بمفردها؟ وماذا عن اختبار التأكيد؟ هل تقبل به

  647. علم الأحياء الرياضي,
    لم أحصل على إجابات
    أنا لا أفهم اختبار الطعن الخاص بك. لماذا، إذا كان التطور قادرًا على إنتاج شوتون، فهل هذا يدحض وجود المصمم الذكي؟ إذا كان بإمكاني الوصول من تل أبيب إلى حيفا عبر الطريق رقم 4، فهل يعني ذلك أن الطريق رقم 2 غير موجود؟
    الرجاء إرسال رابط مكتوب يدعي أنه سيتم دحض التصميم الذكي إذا تم إنشاء Shoton في المختبر. قال بيهي هذا وصاغه بعناية: قال: "التصميم الذكي، كما أفهمه..." هل هذا مبدأ أساسي للتصميم الذكي وأين يمكنك الحصول على تفسير مكتوب (لأنه غير مكتوب في ويكيبيديا) ل سبب هذا القيد الغريب والاعتباطي الذي يطبق على المصمم الذكي وعلى هذه الخلفية أسأل مرة أخرى: إذا كنت أؤمن بمصمم ذكي قادر على إنتاج التطور كما نفهمه، فهل مصممي الذكي يختلف عن ذكائك؟ مصمم؟

  648. ببليوغرافيا مثيرة للشفقة:
    التطور لديه ولا يزال لديه اختبارات تفنيد. في الواقع - إذا كان عدد الجينات N، فهذا فقط على أساس الفكرة المقدمة כאן لديها 2 إلى قوة N من اختبارات الدحض

  649. علم الأحياء الرياضي. لقد صرح الحكيم والأهم مني مرارًا وتكرارًا قانونيًا أن التصميم الذكي هو خلق مقنع أو خلق بطريقة صحيحة سياسيًا. لن يغير ذلك أي قدر من المراوغة النحوية.

  650. حسنًا، أولاً سأبدأ بالقول إنني لا أتحدث عن نظرية الخلق. لذا فإن العثور على أحفورة الكنغر لا ينتمي إلى التخطيط العقلاني والديني، ولا ينتمي إلى نظرية الخلق. على سبيل المثال، عاشت الأسود في إسرائيل. لكن على حد علمي، لم يتم العثور على حفرياتهم في إسرائيل مطلقًا. وهو ما قلته - لا يوجد اختبار دحض للتطور.

    إلى إريك-

    اليوم، حتى أعظم مؤيدي التطور يتعاملون بحذر شديد مع كلمة "الجينات الكاذبة". هناك اسباب كثيرة لهذا. ومن بين أمور أخرى، اكتشفوا أن الكثير مما يعتبر زائفًا مهم جدًا في التحكم بالجينات والتعبير عنها. وانظر أنها معجزة:

    http://www.newscientist.com/article/dn19211-how-does-a-bowhead-whale-smell-quite-well-actually.html

  651. أما بالنسبة لاقتراح موقع المعرفة لوزارة التربية والتعليم – أقترح أن يبادروا بعريضة وطنية. وربما يحفزه الكثير من ناخبي الوزير على إجراء التغيير كما وعد.
    بالنجاح

  652. السيرة الذاتية
    مجرد نماذج من الأسئلة:
    ما السبب حسب نظريتك الخيالية: الدلافين لديها جينات تشم الهواء مثل الثدييات خارج الماء (رغم أنها في الماء) وليست جينات تشم الماء مثل الأسماك. وهذه الجينات هي جينات كاذبة يتم إسكاتها بواسطة الطفرات. ما هو "منطق التخطيط" وراء ذلك؟

  653. شموليك
    لا أفهم ما هو المعقد هنا. انظر إلى الأدلة الموجودة في العالم وانظر إلى معرفتنا بالعلوم الكيميائية والبيولوجية. والآن - لنبحث عن تفسير لما نراه بناءً على ما نعرفه. و …. انتظرها... دعونا نختار التفسير الوحيد الذي وجدناه ونواصل تطويره. وسوف نتحقق معه ونشرح له جميع الملاحظات. سوف نتحقق مما إذا كان الشرح لا يتعارض مع شيء من مجال آخر. سنحاول دحض التفسير.

    هذا كل شيء…. لقد فعلت ذلك بالفعل ووجدت أن التطور الدارويني هو التفسير الذي كنا نبحث عنه.

    الآن - للتوصل إلى تفسير جديد، تحتاج إلى سبب وجيه للغاية. و"لأنه هكذا هو مكتوب في التوراة" ليس سببا وجيها.

  654. علم الأحياء الرياضي,
    أنا لا أفهم اختبار الطعن. لماذا، إذا كان التطور قادرًا على إنتاج شوتون، فهل هذا يدحض وجود المصمم الذكي؟
    إذا كان بإمكاني الوصول من تل أبيب إلى حيفا عبر الطريق رقم 4، فهل يعني ذلك أن الطريق رقم 2 غير موجود؟

    شيء صغير آخر أود الحصول على إجابة مفصلة له: أنا أؤمن بمصمم ذكي قادر على إنتاج التطور كما نفهمه. هل يختلف المخطط الذكي الخاص بي عن المخطط الذكي الخاص بك؟

    فيما يتعلق بسؤالك، يمكننا أن نفعل ذلك بخطوات صغيرة كما نشاء، ذرة بذرة أو بخطوات أكثر خشونة. إذا كان بإمكاننا القيام بذلك، كنوع مصمم ذكي، فلماذا لا نفعل ذلك المصمم الذي خلقنا؟

  655. علم الأحياء الرياضي
    ما هذا الهراء "انتبه مرة أخرى - مبدأ التأكيد لا يقل أهمية عن مبدأ الدحض".
    ؟

  656. علم الأحياء الرياضي
    لقد تم فضح مثال العصا البكتيرية منذ سنوات مضت... أنت لا تعود حقًا إلى ذلك، أليس كذلك؟

    بيهي نفسه يتحدث كثيرًا من الهراء وهو (وديمبسكي) يحرفون قوانين الفيزياء للترويج لأجندتهم المسيحية.

    إذا كنت يهوديًا مؤمنًا - فابتعد عنهم. اختر لك دجالاً آخر، أمنون يتسحاق، هكذا...

  657. هنا على الموقع يتم إثبات ذلك باستمرار. إن تسمية نفسك بـ "علم الأحياء الرياضي" والإثبات في كل رد أنك لا تفهم شيئًا لا في علم الأحياء ولا في الرياضيات لا يمكن أن يكون إلا أمرًا دينيًا على ما يبدو.
    http://www.stephenjaygould.org/ctrl/news/file002.html
    http://www.nrg.co.il/online/55/ART1/748/479.html
    http://www.calcalist.co.il/local/articles/0,7340,L-3480323,00.html
    http://articles.latimes.com/2012/apr/26/science/la-sci-religion-analytical-thinking-20120427

  658. علم الأحياء الرياضي
    وحسب ما نعرفه وفقا لنظرية التطور، فإن الجرابيات لم تعيش في منطقتنا أبدا. وفقا للديانة اليهودية كانوا يعيشون هنا.

  659. شموليك
    هناك دراسات وجدت علاقة عكسية بين الذكاء والمعتقد الديني. وهذا لا يعني أن كل مؤمن أحمق. ولكن تم إثبات الاتصال

  660. قطعاً. ولهذا السبب ادعى أن هذا هو ما الذكي وهذا هو اختبار دحض لها. مرحبًا بك في دحض ذلك ولا مرحب بك في عدم ذلك.

    "إذا تمكنا من إجراء مثل هذه التغييرات بالفعل اليوم، فإن التغييرات في المستقبل ستكون أكثر دراماتيكية، والتي سيتم إجراؤها تلقائيًا، دون تدخل بشري، "-

    هل تقصد شيئا أسلوب المحولات؟ وما هي التكنولوجيا المطلوبة لذلك؟ وبالإضافة إلى ذلك، هل من الممكن بخطوات صغيرة؟

  661. علم الأحياء الرياضي,
    عندما كتبت أنني أريد الرابط، كنت أقصد رابطًا مكتوبًا وليس فيديو.
    لا أفهم ادعائه بأن تكوين شوتون في المختبر يتناقض مع وجود التخطيط الذكي. إذا تم خلق شوتون في المختبر، عن طريق التطور، فلماذا يدحض هذا وجود المصمم الذكي؟ ربما في الماضي، ومن الناحية العملية، كان المخطط الذكي هو الذي خلق اللقطة ويمكن للتطور أن يفعل ذلك أيضًا. ما هي المشكلة هنا؟ على أية حال، هل تتفقون معي على أن المخطط الذكي الذي أعرضه لم يخف عليه خلق شوتون في المختبر؟
    إذا كنت أؤمن بمصمم ذكي قادر على تطوير نظام يتطور بدونه، فهل يعني ذلك أنني أؤمن بنظرية التصميم الذكي التي تختلف عن النظرية التي تؤمن بها؟

    نحن، ككائنات مخططة ذكية، نستطيع اليوم بالفعل زرع مواد وراثية جديدة في كائنات مختلفة
    http://en.wikipedia.org/wiki/Genetically_modified_organism
    هل إدخال مادة وراثية جديدة إلى كائنات مختلفة يجعلها مخلوقات مختلفة في نظرك؟
    ومن الواضح أنه لن يمر وقت طويل حتى نتمكن من الانطلاق بهذه القدرات. إذا تمكنا من إجراء مثل هذه التغييرات بالفعل اليوم، التغييرات التي ستكون في المستقبل أكثر دراماتيكية، والتي سيتم إجراؤها تلقائيًا، دون تدخل بشري، فلماذا لم يتمكن المصمم الذكي من القيام بذلك، بعد كل شيء، لقد خلقنا وهو كثير أكثر تعقيدًا من مجرد صنع سمكة متوهجة؟

  662. ومن غير السار جدًا بالنسبة لي أن أضيف، لكن عندما كنت أتعامل مع إرجاع الوقت رأيت أشياء "تم حذفها" حول هذا الموضوع

  663. إنه أمر مزعج للغاية بالنسبة لي أن أقول ذلك، لكن إذا تحدث إليك التصميم الذكي، فسيكون هذا "دليلًا" على وجوده

  664. علم الأحياء الرياضي,
    لا أريد أن أتحدث عن التطور. أنا معك. لا يوجد مثل هذه التوراة.

    حد اعتباطي، لأنه لا يوجد سبب لافتراض مثل هذا الحد، خاصة في ظل الإنجازات الهندسية للبشرية، كمخططين أذكياء (سنتحدث عن ذلك لاحقًا).
    1. من فضلك أرني رابطًا واحدًا يطالب باختبار الدحض هذا.
    2. من فضلك اشرح لي سبب وضع هذا الحد ولماذا تقبل أنت بنفسك هذا الحد التعسفي؟ لماذا لا نحرر المصمم الذكي من هذا القيد الغريب ونقول: إنه قادر على خلق كل شيء: كائنات لا تتغير وكائنات تتغير. أليس الأمر أبسط؟
    3. إذا كنت أؤمن بمصمم ذكي قادر على تطوير نظام يتطور بدونه، فهل يعني ذلك أنني أؤمن بنظرية التصميم الذكي التي تختلف عما تؤمن به؟

    من الناحية الهندسية، ستتمكن الطابعة ثلاثية الأبعاد قريبًا جدًا من أخذ المواد الخام وصنع سيارة منها. واليوم بالفعل، تقوم الطابعات بأخذ المواد الخام وتصنيعها في أجهزة ذات أجزاء متحركة. انتظر 5 سنوات أخرى.
    البشرية، إن لم يكن بالفعل اليوم، ستكون قادرة في غضون سنوات قليلة على الانطلاق، وإجراء تغييرات على مستوى النانومتر، إن لم يكن أقل من ذلك، على أي مادة تختارها وبمساعدة هذه القدرة والخوارزميات المتطورة، الروبوتات (ليس فقط في المحاكاة الحاسوبية) ستتمكن من برمجة الروبوتات الأخرى التي ستكون مختلفة عن الروبوتات الأصلية كما يحلو لك. وسوف تصبح مخلوقات أخرى. لا يوجد حد هندسي لهذا. هل هناك نصيب للعاقل الذي خلقنا؟

    بالمناسبة، هذه هي بالضبط الطريقة التي يتم بها بناء السيارة - تقوم الروبوتات ببناء السيارة من الألف إلى الياء حتى آخر مكون (المرآة؟). ما الذي يحاول هذا المثال قوله؟

  665. ما هو التعسفي هنا؟ لقد اقترح أنصار التصميم الذكي هذا الاختبار الدحض. حتى أنت، كعميل ذكي، غير قادر على إنشاء نظام معقد تدريجيًا. خذ على سبيل المثال سيارة. تتطلب أبسط سيارة عددًا من المكونات لتعمل: العجلات، والمحرك، والمحاور، وما إلى ذلك. إنه شيء يمكن اختباره، أي تجريبيا.

    ومن ناحية أخرى، أود أن أراك تشير إلى مبدأ التأكيد. بل ويقدم أيضًا اختبارًا دحضًا للتطور.

  666. علم الأحياء الرياضي,
    لم أفهم لماذا.
    من أين تستمد هذا الحد التعسفي؟ أرني رابطًا واحدًا لاختبار الدحض هذا.
    نحن، الجنس البشري، سنكون قادرين في غضون 10 سنوات على إنتاج روبوتات تبرمج نفسها لتصبح أشياء لم نفكر فيها أبدًا (وإن لم يكن في 10 سنوات ففي 20 عامًا)، بما في ذلك الكائنات الأخرى، بما في ذلك دمج العناصر العضوية فيها و ما لا. إذن هنا مثال على التصميم الذكي، الذي يسمح بتطور غير محدود. مرحبا بكم في الاعتقاد بأننا لم نصل إلى هناك، ولكن ليس هناك حد هندسي لذلك ونحن نتسابق نحوه بوتيرة مذهلة. إذا كنا نستطيع فعل ذلك، لماذا المصمم الذي خلقنا، ما هي المعلومات التي لديك والتي لا أملكها، والتي تخبرك أن المصمم لم يكن بإمكانه تصميم نظام يقوم بالتطور؟

  667. حسنًا، هذا سهل جدًا.

    هنا روبوت مكرر. هل يدل على التخطيط؟

    السبب الذي يجعل المخلوق الذي يطور نظامًا معقدًا يدحض التصميم الذكي، لأنه وفقًا للتصميم الذكي لا توجد خطوات وظيفية في تكوين مثل هذا النظام. لذا، إذا تمكنت من إثبات خلاف ذلك، فسيتم دحض النظرية. لاحظ مرة أخرى - مبدأ التأكيد لا يقل مساواة عن مبدأ الدحض.

  668. علم الأحياء الرياضي,
    نظام يشير إلى التخطيط –> التخطيط، وهذا في نظر المراقب
    كلمة معقدة - هي في نظر الناظر
    لا يهم.
    ما لم أفهمه هو السبب، إذا تبين أن الإنتاج يمكن أن يطور تدريجيًا نظامًا معقدًا (مهما كان معنى ذلك)، فإن ذلك يدحض التصميم الذكي. هل التصميم الذكي ليس ذكيا بما فيه الكفاية؟

  669. شموليك، التصميم الذكي يمكن دحضه أو تأكيده ويقدم أيضًا نبوءات. أحد التنبؤات، على سبيل المثال، هو أننا سنجد أنظمة تشير إلى التخطيط. النبوءة التي تحققت مرارا وتكرارا. توقع آخر هو أن المخلوق لا يمكنه تطوير نظام معقد تدريجيًا. وبمجرد دحض ما سبق، فإنك تدحض التخطيط الذكي. لاحظ أن هذه هي المعايير التي تقترحها النظرية. من فضلك لا تخلط بينه وبين الخلق.

  670. علم الأحياء الرياضي,
    سأمنحه المعجزات، إذا أراد أن يشرح عن أحفورة الكنغر.
    أطرح ثلاثة أسئلة بسيطة ودعنا نرى مدى صعوبة عدم الإجابة عليها:
    1. هل يقدم التصميم الذكي دحضه، أي كيف يمكن دحض التصميم الذكي؟
    2. ما هي التنبؤات الحصرية التي يقدمها التصميم الذكي، أي هل هناك تنبؤات يقدمها التطور ويقدم التصميم الذكي تنبؤات أخرى؟
    3. ما هي الأشياء غير الممكنة في ظل التصميم الذكي. هل المعجزات والعجائب ممكنة في ظل التصميم الذكي؟

    الحقيقة هي، إذا لم يكن لديك الوقت/القوة/الرغبة، فأنا مهتم في الغالب بإجابة السؤال رقم 1.

    المعجزات:
    ليس لدي أي فكرة عن كيفية الإجابة على سؤالك لأنني لا أستطيع إلا استخدام افتراضاتي الشخصية وحكاياتي والشيء الأكثر ملاءمة الذي فكرت به هو الذهاب إلى الويكي... (كم هو غبي ومراوغ مني)
    http://en.wikipedia.org/wiki/Religiosity_and_intelligence

  671. شموليك ونسيم-

    أولاً، لماذا قد تدحض أحفورة الكنغر الموجودة في جبال هاريت نظرية التطور؟

    ثانيًا، يمكن تصنيف التصميم الذكي على أنه علم لأنه يحتوي على أدلة علمية. لا يمكن للنظرية العلمية أن تقدم معيار الدحض فحسب، بل معيار التأكيد أيضًا. هل الادعاء بأن تكرار الساعة يتطلب التخطيط غير علمي؟

  672. صحيح أنه لو كان ذلك صحيحا لما أشركك التخطيط الذكي في التخطيط، ولعاشت في فقاعة - وبالتالي، علميا، لم تكن لديك الأدوات العلمية للتحقق منها، إلا الأشياء الخارجية التي هي مشابهة، مثل تطور أجهزة الكمبيوتر وتطورها والتقدم شبه العشوائي لتطورها، وبالإضافة إلى ذلك فإن "أجهزة الكمبيوتر" لم تكن على علم بذلك. شكرا

  673. لم أقرأ المقال لكن لي رأي فيه.
    الناس الذين هم أسرى آرائهم، ما لا يضيف وحتى لو كان موجودا وصحيحا، سوف يفرضون رأيهم بالقوة، يمكنك التحدث بلطف عن كلمة العلم، ولكن ما ليس محل إجماع، لن يتم التحقيق فيه حتى بالطرق "العلمية".

  674. متشكك
    دون التعمق في النقاش الذي تجريه، إذا أخرسوه، فمن المحتمل أن كلماته لن يكون لها وزن كبير.
    وحقيقة أنهم سمحوا له بتعليم الآخرين ما هي إلا نتيجة للتعددية الغربية.
    إلى جانب ذلك هناك أساتذة يؤمنون أيضًا بالكائنات الفضائية. بالطبع، كل هذا لا يعني أن الأساتذة متمسكون بالحقيقة، بل أنه حتى الأساتذة يمكنهم دراسة وحفظ المادة الأكاديمية وما زالوا متمسكين بخرافاتهم.

  675. http://www.evolutionnews.org/2013/08/what_does_eric075081.html

    فيما يلي معلومات غير خاضعة للرقابة حول ما فعله إريك هادين (ربما أستاذ الفيزياء) في ندوته حول فلسفة العلوم (كانت الندوة تسمى "حدود العلم"). هذه هي الندوة التي تم الافتراء عليها في المقال هنا. سمح أريك هادين للطلاب بتشكيل مناصب بشكل مستقل، عندما عرض أمامهم الأدبيات التي نشرها مؤيدو "التخطيط الذكي" (الاسم المختصر للتخطيط الذكي هو ID). نادرًا ما شارك أريك هادين نفسه في المناقشات ولم يتخذ موقفًا إلا في المواقف التي تم فيها تقديم حجج لا أساس لها من الصحة. تمكن المشاركون من الحصول لأنفسهم على مواد عالية الجودة ضد حجج التصميم الذكي، والتي كان من السهل الحصول عليها بفضل ريتشارد دوكينز ورفاقه.

    كان جميع المشاركين في ندوته طلابًا ممتازين، أي قادرين على التفكير بأنفسهم. حسب ما فهمت - هذه ندوة إذنية، لذلك كان أولئك الذين حضروا الندوة مستعدين عقليًا لسماع آراء غير مقبولة بل وأرادوا سماع مثل هذه الآراء.

    نظرًا لأن العديد من أتباع التطور غير قادرين على الجدال على مستوى ما. إنهم سعداء فقط بالتجادل مع خصم ضعيف (مثل شينغو)، فهذا يمنحهم شعورًا جيدًا بأنهم في الجانب الفائز وأنه ليس عليهم بذل أي جهد لتحقيق الفوز. ولكن عندما يظهر أمامهم خصم قوي مثل البروفيسور إيريك هيدين، فإنهم يهربون من النقاش الحقيقي بإسكاته.

  676. ماشيل
    لا أريد أن أتساءل عن جرة Hingoe (بالطبع سيفعلها Molk أو Nisim) لكني أحببت منطقك :)
    وفيما يتعلق بإجابتك حول الانتقاء الطبيعي: في رأيي، الانتقاء الطبيعي هو إعادة إنتاج الجهل للجيل القادم.
    ببساطة، من أجل الجيل المتضاعف - أدوات "التقليل" من الجهل. (ولكن بسبب الجهل لا يعرفون كيفية استخدامها)

  677. شموليك
    لا أعرف شينوجاوا....
    لكن لدي سؤال لك (وأنت فقط): هل هناك علاقة بين الذكاء والمعتقد الديني؟

  678. بيولوجيا،
    لقد كتبت بالفعل عن الإحراج المتوقع من جانبك، وها أنت تأتي وتثبت لي أنني على حق. هل تقبل حقيقة أن التصميم الذكي ليس نظرية علمية؟

    الرد المتوقع: تهرب وبدلا من الإجابة سأسأل إذا كنت أقبل حقيقة أن التطور ليس نظرية علمية. أنا سألت أولا.

  679. علم الأحياء الرياضي
    من المؤكد أن التطور يتم مناقشته من أجل دحضه. العثور على أحفورة للكنغر في جبال أرارات ودحض التطور.

    وما هي بالضبط فكرة تحول القطة إلى كلب؟

  680. التبشيري,
    اسمك بحد ذاته يمثل مشكلة، لأن اسمك يشير إلى أنك تأتي بجدول أعمال بدلاً من العقل المنفتح.

    تواضع؟ تظهر الأدلة أنه ليس كل شيء له علاقة بالجنس البشري، ولكنه يقول إننا نوع آخر، واحد من العديد من الأنواع الموجودة على كوكب عالق على حافة المجرة، واحد من العديد (في الكون الذي يمكن رؤيته فيه) المادة حوالي 1% والباقي عبارة عن مادة مظلمة وطاقة مظلمة) وأن تطور البكتيريا هنا على كوكبنا الصغير يحدث أمام أعيننا حتى بدون مساعدتنا وبدوننا، بينما تزعمون أن كل شيء يدور حولنا بينما كنت أقتبس ديكارت وكأنه يثبت شيئاً ومازلت أتعلم منك درساً في التواضع؟؟؟
    لا تتردد في أن تكون مهووسًا ومغرورًا كما تريد، سألتزم بالحقائق.

    بالمناسبة، التواضع قيمة إشكالية للغاية وأنا شخصيا أفضل التضامن. ولم ينج الجنس البشري لأنه كان متواضعا، بل بسبب التضامن الذي طوره

  681. والحقيقة هي أن التطور هو اعتقاد ولا يتوافق مع مبدأ الدحض العلمي. وبالتالي ليس علميًا تمامًا. لا توجد طريقة يمكن التحقق منها لدحض التطور. وحتى لو تحولت قطة فجأة إلى كلب، فإن أنصار التطور سيزعمون أن "هذا ما حدث بالفعل". بعد كل شيء، هناك عدد لا يحصى من الأكوان المتوازية وكل شيء ممكن. طوبى للمؤمن.

  682. ويفشلون في فهم ما يُقرأ عند كل منعطف.
    وحتى على سبيل الاقتباس، فإنهم لا يعرفون أو لا يفهمون الفرق الشاسع بين الادعاء "إذا لم يكن أ ثم ب" والادعاء "إذا لم يكن أ ثم لا ب"

    اسمحوا لي أن آخذ "منطق" الأحمق وأقلبه ضده.
    البشر فقط يعتقدون أن هناك تطور. هذه حقيقة. لهذا السبب يمكنك أن تكتب: "إذا كنت تعتقد أن هناك تطور، فأنت إنسان"
    والآن لنأخذ اللامنطقية التي قالها المبشر ونستمد منها زعمه "إذا كنت لا تعتقد بوجود تطور فأنت لست إنساناً"

  683. المبشر اسمه
    قال ديكارت: إذا لم يكن هناك فكر، فأنت غير موجود
    في الختام، سأعرف ما كان من قبل وأخمن كما لو كنت هناك، حيث سنكون أكثر تواضعا قليلا.

  684. سفكان
    حقا،
    التصميم الذكي ليس نظرية علمية لأنه غير قابل للدحض وهو مستقل عن التطور، أي أنه لا يهم حقًا ما هو رأيك في التطور (صحيح أو غير صحيح)، كل نظرية علمية يجب أن تقف بمفردها، وتقدم تنبؤات وتسمح إن دحضه وتصميمه الذكي لا يفعل شيئًا من ذلك إذا كنت ترغب في ذلك، افترض أن التطور غير صحيح وغير موجود، فهل هذا يتقدم بالتخطيط الذكي بمقدار ملليمتر؟ هل يوفر التصميم الذكي تنبؤًا واحدًا؟ هل يشرح لك التصميم الذكي ما يجب فعله لدحضه؟ بالتأكيد لا، وبالتالي التصميم الذكي ليس نظرية علمية، لماذا يصعب فهم ذلك؟

    الرد المتوقع: التذمر من التطور ولكن مرة أخرى، لا علاقة له بالتطور. خطأان لا يصنعان صوابًا، وإذا ناقش الأستاذ التصميم الذكي في فصل العلوم، إلا كتفسير لسبب منع تدريس التصميم الذكي في دروس العلوم، فمن الجيد أنه أُمر بالتوقف عن تدريس التصميم الذكي .

    التبشيري,
    التطور موجود بالفكر الروحي وبدونه (مهما كان معنى ذلك). تراه في البكتيريا كل يوم في عدد لا يحصى من المختبرات وقد فعلوا ذلك قبل وجود الإنسان أو قبل أن يفكر ويصوغ هذه الكلمة. ما الذي يصعب فهمه؟

  685. كل عادة
    - ينكر الناس التطور دون أن يفهموا على الإطلاق ما هو عليه، لذلك يتحدثون هراء.
    بون تون - سأبدأ معك..
    الكاميرا هي نتيجة تخطيط ذكي وهناك عدة علامات على ذلك (تجاهل اسم الشركة المصنعة المحفور عليها بشكل بارز للحظة).

    أولاً - أنها مبنية لغرض محدد. الافتراض الأساسي في التخطيط هو أن له هدفًا. مخطط الكاميرا لديه متر. لم يجد أحد حتى الآن غرضًا لمخطط الحياة. ولم يجد أحد الميكانيكي نفسه بصراحة.

    ثانياً- يتم بناء الكاميرا من عدد صغير (نسبياً) من الأجزاء البسيطة التي لكل منها دور محدد. وهذه علامة واضحة على التخطيط الذكي. وهذا ليس صحيحا بالنسبة للكائنات الحية.

    ثالثاً- أن تكون أجزاء الكاميرا مصنوعة من مادة متجانسة ومناسبة للغرض الذي صنعت من أجله. علامة أخرى واضحة على التخطيط. في الجسم الحي ليس الأمر كذلك.

    دعونا ننظر للحظة إلى مثال بسيط من الحياة. إذا قام شخص ما بتصميم الفقاريات، فمن المنطقي أن يكون للذيل غرض. لكن الوضع ليس كذلك - حيث يتم استخدام الذيل لما لا يقل عن 10 استخدامات مختلفة. حقا لا يعني التخطيط.

    ما أحاول قوله هو أنه لا يوجد سبب للاعتقاد بأن الحياة مخططة، أو أن العالم مخطط له. إذا أتيت بموقف ديني وقررت مسبقًا وجود إله، فلا فائدة من الحديث. لقد اتخذت قرارك بالفعل، بغض النظر عن الأدلة.

    ونقطة أخيرة - أي شخص يقول أن هناك أشياء لا يستطيع التطور تفسيرها فهو ببساطة غير صادق مع نفسه. ولم يعثروا حتى الآن على أي شيء قريب منه.

  686. التصميم الذكي ليس علماً، بل هو دين. ليس من الضروري أن تصدقني (أن تصدق - الحديث عن الدين...). وقد قرر ذلك أحد القضاة في الولايات المتحدة الأمريكية منذ عدة سنوات عندما كان عليه أن يتخذ قرارًا بشأن هذه القضية. لكن حتى العلم ليس كما يبدو تمامًا. لأن العلم لا يقدم دليلاً على صحة الادعاءات بل يقدم فقط دعماً للفرضيات. والفرضيات تعتبر صحيحة حتى يثبت العكس. ولكننا سبق أن رأينا العديد من النظريات التي أصبحت صديقة مقربة، بل وتم دحضها، فلا ثقة في فرضيات اليوم بأنها لن تتغير غداً. وإذا سألت علماء الفيزياء ما هي النظرية الأساسية المقبولة اليوم، فلن تحصل على إجابة واحدة. فحتى العلم له حدود ومن يقول غير ذلك لا يختلف عن كاهن متدين يطلب إيماننا.
    ومن أراد أن يسمع المزيد عن الموضوع فليتابع محاضرتي على الرابط: http://www.youtube.com/watch?v=lr3NEJZHyOA

  687. تطور الفكر
    الفكر الروحي خلق التطور - وبدون الفكر لن تكون هناك نظرية التطور، وهي جملة في العقل المفكر
    - التخطيط الذكي هو الفكر + التكنولوجيا ومنتج اسمه الحياة، الحياة هي صناعة الذكاء فن الذكاء والفكر.

  688. إسكات "دين التطور" تجاه من يفكر خارج الصندوق.

    ووفقا للقليل الذي كتبه، قام البروفيسور إريك هادين بتدريس مقرر دراسي في فلسفة العلوم. كجزء من هذه الدورة، يناقش نظرية بديلة لنظرية التطور، والتي لا يمكن استبعادها تماما، أي أنه يناقش نظرية التصميم الذكي، وربما يناقش أيضا نقاط الضعف في نظرية التطور. وحسب ما فهمت، فإنه لم يناقش إطلاقا إمكانية أن يكون الكائن الذكي الذي يتحكم في الكون يتطلب متطلبات ملموسة لسلوك البشر، لذلك من المستحيل أن نرى مساره الفلسفي على أنه وعظ ديني.

    ماذا ستكون الخطوة التالية لدين التطور؟ هل سيستبعدون كل نظريات وجود كائنات فضائية بحجة أن هذا أيضًا دين؟ ومن الواضح أنهم لن يفعلوا ذلك لأن مثل هذا الطلب من شأنه أن يبرر من يطالب به.

    ولكن - ما هو الفرق الكبير بين النظريات حول الكائنات الفضائية والنظريات حول الكائنات الفضائية التي تتحكم في التطور على الأرض (أي التصميم الذكي)؟ الفرق مجهري فقط.

  689. إذا كانت هناك تجربة جديدة للقردة، وخسرت جورا، فإنني أوصي بأن تقوم في نفس الوقت بتدريس بقية قصص الخلق لجميع الأديان. دعهم يكسروا الرؤوس الدينية وحدهم.

  690. إن معارضي التخطيط الذكي يظهرون قدراً كبيراً من الجهل والغباء لدرجة أنه يزيل أي رغبة في الجدال معهم.
    ومن المأمول أن يعتني الانتقاء الطبيعي بالصفات التي تسمح بوجودها عبر الأجيال.

  691. ليفون تون - أنت على حق. لا يوجد فرق بين الكاميرا والنظام البيولوجي مثل آلية الدوران لمحرك سينسيز ATP على سبيل المثال. يزعم أنصار التطور أن الاختلاف الرئيسي هو أن الكاميرا مصنوعة من مادة غير حية ولا تتكاثر. ومن ثم لا يمكن مقارنتها بالكائنات الحية. أي، وفق هذا المنطق، إذا وجدنا كاميرا مصنوعة من مادة عضوية (خشب مثلاً؟) وتحتوي أيضاً على الحمض النووي وآلية استنساخه. علينا أن نستنتج أن الجرس تطور من تلقاء نفسه. ومن ثم فإن الكاميرا المكررة أو الساعة المكررة أو الروبوت المكرر (القرد؟) ليس مطلوبًا للمبدع.

  692. ولماذا تعتبر الكاميرا تصميمًا ذكيًا والنظام البيولوجي الوظيفي ليس تصميمًا ذكيًا؟ شخص ما يزعج ويعطي تفسيرا ذكيا. ربما تنشأ العمليات التطورية في العقل.

  693. ووفقاً لهذا المنطق، فحتى بلورات الكوازو لم تكن مقبولة لدى المجتمع العلمي. ومن ثم فإن بلورات الكوازو غير موجودة.

    الجميل في العلم هو أنه من الممكن والمرغوب أن نختلف حول النظريات الموجودة. وإلا فإنه ليس العلم. لقد كنت أتابع الموضوع والتبادل منذ أن طرحه كوين. والطريف أن كوين نفسه يطرح ادعاءات تم دحضها في كتابه. على سبيل المثال من هذه المقالة:

    https://www.hayadan.org.il/return-of-evolution-220313/

    "يمكننا التنبؤ متى (أي، في أي وقت في الماضي) ظهر أسلاف مشتركون (على سبيل المثال، اكتشاف "داجريجيل"، وهي سمكة ذات براعم أرجل في الهيكل العظمي، تيكتاليك، في صخور عمرها 370 مليون عام، تم وصفها في الفصل الثاني)"-.

    هل أنت متأكد جيري لأنه في آخر مرة قمت فيها بالتحقق، وجدوا أحفورة لزاحف أرضي قبل حوالي 19 مليون سنة من وجود التيكتاليك:
    http://news.bbc.co.uk/2/hi/science/nature/8443879.stm

    دعنا ننتقل إلى مطالبتك التالية جيري:
    "كل جزيء DNA نقوم بتسلسله، وكل نظام عضوي نقوم بتشريحه - كلها تدعم فكرة أن الأنواع البيولوجية تتطور عن طريق التطور من أسلاف مشتركة."

    هل أنت متأكد جيري لأنه في المرة الأخيرة التي تحققت فيها كانت هناك بعض التناقضات الجزيئية الخطيرة. على سبيل المثال، في دراسة نشرت عام 2009 في أبحاث الجينوم، تم فحص مجموعة مكونة من 20 جينًا مختلفًا من المدرع والفيلة والبشر. كشف اختبار النشوء والتطور عن جميع التناقضات المحتملة:
    وبحسب مجموعة معينة من الجينات تبين أن المدرع أقرب إلى الفيل منه إلى الإنسان، وبحسب مجموعة أخرى تبين أن المدرع أقرب إلى الإنسان منه إلى الفيل، وبحسب مجموعة أخرى تبين أن الفيل في الواقع أقرب إلى البشر منه إلى المدرع. باختصار، ليست شجرة مغطاة بل سلطة كاملة، والتي، بطريقة مضحكة إلى حد ما، تنبأت بها نظرية الخلق.

    "صحيح أن هناك، من حيث المبدأ، عدد لا حصر له من الملاحظات المحتملة التي يمكن أن تثبت أن التطور غير صحيح، ولكن حتى الآن لم يتم اكتشاف ولو ملاحظة واحدة من هذا القبيل."

    كما قيل أعلاه. هذا ببساطة غير صحيح. وتبين أن التطور تم دحضه أيضًا وفقًا للمعايير التي قدمها جيري نفسه. إذن أضحك أم أبكي؟

ترك الرد

لن يتم نشر البريد الإلكتروني. الحقول الإلزامية مشار إليها *

يستخدم هذا الموقع Akismat لمنع الرسائل غير المرغوب فيها. انقر هنا لمعرفة كيفية معالجة بيانات الرد الخاصة بك.